You are on page 1of 522

1 | Bản quyền thuộc về Tạp chí Olympiad Hóa học KEM

2 | Bản quyền thuộc về Tạp chí Olympiad Hóa học KEM


50 chuyên đề Olympiad Hóa học

5
Cấu trúc, hoạt tính &
Cơ chế phản ứng

3 | Bản quyền thuộc về Tạp chí Olympiad Hóa học KEM


Lời mở đầu
Các bạn độc giả thân mến. Trên tay bạn là bộ sách 50 CHUYÊN ĐỀ
OLYMPIAD HÓA HỌC - là tuyển tập các câu hỏi trong đề thi Olympiad quốc
tế và nhiều quốc gia trên thế giới trong những năm gần đây, được phân
chia chi tiết thành 50 chuyên đề nhỏ.
Từ cách đây 15 năm, các [cựu] quản trị viên box Hóa học OlympiaVN (nay
là Tạp chí KEM & website sachhoahoc.xyz) đã bắt đầu biên soạn các tài
liệu tương tự, được lưu hành nội bộ - gọi là các Compilation. Tuy nhiên, 3
bộ Compilation trước đây bị giới hạn về mặt nội dung (chủ yếu là đề thi
HSGQG Việt Nam và IChO, cùng với đề thi Olympiad của khoảng 3, 4
nước), cũng như nhân lực và thời gian có hạn nên sự phân chia các chuyên
mục chưa thực sự chi tiết, chỉ chia thành 7 phần lớn chứ chưa chia nhỏ
thành các mảng chuyên đề sâu hơn. Chính vì vậy, trong năm 2018-2019,
chúng tôi quyết định biên soạn lại bộ sách này, với cập nhật thêm đề thi
từ rất nhiều quốc gia trên thế giới (đặc biệt là những nước có truyền thống
về Olympiad Hóa học như Trung Quốc, Nga và các nước Soviet cũ, các
quốc gia khu vực Baltic, ... ) và quan trọng hơn là phân chia nội dung chi
tiết hơn, với 6 lĩnh vực, 50 chuyên đề - cố gắng bám sát khung chương
trình IChO trong khả năng có thể. Hi vọng rằng, với tuyển tập này, lời đáp
cho câu hỏi: "Có những gì trong đề thi Olympiad Hóa học?" mà rất nhiều
độc giả, đặc biệt là những bạn học sinh THPT, vốn thường thắc mắc - sẽ
phần nào sáng tỏ.
Lưu ý rằng tuyển tập này chọn lọc những câu hỏi từ các đề thi Olympiad,
do đó bạn sẽ cần phải có một nền tảng kiến thức tương đối vững chắc về
Hóa học phổ thông chuyên sâu để trước khi bắt đầu với hành trình chinh
phục kiến thức này. Ngoài ra, do tổng hợp từ nhiều nguồn tài liệu nên
tuyển tập chưa có được sự thống nhất về mặt danh pháp, mong bạn bỏ
qua cho sự bất tiện này.
Chúc bạn tìm thấy những niềm vui trong học tập.

4 | Bản quyền thuộc về Tạp chí Olympiad Hóa học KEM


Mục lục
Chuyên đề 37: Cấu trúc hóa học 11
Bài 1 11
Bài 2 13
Bài 3 14
Bài 4 19
Bài 5 21
Bài 6 22
Bài 7 24
Bài 8 25
Bài 9 28
Bài 10 29
Bài 11 30
Bài 12 31
Bài 13 32
Bài 14 33
Bài 15 34
Bài 16 38
Bài 17 40
Bài 18 41
Bài 19 43
Bài 20 46
Bài 21 48
Bài 22 49
Bài 23 51
Bài 24 54
Bài 25 55
Bài 26 56
Bài 27 57
Bài 28 61
Bài 29 63
Bài 30 66
Bài 31 71
Chuyên đề 38: Ảnh hưởng của cấu trúc đến tính chất 73
Bài 1 73
Bài 2 74
Bài 3 75
Bài 4 76
Bài 5 78
Bài 6 79
Bài 7 82
Bài 8 95
Bài 9 98
Bài 10 99
Bài 11 105

5 | Bản quyền thuộc về Tạp chí Olympiad Hóa học KEM


Bài 12 107
Bài 13 109
Bài 14 111
Bài 15 114
Bài 16 116
Bài 17 118
Bài 18 121
Bài 19 122
Bài 20 124
Bài 21 131
Bài 22 132
Bài 23 133
Bài 24 134
Bài 25 136
Bài 26 137
Bài 27 138
Chuyên đề 39: Phổ học 139
Bài 1 139
Bài 2 143
Bài 3 146
Bài 4 153
Bài 5 155
Bài 6 163
Bài 7 171
Bài 8 175
Bài 9 178
Chuyên đề 40: Cơ chế phản ứng 187
Bài 1 187
Bài 2 188
Bài 3 196
Bài 4 198
Bài 5 206
Bài 6 212
Bài 7 221
Bài 8 223
Bài 9 224
Bài 10 229
Bài 11 232
Bài 12 234
Bài 14 236
Bài 15 237
Bài 16 238
Bài 17 240
Bài 18 241
Bài 19 243
Bài 20 244

6 | Bản quyền thuộc về Tạp chí Olympiad Hóa học KEM


Bài 21 246
Bài 22 247
Bài 23 249
Bài 24 250
Bài 25 251
Bài 26 252
Bài 27 254
Bài 28 256
Bài 29 257
Bài 30 259
Bài 31 260
Bài 33 264
Bài 34 267
Bài 35 270
Bài 36 271
Bài 37 272
Bài 38 273
Bài 39 275
Chuyên đề 41: Xác định sản phẩm phản ứng 276
Bài 1 276
Bài 2 284
Bài 3 287
Bài 4 290
Bài 5 297
Bài 6 299
Bài 7 313
Bài 8 315
Bài 9 316
Bài 10 317
Bài 11 318
Bài 12 319
Bài 13 320
Bài 14 321
Bài 15 322
Bài 16 324
Bài 17 326
Bài 18 327
Bài 19 328
Bài 20 329
Bài 21 331
Bài 22 333
Chuyên đề 42: Chuỗi phản ứng từ chất đầu đã biết 338
Bài 1 338
Bài 2 341
Bài 3 343
Bài 4 345

7 | Bản quyền thuộc về Tạp chí Olympiad Hóa học KEM


Bài 5 347
Bài 6 348
Bài 7 350
Bài 8 351
Bài 9 353
Bài 10 355
Bài 11 356
Bài 12 358
Bài 14 362
Bài 15 363
Bài 16 365
Bài 17 367
Bài 18 369
Bài 19 372
Bài 20 373
Bài 21 375
Bài 22 376
Bài 23 377
Bài 24 379
Bài 25 380
Bài 26 381
Bài 27 382
Bài 28 383
Bài 29 384
Bài 30 385
Bài 31 387
Bài 32 394
Bài 33 396
Bài 34 397
Bài 35 399
Bài 36 400
Bài 37 408
Bài 38 410
Bài 39 411
Bài 40 413
Bài 41 416
Bài 42 417
Bài 43 418
Bài 44 420
Bài 45 422
Bài 46 423
Bài 47 424
Bài 48 426
Bài 49 427
Bài 50 430
Bài 51 432

8 | Bản quyền thuộc về Tạp chí Olympiad Hóa học KEM


Bài 52 433
Bài 53 434
Bài 53 435
Bài 54 437
Bài 55 439
Bài 56 440
Bài 57 441
Bài 58 442
Bài 59 443
Bài 60 444
Bài 61 445
Bài 62 446
Bài 63 447
Bài 64 448
Bài 66 450
Bài 67 451
Bài 68 452
Bài 69 453
Bài 70 454
Bài 71 455
Bài 72 456
Bài 73 457
Bài 74 458
Bài 75 460
Bài 76 462
Bài 77 465
Bài 78 467
Bài 79 468
Bài 80 469
Bài 81 471
Bài 82 472
Bài 83 473
Bài 84 474
Bài 85 475
Bài 86 476
Bài 87 477
Bài 88 478
Bài 89 480
Bài 90 484
Bài 91 485
Bài 92 486
Bài 93 492
Bài 94 493
Bài 95 494
Bài 96 495
Bài 97 496

9 | Bản quyền thuộc về Tạp chí Olympiad Hóa học KEM


Bài 98 500
Bài 99 504
Bài 100 513
Bài 101 518
Bài 102 523
Bài 103 531
Bài 104 534
Bài 105 537
Bài 106 544
Bài 107 549
Bài 108 551
Bài 109 556
Bài 110 562
Bài 111 567
Bài 112 573
Bài 113 578
Bài 114 584
Bài 115 587
Bài 116 594
Bài 117 598
Bài 118 608
Bài 119 612
Bài 120 615
Bài 121 619
Bài 122 622
Bài 123 624
Bài 124 626
Bài 125 632
Bài 126 635
Bài 127 636
Bài 128 638
Bài 129 640
Bài 130 643
Bài 131 646
Bài 132 649
Bài 133 652
Bài 134 655
Bài 135 658
Bài 136 660
Bài 137 662
Bài 138 667
Bài 139 670
Bài 140 672
Bài 141 675
Bài 142 677
Bài 143 679

10 | Bản quyền thuộc về Tạp chí Olympiad Hóa học KEM


Bài 144 681
Bài 145 682
Bài 146 685
Bài 147 688
Bài 148 691
Bài 149 695
Bài 150 697
Bài 151 699
Bài 152 701
Bài 153 704
Bài 154 706
Bài 155 709
Bài 156 712
Bài 157 715
Bài 158 719
Bài 159 723
Bài 160 728
Bài 161 730
Bài 162 734
Bài 163 738
Bài 164 740
Bài 165 743
Bài 166 745
Bài 167 747
Bài 168 749
Bài 169 751
Bài 170 754
Bài 171 758
Bài 172 760
Bài 173 763
Bài 174 766
Bài 175 769
Bài 176 772
Bài 177 775
Bài 178 779
Bài 179 783
Bài 180 785
Bài 181 787
Bài 182 789
Bài 183 791

11 | Bản quyền thuộc về Tạp chí Olympiad Hóa học KEM


Chuyên đề 37: Cấu trúc hóa học

Bài 1
Trong các bài giảng Hoá học ở trường, đa số hợp chất hữu cơ được vẽ
dưới dạng công thức cấu tạo. Tuy đây là phương án tốt để biểu diễn các
phân tử nhỏ, nhưng nó lại trở nên phức tạp với các phân tử lớn hơn. Với
những phân tử phức tạp, như Progesterone, thì nên biểu diễn chúng theo
công thức cấu tạo thu gọn (hay công thức khung sườn).

Bảng 1 dưới đây là những cách biểu diễn khác nhau của một số phân tử
hữu cơ:

Những phân tử nào ở bảng 1 giống với các phân tử sau?


i) Ethane
ii) Butane

iii)

12 | Bản quyền thuộc về Tạp chí Olympiad Hóa học KEM


iv) trans-but-2-ene

13 | Bản quyền thuộc về Tạp chí Olympiad Hóa học KEM


Bài 2
Do khí propane không có mùi nên cần bổ sung thêm một lượng nhỏ hợp
chất khác, có mùi để nhận biết sự rò rỉ khí gas. Hợp chất thường dùng là
ethyl mercaptan (hay ethanethiol, C 2H5SH). Chất này được chọn dùng bởi
mũi người có thể phát hiện sự có mặt của nó ở hàm lượng chỉ khoảng
0.02 ppb (ppb = phần tỉ).
i) Vẽ cấu trúc của ethyl mercaptan và dự đoán góc liên kết quanh
nguyên tử lưu huỳnh.
ii) Tính khối lượng ethyl mercaptan cần thêm vào 13 kg propane để
thu được tỉ lệ 0.02 phân tử ethyl mercaptan trên 1 tỉ phân tử
propane.

14 | Bản quyền thuộc về Tạp chí Olympiad Hóa học KEM


Bài 3
Nhiều học sinh lầm tưởng rằng các cấu tạo hoá học chỉ được xác định
chính xác sau khi các thiết bị và công cụ phân tích hiện đại ra đời. Sẽ thật
bất ngờ khi biết rằng tới năm 1910, đã xác định được cấu tạo của hơn
200,000 hợp chất hữu cơ! Một trong những hợp chất hữu cơ có lịch sử
thú vị nhất là benzene (C 6H6), có cấu tạo như dưới đây. Benzene là một
hydrocarbon vòng thơm, trong đó tất cả các liên kết carbon-carbon đều
bằng nhau.

a) Xác định công thức thực nghiệm của benzene.


Michael Faraday đã phát hiện benzene và công bố khám phá của mình với
Hiệp hội Hoá học Hoàng gia Anh vào 16 tháng Sáu, 1825. Ông đã phân lập
được một chất gọi là “bicarburet of hydrogen” từ một sản phẩm phụ tạo
ra khi sản xuất “khí thắp sáng”. Sau khi tinh chế hợp chất này bằng cách
chưng cất phân đoạn và kết tinh, ông đã tiến hành một số thí nghiệm để
xác định nó.
Để xác định công thức thực nghiệm, Faraday đã phân tích các sản phẩm
đốt cháy by weight khi dẫn hơi benzene qua đồng(II) oxide nung nóng.
Đồng(II) oxide có thể được tạo thành bằng cách đốt nóng đơn chất đồng
trong oxygen tinh khiết, nhưng phương pháp này cũng có thể tạo thành
một lượng đồng(I) oxide.
b) Viết các phương trình phản ứng tạo ra đồng(I) oxide và đồng(II) oxide
từ các đơn chất.
Dưới đây là một đoạn trích từ bài báo của Faraday, trong đó ông đã cho
hợp chất chưa biết phản ứng hoàn toàn với đồng(II) dioxide, chỉ thu được
carbon dioxide và nước.
0.776 grain chất này tạo ra 5.6 inch3 khí carbonic (ở 60 oC và áp
suất 29.98 inch) và 0.57 grain nước.

15 | Bản quyền thuộc về Tạp chí Olympiad Hóa học KEM


Khí carbonic là tên cũ của carbon dioxide, do nó tan trong nước tạo thành
carbonic acid. “grain” là một đơn vị đo lường cũ, và trong bài toán này
cũng không cần quy đổi nó về đơn vị SI.
c) Tính hàm lượng hydrogen trong nước.
d) Trong hợp chất phân tích có bao nhiêu grain hydrogen?
5.6 inch3 khí tạo thành trong thí nghiệm tương đương với 0.761704 grain
carbon.
e) Tính tỉ lệ khối lượng carbon : hydrogen trong mẫu chất này.
f) Công thức thực nghiệm của benzene được xác định theo phương
pháp này là gì?
Faraday cũng xác nhận những kết quả của mình bằng cách cho nổ hơi
benzene trong oxygen.
g) Viết phương trình đốt cháy hoàn toàn (i) carbon, (ii) hydrogen, (iii) hơi
benzene trong oxygen dư.
Dưới đây là ghi chép của Faraday về thí nghiệm nổ hỗn hợp 1 phần hơi
benzene với 19.6 phần khí oxygen. Không phải toàn bộ oxygen đều được
dùng tới, bởi 19.6 phần oxygen là quá dư. Dưới đây là một trích dẫn khác:
7 đơn vị thể tích hỗn hợp này được kích nổ trong một ống nghiệm
bởi một tia lửa điện, và tổng khối lượng giảm còn 6.13 đơn vị thể
tích. Hỗn hợp khí tạo thành được dẫn vào bồ tạt thì lượng chất khí
giảm tiếp còn 4 đơn vị thể tích. Lượng khí còn lại này chính là
oxygen tinh khiết.
Nước lỏng được tạo thành trong phản ứng này và không được tính vào
thể tích khí. Bồ tạt (potash) là tên gọi gọi cũ của potassium hydroxide,
chất này phản ứng với carbon dioxide tạo thành potassium carbonate rắn.
h) Có bao nhiêu đơn vị thể tích carbon dioxide tạo thành trong phản ứng
đốt cháy này?
i) Phản ứng này cần dùng bao nhiêu đơn vị thể tích oxygen?
Thể tích khí tỉ lệ thuận với số mol chất.
j) Từ dữ kiện phản ứng đốt cháy, tính tỉ lệ khối lượng của carbon và
hydrogen trong hợp chất chưa biết này. Từ tỉ lệ này xác định công thức
thực nghiệm của hợp chất.

16 | Bản quyền thuộc về Tạp chí Olympiad Hóa học KEM


Ở thời kì của Faraday, hydrogen chưa được biết tới là một chất khí lưỡng
phân tử, mà được xem là khí đơn nguyên tử. Nghĩa là carbon được gắn
với khối lượng mol 6.00, còn oxygen là 8.00 so với hydrogen – chất có khối
lượng mol 1.00.
k) Faraday đã xác định được khối lượng mol benzene bằng bao nhiêu
nếu chỉ sử dụng dữ kiện phản ứng đốt cháy.
Sau khi xác định được khối lượng mol benzene, thử thách tiếp theo của
các nhà hoá học là hiểu được cấu tạo của nó. Do cấu trúc có độ bất bão
hoà cao và hoạt tính hoá học bất thường, nên đã có rất nhiều tranh cãi về
cấu tạo hoá học thật sự của benzene. Nhiều cấu trúc đã được đề xuất cho
benzene, như dưới đây, và tất cả đều thoả mãn hoá trị carbon.

l) Vẽ một trong các cấu tạo khác của C 6H6 cũng thoả mãn các quy tắc
hoá trị của carbon.
Một trong những cơ sở xác định cấu tạo chính xác là số lượng đồng phân
tạo thành khi thay thế các nguyên tử hydrogen bởi các nguyên tử chlorine.
Các nhà nghiên cứu thấy rằng khi thay thế 1 hydrogen trong benzene bởi
1 chlorine thì chỉ tạo ra duy nhất 1 cấu tạo mới. Nghĩa là tất cả các nguyên
tử hydrogen trong benzene là tương đương nhau.
m) Cấu tạo nào ở trên chỉ tạo ra một sản phẩm thế mono duy nhất?
Sự thay thế 2 nguyên tử hydrogen bởi 2 nguyên tử chlorine sẽ tạo thành
3 sản phẩm thế. Quan sát này cho thấy rằng chỉ có cấu tạo Kekulé và
prismane có khả năng là cấu tạo thật.
n) Vẽ các đồng phân hai lần thế với mỗi cấu tạo Kekulé và prismane.
Ngày nay, chúng ta đã biết cấu tạo Kekulé là chính xác, bởi Van’t Hoff đã
chỉ ra rằng một trong những cấu tạo ở trên có tính quang hoạt, và phản
ứng thế vào benzene thì chẳng có sản phẩm nào có tính quang hoạt cả.
Có thể xem rằng một hợp chất quang hoạt thông thường thì sẽ có 1

17 | Bản quyền thuộc về Tạp chí Olympiad Hóa học KEM


nguyên tử carbon với 4 liên kết đơn, và mỗi liên kết đó gắn với các nhóm
khác nhau.
o) Đồng phân nào bạn vẽ ở trên có tính quang hoạt?

18 | Bản quyền thuộc về Tạp chí Olympiad Hóa học KEM


Bài 4
1) Khi các phân tử polyacetylene gắn với các thuốc, amino acid, đường
hoặc các phân tử nhỏ khác thì nó sẽ có các hoạt tính sinh học nhất
định. Dưới đây là một vài dẫn xuất của polyacetylene được tổng hợp
trong những năm gần đây:

a) Hãy viết cấu trúc của phân tử thuốc carboxylic acid trong A và gọi tên
theo danh pháp.
b) Hãy viết cấu trúc của amino acid trong B và gọi tên theo danh pháp.
c) Hãy viết công thức hình chiếu Fischer cho phân tử đường trong C và
gọi tên theo danh pháp.
2) Liên kết C-O nào dài nhất trong số 4 chất sau. Giải thích.

3) Năm 1995, nhà hóa học Pháp Remi Chauvin - con trai của Yves Chauvin
(chủ nhân giải Nobel Hóa học 2005) - đã đề ra thuật ngữ “carbo-
benzene1”. Carbobenzene là những phân tử được mở rộng, tạo thành
bằng cách chèn các đơn vị C(sp)-C(sp) giữa các liên kết của benzene
mà vẫn duy trì sự đối xứng của phân tử. Các phân tử này đã thu hút
được nhiều sự chú ý của các nhà hóa học, vật lí, sinh học do tiềm năng

1 chính xác “carbomer” - thuật ngữ mang ý nghĩa rộng hơn.

19 | Bản quyền thuộc về Tạp chí Olympiad Hóa học KEM


ứng dụng trong lĩnh vực quang học phi tuyến tính và quang động học
trị liệu.
a) Viết công thức cấu tạo của hai đồng phân carbobenzene (C 18H6) thế
hệ đầu tiên.
b) Trong mỗi phân tử carbobenzene này có bao nhiêu π-electron?

20 | Bản quyền thuộc về Tạp chí Olympiad Hóa học KEM


Bài 5
Khung cacbon của các hợp chất
tecpen được tạo thành từ các phân
tử isopren kết nối với nhau theo quy
tắc «đầu – đuôi». Ví dụ, nếu tạm quy
ước: (đầu) CH2=C(CH3)-CH=CH2
(đuôi) thì phân tử -myrcen (hình
bên) được kết hợp từ 2 đơn vị
isopren.
Dựa vào quy tắc trên, hãy cho biết các chất nào sau đây là tecpen và chỉ
ra các đơn vị isopren trong khung cacbon của các tecpen này.
OH O
O
O
O OH
O
O
O OH
O O
Prostaglan®in PG-H2 O
Acoron
O O HO Axit abi eti c
O
Po®ophyllotoxin

21 | Bản quyền thuộc về Tạp chí Olympiad Hóa học KEM


Bài 6
Xitral (CH3)2C=CHCH2CH2C(CH3)=CHCH=O có trong tinh dầu chanh, gồm 2
đồng phân a và b.
a) Cấu tạo phân tử xitral có tuân theo qui tắc isoprenoit hay không? Hai
chất a và b thuộc loại đồng phân nào? Hãy viết công thức cấu trúc và
gọi tên hệ thống hai đồng phân đó.
b) Để tách riêng hai đồng phân a và b, người ta sử dụng semicacbazit và
axit vô cơ. Hãy nêu vắn tắt quá trình thực nghiệm đó.

22 | Bản quyền thuộc về Tạp chí Olympiad Hóa học KEM


Bài 7
Fanezol (tách được từ hoa linh lan) và các đồng phân lập thể của nó
(C 15H26O, tất cả kí hiệu là Pi với i: 1, 2, 3, …) khi bị ozon phân chế hóa khử
thì đều cho axeton, 2-hiđroxyetanal và 4-oxopentanal. Khi Pi bị đun nóng
với axit thì đều tạo ra hỗn hợp các chất có công thức C 15H24 là sản phẩm
chính của mỗi phản ứng, gọi chung là Qi với i: 1, 2, 3, … Hãy vẽ công thức
cấu trúc các hợp chất Pi.

23 | Bản quyền thuộc về Tạp chí Olympiad Hóa học KEM


Bài 8
Đặc trưng quan trọng nhất của một phân tử, do sự phân bố trong không
gian của điện tích các nguyên tử, là moment lưỡng cực của phân tử, μ.
Moment lưỡng cực được biểu diễn như một vector hướng từ tâm tất cả
các điện tích dương đến tâm tất cả các điện cực âm. Vector này có thể
được biểu diễn như tổng vector của các moment lưỡng cực của các liên
kết hóa học. Ví dụ (xem hình), phân tử CO2 không phân cực (μ = 0), trái
ngược với phân tử nước phân cực (μ ≠ 0).

1) Mỗi cột trong bảng sau đây liệt kê cấu trúc hoặc tên của 3 phân tử
phân cực (μ ≠ 0) hoặc không phân cực (μ = 0). Chỉ rõ chất nào trong số
chúng có moment lưỡng cực khác 0.

2) Với mỗi hợp chất dưới đây, hãy vẽ công thức cấu tạo của dẫn xuất
dichloro không phân cực hoặc chỉ rõ “0” nếu không có cấu trúc phù
hợp.

Để ước lượng tương đối moment lưỡng cực của phân tử, các giả định sau
đã được sử dụng: 1) thế một nguyên tử bằng một nguyên tử halogen
không làm thay đổi góc hóa trị trong phân tử; 2) moment lưỡng cực tổng
chỉ được cộng từ các moment lưỡng cực của các liên kết carbon-halogen.
3) Sử dụng các giả định này để so sánh moment lưỡng cực trong các cặp
chất sau: a) CHF3 và CH3F; b) 1,2-difluorobenzene và 1,3-
difluoropropa-1,2-diene. Câu trả lời được xác thực bằng cách tính giá
trị của μ, giả sử rằng moment lưỡng cực của liên kết C-F, bất kể vị trí
nào của nó trong phân tử, là a.

24 | Bản quyền thuộc về Tạp chí Olympiad Hóa học KEM


Việc xác định bằng thực nghiệm moment lưỡng cực của một phân tử dựa
vào sự đo các tính chất điện của các hỗn hợp khí và dung dịch chứa chất
cần xác định.
4) Moment lưỡng cực đo được của các đồng phân dichloroethane phụ
thuộc chủ yếu vào nhiệt độ, và với các đồng phân khác sự phụ thuộc
ấy gần như biến mất. Xác định bản chất mối liên hệ này.

25 | Bản quyền thuộc về Tạp chí Olympiad Hóa học KEM


Bài 9
Cho các hợp chất X và Y có công thức như trong hình vẽ. So sánh (có giải
thích) momen lưỡng cực của các hợp chất X và Y.

26 | Bản quyền thuộc về Tạp chí Olympiad Hóa học KEM


Bài 10
a) Các alkene có thể tạo ra đồng phân cấu hình (đồng phân hình học).
Các alkene có hai yêu cầu cần thỏa mãn để tồn tại loại đồng phân này.
Giải thích ngắn gọn tại sao 1-chloropropene có đồng phân hình học
còn 2-chloropropene thì không.
b) Vài nhóm hợp chất khác cũng thỏa mãn các điều kiện có đồng phân
hình học, một trong số đó là các cycloalkane. Hãy cho biết các hợp
chất sau có hay không có đồng phân hình học. Vẽ cấu trúc các cặp
đồng phân hình học đó và cho biết chúng có cấu hình cis hay trans.

27 | Bản quyền thuộc về Tạp chí Olympiad Hóa học KEM


Bài 11
Vẽ công thức phối cảnh và công thức chiếu Niumen của hợp chất 7,7-
dimetylbixiclo[2.2.1]heptan.

28 | Bản quyền thuộc về Tạp chí Olympiad Hóa học KEM


Bài 12
Một bài báo công bố vào tháng 5/2003 cho biết một phân tử alkane mới
được tìm thấy trong dầu có cấu trúc tương tự kim cương, được gọi là “kim
cương phân tử” và quy trình tổng hợp chất này có thể dùng làm hình mẫu
lí tưởng cho tổng hợp vật liệu nano. Cấu trúc phân tử này là

a) Xác định công thức phân tử hydrocarbon.


b) Phân tử này có tâm đối xứng không?
c) Có bao nhiêu loại nguyên tử carbon khác nhau trong phân tử này?
d) Phân tử này có chiral carbon (carbon bất đối) không?
e) Phân tử này có tính quang hoạt không?

29 | Bản quyền thuộc về Tạp chí Olympiad Hóa học KEM


Bài 13
Các hợp chất A, B, và C là các đồng phân. Phân tích nguyên tố thì ta thấy
hàm lượng của carbon chiếm 92.3% và hydrogen chiếm 7. 7%. Đốt cháy
hoàn toàn 1 mol A trong oxygen thu được 179.2 L carbon dioxide (điều
kiện chuẩn). Biết rằng:
- A là một hợp chất thơm, tất cả các nguyên tử của phân tử A nằm
trên cùng một mặt phẳng.
- B là một hợp chất mạch thẳng có hai nhánh, có hai loại nguyên tử
hydrogen trong phân tử, moment lưỡng cực của phân tử này bằng
0.
- C là ankan, chỉ có một loại carbon duy nhất trong phân tử.
1) Viết công thức phân tử của hợp chất A, B và C.
2) Viết công thức cấu tạo của hợp chất A, B và C.

30 | Bản quyền thuộc về Tạp chí Olympiad Hóa học KEM


Bài 14
Hợp chất A là hóa chất quan trọng để sản xuất thuốc nhuộm, vật liệu
quang điện và thuốc điều trị mụn cóc. A chứa các nguyên tố thuộc chu kì
thứ nhất và thứ hai, là tinh thể trắng có khối lượng mol 114.06 gam mol -
1 , điểm nóng chảy 293 oC và có hằng số acid pK = 1.5, pK = 3.4. Anion
a1 a2
A2- chỉ có 2 loại liên kết hóa học và có trục đối xứng bậc 4.
a) Vẽ cấu tạo của A.
b) Tại sao trong A2- chỉ có 2 loại liên kết?
c) Anion A2- có bao nhiêu ảnh gương?

31 | Bản quyền thuộc về Tạp chí Olympiad Hóa học KEM


Bài 15
Trong số các hợp chất có công thức phân tử C4H6 thì nổi tiếng nhất là buta-
1,3-diene, với sản lượng toàn cầu vào khoảng 12,3 triệu tấn mỗi năm. Hợp
chất này được nhà hóa học Pháp Joseph Caventou phân lập lần đầu tiên
vào năm 1863, và tới năm 1886 thì nhà hóa học Anh Henry Armstrong đã
xác định được cấu trúc. Tuy nhiên, với giới học giả, hợp chất này gắn liền
với tên tuổi của hai nhà hóa học Nga: Sergei Vasilyevich Lebedev and Ivan
Ivanovich Ostromyslensky. Hai nhà khoa học này đã phát triển các
phương pháp tổng hợp buta-1,3-diene, tạo tiền đề cho việc sản xuất cao
su công nghiệp.

Sergei Lebedev và Ivan Ivanovich Ostromyslensky


Phương pháp của Lebedev được công bố ở liên bang Soviet và sử dụng ở
toàn Đông Âu và Brazil cho tới cuối thập niên 1970. Trong phương pháp
này, ethanol được dẫn qua xúc tác SiO 2-Al2O3 ở 370-450 oC. Ưu điểm của
quá trình này nằm ở tính đơn giản và độ chọn lọc đạt tới 70 %. Phương
pháp của Ostromislensky được giới thiệu lần đầu vào năm 1942 ở Mỹ, và
hiện vẫn được sử dụng ở Trung Quốc và Ấn Độ. Đây là một quá trình hai
giai đoạn: Ở giai đoạn đầu, ethanol bị oxid hóa với xúc tác Cu2Cr2O5 ở 200
oC, tạo thành acetic aldehyde. Tiếp đó, hỗn hợp đẳng mol của acetic

aldehyde và ethanol được dẫn qua xúc tác SiO 2-Ta2O5 để thu được
butadien với hiệu suất cao.
1) Viết các phương trình phản ứng tương ứng với các phương pháp trên.

32 | Bản quyền thuộc về Tạp chí Olympiad Hóa học KEM


2) Xác định vai trò của các xúc tác trong những quá trình trên.
Xúc tác/Vai trò Dehydrogen hóa Dehydrate hóa
SiO2-Al2O3
Cu2Cr2O5
SiO2-Ta2O5
Mặc dù hai quy trình này đã được biết đến và thương mại hóa từ lâu
nhưng cơ chế chi tiết của chúng vẫn đang được nghiên cứu. Ví dụ, tính
toán cơ lượng tử của các phản ứng trên bề mặt xúc tác đã được các nhà
hóa học Trung Quốc thực hiện vào năm 2015. Tuy nhiên, tất cả các nghiên
cứu đều chỉ ra rằng các quy trình của Lebedev và Ostromyslensky đều diễn
ra theo cùng một sơ đồ, và giai đoạn đầu tiên trong quy trình của Lebedev
là phản ứng oxid hóa ethanol. Nhìn chung thì sơ đồ này được tóm lược
như sau:
2СH3CHO → A;
A → B + H2O;
B + C 2H5OH → C + CH3CHO;
С → CH2=CH–CH=CH2 + H2O.
3) Xác định công thức cấu tạo của các chất A-C.
4) Ngoài buta-1,3-diene, vẫn còn có một số hydrocarbon khác cùng công
thức C4H6. Xác định công thức cấu tạo của chúng. Những hợp chất này
có tính quang hoạt không?
5) Ở thời điểm hiện tại, tất cả các đồng phân C 4H6 có thể có đều đã được
tổng hợp và nghiên cứu chi tiết. Dưới đây là bảng dữ liệu về một số
tính chất của chúng. Hãy bổ sung vào các ô trống.
Số loại Số loại
Sự tồn tại Sự tồn tại
Cấu proton carbon
STT trục đối mặt phẳng
tạo tương tương
xứng đối xứng
đương đương
1 3 2 Có Có
2 3 4 Không Có
3 3 4

33 | Bản quyền thuộc về Tạp chí Olympiad Hóa học KEM


4 3 3
5 2 3
6 2 2
7
8 1

34 | Bản quyền thuộc về Tạp chí Olympiad Hóa học KEM


Bài 16
Có rất nhiều hợp chất chứa nitrogen được tìm thấy trong thế giới động-
thực vật, điển hình là: alkaloid, nucleic acid, vitamin, … và các hợp chất
này đều có ảnh hưởng sinh lý.
Trung tâm lập thể có thể là nguyên tử hoặc nhóm nguyên tử, trong đó sự
thay đổi vị trí của 2 nhóm bất kì tạo thành một đồng phân lập thể khác.
Các hợp chất chứa nitrogen cũng có thể có cấu trúc bất đối và có tính
quang hoạt.
Cho ba hợp chất: (I): không quang hoạt; (II): quang hoạt nhưng không tách
riêng các đối quang được; (III): quang hoạt và có thể tách riêng các đối
quang.

1) Hãy cho biết I, II, III ứng với những hợp chất nào ở trên.
Alkaloid là nhóm hợp chất chứa nitrogen có tính base, có nguồn gốc thực
vật. (+)-Muscarine là một alkaloid có độc tính, được tìm thấy trong một
số loại nấm.

2) Vẽ và xác định cấu hình tuyệt đối của đối quang (+)-Muscarine.
Trong thuốc phiện có nhiều alkaloid, một trong số đó là morphine - có thể
dùng làm thuốc giảm đau.

35 | Bản quyền thuộc về Tạp chí Olympiad Hóa học KEM


3) Trong morphine có bao nhiêu tâm bất đối?

36 | Bản quyền thuộc về Tạp chí Olympiad Hóa học KEM


Bài 17
1) Phân tử twistane có cấu trúc như sau. Hãy cho biết nó có tính quang
hoạt hay không. Tại sao?

2) Phân tử nào sau đây có tính quang hoạt:

37 | Bản quyền thuộc về Tạp chí Olympiad Hóa học KEM


Bài 18
trans-Cyclooctene là một phân tử quang hoạt với hàng rào năng lượng
racemic hóa cao. Liên kết đôi của trans-cyclooctene bị vặn xoắn, và kết
quả là phân tử thể hiện hoạt tính bất thường trong các phản ứng cộng
vòng.
Năm 2011, Fox và các cộng sự đã phát triển một quy trình tổng hợp quang
hóa đối với một số dẫn xuất của trans-cyclooctene. Quy trình này không
có tính đặc thù lập thể và sơ đồ tổng hợp được cho dưới đây:

1) Vẽ tất cả các đồng phân lập thể có thể có của hợp chất 3 có thể nhận
được từ phản ứng khử hợp chất 2. Không cần xác định các cấu hình
tuyệt đối.
2) Giả sử rằng một trong các đồng phân lập thể của hợp chất 3 bị chuyển
thành hợp chất 4. Hãy cho biết có thể nhận được bao nhiêu dạng đồng
phân lập thể của hợp chất 4.
3) Bây giờ, giả sử rằng có nhiều hơn 1 đồng phân lập thể, vậy có thể tách
các đồng phân lập thể của hợp chất 4 bằng phương pháp sắc kí achiral
(không quang hoạt) được không?

38 | Bản quyền thuộc về Tạp chí Olympiad Hóa học KEM


Bài 19
Xác định công thức cấu tạo các hydrocarbon I - X, tương ứng với những
đặc điểm cho trong bảng dưới đây (trong mỗi trường hợp chỉ cần một câu
trả lời là đủ).
Tổng số đồng phân (tính cả
Thành phần phân Sự tồn tại đồng phân lập thể) trong
Hydro-
tử và đặc điểm đồng phân dãy các dẫn xuất thế của
carbon
cấu trúc quang học hydrocarbon nếu các nhóm
thế giống nhau
C5 Mono-một, di-hai, tri-ba,
I Không
mạch hở tetra-bốn
C7 Mono-một, di-bốn (trong
II mạch hở phân Không đó có hai đồng phân quang
nhánh học)
Mono-hai, di-bảy (trong đó
III C5 Không có hai đồng phân quang
học)
C10 Mono-hai (không quang
IV Không
Ba vòng hoạt)
C9
V Không Mono-hai (quang hoạt)
Bốn vòng

C12 Mono-hai, di-năm (trong


VI Không đó có hai đồng phân quang
Bốn vòng học)
C17
VII Không Mono-hai (quang hoạt)
Sáu vòng
C24
VIII Không Mono-một
Bảy vòng
IX C50H100 Không Mono-một

39 | Bản quyền thuộc về Tạp chí Olympiad Hóa học KEM


Hai vòng, tất cả
các nguyên tử
cacbon đều bậc 2
C50H100

X Một vòng, tất cả Có


các nguyên tử
cacbon đều bậc 2

40 | Bản quyền thuộc về Tạp chí Olympiad Hóa học KEM


Bài 20
1) Hãy viết một công thức cấu tạo của hiđrocacbon X (C 14H26) mà có hơn
50 đồng phân quang học.
2) Vẽ các cấu trúc đồng phân có cùng công thức phân tử C4H8O trong các
trường hợp sau:
a) Là các đồng phân hình học.
b) Là các đồng phân quang học.
c) Vừa là đồng phân hình học, vừa là đồng phân quang học.
3) Artemisinin là một dược chất được chiết xuất bởi các nhà khoa học
Trung Quốc vào năm 1972 từ cây thanh hao hoa vàng (Artemisia annua)
và cấu trúc của nó đã được khẳng định lại vào năm 1976.Theo một bài
báo đăng đăng trên tạp chí Nature của Anh vào tháng 8 năm 2003,
artemisinin tác dụng trực tiếp lên màng tế bào tế bào ký sinh trùng sốt rét
và có tính kháng khuẩn. Vì thời gian bán huỹ ngắn nên ký sinh trùng sốt
rét không có khả năng kháng thuốc. Thêm vào đó, arteminisin không có
các tác dụng phụ của thuốc chống sốt rét quinine. Năm 2004 Artermisin
đã được trao giải thưởng cao quý ở Thái Lan để ghi nhận những hiệu quả
mang lại của thuốc này. Cấu trúc của artemisinin được cho trong hình
dưới đây.
a) Cho biết các nhóm chức có trong phân tử artemisinin.
b) Phân tử artemisinin có bao nhiêu nguyên tử carbon bất đối xứng?

41 | Bản quyền thuộc về Tạp chí Olympiad Hóa học KEM


Bài 21
LSD hay đietylamit của axit lysergic, có
công thức cấu trúc ở dưới, là một chất
gây ảo giác rất mạnh và bị nghiêm cấm sử
dụng.
a) Xác định cấu hình tuyệt đối của LSD.
b) Chỉ rõ nguyên tử nitơ có tính bazơ
mạnh nhất.
c) Phân tử LSD chứa những nhóm chức
gì?

42 | Bản quyền thuộc về Tạp chí Olympiad Hóa học KEM


Bài 22
1) Verapamile là một loại thuốc ngăn ngừa canxi được sử dụng để điều trị
cao huyết áp và rối loạn nhịp tim. Cấu trúc của Verapamile được trình bày
dưới đây, nó có một trung tâm lập thể.

Xác định cấu hình tuyệt đối của trung tâm lập thể này.
2)
a) Hợp chất sau đây có bao nhiêu đồng phân lập thể:

b) Hãy vẽ cấu trúc một đồng phân của chất này và xác định rõ cấu hình
tuyệt đối tại các nguyên tử carbon bất đối xứng.

43 | Bản quyền thuộc về Tạp chí Olympiad Hóa học KEM


Bài 23
1) cis-1-Đecyl-2-(5-metylhexyl)etilenoxit (hay disparlure) là một
pheromon của một loài bướm. Hãy: a) Vẽ công thức các đồng phân
lập thể của dispalure.
2) Viết công thức các sản phẩm có thể tạo thành khi thực hiện quá trình
đồng phân hoá B (có công thức cấu tạo cho dưới) trong môi trường
axit. Gọi tên cấu hình (nếu có) của các chất và cho biết đồng phân nào
có tính quang hoạt.

B CH3

3) Viết các đồng phân lập thể của metylxiclohexanon. Đồng phân nào có
tính quang hoạt? Giải thích vì sao dưới tác dụng của bazơ, xeton
quang hoạt bị raxemic hoá?
4) Hãy cho biết hợp chất dưới đây có bao nhiêu đồng phân lập thể và
biểu diễn cấu trúc không gian của các đồng phân lập thể đó:

5) Vẽ công thức cấu trúc của các dẫn xuất 1,4-đioxan là sản phẩm đime
hóa hợp chất (R)-1,2-epoxi-2-metylpentan.

44 | Bản quyền thuộc về Tạp chí Olympiad Hóa học KEM


Bài 24
Gọi tên các chất sau theo danh pháp IUPAC, kèm theo kí hiệu cấu hình
tuyệt đối (R/S).
Hợp chất X A B

Cấu trúc

Hợp chất C D E

Cấu trúc

45 | Bản quyền thuộc về Tạp chí Olympiad Hóa học KEM


Bài 25
a) Styryllacton được phân lập từ thực vật có công thức.
6 5 4
O
7 3
9 O
8
O
1 2
HO

Viết công thức cấu dạng các cặp đồng phân đối quang và gọi tên
styryllacton theo danh pháp IUPAC.
b) Khi hiđro hóa naphtalen thu được đecalin (C 10H18), oxi hóa đecalin thì
được hỗn hợp các đecalon (C 10H16O). Viết các đồng phân lập thể của
1-đecalon.

46 | Bản quyền thuộc về Tạp chí Olympiad Hóa học KEM


Bài 26
a) Cho hợp chất CH3CH=C(CH3)COCH3. Vẽ tất cả các công thức cấu trúc
bền và viết tên của một trong các công thức cấu trúc tìm được của
hợp chất.
b) Hợp chất A có công thức:

Hãy gọi tên A và cho biết A có bao nhiêu dạng cấu trúc không gian tương
đối bền, các dạng đó khác nhau về các yếu tố lập thể nào? Hãy viết công
thức cấu trúc của hai dạng tiêu biểu, có ghi đầy đủ các ký hiệu lập thể
thích hợp.

47 | Bản quyền thuộc về Tạp chí Olympiad Hóa học KEM


Bài 27
(-)-Menthol là hợp chất thiên nhiên quan trọng, được sử dụng trong nhiều
ngành công nghiệp như thực phẩm, dược phẩm, mỹ phẩm và thuốc lá.
Sản lượng toàn cầu của hợp chất này vượt quá 10000 tấn mỗi năm. (-)-
Menthol là một trong các đồng phân lập thể của 2-isopropyl-5-
methylcyclohexanol.
1. Vẽ cấu tạo của 2-isopropyl-5-methylcyclohexanol và xác định số đồng
phân lập thể của hợp chất này.
2. Biểu diễn cấu trúc của (-)-menthol biết rằng tên gọi đầy đủ của hợp
chất này là (1R, 2S, 5R) -2-isopropyl-5-methylcyclohexanol.
3. Độ quay quang riêng của (-)-menthol là -50.2o. Vẽ cấu trúc của đồng
phân lập thể có độ quay quang riêng là +50.2o.
4. Từ (-)-menthol có thể điều chế đồng phân lập thể B của nó qua quá
trình đồng phân hoá hai giai đoạn, sử dụng phản ứng Mitsunobu. Hình
dưới là sơ đồ chuyển hoá và phổ IR của (-)-menthol và hợp chất A.
Đánh giá cả hai phổ này bằng cách tập trung vào những khác biệt của
chúng và giải thích cho các khác biệt đó. Xác định nhóm chức nào
được thêm vào hợp chất A. Vẽ công thức cấu tạo của hợp chất A và
đồng phân lập thể B. Có thể nhận xét gì về hoá lập thể của phản ứng
Mitsunobu? (Chú ý rằng trong phản ứng này, cấu hình ở vị trí 2 và 5
trong vòng menthol không thay đổi.)

reakcja Mitsunobu = phản ứng Mitsunobu; hydroliza = thuỷ phân


DEAD - diethyl azodicarboxylate (C 2H5OOC-N=N-COOC 2H5); PPh3 -
triphenylphosphine

48 | Bản quyền thuộc về Tạp chí Olympiad Hóa học KEM


5. (-)-Menthol và đồng phân B được chuyển thành các chloride tương
ứng (chuyển nhóm -OH thành -Cl) mà không làm thay đổi cấu hình.
Thực hiện phản ứng tách với hai dẫn xuất halide này. Một trong số các
chất đầu chỉ tạo ra duy nhất một sản phẩm, trong khi đó chất còn lại
tạo ra hỗn hợp sản phẩm với tỉ lệ 3:1. Viết các phản ứng tách của hai
hợp chất, và giải thích sự tạo thành các sản phẩm. Trong trường hợp
thứ hai, hãy cho biết đâu là sản phẩm chính.

49 | Bản quyền thuộc về Tạp chí Olympiad Hóa học KEM


Bài 28
Khi đun nóng nhanh tới 850 oC thì 1,2-dichloroethane, ClCH2-CH2Cl, bị
phân hủy nhanh, tách ra 1 phân tử hydrogen chloride. Đây là phản ứng
bậc 1. Tốc độ của phản ứng bậc 1 tỉ lệ thuận với nồng độ c của chất phản
ứng: v = kc, trong đó k là hằng số tốc độ và biến thiên nồng độ chất phản
− kt
ứng được mô tả theo phương trình: c t = c0e - trong đó c0 là nồng độ ban
đầu, còn ct là nồng độ sau khi trải qua thời gian t.
1) Viết phương trình phản ứng phân hủy dichloroethane.
Dưới những điều kiện này, chất X được tạo thành dưới dạng sản phẩm
phụ. Do đó, trong công nghiệp, phản ứng được tiến hành ở 500 oC và áp
suất 1 - 3 MPa để tối thiểu hóa sự tạo thành X.
2) Xác định công thức của X.
Phân tử dichloroethane có 2 cấu dạng bền.
3) Vẽ cấu tạo 2 cấu dạng này. Cấu dạng nào có năng lượng thấp hơn? Tại
sao?
Hằng số tốc độ phân hủy của hai cấu dạng dichloroethane này trong cùng
điều kiện phản ứng là khác nhau, k1 = 690 s-1, k2 = 335 s-1. Tỉ lệ nồng độ
của các cấu dạng dichloroethane ở nhiệt độ cao là khoảng 2:1. Tỉ lệ này
không thay đổi trong quá trình phản ứng, do cân bằng giữa các cấu dạng
được thiết lập cực kì nhanh.
4) Sau bao lâu thì ½ lượng dichloroethane bị phân hủy?
5) Nồng độ ở cấu dạng nào cao hơn ở nhiệt độ cao hơn? Tại sao?

50 | Bản quyền thuộc về Tạp chí Olympiad Hóa học KEM


Bài 29
Cấu dạng là một trong những khái niệm cơ bản của Hoá lập thể hiện đại.
Khái niệm này đặc trưng cho sự sắp xếp tương hỗ giữa các nguyên tử
trong một phân tử, có thể bị thay đổi bởi sự quay và bẻ cong các liên kết.
Có hai loại công thức chiếu được sử dụng để biểu diễn các cấu dạng:

Khi thảo luận về conformational behavior của phân tử, các cấu dạng quan
trọng nhất tương ứng với giá trị năng lượng nhỏ nhất (các cấu dạng bền)
và lớn nhất (các trạng thái chuyển tiếp) trên mặt thế năng. Sự phụ thuộc
thế năng vào góc quay của ethane, propane, n-butane và cyclohexane
được cho dưới đây:

1) Gán các hợp chất trên với các biểu đồ I - IV tương ứng. Chú ý rằng
năng lượng tương tác giữa các nhóm không liên kết H-CH3 (EH-CH3) >
H-H (EH-H).
2) Vẽ công thức chiều Newman của các cấu dạng không tương đương và
các trạng thái chuyển tiếp của butane và propane. Xác định năng
lượng tương đối của các cấu dạng này.

51 | Bản quyền thuộc về Tạp chí Olympiad Hóa học KEM


3) Tính EH-H, EH-CH3, ECH3-CH3 trong các cấu dạng che khuất của các alkane
và E’CH3-CH3 trong các cấu dạng xen kẽ (biết rằng các năng lượng này
trong các alkane là bằng nhau).
4) ∆So của sự chuyển n-butane từ cấu dạng đối (bền) thành cấu dạng xen
kẽ (giả bền) là -5.76 kJ/mol·K. Tính tỉ lệ các cấu dạng của n-butane khi
đạt cân bằng ở 25 oC.
5) Ở 800 oC, cyclohexane tồn tại dưới dạng hỗn hợp cân bằng của dạng
xoắn (25 %) và dạng ghế (75 %). Tính phần mol của dạng xoắn của
cyclohexane ở 25 oC. Sử dụng phương trình van’t Hoff cho các tính
toán (giả thiết ∆Ho phụ thuộc vào T).

52 | Bản quyền thuộc về Tạp chí Olympiad Hóa học KEM


Bài 30
Hợp chất có cấu trúc vòng 3 cạnh, tạo nên từ 2 nguyên tử carbon và 1
nguyên tử oxygen thường được gọi là epoxy. Dưới đây là mô tả quy trình
tổng hợp hợp chất epoxy 1.

“Cho hydrocarbon X vào dung dịch chlorine ở nhiệt độ khoảng 35-50 oC,
sau khi phản ứng xảy ra thu được sản phẩm chính là hợp chất 2. Hợp chất
2 được xử lí với một base mạnh, như calcium hydroxide, thì xảy ra phản
ứng tách hydrogen chloride và hợp chất 1 được tạo thành.”
1) Xác định cấu trúc hydrocarbon X.
Hợp chất có bộ khung Cl-C-C-OH, như hợp chất 2 ở trên, được gọi là
chlorohydrin. Khi cho chlorohydrin phản ứng với base mạnh, thu được
một hợp chất epoxy.
2 nguyên tử carbon liên kết với các nhóm hydroxyl và chloro trong hợp
chất 2 được nối liên với nhau bởi một liên kết đơn và có thể quay tự do
quanh trục liên kết. Lúc này, khi một nguyên tử carbon được cố định và
nguyên tử còn lại quay thì cấu trúc mà trong đó các nhóm thế (nguyên
tử/nhóm nguyên tử) xen kẽ (2a và 2’’a) và che khuất (2’a) lẫn nhau khi sự
quay diễn ra được cho dưới đây. Trong hình này, các đường nét đậm biểu
diễn các liên kết hướng lên trên mặt phẳng giấy (mặt phẳng chứa liên kết
C-C trung tâm), còn đường nét đứt biểu diễn các liên kết hướng xuống
dưới.

“Công thức chiếu Newman” là phương pháp đơn giản và thuận tiện để
biểu diễn sự quay giống như sự quay quanh trục C-C trên. Như mô tả
trong hình 1, công thức chiếu Newman là cách biểu diễn phân tử được
nhìn theo hướng xuyên quatrục quay và nguyên tử carbon đặt xa hơn
điểm quan sát được biểu diễn ở dạng quả cầu đặc.

53 | Bản quyền thuộc về Tạp chí Olympiad Hóa học KEM


Hình 1: Cách vẽ công thức chiếu Newman.
Chlorohydrin 2a, 2’a,2’’a và hợp chất epoxy 1a ở trên được biểu diễn bởi
các công thức chiếu Newman tương ứng 2b, 2’b, 2’’b và 1b dưới đây. (Với
các dạng che khuất, để thuận tiện [dễ nhìn] thì sẽ được biểu diễn theo
kiểu hơi lệch một chút.)

Còn nếu xử lí hợp chất 3 - một chlorohydrin khác với hợp chất 2 - với base
mạnh thì chỉ có duy nhất epoxy 4 được tạo thành.

2) Với các hợp chất 5 và 6, mỗi trường hợp tạo ra một hợp chất epoxy.
Nếu bạn biểu diễn các cấu trúc đó như dạng liên kết nét đậm-nét đứt
kiểu hợp chất 7, thì hãy cho biết các nhóm (a), (b), (c) là gì?

3) 8 có 2 nguyên tử chlorine. Khi phản ứng với base mạnh thì chỉ có duy
nhất một nguyên tử chlorine phản ứng, do đó epichlorohydrin - là hợp
chất epoxy có 1 nguyên tử chlorine - có thể được tạo thành. Liệu các
epochlorohydrin được tạo thành bởi phản ứng của mỗi nguyên tử
chlorine trong 8 sẽ giống hay khác nhau? Giải thích. (Ở đây chúng ta
không nói đến sự khác biệt gây ra bởi các đồng vị của các nguyên tử
chlorine.)

54 | Bản quyền thuộc về Tạp chí Olympiad Hóa học KEM


Hợp chất epoxy có sự biến dạng không gian (ba chiều) bởi kích thước nhỏ
của vòng 3 cạnh và nó có xu hướng gây ra phản ứng mở vòng để loại bỏ
sự biến dạng đó. Ví dụ, hợp chất epoxy 9 phản ứng với phenol khi có mặt
base mạnh, tạo thành hợp chất 10.

Nghiên cứu kĩ về phản ứng giữa phenol và hợp chất epoxy 9 thì thấy rằng
hiện tượng trong đó phenoxide ion (C 6H5O-) đã tiếp cận với vòng 3 cạnh
từ hướng bên kia (so với oxygen).

Hình 2: Phản ứng của hợp chất epoxy 9 với phenol.


Còn nếu phản ứng với phenol, có mặt base mạnh, được tiến hành với hợp
chất 4 thì tạo thành hợp chất 11.

4) Biểu diễn cấu trúc sản phẩm của phản ứng giữa hợp chất epoxy 12 với
phenol khi có mặt base mạnh theo công thức chiếu Newman (kiểu
như hợp chất 13). Xác định các nhóm (a), (b), (c), (d), (e).

55 | Bản quyền thuộc về Tạp chí Olympiad Hóa học KEM


Epichlorohydrin có hoạt tính độc đáo và phản ứng lần lượt với 2 phân tử
phenol khi có mặt base như sau:

Khi phản ứng tương tự được tiến hành với các phenol có 2 nhóm hydroxyl
thay vì phenol đơn giản thì phản ứng trùng hợp diễn ra và một hợp chất
epoxy gọi là nhựa resin được tạo thành. Nhựa resin được dùng làm sơn,
vật liệu bán dẫn, keo dính, …. và các hợp chất polymer quen thuộc trong
đời sống thường này. Một trong các phương pháp tổng hợp được cho
dưới đây. (NaCl và H2O tạo thành bởi các phản ứng được loại bỏ [không
biểu diễn] trong sơ đồ.)

5) Hoàn thành phản ứng tổng hợp nhựa resin ở trên. Chỉ cần biểu diễn
phần cấu trúc trong đường gợn sóng.

56 | Bản quyền thuộc về Tạp chí Olympiad Hóa học KEM


Bài 31
Mặc dù cấu trúc khung của cyclohexane (C 6H12) thường được biểu diễn ở
dạng lúc giác đều nhưng thực chất thì nó là một phân tử tồn tại ở nhiều
dạng hình học khác nhau, gọi là các cấu dạng. Hai trong số các cấu dạng
đó là dạng ghế và thuyền xoắn (twist-boat), được biểu diễn dưới đây. Các
nguyên tử carbon trong vòng 6 cạnh được đánh số 1-6 để biểu diễn mối
liên hệ giữa chúng.

Dạng thuyền là cấu dạng có mức năng lượng thấp nhất của cyclohexane
với tất cả các góc liên kết gần như bằng góc liên kết lí tưởng (đối với
nguyên tử carbon tứ diện). Dạng thuyền xoắn có mức năng lượng cao
hơn.
a) Góc liên kết C-C-C lí tưởng trong dạng ghế của cyclohexane khác bao
nhiêu độ so với góc của lục giác đều.
Phân tử adamantane có thể được biểu diễn trực quan bằng cách thêm 4
nguyên tử carbon vào cấu dạng ghế của cyclohexane như dưới đây. Khi
đó, các vòng 6 cạnh khác được tạo thành. Trong adamantaen, tất cả các
vòng sáu dạng đều bị “khóa” theo cấu dạng ghế.

b) Với mỗi vòng 6 cạnh trong adamantane, hãy viết số thứ tự của 6
nguyên tử carbon trong vòng theo trật tự liên kết với nhau, bắt đầu
từ số nhỏ nhất. Ví dụ -1-2-3-4-5-6-
c) Có bao nhiêu tín hiệu trong phổ 13C NMR của adamantane (nghĩa là có
bao nhiêu loại carbon khác nhau)?
Phân tử twistane có thể được biểu diễn trực quan bằng cách thêm 4
nguyên tử carbon vào cấu dạng thuyền xoắn của cyclohexane như dưới
đây. Khi đó, các vòng 6 cạnh khác được tạo thành. Trong twistane, tất cả

57 | Bản quyền thuộc về Tạp chí Olympiad Hóa học KEM


các vòng sáu dạng đều bị “khóa” theo cấu dạng thuyền xoắn - đây chính
là nguồn gốc tên gọi của nó.

d) Với mỗi vòng 6 cạnh trong twistane, hãy viết số thứ tự của 6 nguyên
tử carbon trong vòng theo trật tự liên kết với nhau, bắt đầu từ số nhỏ
nhất. Ví dụ -1-2-3-4-5-6-
e) Có bao nhiêu tín hiệu trong phổ 13C NMR của twistane (nghĩa là có bao
nhiêu loại carbon khác nhau)?
f) Adamantane và twistane là các đồng phân của nhau. Hãy xác định
công thức phân tử.

58 | Bản quyền thuộc về Tạp chí Olympiad Hóa học KEM


Chuyên đề 38: Ảnh hưởng của cấu trúc đến tính chất

Bài 1
Axit fumaric và axit maleic có các hằng số phân ly nấc 1 (k1), nấc 2 (k2).
Hãy so sánh các cặp hằng số phân ly tương ứng của hai axit này và giải
thích

59 | Bản quyền thuộc về Tạp chí Olympiad Hóa học KEM


Bài 2
1) Hằng số acid (Ka) của cis- và trans-butenedioic acid (hay maleic acid và
fumaric acid) là 1.17·10-2, 9.3·10-4, 2.9·10-5 và 2.60·10-7. Hãy gán các
hằng số này vào vị trí tương ứng trong cấu tạo của các acid. Giải thích
ngắn gọn.
2) Tính toán lý thuyết cân bằng hai dạng axit formic cho kết quả như sau

Biết pKa dạng Z là 3.77, như vậy pKa dạng E sẽ là: a) > 3.77; b) < 3.77;
c) = 3.77; d) không xác định.

60 | Bản quyền thuộc về Tạp chí Olympiad Hóa học KEM


Bài 3
Hãy qui kết các giá trị pKa 3,15 và 8,23 cho từng nhóm chức trong phân
tử đipeptit Gly-Ala. Viết công thức cấu tạo của đipeptit này khi ở pH= 4,0
và pH= 11,0.

61 | Bản quyền thuộc về Tạp chí Olympiad Hóa học KEM


Bài 4
a) So sánh tính bazơ của các hợp chất sau và giải thích:
CH3-CH(NH2)-COOH (I) ; CHC-CH2-NH2 (II) ; CH2=CH-CH2-NH2 (III) ; CH3-
CH2-CH2-NH2 (IV).
b) So sánh (có giải thích) tính bazơ của các hợp chất A và B dưới đây:

N
C6H5-CHOH-CH2NH- C6H5-CHOH-CH2NH-
A N
B
c) So sánh (có giải thích) tính bazơ của hai hợp chất X, Y dưới đây:

d) Viết công thức cấu tạo của các chất sau đây và sắp xếp theo thứ tự
tăng dần tính bazơ: pyridin, các aminopyridin, 3-clopyridin, 3-
nitropyridin và giải thích ngắn gọn.

62 | Bản quyền thuộc về Tạp chí Olympiad Hóa học KEM


Bài 5
1) Trong số các chất: B1, B2 và B3, chất nào có lực bazơ lớn nhất? Giải
thích.

2) Cho biết chất nào có tính base mạnh nhất trong số các chất sau đây.
Giải thích.

63 | Bản quyền thuộc về Tạp chí Olympiad Hóa học KEM


Bài 6
Các bioisostere (hay các chất đẳng cấu sinh học2) là những hợp chất có
cấu trúc hoá học khác nhau nhưng có hoạt tính sinh lí tương tự.
Ví dụ, các dẫn xuất phenolic và sulfonylamide của phenylethanolamine
(các chất tác động đến thụ thể giao cảm) là những chất đẳng cấu sinh học.
Nguyên nhân của sự tương đồng này là do tính acid của các nhóm
phenolic và sulfonylamide là tương tự nhau. Ngoài ra, NH của
sulfonylamide có thể liên kết với các thụ thể (như trong hình dưới đây)
cũng hiệu quả như OH (phenol) trong ethanolamine.

Các bioisostere được biết đến nhiều là p-aminobenzoic acid (I) và


streptocid trắng (II):

Ở thời điểm hiện tại (năm 2002), kĩ thuật “bioisosteric analogy” (tìm kiếm
đẳng cấu sinh học) được sử dụng phổ biến để nghiên cứu trực tiếp các
loại thuốc mới. Cấu trúc của hợp chất mới được xây dựng từ cấu trúc của
một hợp chất có hoạt tính sinh học đã biết. Một phần của phân tử bị biến
đổi theo cách đặc biệt. Trong chuyển hoá như vậy cần phải biết các thông
số của phân tử đầu và đẳng cấu sinh học của nó: “kích thước” của các
phân tử, khoảng cách giữa các nhóm chức và tính acid-base của chúng.
Chất trung gian gây ức chế nội sinh trong hệ thần kinh trung ương của
người và động vật là 𝛾-aminobutyric acid (GABA). Chất này có mặt trong
tất cả các tế bào thần kinh. Một số chất tương đồng với GABA đã được
nghiên cứu chuyên sâu. Một trong những kĩ thuật của các nghiên cứu này
là tìm kiếm các bioisostere của GABA.

2 Bioisosteric analogue

64 | Bản quyền thuộc về Tạp chí Olympiad Hóa học KEM


1. Xác định công thức cấu tạo của 𝛾-aminobutyric acid.
2. Cho 6 cấu trúc hoá học dưới đây. 3 trong số chúng là các bioisostere
của GABA. Xác định 3 hợp chất này và giải thích lựa chọn của bạn.
Những tính chất nào của các phân tử được chọn xác định tính tương
đồng sinh học của chúng với GABA?

3. Vì sao các cấu trúc còn lại không phải bioisoster của GABA?
4. Các nhóm thế nào trong vòng benzene của các chất A, B ảnh hưởng
đến tính acid-base của các phân tử này.

65 | Bản quyền thuộc về Tạp chí Olympiad Hóa học KEM


Bài 7
Đọc và trả lời các câu hỏi từ (105)-(131).
Khi đề cập đến acid và base, nhiều người sẽ nghĩ đến các hợp chất vô cơ,
ví dụ như hydrogen chloride - một acid mạnh, và sodium hydroxide - một
base mạnh. Mặt khác, cũng có những chất được phân loại hữu cơ nhưng
cũng thể hiện tính acid hoặc base - chúng thường được gọi là các acid và
base hữu cơ. Nhiều phản ứng có sự tham gia của các hợp chất hưu cơ này
cũng được coi là phản ứng acid-base. Trong bài này, chúng ta hãy tìm hiểu
về acid và base trong hóa hữu cơ.
S. A. Arrhenius là người đầu tiên đề ra khái niệm acid-base. Ông nói một
chất giải phóng hydrogen ion (H+ , proton - nhưng thực chất phải là
oxonium, H3O+ , trong dung dịch nước) trong dung dịch nước là acid, còn
chất giải phóng hydroxide ion (OH-) là base. Ví dụ, khi hydrogen chloride
và sodium hydroxide tan trong nước thì xảy ra các quá trình:

Lần lượt, tạo thành H+ và OH-. Định nghĩa của Arrhenius chỉ giới hạn với
các phản ứng trong dung dịch nước và không thể áp dụng với các hệ phi-
nước như dung môi hữu cơ. Mặt khác, hydrogen chloride và ammonia
phản ứng với nhau trong benzene thì có ammonium chloride (NH 4Cl) kết
tủa. Đây rõ ràng là một phản ứng acid-base. Do đó, J. N. Brønsted và T. M.
Lowry đã mở rộng định nghĩa của Arrhenius thành “acid có thể nhường
H+ còn base có thể nhận H+ .” Ví dụ như trong phản ứng của hydrogen
chloride và ammonia trong benzene, thì hydrogen chloride giải phóng H+
và ammonia nhận H+ . Khoảng áp dụng của định nghĩa này rộng hơn và có
thể bao trùm không chỉ phase lỏng mà còn cả phản ứng trong phase khí
và phase rắn. Trong định nghĩa của Brønsted và Lowry, H+ đóng vai trò
chủ động, nhưng G. N. Lewis đã mở rộng định nghĩa này với các phản ứng
thậm chí không có H+ . Ông định nghĩa base là tác nhân nhường (donor)
một cặp electron, còn acid là tác nhân nhận (acceptor) cặp electron. Định
nghĩa của Lewis bao hàm cả định nghĩa của Brønsted và Lowry nhưng vượt
quá những giới hạn. Ví dụ, BH3 và BF3 là các Lewis acid điển hình - cách
phân loại có thể không phù hợp với những chất này nếu xét theo các định
nghĩa cũ, bởi chúng trung hòa điện và không tạo thành các cation. Hãy tìm
hiểu về Lewis acid với ví dụ về BH3.

66 | Bản quyền thuộc về Tạp chí Olympiad Hóa học KEM


BH3 kết hợp với ammonia tạo thành sản phẩm cộng bền (cặp Lewis) - theo
định nghĩa của Lewis, đây là một phản ứng acid-base.

Lớp vỏ ngoài cùng của nguyên tử boron có 3 electron, và phân tử được


tạo thành bởi sự chia sẻ các electron này với 3 nguyên tử hydrogen. Do
lớp vỏ ngoài cùng của boron có thể chứa tối đa 8 electron nên nó có thể
tiếp nhận thêm 1 cặp electron nữa. Bằng cách tiếp nhận cặp electron chưa
liên kết của nguyên tử nitrogen trong ammonia, cấu hình electron của
nguyên tử boron được xem là giống như của khí hiếm neon - một trong
những trạng thái bền vững nhất.
Khi xét các phản ứng acid-base, thì “lực” [độ mạnh] của mỗi tác nhân là
vấn đề quan trọng. Một thước đo là mức độ điện li. Khi một chất điện li
như acid hoặc base tan trong nước, nó bị điện li tạo thành các ion. Acid
mạnh như hydrogen chloride gần như bị điện li hoàn toàn, còn acid yếu
như acetic acid thì xảy ra theo cân bằng sau:

Hằng số cân bằng của phản ứng này được kí hiệu là K. Trong dung dịch
loãng, đặt K·[H2O] là Ka (hằng số ion hóa của acid) và [H3O+ ] là [H+ ], và viết
lại biểu thức hằng số cân bằng:

Ka có thể xem là không đổi nếu nhiệt độ không đổi. Trong trường hợp acid
yếu, giá trị của hằng số ion hóa rất nhỏ. Ví dụ với acetic acid thì Ka = 1.7·10-
5 mol L-1 . Ngoài ra, giá trị hằng số ion hóa biến đổi nhiều bậc [theo số mũ],

tùy thuộc vào bản chất của chất điện li. Do đó, thay vì mô tả cách hằng số
ion hóa trên một khoảng rất rộng như vậy thì chúng ta thường sử dụng
thang logarithm, với đại lượng pKa = -log10Ka. Ví dụ đối với acetic acid thì
pKa = 4.8.
Như đã thấy rõ từ định nghĩa, giá trị pKa càng nhỏ thì lực acid càng mạnh.
Bảng 1 cho biết giá trị pKa của một số carboxylic acid. So với acetic acid
thì chloroacetic acid và fluoroacetic acid là những acid mạnh hơn. Do các
nguyên tử chlorine và fluorine hút electron mạnh nên chúng có vai trò
phân tán điện tích âm của carboxylate anion (-COO-), dẫn đến tăng độ bền
của anion và tăng khả năng ion hóa. Hiệu ứng này có tác động rất lớn,

67 | Bản quyền thuộc về Tạp chí Olympiad Hóa học KEM


thậm chí các acid hữu cơ này còn mạnh hơn những acid vô cơ như HF (pKa
= 3.2). Các dẫn xuất với nhóm nitro (-NO2), cyano (-CN), acetyl (CH3CO-)
và hydroxyl (-OH) cũng có lực acid mạnh hơn acetic acid, đó là bởi tính
chất hút electron. Các nhóm chức như vậy được gọi là nhóm hút electron.
Mặt khác, pKa của propionic acid (CH3CH2COOH) lớn hơn một chút so với
acetic acid, điều này dường như ngược với tác động của nhóm thế hút
electron như trên. Đó là do khi thay thế hydrogen bởi nhóm methyl thì
electron sẽ bị đẩy về điện tích âm, làm cho anion trở nên kém bền và quá
trình ion hóa khó xảy ra hơn. Các nhóm alkyl như methyl được gọi là nhóm
đẩy electron.
Bảng 1
Chú thích: カルボン酸 = carboxylic acid

Câu hỏi ア: Dựa vào các diễn giải ở trên và dữ kiện trong bảng 1, hãy chọn
ra một phát biểu đúng dưới đây. (105)
① Formic acid (HCOOH) mạnh hơn acetic acid.
② CH3CH2CH2COOH là acid mạnh hơn acetic acid.
③ CH3CH2CHClCOOH là acid yếu hơn acetic acid.
④ BrCH2COOH là acid mạnh hơn FCH2COOH.
Từ các thảo luận tới thời điểm hiện tại, chúng ta thấy rằng lực acid của
carboxylic acid bị ảnh hưởng nhiều bởi hiệu ứng hút electron của nhóm
thế có trong cấu trúc. Tiếp theo, hãy cùng xem các nhóm thế ảnh hưởng
như thế nào đến hoạt tính và độ chọn lọc trong các phản ứng acid-base
có sự tham gia của hợp chất hữu cơ.
Theo định nghĩa của Lewis thì có rất nhiều phản ứng hóa học có thể được
xem là phản ứng acid-base, như phản ứng thế của benzene dưới đây. Khi
benzene phản ứng với một tác nhân electrophile [ái điện tử] - là chất dễ
tiếp nhận electron - thì phản ứng thế với hydrogen diễn ra tương đối dễ
dàng. Phản ứng này được gọi là phản ứng thế electrophile. Ví dụ, xét phản
ứng nitro hóa benzene. Khi trộn nitric acid đặc với sulfuric acid đặc thì

68 | Bản quyền thuộc về Tạp chí Olympiad Hóa học KEM


nitronium ion (NO 2+ ) được tạo thành và đóng vai trò là tác nhân
electrophile. Nitronium ion nhận cặp electron từ liên kết π của vòng
benzene và tạo thành liên kết carbon-nitrogen trong hợp chất trung gian
acarbocation I. Tiếp đó, H+ bị tách khỏi I, tạo thành nitrobenzene. Do đó,

trong phản ứng này, tác nhân electrophile (nitronium ion) là Lewis acid và
benzene là Lewis base.
Câu hỏi イ: Chọn ra hai phát biểu không đúng khi mô tả tính chất của
benzene. (106) (107)
① Nó là chất độc và tác nhân gây ung thư.
② Sodium chloride khó tan trong chất này.
③ Trong không khí, nó bị đốt cháy không hoàn toàn và tạo ra nhiều muội.
④ 6π electron được giải tỏa.
⑤ Tất cả các nguyên tử carbon và hydrogen nằm trên cùng mặt phẳng.
⑥ Hàm lượng hydrogen cao hơn các hydrocarbon no.
⑦ Tất cả các liên kết carbon-carbon có độ dài bằng nhau.
⑧ Nhiệt độ sôi thấp hơn toluene.
Câu hỏi ウ: Chọn ra 2 trong số các cấu trúc dưới đây không phải của
carbocation I (ở phần gạch chân a ở trên). (108) (109)

Câu hỏi エ: Chọn ra 3 phản ứng có thể tạo thành electrophile:

69 | Bản quyền thuộc về Tạp chí Olympiad Hóa học KEM


Chú thích: 紫外線 = ánh sáng tử ngoại (2); 鉄触媒 = xúc tác sắt (3); 濃硫
酸 = sulfuric acid đặc (4); ニッケル触媒 = xúc tác nickel (5)
Trong benzene, nếu 1 nguyên tử hydrogen bị thay thế bởi 1 tác nhân
electrophile thì sản phẩm là benzene 1 lần thế (thế mono). Kí hiệu nhóm
thế này là G. Nếu phản ứng thế với tác nhân electrophile được thực hiện
với dẫn xuất mono C 6H5G thì nhóm thế G đã tồn tại có 2 kiểu tác động với
vòng benzene.
1) Nhóm thế G tác động đến hoạt tính của vòng benzene, một số nhóm
làm tăng tốc độ phản ứng [hoạt hóa], một số khác thì lại làm chậm
[phản hoạt hóa].
- Các nhóm hoạt hóa: -NH2, -CH3, -OH, -OCH3, …
- Các nhóm phản hoạt hóa: -F, -Cl, -NO2, -COOH, …
2) Nhóm thế G xác định vị trí của nhóm thế tiếp theo (đây gọi là sự định
hướng của nhóm thế.)
Như đã mô tả ở trên, tác nhân electrophile được xem là Lewis acid và
vòng benzene là Lewis base. Vòng benzene sẽ nhường cặp electron. Khi
mật độ electron trong vòng benzene tăng lên hoạt tính tăng, và ngược lại,
hoạt tính sẽ giảm khi mật độ giảm. Do fluorine và chlorine có độ âm điện
cao hơn hydrogen, có thể hiểu rằng nó đóng vai trò nhóm giảm hoạt tính.
Tuy nhiên, với trường hợp oxygen, tuy có độ âm điện tương đối cao
nhưng các nhóm thế có liên kết qua oxygen lại được xếp vào nhóm làm
tăng hoạt tính. Điều này không thể được giải thích chỉ bởi bằng độ âm
điện. Trong trường hợp này, mật độ electron tăng lên bởi cặp electron
của oxygen đã giải tỏa vào vòng benzene. Nguyên tắc này cũng được áp
dụng với NH2. Có thể thấy được điều này từ thực tế rằng ammonia đóng
vai trò như một base, nguyên tử nitrogen có khả năng nhường cặp

70 | Bản quyền thuộc về Tạp chí Olympiad Hóa học KEM


electron chưa liên kết, dẫn đến tăng khả năng phản ứng. Còn nguyên nhân
tại sao nhóm nitro (-NO2) thể hiện tính chất trái ngược hoàn toàn là bởi 2
nguyên tử oxygen với độ âm điện lớn có liên kết với nguyên tử nitrogen,
dẫn đến khả năng hút electron cực mạnh và làm giảm khả năng phản ứng
của vòng benzene.
Câu hỏi オ: Chọn ra 4 nhóm thế làm giảm khả năng phản ứng thế
electrophile của vòng benzene. (113) (114) (115) (116)

Tiếp theo, liên quan đến sự định hướng của nhóm thế G, hãy xét phản
ứng của phenol với tác nhân electrophile.

Cặp electron chưa liên kết của oxygen trong phenol có thể được giải tỏa
khi tham gia vào hệ electron π liên hợp của vòng benzene (như biểu diễn
trong các cấu trúc cộng hưởng trên). Sự giải tỏa này đóng vai trò then
chốt trong việc làm bền hóa hệ (“bền hóa cộng hưởng”). Khi nitronium
ion phản ứng với phenol, nếu phản ứng thế diễn ra ở (117) và (118) của
nhóm hydroxyl, cấu trúc cộng hưởng có thể giải tỏa điện tích dương của
trung gian và có đóng góp lớn vào sự bền hóa cấu trúc này. Còn nếu phản
ứng thế ở vị trí (119) thì không có hiệu ứng như vậy. Thực tế, phản ứng
này gần như chỉ xảy ra ở các vị trí (117) và (118). Sản phẩm mà tất cả vị trí
(117) và (118) được nitro hóa gọi là (120) và được sử dụng làm nguyên
liệu thô cho thuốc nổ. Ngoài ra, sodium phenoxide - được tạo thành khi
xử lí phenol với sodium hydroxide - có hoạt tính cao hơn phenol, bởi điện
tích âm có xu hướng được giải tỏa vào vòng benzene, và có thể phản ứng
với cả electrophile kém hoạt động như carbon dioxide ở vị trí (117). Sản
phẩm sau khi xử lí với sulfuric acid loãng được gọi là (121) và được dùng
làm nguyên liệu cho thuốc giảm đau, chống viêm.
Tiếp theo, chúng ta hãy xét đến sự phản hoạt hóa gây ra bởi nhóm nitro.
Hợp chất này (nitrobenzene) có các cấu trúc cộng hưởng mà điện tích
dương xuất hiện ở các vị trí (117) và (118) nên rất khó để thực hiện phản
ứng thế ở đây. Do đó phản ứng ở vị trí (119) chiếm ưu thế hơn. Tiếp theo,
hãy xét mối quan hệ giữa hoạt tính của nhóm thế và sự định hướng vị trí.

71 | Bản quyền thuộc về Tạp chí Olympiad Hóa học KEM


Dựa vào các thảo luận vừa qua, có thể dự đoán rằng
nhóm thế hoạt hóa định hướng vị trí (117), (118) còn
các nhóm phản hoạt hóa định hướng vị trí (119). Xu
hướng này gần như luôn đúng, tuy nhiên vẫn có những
ngoại lệ, đó là fluorine và chlorine. Chúng đóng vai trò các nhóm thế phản
hoạt hóa, nhưng mặt khác, cũng có (Q122) cặp electron chưa liên kết
quanh nguyên tử gắn trực tiếp với vòng benzene, do đó, tác nhân
electrophile dễ phản ứng vào vị trí (117) và (118).
Câu hỏi カ: Xác định từ/cụm từ phù hợp với (117)-(119).
① orth ② meta ③ para
Câu hỏi キ: Chọn tên hợp chất phù hợp với (120), (121).
① Salicylic acid ② Phthalic acid
③ Picric acid ④ Benzoic acid
Câu hỏi ク: Chọn số phù hợp với (112).
Cũng như benzene, các alkane cũng có thể được xem là base. Từ quan
điểm của sự nhường và nhận cặp electron, phản ứng cộng với hydrogen
halide (HX) có thể được xem là phản ứng acid-base. Trong phản ứng thế
của benzene thì các nhóm thế có ảnh hưởng đến sự chọn lọc sản phẩm,
ảnh hưởng tương tự cũng xảy ra với phản ứng của alkene. Trong phần
tiếp đây, hãy xét hiệu ứng điện tử của nhóm thế đến độ chọn lọc của sản
phẩm cộng. Trong phản ứng của 2-methylpropene và hydrogen chloride,
có thể dự doán 2 sản phẩm sẽ được tạo thành, nhưng thực tế chỉ thu
được A. Sự chọn lọc vùng như vậy cũng được quan sát thấy với các alkene
bất đối và được gọi là quy tắc Markovnikov (đặt theo tên nhà hóa học đầu
tiên phát hiện ra, V. V. Markovnikov). Nguyên nhân tại sao sự chọn lọc
vùng như vậy xảy ra có thể được hiểu khi xét đến ảnh hưởng điện tử của
nhóm alkyl và xét trung gian bền hơn trong tiến trình phản ứng.
Câu hỏi ケ: Chọn tiểu phân trung gian phù hợp để tạo thành các sản phẩm
A và B trong phản ứng sau đây.

A: (123), B: (124)

72 | Bản quyền thuộc về Tạp chí Olympiad Hóa học KEM


Câu hỏi コ: Sự chọn lọc tương tự cũng được quan sát thấy trong phản
ứng cộng HBr. Xác định các sản phẩm của phản ứng (1)-(3). (Q125)

Từ phản ứng trung hòa của hydrogen chloride với sodium hydroxide, tạo
thành sodium chloride trung tính và nước, có thể suy đoán phản ứng acid-
base tạo thành sản phẩm bền với hoạt tính kém. Tuy nhiên, với sự kết hợp
của Lewis acid và base, có những trường hợp mà chất tạo thành lại có
hoạt tính hơn nhiều. Hãy xét những ví dụ như vậy. Cặp Lewis ammonia
borane (HN3-BH3) là chất rắn bền ở nhiệt độ và áp suất thường, có hàm
lượng hydrogen cao, và được nghiên cứu sử dụng làm môi trường lưu trữ
hydrogen cho pin nhiên liệu. Nếu thay thế hydrogen của ammonia borane
bởi một nhóm alkyl thì độ bền của cặp Lewis sẽ thay đổi. Ví dụ, trong cặp
Lewis của tripropylamine và triethylborane, mỗi nhóm alkyl chiếm một

73 | Bản quyền thuộc về Tạp chí Olympiad Hóa học KEM


vùng không gian đáng kể, do đó các tương tác đẩy không gian có thể diễn
ra (xem hình bên dưới) và độ bền sẽ giảm. Hiện tượng này được gọi là
“sức căng trước “ (front strain, hay F-strain).

Một trong những ví dụ kinh điển nhất là sự kết hợp của B(C 6F5)3 và
[(CH)3C]3P được thực hiện bởi D. W. Stephan.

Phosphorus đóng vai trò như một Lewis base tương tự nitrogen bởi trong
bảng tuần hoàn nó nằm ngay dưới nitrogen và cũng có một cặp electron
có thể nhường như nitrogen. Do nhóm [(CH) 3C]- là nhóm nhường electron
nên tính base tăng lên. Mặt khác, nhóm pentafluorophenyl của B(C 6F5)3 là
nhóm hút electron mạnh và làm tặng lực acid. Do đó, hai chất này có xu
hướng kết hợp tạo thành cặp Lewis. Tuy nhiên, do lực đẩy không gian của
các nhóm C 6F5- với [(CH)3C]- là rất lớn nên không thể tạo thành sản phẩm
cộng như ammonia borane. Chính vì vậy mà các hợp chất kiểu như vậy
được gọi là “Frustrated Lewis pair” hay FLP (“frustrated” có nghĩa là “thất
bại, mất tác dụng”). FLP có khả năng phản ứng mạnh, nó hoạt hóa các
phân tử hydrogen trung hòa bền và phân hủy thành acid (H+ ) và base (H-
). H+ liên kết với phosphorus và H- nhường cặp electron cho boron, tạo
thành một cặp ion [hợp chất ion]. Cặp ion này bền và không phân hủy giải
phóng hydrogen, thậm chí là khi đun nóng tới 100 oC.
Câu hỏi サ: FLP cũng phản ứng với nhiều chất. Hãy dự đoán sản phẩm
chính của phản ứng dưới đây. (Trong các công thức cấu tạo, t-Bu là kí hiệu
của nhóm [(CH3)3C-].
(126) Khi xảy ra phản ứng cộng:

74 | Bản quyền thuộc về Tạp chí Olympiad Hóa học KEM


(127) Khi liên kết C-H bị hoạt hóa và phân cắt:

(128) Khi xảy ra phản ứng cộng:

Stephan đã tiếp tục thiết kế các FLP có các tâm Lewis acid và Lewis base
trong cùng phân tử. FLP này có thể đóng vai trò xúc tác hydrogen hóa cho
imine (xem hình dưới).

Phản ứng hydrogen hóa xúc tác của imine đã được thực hiện sử dụng các
kim loại chuyển tiếp như platiunum. Phương pháp sử dụng FLP mới này
đã thu hút nhiều sự quan tâm, gọi là phản ứng xúc tác không cần kim loại
(metal-free), và hiện nay vẫn được nghiên cứu tích cực.
Câu hỏi シ: Lựa chọn nhóm gồm tất cả các chất đều phù hợp để trở thành
FLP như trong phần gạch chân. (129)

75 | Bản quyền thuộc về Tạp chí Olympiad Hóa học KEM


76 | Bản quyền thuộc về Tạp chí Olympiad Hóa học KEM
Bài 8
Hydroxide ion là base mạnh nhất có
thể có trong dung dịch nước, tuy nhiên
trong dung môi hữu cơ thì có thể có
các base mạnh hơn. Trong nhiều năm
liền, base mạnh nhất từng được biết
đến là methyl anion, nhưng vào năm
2008, một nhóm khoa học gia đã tổng
hợp được lithium monoxide anion có
tính base mạnh hơn. Năm 2016 đã ghi
nhận một kỉ lục mới khi các nhà nghiên
cứu của Úc công bố về việc tạo thành
một dianion hữu cơ phase khí (DEB 2-)
có ái lực proton lớn nhất từ trước tới nay - nghĩa là có tính base mạnh
nhất.
Độ mạnh của một base có thể được định nghĩa bởi ái lực với proton. Ái
lực proton của tiểu phân X-, kí hiệu PA(X-), được xác định bởi biến thiên
enthalpy chuẩn của phản ứng: HX → H+ + X-
a) Sử dụng dữ kiện trong bảng dưới đây, hãy tính ái lực proton của
methyl anion, PA(CH3-) theo đơn vị kJ mol-1.

Với ái lực proton 1778 kJ mol -1, lithium monoxide anion được xem là base
mạnh hơn methyl anion. Lithium monoxide anion được tạo thành trong
một khối phổ kế khi lithium oxalate anion, LiC 2O4-, ban đầu bị mất đi một
phân tử trung hòa P có khối lượng 44, rồi mất tiếp một phân tử trung hòa
Q khác có khối lượng 28.
b) Vẽ cấu tạo oxalate anion (C 2O42-) và xác định công thức P, Q.
Dianion hữu cơ phase khí DEB 2- có ái lực proton lớn nhất từng được báo
cáo từ trước tới nay, là 1843 kJ mol -1. Dianion này được tạo thành từ hợp
chất A thuộc nhóm benzene hai nhóm thế, C6H4R2, trong đó hai nhóm thế
R giống nhau. Hợp chất A có công thức C 12H6O4 và sủi bọt khi được thêm
sodium hydrogen carbonate.

77 | Bản quyền thuộc về Tạp chí Olympiad Hóa học KEM


c) Đề xuất nhóm chức có trong R gây ra phản ứng sủi bọt. Xác định cấu
trúc của R.
d) Vẽ tất cả các cấu trúc có thể có của benzene hai nhóm thế C6H4R2 và
xác định số tín hiệu có thể có trong phổ 13C NMR của mỗi cấu trúc.
DEB2- được quan sát thấy trong khối phổ ion âm của hợp chất A. Nó được
tạo thành qua các tiểu phân B2- và C 2-. Hợp chất A có 6 tín hiệu trong phổ
13 C NMR.

e) Xác định cấu tạo các tiểu phân trung gian B2-, C 2- và DEB2-.

78 | Bản quyền thuộc về Tạp chí Olympiad Hóa học KEM


Bài 9
Cho các chất sau đây:

a) Giữa A1 và A2, chất nào dễ tham gia phản ứng với Br 2 theo cơ chế
cộng electrophin (vào liên kết đôi C=C)? Giải thích.
b) So sánh lực axit giữa A2 và A3. Giải thích.

79 | Bản quyền thuộc về Tạp chí Olympiad Hóa học KEM


Bài 10
Giải Nobel Hóa học năm 2000
được tao cho giáo sư Hideki
Shirakawa. Năm 1976, ngài
Shirakawa đã thành công
trong việc dẫn dòng điện qua
vật liệu hữu cơ polyacetylene
(1). Nghiên cứu này đã được
phát triển theo nhiều hướng và các hợp chất hữu cơ dẫn điện đã được áp
dụng rộng rãi trong công nghệ pin, màn hình, … nhờ ưu thế cực nhẹ và có
thể thu nhỏ kích cỡ.
Có thể xem polyacetylene là phân tử trong đó nhiều ethylene (2) được
liên kết với nhau. Nguyên tử carboncuar ethylene có các electron liên kết
yếu với mỗi hạt nhân nhưng các electron này chịu trách nhiệm cho tính
dẫn điện của polyacetylene. Như biểu diễn trong hình 1, các electron này
phân tán và tồn tại theo hướng trực giao [vuông góc] với mặt phẳng phân
tử của ethylene. Vùng xuất hiện các electron gọi là “orbital”.

Hình 1: Giản đồ biểu diễn sự xuất hiện của electron của nguyên tử
carbon trong ethylene và các electron tồn tại thế nào trong các orbital.
Chú thích: エチレンの分子平面 = Mặt phẳng phân tử của ethylene; 核
による束縛が弱い電子 が存在する領域(軌道)= Vùng mà các
electron yếu được liên kết với các hạt nhân (orbital)
Kí hiệu màu sắc ở phần trên và phần dưới của hình biểu diễn sự khác nhau
về trạng thái của các electron. Sự bền hóa xảy ra khi các phần cùng màu
tương tác với nhau, và ngược lại sự kém bền hóa xảy ra khi những phần
khác màu cạnh nhau. Như trong hình 2, các electron của mỗi nguyên tử
carbon thuộc ethyelen đi vào các orbital tạo thành bởi sự bền hóa, và một
liên kết được tạo thành (đây là liên kết π).

80 | Bản quyền thuộc về Tạp chí Olympiad Hóa học KEM


Hình 2: Sự tạo thành orbital π của ethylene (2).
Chú thích: エネルギー = Năng lượng; 化によって 形成された軌道 =
Orbital tạo thành bởi sự kém bền hóa; 化によって 形成された軌道 (
π軌道)= Orbital π tạo thành bởi sự bền hóa.
Liên kết π được tạo thành theo cách này. Để xem xét tính chất dẫn điện
của polyacetylene, chỉ cần hiểu về các electron có trong liên kết này.
Khi hai đơn vị ethylene được nối với nhau thì tạo thành butadiene (3).
Orbital π của butadiene được tạo thành từ 4 electron (mỗi electron từ 1
nguyên tử carbon). Theo tính toán lí thuyết, có 4 kiểu sắp xếp màu sắc
orbital như trong hình 3. Trong đó, năng lượng của các orbital càng thấp
(bền) khi sự nghịch đảo màu sắc trên-dưới là nhỏ nhất. Khi mỗi quỹ đạo
được xem là một sóng biểu diễn bởi đường nét đứt trong mỗi orbital π
trong hình 3 thì nó sẽ được xem là càng bền khi số giao điểm với đường
thẳng nối các nguyên tử carbon càng nhỏ.

81 | Bản quyền thuộc về Tạp chí Olympiad Hóa học KEM


Hình 3: Orbital π của butadiene.
Số nút sóng được biểu diễn bởi các điểm giao cắt giữa đường nét đứt
với đường thẳng nối các nguyên tử carbon.
Mỗi orbital là một quỹ đạo trải rộng khắp phân tử và các electron có thể
phân bố khắp phân tử qua các liên kết π theo cách này. Như trong hình 3,
orbital với năng lượng cao nhất trong số các orbital có chứa electron được
gọi là “orbital bị chiếm cao nhất” (HOMO),
và orbital với năng lượng thấp nhất trong số
các orbital không có electron gọi là “orbital
chưa bị chiếm thấp nhất” (LUMO). Hãy xét
polyacetylene trong đó có N đơn vị ethylene
được nối với nhau.

Hình 4: Orbital π của decapentaene.


1) Hình 4 biểu diễn 10 orbital π có thể hình dung được với phân tử
decapentaene có 5 đơn vị ethylene. Hãy chọn ra orbital bền nhất
trong số các phương án từ (ア) đến (コ).
2) Chọn ra HOMO và LUMO của decapentaene trong số các phương án
từ (ア) đến (コ).
3) Biết rằng chênh lệch năng lượng ΔE giữa HOMO và LUMO và số đơn
vị ethylene (N) của polyacetylene có mối liên hệ như trong hình 5.
Trong đó eV (electronvolt) là đơn vị năng lượng, và 1 eV = 1.6·10-19 J.
Xác định ΔE của decapentaene theo đơn vị J.

82 | Bản quyền thuộc về Tạp chí Olympiad Hóa học KEM


Hình 5: Mối liên hệ giữa số đơn vị ethylene N với chênh lệch năng lượng
ΔE.
4) Ethylene và butadiene không có màu, nhưng các polyacetylene có
màu khi N tăng cao. Hợp chất có màu bởi cơ chế sau: Khi bị chiếu xạ
bởi ánh sáng, 1 electron của HOMO chuyển lên LUMO và năng lượng
ΔE được hấp thụ. Có mối liên hệ ΔE = hc/λ giữa bước sóng của ánh
sáng bị hấp thụ và ΔE, trong đó h là hằng số Planck và c là tốc độ ánh
sáng (h = 6.6·10-34 J s, c = 3.0·108 m s-1.) Khoảng bước sóng của ánh
sáng mà con người có thể nhìn được [khả kiến] là 400 đến 800 nm (1
nm = 10-9m), và chất có thể hấp thụ ánh sáng trong khoảng này thì có
màu. Mối liên hệ giữa bước sóng và màu sắc của ánh sáng mà chất
hấp thụ được biểu diễn như trong hình 6.

Màu sắc xuất Xanh


Xanh Xan Vàn
hiện khi hấp Tím dươn Cam Đỏ
chàm h lục g
thụ g

Hình 6: Mối liên hệ giữa bước sóng và màu sắc.


Chú thích: 吸収すると見える色 = màu sắc xuất hiện khi hấp thụ.
a) Xác định bước sóng của ánh sáng bị hấp thụ bởi decapentaene theo
nm và đánh giá xem liệu decapentane có màu hay không?

83 | Bản quyền thuộc về Tạp chí Olympiad Hóa học KEM


b) Từ hình 6 có thể thấy rằng các chất hấp thụ ánh sáng ở chủ yếu ở 500
nm thì có màu đỏ. Polyacetylene có bao nhiêu đơn vị ethylene thì hấp
thụ ánh sáng ở bước sóng này? Giải thích.
5) Khi N tăng, số orbital cũng tăng. Chênh lệch năng lượng giữa các
orbital giảm và thậm chí có thể được xem như là liên tục. Đây được
gọi là “cấu trúc vùng”, vùng chứa các electron được gọi là “vùng hóa
trị”, còn vùng không chứa electron được gọi là “vùng trống” (hình 7).

Hình 7: Số đơn vị ethylene của polyacetylene (N) và năng lượng


orbital π.
Với các chất có vùng hóa trị được lấp đầy electron, các electron không thể
dịch chuyển thậm chí là khi có điện trường được áp vào, do đó dòng điện
không thể chạy qua [dẫn qua] các chất này. Tuy nhiên, nếu có một khoảng
trống trong vùng hóa trị hoặc một chất có các electron trong vùng trống
thì các electron có thể di chuyển tự do khi đeiẹn trường được áp vào và
dòng điện có thể dẫn qua chất. Giáo sư Shirakawa đã chế tạo
polyacetylene với độ dẫn điện cao bằng cách thêm vào các tạp chất ở
dạng vi lượng. Ví dụ, khi trộn một lượng nhỏ lithium kim loại với
polyacetylene thì độ dẫn điện tăng lên đáng kể. Hãy giải thích hiện tượng
này.

84 | Bản quyền thuộc về Tạp chí Olympiad Hóa học KEM


Bài 11
Các hydrocarbon đơn vòng, liên hợp hoàn toàn với công thức tổng quát
(CH2)m (các annulene) có thể được chia thành 3 nhóm: thơm (aromatic,
A), phản thơm (antiaromatic, AA) và không thơm (nonaromatic, NA.) Cách
phân nhóm này cũng có thể được áp dụng cho các ion vòng với một hệ
liên kết đôi liên hợp.
1) Dưới đây là một số cấu trúc vòng. Hãy cho biết các cation và anion
tương ứng với các cấu trúc này (thay thế * bằng cấu điện tích) thuộc
nhóm nào (A, AA, NA)?

Các nguyên tử dị tố cũng có thể tham gia vào hệ giải tỏa electron π trong
các cấu trúc vòng tương tự.
2) Dưới đây là 6 dị vòng d-i. Hãy cho biết đâu là dị vòng thơm, không
thơm, phản thơm. Trong tất cả các trường hợp, hãy cho biết orbital
p của nguyên tử dị tố (orbital tự do hoặc orbital chưa bị chiếm) có
tham gia vào sự hình thành hệ electron liên hợp không?

85 | Bản quyền thuộc về Tạp chí Olympiad Hóa học KEM


Bài 12
1) Hợp chất nào dưới đây có tính thơm:

2) Cho các hợp chất sau:

Hãy cho biết những chất nào


a) có tính thơm?
b) phản thơm?
c) không thơm?
d) không thơm nhưng có cấu trúc cộng hưởng có tính thơm?
e) không thơm nhưng có base liên hợp có tính thơm?
f) không thơm, có pKa xấp xỉ -3.8.

86 | Bản quyền thuộc về Tạp chí Olympiad Hóa học KEM


Bài 13
Cho dãy hợp chất sau:

a) Hãy chỉ rõ đặc điểm cấu trúc cho thấy A là hợp chất thơm. So sánh khả
năng phản ứng thế electrophin của A với benzen và cho biết vị trí phản
ứng ưu tiên ở A. Giải thích.
b) So sánh (có giải thích) momen lưỡng cực của A, B, C về độ lớn.
c) So sánh (có giải thích) nhiệt độ nóng chảy, nhiệt độ sôi của dãy hợp
chất trên.

87 | Bản quyền thuộc về Tạp chí Olympiad Hóa học KEM


Bài 14
Cho các chất sau:

A1 A2 A3 A4 A5 A6 A7
a) Mỗi chất trên trong phản ứng cộng electrophin với hợp chất loại E-Nu
(E+ : tác nhân electrophin, Nu-: tác nhân nucleophin) theo tỉ lệ mol 1:1
ở nhiệt độ thấp và ở nhiệt độ cao tạo ra các sản phẩm khác nhau. Hãy
cho biết sản phẩm chính và giải thích. Giả thiết trong quá trình phản
ứng, khung cacbon của các chất ban đầu không thay đổi.
b) Những chất nào phản ứng cộng được với anhiđrit maleic theo tỉ lệ mol
1:1 ở nhiệt độ cao? Vẽ công thức lập thể của sản phẩm thu được. Biết
rằng phản ứng đóng vòng có sự tham gia đồng thời của 4n+2 electron
π (n = 1, 2,...) thì dễ xảy ra.

88 | Bản quyền thuộc về Tạp chí Olympiad Hóa học KEM


Bài 15
Hợp chất A có công thức phân tử CHNO, ở thể khí, độ dài liên kết CN bằng
121 pm (1 pm = 10-12m), độ dài liên kết CO bằng 117 pm. A tan trong
nước tạo thành dung dịch axit với Ka = 1,2 . 10-4. ở 00C, A tự biến đổi thành
chất rắn X không tan trong các dung môi thông thường, độ dài liên kết CN
bằng 140 pm. Trong dung dịch đặc, A tự biến đổi thành chất rắn Y có vòng
6 cạnh với độ dài liên kết CN bằng 135 pm. Biết độ dài liên kết trung bình
ở các hợp chất như sau:
Liên kết C–C C–N C–O C=N C=O CC cacbon
oxit
Độ dài, 154 147 143 130 123 116 112
pm
a) Hãy viết công thức cấu tạo các đồng phân ứng với công thức phân tử
CHNO. Công thức nào phù hợp với A ở thể khí, vì sao ?
b) b. Viết công thức cấu tạo và phương trình phản ứng điện li của A trong
dung dịch.
c) Hãy đề nghị phương trình phản ứng tạo thành X, Y và công thức cấu
tạo của chúng.

89 | Bản quyền thuộc về Tạp chí Olympiad Hóa học KEM


Bài 16
Cho các hợp chất sau:

a) So sánh khả năng enol hóa của các hợp chất trên. Giải thích ngắn gọn.
b) Hợp chất (V) dễ chuyển thành hợp chất T có công thức C 10H8O2. Hãy
cho biết công thức cấu tạo của T và giải thích.
c) Hợp chất (VIII) tạo được oxim khi phản ứng với hydroxylamin, hợp
chất (VI) thì không. Giải thích.

90 | Bản quyền thuộc về Tạp chí Olympiad Hóa học KEM


Bài 17
Cho các hợp chất cacbonyl sau:
O O O
*
H * * O O O O O
H H *
Me Me H
Me Ph Me Me OEt Me Me
Me Ph
A B C D E F

Các hợp chất cacbonyl có nguyên tử hiđro ở vị trí α có thể chuyển hóa
sang dạng enol theo cân bằng sau:
O OH
1
R2 R2 Ke
R 1
R
H

Nguyên tử hiđro trong nhóm OH enol phân li tạo anion enolat theo cân
bằng sau:
OH O
R2 R2 + H
+
Ka(OH)
R1 1
R

a) So sánh (có giải thích) giá trị pKe của các hợp chất A, B, C và D đối với
quá trình enol hóa xảy ra với các nguyên tử hiđro được đánh dấu (*).
b) Cho:
Chất D E F
pKe 8,33 1,00 -0,41
pKa(OH) 10,94 10,00 9,41
Tính giá trị pKa(CH) đối với sự phân li nguyên tử hiđro linh động nhất
thuộc liên kết C-H của các hợp chất D, E và F.
c) Dựa trên các giá trị cho ở câu (b), tính gần đúng giá trị pKe , pKa(OH) và
pKa(CH) của đietyl malonat.

91 | Bản quyền thuộc về Tạp chí Olympiad Hóa học KEM


92 | Bản quyền thuộc về Tạp chí Olympiad Hóa học KEM
Bài 18
Từ tinh dầu bạc hà người ta tách được (-)-menton (trans-2-isopropyl-5-
metylxiclohexanon). Khi chế hóa (-)-menton với axit hoặc kiềm, nó chuyển
một phần thành xeton đồng phân (+)-isomenton. Khi chế hóa (-)-menton
với anhiđrit axetic trong dung dịch natri axetat thì thu được hai đồng phân
A và B có công thức phân tử C12H20O2.
a) Vẽ các cấu trúc đồng phân lập thể của (-)-menton.
b) Dùng công thức cấu trúc, hãy giải thích sự tạo thành (+)-isomenton, A
và B từ (-)-menton.

93 | Bản quyền thuộc về Tạp chí Olympiad Hóa học KEM


Bài 19
Cho các công thức cấu tạo sau:
Me 1 2 4 5 Me Me OH Me O COOH
C(CH=CHF)2 3 N
Et O O Me
CH CH2F O 2N O O 2N OH NH2
O N
(A) (B) (C) (D) H (E)

1) Hãy vẽ công thức các đồng phân lập thể ứng với cấu tạo A.
2) Ứng với công thức cấu tạo B có bao nhiêu đồng phân lập thể, vì sao?
Dùng các kí hiệu thích hợp để chỉ rõ cấu hình của mỗi đồng phân đó
3) Hãy viết cơ chế phản ứng để giải thích vì sao C và D khi tương tác với
dung dịch NaOH thì đều tạo thành natri 3-metyl-4-nitrobenzoat.
4) Hãy chỉ rõ trạng thái lai hóa của từng nguyên tử N ở cấu tạo E và ghi
giá trị pKa (ở 25 oC): 1,8; 6,0; 9,2 vào từng trung tâm axit trong công
thức tương ứng với E, giải thích.

94 | Bản quyền thuộc về Tạp chí Olympiad Hóa học KEM


Bài 20
Hai hợp chất có cùng công thức phân tử nhưng khác về sự sắp xếp các
nguyên tử được gọi là các đồng phân cấu tạo, ví dụ như C 2H6O có thể là
ethyl alcohol hoặc dimethyl ether. Một dạng đồng phân cấu tạo quan
trọng là hỗ biến (tautomer hóa), trong đó cùng một phân tử tự động
chuyển hóa qua lại giữa hai hoặc nhiều dạng cấu trúc. Các hợp chất
carbonyl có hỗ biến keto-enol, được mô tả như sau:

1) Viết các dạng hỗ biến có thể có của hợp chất sau:

Nhiều hợp chất carbonyl với nguyên tử alpha hydrogen có tính acid có vai
trò quan trọng trong tổng hợp nhiều hóa chất công nghiệp. Ethyl
acetoacetate (EAA), hay ethyl-3-oxo butanoate là một trong các hợp chất
được sử dụng phổ biến như vậy. Trong dung dịch, EAA là một hỗn hợp
cân bằng của các dạng keto và enol.
2) Vẽ cấu trúc các dạng enol có thể có của EAA và chỉ ra dạng bền nhất.
Dạng enol bền nhất được quan sát thấy trong thực nghiệm. Để nghiên
cứu về động học của phản ứng sau đây, một thí nghiệm chi tiết đã được
tiến hành.
kF
EAA(keto) kB
EAA(enol) (1)

Lấy 4 bình cầu (A - D), mỗi bình chứa 25 ml dung dịch S của EAA (15 gam
L-1) trong methanol khan. Các bình cầu được giữ ở 20 oC trong suốt thí
nghiệm này. Lấy lượng dư dung dịch bromine trong methanol (25 mL) vừa
điều chế đổ nhanh vào mỗi bình ở thời điểm t = 0 giây. Dạng enol tạo
thành phản ứng cực kì nhanh với bromine, trong khi đó dạng keto không

95 | Bản quyền thuộc về Tạp chí Olympiad Hóa học KEM


có phản ứng. Phản ứng này tạo thành dung dịch có tính acid. Sự bromine
hóa enol là phản ứng rất nhanh khi so với sự enol hóa của EAA. Ester
monobromo hóa được tạo thành có tính bền và không phản ứng tiếp dưới
những điều kiện này.
3) Viết phương trình của phản ứng diễn diễn ra khi thêm bromine vào
dung dịch S.
Thêm nhanh lượng dư 2,4,4-trimethyl-1-pentene vào các bình A, B, C, D
ở các thời điểm t = 10, 40, 75, 100 tương ứng.
4) 2,4,4-trimethyl-1-pentene được thêm vào sẽ
i) Tăng tốc độ chuyển hóa keto thành enol.
ii) Tiêu thụ lượng bromine chưa phản ứng.
iii) Phản ứng với ester monobromo hóa và do đó đảm bảo
bromine hóa hoàn toàn EAA.
iv) Ngừng phản ứng enol hóa.
Sau khi thêm 2,4,4-trimethyl-1-pentene, thì thêm khoảng 10 mL dung
dịch KI 10 % vào mỗi bình A - D rồi giữ yên trong 30 phút. Hỗn hợp phản
ứng vẫn có tính acid. Trong thời gian này, ester bromo hóa bị chuyển trở
lại thành EAA. Sau đó, iodine giải phóng được chuẩn độ bởi dung dịch
sodium thiosulfate 0.1 M với chỉ thị hồ tinh bột. Giả sử rằng không có
phản ứng giữa iodine với bất kì dạng nào của EAA.
5) Viết phương trình phản ứng giải phóng iodine.
6) Viết phương trình phản ứng giữa iodine với sodium thiosulfate.
Thí nghiệm có thể được tóm tắt như sau:

96 | Bản quyền thuộc về Tạp chí Olympiad Hóa học KEM


Các thể tích sau sodium thiosulfate sau đã được sử dụng trong chuẩn độ
với dung dịch của các bình A, B, C ở 20 oC.
Thể tích dung dịch
Bình
Na2S2O3 đã sử dụng (mL)
A 3.78
B 4.02
C 4.30
7) Từ dữ kiện này, tìm nketo (số mol của dạng keto của EAA) của các bình
A, B, C ở thời điểm thêm 2,4,4-trimethyl-1-pentene vào. Bạn có thể
giả sử các thể tích được thêm vào trong quá trình khuấy dung dịch.
Dưới đây là đồ thị -ln(nketo) - thời gian t. Điểm tương ứng với bình D được
đánh dấu trong đồ thị.

97 | Bản quyền thuộc về Tạp chí Olympiad Hóa học KEM


8) Xác định các giá trị tương ứng với các bình A, B, C trong đồ thị trên.
Sử dụng đồ thị này, tìm các đại lượng sau đối với chuyển hóa
EAA(keto) thành EAA(enol) trong phản ứng (1).
a) Bậc phản ứng.
b) Hằng số tốc độ kF.
c) Hằng số cân bằng Keq. Lấy kết quả với 5 chữ số có nghĩa.
Khi tiến hành thí nghiệm trên ở 40 oC, giá trị KF và Keq lần lượt là 5.25·10-4
giây-1 và 0.06272.
9) Sử dụng thông tin trên, tìm các giá trị ∆Ho (kJ mol-1) và ∆So (kJ mol-1 K-
1 ) với phản ứng enol hóa (1). Giả sử ∆Ho và ∆So không phụ thuộc vào

nhiệt độ.
Động học phản ứng có ý nghĩa rất quan trọng trong các ứng dụng công
nghiệp của một phản ứng, do tốc độ phản ứng tác động đến thời gian mà
một lượng sản phẩm nhất định có thể được tổng hợp. Do đó thời gian
phản ứng sẽ ảnh hưởng đến giá thành sản phẩm cuối.

98 | Bản quyền thuộc về Tạp chí Olympiad Hóa học KEM


Một nhà tư bản công nghiệp có 8.5 kg EAA và hi vọng tạo thành sản phẩm
Y (khối lượng mol 174 gam mol -1) với lượng đẳng mol từ dạng enol của
EAA.
kF
EAA(keto) kB
EAA(enol) ⎯⎯
k'
→Y

10) Với giá trị nào của k’ thì nhà tư bản công nghiệp có thể nhận được
lượng Y mong muốn trong thời gian cực tiểu?
a) k’ ≈ 0.
b) k’ → vô cùng.
11) Nhà tư bản công nghiệp không biết giá trị của k’ nhưng muốn biết thời
gian cực tiểu cần để sản xuất 10 kg Y sử dụng lượng EAA sẵn có trong
dung môi methanol ở 40 oC. Ước lượng thời gian này, giả sử rằng (i)
động học của phản ứng enol hóa EAA trong quy mô lớn cũng giống
như trong phản ứng phòng thí nghiệm đã mô tả ở trên và ii) các ảnh
hưởng khác như sự biến đổi nhiệt độ và thể tích dung dịch khi tiến
hành phản ứng được bỏ qua.

99 | Bản quyền thuộc về Tạp chí Olympiad Hóa học KEM


Bài 21
Ba hợp chất dị vòng 5 cạnh 2 dị tử thường gặp là imidazol, oxazol và thiazol
đều là những tiền chất được sử dụng trong tổng hợp các hợp chất có hoạt
tính sinh học. Hãy so sánh nhiệt độ sôi, nhiệt độ nóng chảy của ba hợp
chất nêu trên và giải thích ngắn gọn.

100 | Bản quyền thuộc về Tạp chí Olympiad Hóa học KEM
Bài 22
2,3-pyridinedicarboxylic acid, thường được gọi là quinolinic acid, là một
độc tố thần kinh trung ương làm tiến triển các bệnh như Alzheimer hoặc
Parkinson. Ở điều kiện nhiệt độ phòng, quinolinic acid là chất rắn. Khi đun
nóng tới 185 - 190 oC thì giải phóng CO 2 tạo thành niacin.
1) Ở dạng tinh thể, quinolinic acid tồn tại ở cấu trúc với mức năng lượng
thấp nhất. Hãy vẽ cấu trúc này.
2) Dung dịch quinolinic acid có pKa1 = 2.41. Hãy viết phương trình phản
ứng phân li nấc thứ nhất của acid này.
3) Xác định cấu tạo của niacin.

101 | Bản quyền thuộc về Tạp chí Olympiad Hóa học KEM
Bài 23
a) Xác định cấu trúc nào sau đây là của polyethylene glycol

b) Vì sao polyethylene glycol có thể giữ nước đường ruột?


c) Polyethylene glycol có thể được trùng hợp từ ethylen oxide trong môi
trường acid. Viết phương trình phản ứng.

102 | Bản quyền thuộc về Tạp chí Olympiad Hóa học KEM
Bài 24
Các hợp chất A, B, C, D là đồng phân, có khối lượng phân tử 136, trong
phân tử chỉ chứa carbon, hydrogen và oxygen với hàm lượng oxygen là
23.5 %. Kết quả thực nghiệm cho thấy: A, B, C, D là các hợp chất thơm
đơn giản, trong đó mỗi vòng thơm chỉ có 1 nhóm thế. 4 hợp chất này có
thể tham gia phản ứng trong môi trường base như sau:
A ⎯⎯⎯⎯
dd NaOH
→ ⎯⎯⎯⎯
acid hãa
→ E (C7H6O2 ) + F
B ⎯⎯⎯⎯
dd NaOH
→ ⎯⎯⎯⎯
acid hãa
→ G (C7H8O) + H
C ⎯⎯⎯⎯
dd NaOH
→ ⎯⎯⎯⎯
acid hãa
→ I (hî p chÊt th¬m) + J
D ⎯⎯⎯⎯
dd NaOH
→ K + H2O

1) Xác định công thức phân tử các chất A - D.


2) Xác định cấu tạo các chất A - D.
3) Các phản ứng của A và D với NaOH thuộc loại nào?
4) Xác định tên gọi các nhóm chức trong B.
5) Cho các dung dịch sau: HCl, HNO 3, NH3·H2O, NaOH, NaHCO 3, nước Br2
bão hòa, FeCl 3, NH4Cl. Hãy chọn ra những thuốc thử phù hợp và thiết
kế một quy trình thực nghiệm để xác định các chất E, G, I.

103 | Bản quyền thuộc về Tạp chí Olympiad Hóa học KEM
Bài 25
Hợp chất C được tổng hợp theo cách sau:

Sau phản ứng, sản phẩm vẫn chứa một lượng A và B chưa phản ứng.
1) Trình bày quy trình khả thi để tách C khỏi hỗn hợp. Mô tả ngắn gọn cơ
sở lí thuyết của quy trình này.
2) Phản ứng tạo thành C thuộc loại phản ứng cơ bản nào?

104 | Bản quyền thuộc về Tạp chí Olympiad Hóa học KEM
Bài 26
1) Sự tồn tại của CH5+ đã được phát hiện từ thập niên 1950 và nhiều giả
định về cấu trúc ion này đã được đề xuất. Tuy nhiên, mãi đến năm 1999
thì các nhà nghiên cứu mới xác định được phổ dao động quay của ion này
ở nhiệt độ thấp. Mô hình cấu trúc của ion này: các nguyên tử hydrogen
chuyển động nhanh quanh nguyên tử carbon và các liên kết carbon-
hydrogen có độ dài bằng nhau.
a) Có thể mô tả cấu trúc của ion này theo thuyết liên kết hóa trị cổ điển
không? Giải thích ngắn gọn.
b) Ion CH5+ là: acid nhường proton (protonic acid); Lewis acid; gốc tự do;
nucleophile?
2) Sắp xếp các ion dương dưới đây theo thứ tự giảm dần tính bền.

105 | Bản quyền thuộc về Tạp chí Olympiad Hóa học KEM
Bài 27
Cho các ancol p-CH3-C6H4-CH2OH, p-CH3O-C6H4-CH2OH, p-CN-C6H4-CH2OH
và p-Cl-C6H4-CH2OH. So sánh khả năng phản ứng của các ancol với HBr và
giải thích.

106 | Bản quyền thuộc về Tạp chí Olympiad Hóa học KEM
Chuyên đề 39: Phổ học

Bài 1
Khái niệm “dao động nhóm” được sử dụng để nghiên cứu cấu trúc phân
tử bằng phương pháp phổ hồng ngoại. Nhiều nhóm chức hấp thụ bức xạ
hồng ngoại ở vùng sóng ngắn, tạo thành các dải phổ đặc trưng. Vị trí dải
phổ (ν - số sóng, cm-1) của nhóm A-B có thể tính theo công thức:

(1),
trong đó c = 3⋅1010 cm⋅s-1; π = 3.14; μ = M(A)⋅M(B)/[M(A)+M(B)]; NA =
6.02⋅1023; k - hằng số lực liên kết hóa trị (g/c2). Mặc dù k được dùng để
đánh giá độ bền của liên kết, một mối liên hệ đơn giàn giữa năng lượng
liên kết (E, J/mol) và k chưa được tìm thấy. Thêm vào đó, cho các liên kết
bội carbon-carbon khác nhau có một mối liên hệ tuyến tính:
E = 2.24⋅105 + 0.395⋅k (2).
1) Sử dụng phương trình (1) và (2), hoàn thành bảng sau:
C=O

Liên kết C- C= C= tru aldehy C- H- H-


A-B C C C ng keto aldehy de α-β O O C
bìn ne de không
h no
3 3
E, 60 46 40
5 750 5
kJ/mol 3 0 0
0 8
k⋅10-5 11.8
12.10 11.65
g/s2 9
9
16 20
νtb , cm-1 5
46 99
1
Aldehyde α,β không no chứa 2 nhóm thế β không thể thu được từ phản
ứng ngưng tụ croton với xúc tác base. Có thể tổng hợp các chất này từ 2-
Z-ethoxyvinyl lithium (I):

107 | Bản quyền thuộc về Tạp chí Olympiad Hóa học KEM
2) Vẽ các chất II, III và IV. Giải thích tính không bền của III bằng cách tính
ΔHo chuyển hóa III → IV (đối với các tiểu phân có thay đổi liên kết, sử
dụng các giá trị năng lượng từ bảng trên)
3) Hợp chất V dễ dàng tham gia phản ứng cộng 1,2 hoặc cộng 1,4 tạo
thành sản phẩm VIa-VIf. Xác định công thức cấu tạo VIa-f, sử dụng
bảng sau:
1.CH3M
gBr Na/Hg
Tác CH2=C= C 2H5OHdư
+ NaHSO3 H2O/H+
nhân 2.H2O/H C2H5OH O /HCl
+

VIa VIc VId VIe


3600- VIb 3600- 3600- 1725- VIf
Phổ IR 3400 1740- 3400 3400 1705
__
1680- 1720 1740- 1740- 1680-
1620 1720 1720 1620

4) Chỉ ra số sóng của các dải phổ trong phổ IR của chất V.
5) Các chất VIa, VIc-f là hợp chất 1,2- hay 1,4-?

108 | Bản quyền thuộc về Tạp chí Olympiad Hóa học KEM
Bài 2
Vị trí của các dải hấp thụ màu (λmax) C=C-C=O, với bước chuyển điện tử π
→ π* trong phổ UV của các hợp chất α, β carbonyl không no, có thể được
tính theo các quy tắc Woodward - Fieser:  max =   i , trong đó Δi là đóng
góp của các mảnh cấu trúc vào λmax, và cường độ hấp thụ Di, theo định
luật Bouguer-Lambert-Beer, tỉ lệ thuận với nồng độ của chất.
C=C- C=C- Nhóm alkyl Nhóm alkyl
Mảnh
C=Oaldehyde C=Oketone vị trí α vị trí β
Δi, nm 207 215 10 12
Có thể nghiên cứu các phản ứng ngưng tụ croton của butanone-2 (I) với
các aldehyde không thơm II - IV trong môi trường acid bằng phương
pháp phổ UV:
Sản λmax,
Chất D lg max
phẩm nm
II A 237 0.72 4.0
B 229 0.48 4.0
III C1 227 0.45 4.0
C2 237 0.90 4.0
C3 185 0.03 3.3
190 ? 3.9
IV D1 237 0.57 4.0
D2 227 0.19 4.0
Khi tính toán, giả sử rằng các giá trị λmax tính được trùng khớp với các giá
trị thực nghiệm.
1) Xác định thành phần (theo % mol) của hỗn hợp sản phẩm của phản
ứng I với các aldehyde II - IV.
Tính độ hấp thụ của hợp chất C3 ở 190 nm.
2) Thêm các nhóm thế H (hydrogen) hoặc R (alkyl) vào mảnh C=C-C=O
để tạo thành các hợp chất với λmax bằng 227 nm, 229 nm, 237 nm.
3) Chỉ ra tác nhân nào (I hay aldehyde; nhóm CH, CH2 hay CH3) đã đóng
vai trò nhóm carbonyl (phản ứng như nhóm carbonyl), và tác nhân

109 | Bản quyền thuộc về Tạp chí Olympiad Hóa học KEM
nào đóng vai trò nhóm methylene trong phản ứng tạo thành hợp chất
có λmax bằng 227, 229 và 237 nm.
4) Biết rằng m(A)/m(B) = 1.7. Tính tỉ lệ khối lượng mol M(A)/M(B) rồi
dựa vào đó để xác định cấu tạo của II, A, B.
5) Xác định cấu tạo các chất III, IV, C1 - C3, D1 và D2, biết:
- Các chất trong dãy C1 - C3, D1 - D2 là đồng phân.
- M(C i) + M(Di) = 2M(A)
- Chất C3 không có đồng phân hình học.

110 | Bản quyền thuộc về Tạp chí Olympiad Hóa học KEM
Bài 3
Ý tưởng về độ bất bão hoà (DBE – double bond equivalent, số liên kết đôi
và/hoặc vòng có trong một hợp chất), rất hữu dụng khi xác định công thức
cấu tạo từ công thức phân tử.

Hình: Mr Bond3, ở hình trên, có DBE bằng 7.


a) Công thức tổng quát của một alkene không vòng là C nH2n+2 . Xác định
công thức tổng quát của các hợp chất sau:
• Alkene hoặc cycloalkane.
• Alkyne
• Cycloalkene.
• Dialkyne.
Khi thêm 1 vòng hoặc liên kết π vào thì phân tử giảm đi 2 hydrogen. Với
các cấu trúc đa vòng, số vòng có thể được xác định bằng cách: đếm số
đường tối thiểu, cắt qua các liên kết để tạo thành cấu trúc không vòng.
b) Bảng dưới đây cho biết những tổ hợp có thể có của các giá trị số vòng,
liên kết đôi và liên kết ba tương ứng với DBE = 1 và DBE = 2. Trong
Phiếu trả lời, hãy mở rộng bảng này với các tổ hợp có thể có với DBE
= 3 và DBE = 4.

3 Một cách chơi chữ, ngụ ý chỉ điệp viên 007 (James Bond).

111 | Bản quyền thuộc về Tạp chí Olympiad Hóa học KEM
Giá trị DBE của một hydrocarbon có thể được tính bằng cách so sánh số
nguyên tử hydrogen thực tế (X) có trong hydrocarbon (công thức C nHX)
với số hydrogen trong alkane có cùng số carbon (C nHA):
DBE = ½ (A-X)
Ví dụ, để xác định DBE của cyclohexene, C 6H10, do số hydrogen trong
hexane (alkane có cùng 6C) là 14 nên DBE(cyclohexene) là ½ (14-10) = 2
c) Xác định các giá trị DBE của

Có thể dùng kĩ thuật phổ, như NMR, để xác định các loại DBE.
Trong kĩ thuật 13C NMR, số tín hiệu trong phổ phụ thuộc vào số loại
carbon4 khác nhau trong cấu trúc. Ví dụ, trong benzene, các nguyên tử
carbon đều tương đương nhau, do vậy phổ đồ chỉ có duy nhất 1 mũi
(peak), trong khi đó 1,3-dimethylbenzene có tới 5 loại carbon khác nhau:

4 Nguyên gốc là “carbon enviroment” (môi trường carbon).

112 | Bản quyền thuộc về Tạp chí Olympiad Hóa học KEM
Trong benzene, tất cả các Trong 1,3-dimethylbenzene, vẽ một mặt
nguyên tử carbon tương phẳng đối xứng (hoặc trục quay) chạy
đương nhau, do vậy trong qua phân tử (biểu diễn bằng đường nét
phổ đồ chỉ xuất hiện duy đứt) thì thấy có 5 loại carbon khác nhau
nhất 1 peak ở 128 ppm. trong phân tử. Do vậy, trong phổ đồ xuất
hiện 5 peak.

Dưới đây là vùng phổ đặc trưng của một số loại nguyên tử carbon:
• carbon trong liên kết ba alkyne: 70-100 ppm.
• carbon trong liên kết đôi: 100-160 ppm.
• carbon no (tạo ra 4 liên kết đơn với hydrogen hoặc nguyên tử
carbon khác): 0-50 ppm.
Các allene (dạng R2C=C=CR2) là trường hợp khá bất thường. Nguyên tử
carbon trung tam tạo ra một peak trên 200 ppm, còn các nguyên tử
carbon liên kết trực tiếp với carbon trung tâm (còn gọi là nguyên tử
carbon “cạnh sườn”) thì nằm cùng khoảng với các nguyên tử carbon của
liên kết ba.
• carbon trung tâm của allen: >200 ppm.
• carbon ở cạnh sườn: 70-100 ppm.
Khi sử dụng các kĩ thuật NMR tiên tiến, ngoài việc xác định được có bao
nhiêu nguyên tử carbon thuộc cùng một loại thì còn có thể biêt có bao
nhiêu nguyên tử hydrogen gắn với mỗi carbon bất kì. Chúng ta kí hiệu
(CH3), (CH2), (CH) hoặc (C) cho các nguyên tử carbon gắn với 3, 2, 1 hoặc
0 nguyên tử hydrogen.
Ví dụ, phổ đồ của 1,3-dimethylbenzene có thể tóm tắt như sau:
2 × 138 (C), 130 (CH), 128 (CH), 2 × 126 (CH), 2 × 21 (CH3)
Các dữ liệu sau được lấy từ phổ 13C NMR của các đồng phân có cùng công
thức C 8H8 (5 DBE).

113 | Bản quyền thuộc về Tạp chí Olympiad Hóa học KEM
Phổ của Các tín hiệu / ppm
A 8 × 132 (CH)
B 8 × 47 (CH)
C 4 × 96 (C), 4 × 20 (CH2)
D 2 × 210 (C), 134 (C), 113 (CH2), 2 × 93 (CH), 2 × 79 (CH2)
E 147 (C), 2 × 138 (CH), 2 × 131 (CH), 2 × 127 (CH), 112
(CH2)
F 142 (CH), 136 (CH), 127 (CH2), 120 (CH2), 117 (CH), 112
(CH), 91 (C), 89 (C)
G 2 × 146 (C), 2 × 127 (CH), 2 × 122 (CH), 2 × 30 (CH 2)
H 154 (C), 151 (C), 2 × 136 (CH), 2 × 128 (CH), 2 × 37 (CH 2)
I 157 (C), 101 (CH2), 2 × 26 (CH), 4 × 19 (CH)
d) Hãy hoàn thành bảng dữ liệu về số loại nguyên tử carbon, từ đó hãy
cho biết có bao nhiêu liên kết đôi, ba và vòng trong mỗi phân tử.
Dự đoán thông tin cấu
Số nguyên tử carbon thuộc loại
trúc
Số Số
Liên kết Liên kết Liên Trung Cạnh
liên liên Số
ba đôi kết tâm sườn
kết kết vòng
(alkyne) (alkene) đơn allene allene
ba đôi

114 | Bản quyền thuộc về Tạp chí Olympiad Hóa học KEM
e) Dự đoán công thức cấu tạo của các chất A-I

115 | Bản quyền thuộc về Tạp chí Olympiad Hóa học KEM
Bài 4
Dưới đây là phổ 1H NMR của hỗn hợp 3 hợp chất tự nhiên có công thức
phân tử C5H10O, C7H6O4 và C 8H11NO đo trong CD3OD (những con số ghi
trên mỗi tín hiệu biểu diễn cường độ tương đối của chúng) và phổ 13NMR
của mỗi thành phần trong hỗn hợp trên theo trật tự ngẫu nhiên.

1) Dựa vào dữ kiện phổ, hãy xác định cấu trúc của mỗi thành phần trong
hỗn hợp và chỉ rõ phổ 13C NMR tương ứng của chúng.
2) Xác định thành phần định lượng của hỗn hợp theo phần trăm số mol.
3) Đề xuất một phương pháp hiệu quả nhất, theo quan điểm của bạn, để
cô lập các chất trong hỗn hợp ban đầu.

116 | Bản quyền thuộc về Tạp chí Olympiad Hóa học KEM
Bài 5
A. Dược phẩm
Các dịch chuyển hoá học đặc trưng của phổ 13C-NMR
Loại carbon Độ dịch Loại carbon (R= Độ dịch chuyển
(R= Alkyl, Ar = chuyển hoá Alkyl, Ar = Aryl) hoá học (ppm)
Aryl) học (ppm)
RCH3 10 – 25 RC≡CR 65 - 85
RCH2R 20 – 35 RCH=CHR 120 - 140
R3CH 25 – 35 Aryl C 120 - 140
RCH2COR 35 – 50 RCOOR 160 - 180
RCH2Br 25 – 35 RCONR2 165 - 180
RCH2Cl 40 – 45 RCOOH 175 - 185
RCH2NH2 30 – 65 RCHO 190 - 205
RCH2OH 60 – 70 RCOR 200 - 215
RCH2OR 65 – 70

Khi nói đến các tâm quang hoạt (tâm thủ tính, hay chiral), chúng ta thường
nghĩ đến các nguyên tử carbon. Nhưng nguyên tử sulfur của nhóm sulfinyl
(S=O) cũng có thể được biểu diễn theo cấu hình R/S. Nó chứa một cặp
electron tự do, được gắn với độ ưu tiên thấp nhất theo quy tắc CIP.
Nguyên tử sulfur của sulfoxide không thể dao động qua mặt phẳng của
các nguyên tử khác liên kết với nó đây là lý do vì sao nó là tâm quang hoạt
khi những nguyên tử đó khác nhau.
Adrafinil là một chất kích thích dạng racemic, có công thức cấu tạo như
dưới đây. Nó có chứa một tâm quang hoạt.

117 | Bản quyền thuộc về Tạp chí Olympiad Hóa học KEM
1) Vẽ các công thức lập thể, chỉ rõ cấu hình R/S, cuả hợp chất này.
Dưới đây là phổ 13C-NMR của Adrafinil trong CDCl 3:

2) Gán tất cả các tín hiệu trong phổ 13C-NMR với các nguyên tử carbon
tương ứng trong Adrafinil.
Phổ 1H-NMR của Adrafinil and 2-[(Diphenylmethyl)-sulfonyl]acetamide
trong CDCl 3 đã được ghi lại. Đáng tiếc là hai phổ đồ (gọi là “Phổ 1” và “Phổ
2”) đã bị lẫn vào nhau, và không biết được phổ nào tương ứng với hợp
chất nào.
Công thức cấu tạo của 2-[(diphenylmethyl)sulfonyl]acetamide:

Phổ 1:

118 | Bản quyền thuộc về Tạp chí Olympiad Hóa học KEM
Phổ 2:

3) Hãy cho biết phổ 1H-NMR Adrafinil là phổ 1 hay 2? Hãy gắn ít nhất hai
độ dịch chuyển hoá học (theo ppm) vào công thức cấu tạo dưới đây
để chứng minh cho lựa chọn của bạn.

4) Giả sử rằng dung môi bị thay thế bởi CD3OD. Trong trường hợp này,
phổ đồ còn giữ nguyên tất cả các tín hiệu hay không? Nếu không, hãy
vẽ công thức cấu tạo tạo thành (đã thay đổi).

119 | Bản quyền thuộc về Tạp chí Olympiad Hóa học KEM
Thuốc giảm đau vedaclidine và thuốc giãn cơ alfoqualone cũng là các hợp
chất quang hoạt.

Vedaclidine
5) Kí hiệu (các) tâm quang hoạt bằng dấu hoa thị. Sau đó, xác định (các)
cấu hình tuyệt đối của Vedaclidine.

Alfoqualone
6) Chỉ rõ lý do tại sao afloqualone có tính quang hoạt.
7) Vẽ công thức lập thể của afloqualone và xác định (các) cấu hình lập
thể tương ứng.
Dưới đây là phổ 13 C-NMR của linezolid, một kháng sinh thuộc nhóm
oxazolidinone:

120 | Bản quyền thuộc về Tạp chí Olympiad Hóa học KEM
Linezolid
8) Chỉ rõ các nguyên tử C trong công thức của linezolid đều tạo thành
cùng một tín hiệu trong phổ 13C-NMR và xác định có bao nhiêu tín hiệu
trong phổ được tạo thành bởi nhiều hơn 1 C.
B: Một hợp chất chưa biết
Tiến hành phân tích nguyên tố 0.5000 gam một hợp chất chưa biết. Ở áp
suất 1,0135 bar và nhiệt độ 25 oC, có 683,9 cm3 CO2 và 0,323825 gam H2O
đã được tạo thành. Hợp chất này có khối lượng mol khoảng 250 gam/mol.
Dưới đây là phổ hồng ngoại, 1H NMR và dữ liệu phân tích phổ 13C NMR
của hợp chất này.
9) Xác định công thức phân tử của hợp chất
Phổ hồng ngoại:

Phổ 1H NMR:

121 | Bản quyền thuộc về Tạp chí Olympiad Hóa học KEM
Dữ liệu phân tích phổ 13C NMR: 13.6; 35.7; 57.8; 59.5; 125.3; 127.6; 128.7;
141.1; 174.5 ppm
10) Vẽ công thức cấu tạo của hợp chất chưa biết. Gán các tín hiệu trong
phổ 1H NMR với các proton trong hợp chất bằng các kí hiệu chữ cái a,
b, c, …

122 | Bản quyền thuộc về Tạp chí Olympiad Hóa học KEM
Bài 6
Phổ 1H NM, 13C NMR và hồng ngoại (IR) của các hợp chất A - C, đều có
công thức phân tử C3H7NO2, được cho ở trang bên.
1) Xác định cấu tạo các chất A - C.
2) Hãy gắn các tín hiệu trong phổ 1H NMR và 13C NMR với các phần tương
ứng trong cấu trúc A - C. Để thuận tiện, hãy đánh số các nguyên tử
trong cấu tạo (tìm được ở ý 1), và đánh số các tín hiệu tương ứng
trong phổ.
3) Trong phổ 1H NMR, hãy xác định độ bội của các tín hiệu.
4) Các nhóm nào tương ứng với các dải trong phổ hồng ngoại.

123 | Bản quyền thuộc về Tạp chí Olympiad Hóa học KEM
Phổ của chất A

124 | Bản quyền thuộc về Tạp chí Olympiad Hóa học KEM
Phổ của chất B

125 | Bản quyền thuộc về Tạp chí Olympiad Hóa học KEM
Phổ của chất C

126 | Bản quyền thuộc về Tạp chí Olympiad Hóa học KEM
Bài 7
Ngày nay, kĩ thuật khối phổ là một phần quan trọng trong nghiên cứu xác
định cấu trúc phân tử. Trong số rất nhiều những ưu điểm của phương
pháp này, thì phải kể đến đầu tiên là độ nhạy cao (10-9 - 10-15 gam chất),
thời gian yêu cầu ngắn (vài phút), giàu hàm lượng thông tin và đáng tin
cậy. Sử dụng kĩ thuật sắc kí/khối phổ, có thể biết được thành phần định
tính và định lượng của một hỗn hợp hữu cơ phức tạp chỉ nặng 1 mg.
Điểm đặc trưng của kĩ thuật khối phổ là có thể xác định khối lượng (hay
chính xác hơn, là tỉ lệ khối lượng trên điện tích, m/z) của các ion tạo thành
khi ion hoá phân tử (các ion phân tử) và sản phẩm của các quá trình phân
mảnh nội phân tử kế tiếp nhau của các ion này (mảnh ion). Cường độ
tương đối của mỗi tín hiệu trong phổ tỉ lệ với số mol ion tương ứng, được
xác định bởi tốc độ tạo thành và phân mảnh ion, và bởi thành phần đồng
vị của các nguyên tố.
Ví dụ, khối phổ của chloromethane gồm các tín hiệu mạnh với m/z = 52,
51, 50, 49, 37, 35, 15 - có thể được giải thích bởi sơ đồ ion hoá và phân
mảnh sau:

Sự tồn tại của các đồng vị chlorine 35Cl và 37Cl với tỉ lệ xấp xỉ 3:1 và một
lượng nhỏ các đồng vị carbon và hydrogen nặng (xem bảng dưới đây) là
nguyên nhân tạo ra các tỉ lệ cường độ sau đây trong phổ:
I(52) I(51) I(37) 1
  
I(50) I(49) I(35) 3

Nguyên tố Đồng vị Hàm lượng, %


Br 79 50.69
81 49.31
C 12 98.90
13 1.10

127 | Bản quyền thuộc về Tạp chí Olympiad Hóa học KEM
Cl 35 75.77
37 24.23
F 19 100
H 1 99.98
2 0.015
I 127 100
Bảng: Hàm lượng tự nhiên của các đồng vị carbon, hydrogen và halogen.
Sử dụng bảng này, có thể dễ dàng dự đoán số lượng và tỉ lệ cường độ của
các tín hiệu từ một ion cụ thể bởi sự có mặt của các đồng vị khác nhau
(các peak đồng vị). Ví dụ, với ion phân tử dibromomethane, tỉ lệ xấp xỉ là:
172(M) : 174(M+2) : 176(M+4) = 1:2:1
1) Tính tỉ lệ cường độ (với độ chính xác tới 4 chữ số có nghĩa) của các tín
hiệu tương ứng với RCl2+ và RCl3+ ion. Biết rằng nhóm R chỉ có các
nguyên tố tinh khiết đồng vị (hay hàm lượng của các đồng vị khác có
thể bỏ qua). Cường độ của tín hiệu có khối lượng nhỏ nhất (M) là 1.00.
2) Xác định độ làm giàu hydrogen bởi deuterium (theo % mol) trong một
mẫu ammonia, nếu biết cường độ các tín hiệu m/z 19 và 20 trong khối
phổ bằng nhau.
3) Xác định công thức phân tử của perfluoroalkane (một alkane với tất
cả các nguyên tử hydrogen bị thay thế bởi fluorine), nếu biết tỉ lệ
cường độ tín hiệu của ion phân tử chính (M) trong khối phổ và tín hiệu
kề cận (M+1) là 1.000 : 0.045.
Dưới đây là khối phổ của chất X, chỉ chứa các nguyên tố được liệt kê trong
bảng đồng vị ở trên.

128 | Bản quyền thuộc về Tạp chí Olympiad Hóa học KEM
4) Xác định công thức của X.
5) Xác định công thức các ion, có cường độ cực đại là (m/z) 225; 190;
155; 141; 118 và 130.

129 | Bản quyền thuộc về Tạp chí Olympiad Hóa học KEM
Bài 8
Phun điện tử là một phương pháp để ion hoá các phân tử trong dung dịch.
Ưu điểm chính của phương pháp này là sự phân mảnh thấp. Một chất
lỏng dẫn qua một ống mao quản mỏng và để lại đầu mũi kim các giọt
(sương) nhỏ mang điện. Những giọt này di chuyển trong điện trường, bị
giảm kích thước bởi sự bay hơi của dung môi và tự phân mảnh thành
những phần nhỏ hơn. Kết quả là các hạt siêu nhỏ được tạo thành, chỉ
chứa một phân tử mang điện (ion). Sau khi làm bay hơi những phần dung
môi cuối cùng, các ion được đưa vào khối phổ kế để phân tích tỉ lệ khối
lượng/điện tích (m/z).
Kĩ thuật phun điện tử được dùng để xác định khối lượng phân tử của các
protein. Các peak (mũi) của những ion bị proton hoá nhiều lần [A+nH]n+
(trong đó n là số proton gắn vào phân tử A) được ghi lại bởi khối phổ kế.
Trong hình là phổ khối lượng của lysozyme A. Điện tích của các ion cạnh
nhau chỉ khác nhau 1 đơn vị.

1) Hãy chọn ra 2 peak bất kì và tính khối lượng phân tử (M) của A từ các
giá trị m/z. Biểu diễn khối lượng theo đơn vị dalton (1 Da = 1 a.m.u. =
1.66054∙10-27 kg) và làm tròn tới số nguyên gần nhát. Khối lượng của
proton là mp = 1.0073 Da.
2) Xác định điện tích của các ion trong phổ khối (theo điện tích nguyên
tố e = 1.602∙10-19 C).
Độ bền của một giọt mang điện liên quan đến sự phân mảnh chịu ảnh
hưởng bởi sự cạnh tranh giữa năng lượng tĩnh điện (electrostatic energy)

130 | Bản quyền thuộc về Tạp chí Olympiad Hóa học KEM
và năng lượng bề mặt dính kết (cohesion surface energy). Với một giọt
hình cầu, Uel = q2 / 8πε0r, trong đó q - điện tích của giọt, r - bán kính, e0 =
8.854∙10-12 J-1C 2m-1, Usurf = σS, trong đó S - diện tích bề mặt giọt, σ - sức
căng bề mặt.
3) Từ những xem xét về mặt năng lượng, hãy rút ra điều kiện bền cho
một giọt hình cầu liên quan đến sự phân mảnh thành hai phần với
hình dạng và kích thước giống nhau (điện tích q/2). Bất đẳng thức biểu
diễn điều kiện nên chứa các biến số q, r và các hằng số σ, ε0.
4) Xác định điện tích cực đại q (theo đơn vị điện tích nguyên tố, e) mà
một giọt nước với r = 10-5 cm có thể mang? Làm tròn kết quả tới số
nguyên. Sức căng bề mặt của nước là σ = 7.28∙10-2 J∙m-2
Khác với kĩ thuật phun electron, các phân tử bị ion hoá do va chạm
electron sẽ bị phân mảng đáng kể. Phổ khối của dẫn xuất fullerene
CN(CX3)n, nhận được từ phase hơi, được cho ở hình dưới. Nguyên tố X là
halogen. Tất cả các mũi đều biểu diễn những ion với điện tích +1, nhưng
khác nhau một số nhóm CX3.
5)
a) Trong phân tử dẫn xuất fullerene này có bao nhiêu (N) nguyên tử
carbon?
b) Xác định nguyên tố X;
c) Xác định số (n) nhóm chức trong hợp chất này.

131 | Bản quyền thuộc về Tạp chí Olympiad Hóa học KEM
Bài 9
X1 là một trong những hydrocarbon có công thức C 4H6, thể hiện những
tính chất rất thú vị trong phổ hồng ngoại. Hợp chất này được tổng hợp
lần đầu tiên vào thập niên 1930, bởi hai nhóm nghiên cứu độc lập từ Mỹ
và Soviet, với hai quy trình tổng hợp có các giai đoạn đầu và cuối khác
nhau. Dưới đây là sơ đồ tổng hợp của họ. Khi nghiên cứu về các phản ứng
đồng phân hoá của X1, các nhà nghiên cứu nhận thấy rằng khi dẫn qua
aluminum oxide tẩm sodium, trong khí quyển nitrogen thì chỉ có
hydrocarbon X2 được tạo thành. Còn khi dẫn qua sét floridine vi tinh thể
thì nó bị chuyển hoá một phần thành buta-1,3-diene và hydrocarbon X3.
Dưới đây là phổ IR của các hydrocarbon X1 - X3.

Sơ đồ 1: Điều chế các hydrocarbon X1, X2, X3.


1. Xác định các hợp chất A - E, X1 - X2, khi biết diện tích các mũi trong
phổ khối lượng của B - E dưới đây.

Phổ khối lượng của B.

132 | Bản quyền thuộc về Tạp chí Olympiad Hóa học KEM
Phổ khối lượng của C.

Phổ khối lượng của D.

Phổ khối lượng của E.

133 | Bản quyền thuộc về Tạp chí Olympiad Hóa học KEM
Phổ hồng ngoại của X1.

Phổ hồng ngoại của X2.

134 | Bản quyền thuộc về Tạp chí Olympiad Hóa học KEM
Phổ hồng ngoại của X3.
2. Giải thích nguyên nhân sự xuất hiện hoặc biến mất của các dải hấp thụ
trong phổ hồng ngoại của các hợp chất X1 - X3. Những nhóm chức
nào liên quan đến các dải hấp thụ này?
3. Giải thích loại peak của các ion phân tử trong các hợp chất B - E.
Một hợp chất thú vị khác có công thức phân tử C4H6 là X4. Phương pháp
điều chế hợp chất này được Gary Lumpman và James Aumiler của ĐH
West Washington công bố năm 1971, theo sơ đồ dưới đây. Một tính chất
phổ học thú vị của hợp chất này là việc nó có hai tín hiệu trong phổ 13C
NMR ở 33 và -3 ppm (!). Ngoài ra, hợp chất này còn có những tính chất
hoá học rất bất thường.

Sơ đồ 2: Điều chế X4.


4. Xác định các hợp chất A1 - E1, biết rằng các chất D1 và E1 đều tồn tại
ở dạng hai đồng phân dia, mỗi đồng phân đều có mặt phẳng đối xứng.

135 | Bản quyền thuộc về Tạp chí Olympiad Hóa học KEM
5. Gán các tín hiệu tương ứng trong phổ 1H và 13C NMR vào cấu tạo hợp
chất X4.

136 | Bản quyền thuộc về Tạp chí Olympiad Hóa học KEM
Bài 10
Tinh thể chất A màu trắng tan nhanh trong nước. Chất lỏng B có thể tan
trong nước theo mọi tỉ lệ. Phản ứng giữa A và B tạo thành chất lỏng C, với
dữ liệu phổ được cho dưới đây (các cường độ tương đối trong phổ NMR
có thể xác định bởi đường cong tích phân trên các tín hiệu; tín hiệu ở 0
ppm tương ứng với chất chuẩn TMS.)
1) Xác định cấu tạo của A, B, C. Gắn các tín hiệu trong phổ 1H NMR với
chất C.
2) Viết hướng phân mảnh chính trong phân tích khối phổ của các ion
phân tử A, C.
3) Xác định các điều kiện trong phản ứng giữa A và B.
4) Viết cơ chế của phản ứng.
Các dữ liệu phổ được cho ở trang sau.

Phổ khối lượng của chất A.

137 | Bản quyền thuộc về Tạp chí Olympiad Hóa học KEM
Phổ 1H NMR của chất C.

Phổ hồng ngoại của chất C.

Phổ khối lượng của chất C.

EtOH
max
lgmax
229 4.08
272 2.90
280 2.85
Dữ liệu phổ UV của chất C

138 | Bản quyền thuộc về Tạp chí Olympiad Hóa học KEM
Chuyên đề 40: Cơ chế phản ứng
Lưu ý các bài tập được sắp xếp theo trình tự ngẫu nhiên

Bài 1
Lí thuyết va chạm đòi hỏi các va chạm phải có đủ năng lượng và sự định
hướng đúng. Các thành phần khác của phân tử có thể chắn, làm giảm khả
năng xảy ra va chạm với định hướng đúng.
Chloroalkane có thể thế nhóm chloro bởi một nhóm OH, điển hình là phản
ứng với dung dịch hydroxide. Có hai cách xảy ra phản ứng, theo cơ chế
SN1 và SN2. Theo hướng SN2, OH- ion cần phải va chạm với nguyên tử
carbon liên kết trực tiếp với nhóm chloro (180o).
a) Xét các hợp chất sau. Hãy sẵp xếp chúng theo trật tự tăng dần khả
năng phản ứng theo cơ chế SN2. Giải thích ngắn gọn.

Ngoài ra, hydroxide có thể tách nhóm chloro trong các phản ứng tương
tự nhưng sử dụng dung môi alcohol. Lần này, va chạm diễn ra với nguyên
tử hydrogen trên một carbon kế cận với C-Cl, tạo thành nước, Cl- ion và
alkene.
b) Phản ứng tách nhóm chloro thường được mô tả là sự loại bỏ HCl. Giải
thích tại sao điểm hợp lí và chưa hợp lí của cách mô tả phản ứng tách
này.

139 | Bản quyền thuộc về Tạp chí Olympiad Hóa học KEM
Bài 2
Các alkene phản ứng với HBr tạo thành bromoalkane
a) Những thuật ngữ nào sau đây có thể dùng để mô tả phản ứng của
alkene với HBr:
Gốc tự do
Nucleophile
Phản ứng thế
Electrophile
Cộng hợp
Phản ứng của HBr với alkene diễn ra qua một tiểu phân trung gian
carbocation như sau:
v
Phản ứng của HBr với một số alkene tạo thành nhiều hơn một sản phẩm
nếu hai cation khác nhau có thể được tạo thành (thường sẽ có sản phẩm
chính và phụ, nhưng chúng ta không xét đến ở đây.)

Từ nhiều alkene khác nhau có thể tạo thành cùng một bromoalkane.

b) Vẽ công thức 3 alkene có thể phản ứng với HBr tạo thành phân tử sau
đây:

c) Hãy chọn ra các bromoalkane trong bảng 2 có thể tạo thành từ phản
ứng của mỗi alkene sau đây với HBr (không cần quan tâm đến các sản
phẩm chính và phụ.)

140 | Bản quyền thuộc về Tạp chí Olympiad Hóa học KEM
Khi trong dung dịch thiếu các tác nhân nucleophile mạnh, như Br -, thì
carbocation có thể bị tấn công bởi các electron của liên kết đôi C=C, ở một
vị trí nào đó trong phân tử, dẫn tới sự vòng hoá phân tử và tạo thành một
carbocation khác. Từ carbocation này cũng có thể tạo thành nhiều sản
phẩm khác nhau.

Đôi khi rất khó để hiểu cách thức các nguyên tử liên kết với nhau trong
phản ứng vòng hoá, tuy nhiên, có một chiến lược chung mà chúng ta sẽ
tóm lược bằng cách xét phản ứng tạo thành hợp chất 19. Cần nhớ rằng
không có liên kết đơn C-C nào bị phá vỡ trong phản ứng.
Tìm những nhóm nguyên tử trong sản phẩm không bị thay đổi về trật tự
liên kết so với chất đầu và đánh dấu chúng. Đây là một ý tưởng tốt để thử
xác định vị trí các nguyên tử ở các vị trí đầu mạch của chất đầu xuất hiện
trong sản phẩm. Trong các phân phức tạp hơn, hãy tìm kiếm những đặc

141 | Bản quyền thuộc về Tạp chí Olympiad Hóa học KEM
điểm như vòng hoặc nhóm methyl. Bằng phương pháp này, có thể đánh
dấu các nguyên tử a, b, d, j trong hợp chất 19.
Hãy bắt đầu một trong các nhóm nguyên tử mà bạn tự tin đã xác định
chính xác và xem liệu bạn có thể xác định đường đi tới nhóm tiếp theo
mà bạn tin tưởng hay không.

Nếu chúng ta vẽ lại chất đầu theo cách gấp khúc như sản phẩm, có thể dễ
dàng thấy được cách phản ứng đã diễn ra

Carbocation A+ dễ được tạo ra trong tự nhiên và có thể phản ứng để tạo


ra nhiều sản phẩm tự nhiên khác nhau bởi một hoặc nhiều quá trình vòng
hoá của các liên kết đôi trên các carbocation. Không có bất kì liên kết đơn
C-C nào bị phá huỷ trong các phản ứng.

d)
i. Chỉ ra các chữ chữ cái (trong A+ ) tương ứng với cặp nguyên tử
carbon đã liên kết với nhau để tạo thành α-terpineol.
ii. Đề nghị công thức cấu tạo cho tiểu phân trung gian carbocation
B+ .

142 | Bản quyền thuộc về Tạp chí Olympiad Hóa học KEM
e)
i. Chỉ ra các chữ chữ cái (trong A+ ) tương ứng với cặp nguyên tử
carbon đã liên kết với nhau để tạo thành borneol từ quá trình vòng
hoá carbocation B + .
ii. Đề nghị công thức cấu tạo cho tiểu phân trung gian carbocation
C+.
f)
i. Chỉ ra các chữ chữ cái (trong A+ ) tương ứng với cặp nguyên tử
carbon đã liên kết với nhau để tạo thành β-pinene từ quá trình
vòng hoá carbocation B + .
ii. Đề nghị công thức cấu tạo cho tiểu phân trung gian carbocation
D+ .
Quy trình sinh tổng hợp các steroid hormone được mô phỏng theo
chuyển đổi của hợp chất T thành hormone Progesterone. Phản ứng bắt
đầu bằng cách xử lí hợp chất T với acid, tạo thành cation U+ , tiểu phân này
sau đó bị mất nước tạo thành carbocation V+ .

g) Xác định công thức cấu tạo carbocation U+ .


Carbocation V+ bị vòng hoá tạo thành carbocation W+ . Không có liên kết
C-C nào bị phá vỡ trong quá trình này.
h) Chỉ ra các chữ chữ cái tương ứng với cặp nguyên tử carbon đã liên kết
với nhau trong quá trình vòng hoá carbocation V+ thành carbocation
W+ .
Çarbocation W+ có thể chuyển thành Progesterone sau 3 giai đoạn. Chú ý
rằng trong quá trình tạo thành hợp chất Y, ozone đã phá vỡ liên kết C=C
và tạo thành 2 liên kết C=O mới.

143 | Bản quyền thuộc về Tạp chí Olympiad Hóa học KEM
i) Vẽ công thức cấu tạo của hợp chất X, Y và anion Z -.

144 | Bản quyền thuộc về Tạp chí Olympiad Hóa học KEM
Bài 3
Xác định hóa lập thể của sản phẩm từ các phản ứng sau và cho biết phản
ứng thuộc loại SN1 hay SN2.
Loại
Sản
STT Chất đầu Tác nhân phản
phẩm
ứng

1 H2 S/KOH

2 K2 CO 3

3 H2 O

4 CH3 OH

145 | Bản quyền thuộc về Tạp chí Olympiad Hóa học KEM
Bài 4
Các bromoalkane được dùng làm nguyên liệu đầu để tổng hợp nhiều loại
thuốc. Trong một số trường hợp, các bromoalkane cần dùng có thể được
tạo thành bởi phản ứng của một alkane với bromine.
Từ phần a-f của câu hỏi này, bạn có thể giả sử rằng chỉ có duy nhất 1
nguyên tử hydrogen của mỗi alkane bị thay thế bởi một nguyên tử
bromine trong phản ứng của các alkane với bromine.
a)
i. 1 mol ethane phản ứng với 1 mol bromine tạo thành
bromoethane. Viết phương trình phản ứng (không cần viết kí hiệu
trạng thái chất).
ii. Phản ứng thuộc loại nào (cộng, tách, thế, thủy phân, cracking,
trùng hợp hóa)?
iii. Phản ứng giữa ethane và bromine cần ánh sáng và không thể diễn
ra trong bóng tối. Ánh sáng cần cho sự phá vỡ một liên kết để bắt
đầu phản ứng. Liên kết nào bị phá hủy bởi ánh sáng?
Khi ethane phản ứng với bromine, bất kì nguyên tử nào trong số 6 nguyên
tử hydrogen cũng có thể bị thay thế. Do tính đối xứng trong phân tử
ethane nên 6 nguyên tử hydrogen này tương đương với nhau, do vậy chỉ
có duy nhất một sản phẩm được tạo thành, bất kể nguyên tử hydrogen
nào bị thay thế.
Với các alkene lớn hơn, thường có sự tạo thành các đồng phân cấu tạo
khác nhau của sản phẩm bromoalkane, tùy thuộc vào nguyên tử hydrogen
nào bị thay thế bởi nguyên tử bromine. Trong phản ứng bromine
methylbutane, có thể tạo thành 4 bromoalkane (kí hiệu A-D).

b) Giả sử rằng khả năng bị thay thế của các nguyên tử hydrogen trong
methylbutane là tương đương nhau, hãy cho biết bromoalkane nào
(từ A-D) được tạo thành với tỉ lệ nhỏ nhất? Cho biết kí hiệu (A, B, C
hay D?) và tên hệ thống của alkane này.
c) Sử dụng giả thiết tương tự như phần b ở trên, tính phần trăm các chất
A-D trong hỗn hợp sản phẩm.

146 | Bản quyền thuộc về Tạp chí Olympiad Hóa học KEM
Khi tiến hành phản ứng này trong phòng thí nghiệm, tỉ lệ thực tế của các
sản phẩm nhận được là

Để xác định vị trí nào hoạt động nhất dưới những điều kiện này (nghĩa là
sự ưu tiên của mỗi nguyên tử hydrogen để bị thay thế bởi bromine),
chúng ta cần so sánh tỉ lệ phần trăm của mỗi sản phẩm thực tế với dự
đoán ở phần 3. Điều này cho phép đánh giá độ hoạt động của mỗi vị trí.
d) Xác định tỉ lệ độ hoạt động của sự thay thế 1 hydrogen để tạo hợp
chất B với hợp chất C.
e) Dựa vào tính đối xứng của các alkane sau, hãy xác định có bao nhiêu
đồng phân cấu tạo bromoalkane khác nhau có thể được tạo thành
bằng cách thay thế 1 nguyên tử hydrogen bởi 1 bromine.

Memantine là một trong những loại thuốc sẵn có để giúp giảm các triệu
chứng của bệnh Alzheimer. Quy trình tổng hợp metantine bắt đầu với
phản ứng bromine hóa 1,3-dimethyladamantane để tạo thành hợp chất
E, với sự thay thế 1 nguyên tử hydrogen bởi 1 bromine.

147 | Bản quyền thuộc về Tạp chí Olympiad Hóa học KEM
f)
i. Có bao nhiêu đồng phân cấu tạo khác nhau của bromoalkane E có
thể được tạo thành bởi sự thay thế 1 hydrogen trong 1,3-
dimethyladamantane bởi bromine.
ii. Trong tổng hợp memantine trong phòng thí nghiệm, chỉ có 1 đồng
phân cấu tạo của E được tạo thành. Dựa vào thông tin trước đó
của câu hỏi này, hãy vẽ cấu trúc của đồng phân này.
Chỉ cần 2 giai đoạn để chuyển hóa E thành memantine. Giai đoạn đầu tiên
là phản ứng Ritter, được mô tả như dưới đây. Kí hiệu R biểu diễn một
nhóm alkyl.

Dưới đây là chuyển hóa E thành memantine.

g) Vẽ công thức cấu tạo của hợp chất F, memantine, và anion G- (có khối
lượng phân tử bằng 59,0).
Thuốc carpipramine được dùng để điều trị tâm thần phân liệt và chứng lo
âu, bồn chồn. Quy trình tổng hợp carpipramine sử dụng một trong các
chất đầu là 1,3-dibromopropane.
h) Vẽ các đồng phân cấu tạo khác của dibromopropane.
Nếu thêm 2 mol bromine vào 1 mol propane, sau đó chiếu sáng hỗn hợp
thì thu được một hỗn hợp các đồng phân dibromopropane.

148 | Bản quyền thuộc về Tạp chí Olympiad Hóa học KEM
i) Tính phần trăm 1,3-dibromopropane có thể được tạo thành trong hỗn
hợp này nếu khả năng bị thay thế của các nguyên tử hydrogen là giống
nhau.
Đáng tiếc, không chỉ có mỗi 1,3-dibromopropane được tạo thành khi tiến
hành phản ứng này trong phòng thí nghiệm, do phản ứng ưu tiên tạo ra
các đồng phân khác. Tuy nhiên, một hydrocarbon khác (H) khi phản ứng
với bromine tạo thành 1,3-dibromopropane. Hydrocarbon này có chứa
85,63 % carbon về khối lượng và khi phản ứng với bromine không tạo
thành sản phẩm nào khác 1,3-dibromopropane.
j)
i. Vẽ công thức cấu tạo của H.
ii. Phản ứng của hydrocarbon H với bromine tạo thành 1,3-
dibromopropane thuộc loại nào (cộng, tách, thế, thủy phân,
cracking, trùng hợp hóa)?
Dưới đây là tổng hợp carpipramine:

k) Vẽ công thức cấu tạo của hợp chất I và tác nhân J.


1,3-dibromopropane cũng được dùng trong tổng hợp thuốc
ethoheptazine, dùng để điều trị các cơn đau và chứng bồn chồn, lo âu.
Dưới đây là quy trình tổng hợp:

149 | Bản quyền thuộc về Tạp chí Olympiad Hóa học KEM
Các nguyên tử carbon trong ethoheptazine không chỉ có từ chất đầu mà
còn có từ 1,3-dibromopropane hoặc ethanol. Không có liên kết C-C nào bị
phá vỡ trong tổng hợp.
l)
i. Khoanh tròn những nguyên tử carbon được tạo thành từ 1,3-
dibromopropane.
ii. Khoanh tròn những nguyên tử carbon được tạo thành từ ethanol.
Tổng hợp ethoheptazine đi qua 5 giai đoạn. Trong giai đoạn đầu tiên, chất
đầu bị deproton hóa tạo thành K-, là một carbanion (nghĩa là ion với
nguyên tử carbon mang điện tích âm).
m) Xác định công thức cấu tạo của carbanion K-, hợp chất L, cation M + và
hợp chất N.

150 | Bản quyền thuộc về Tạp chí Olympiad Hóa học KEM
Bài 5
1) Gọi tên IUPAC (kèm theo kí hiệu lập thể E/Z) của hợp chất sau:

Một trong các phương pháp được dùng nhiều nhất để tạo liên kết đôi
carbon-carbon là sử dụng phản ứng tách β như sau:

Cấu trúc lập thể để phản ứng tách lưỡng phân tử (E2) xảy ra như trên đòi
hỏi sự phân bố hình học dạng trans (trans geometry) của nguyên tử β-
hydrogen và nhóm rời đi (X). Trong đa số trường hợp, sản phẩm chính là
olefin nhiều nhóm thế nhất (ở C=C). Tuy nhiên, trong phản ứng tách của
các muối ammonium và sulphonate thì olefin ít thế nhất mới là sản phẩm
chính.
2) Xác định sản phẩm chính (A-D) trong các phản ứng sau. Xác định hóa
lập thể của sản phẩm nếu có thể.

i.

ii.

iii.

iv.

Phản ứng tách nhiệt là một hướng tổng hợp khác để tạo ra alkene. Phản
ứng được thực hiện đồng bộ qua một trạng thái chuyển tiếp dạng vòng.

151 | Bản quyền thuộc về Tạp chí Olympiad Hóa học KEM
Phản ứng nhiệt phân thường được tiến hành với: carboxylate ester,
sulfoxide, xanthate, … Các phản ứng này diễn ra theo cơ chế syn, với
nguyên tử hydrogen và nhóm X tách ra cùng một phía.
3) Xác định sản phẩm chính (F-H) trong các phản ứng sau. Xác định hóa
lập thể của sản phẩm nếu có thể.

i.

ii.

Các phản ứng phân mảnh, tương tự phản ứng tách β, cũng được sử dung
để tổng hợp các olefin. Dưới đây là một ví dụ:

Dưới đây là chuỗi phản ứng để điều chế một chất trung gian cho quá trình
tổng hợp juvenile hormone – có chức năng điều hóa sinh lý côn trùng.
4) Xác định các sản phẩm (I-K), chỉ rõ hóa lập thể, trong chuỗi phản ứng
sau.
Một trong các hướng tổng hợp E-olefin phổ biến là sử dụng phản ứng
olefin hóa Julia, đi từ các hợp chất sulfone có một nhóm rời đi tốt ở vị trí
β.

152 | Bản quyền thuộc về Tạp chí Olympiad Hóa học KEM
Các sphingolipid là nhóm hợp chất thiên nhiên có vai trò bảo vệ bề mặt tế
bào khỏi các yếu tố nguy hiểm từ môi trường. Dưới đây là sơ đồ tổng hợp
tiền chất của sphingosin - chất tạo ra bộ khung của sphingolipid.
5) Xác định các sản phẩm L-N.

Có thể cộng hợp dễ dàng các diborane vào alkene để điều chế các hợp
chất borane hữu cơ dạng monoalkyl, dialkyl hoặc trialkyl. Borane hữu cơ
có thể chuyển hóa thành các liên kết C-O, C-N, C-C qua phản ứng của
nucleophile, theo sau đó là sự chuyển vị như trong sơ đồ sau đây:

6) Hoàn thành chuỗi phản ứng sau bằng cách xác định các hợp chất O-
Q.

7) Viết công thức cấu tạo các chất R-T trong các phản ứng sau và xác định
hóa lập thể của chúng nếu có thể.
i.

3 đương lượng

153 | Bản quyền thuộc về Tạp chí Olympiad Hóa học KEM
ii.

Các olefin thường có tính nucleophilic (ái nhân), nhưng khi có nhóm hút
electron gắn vào liên kết bội thì nó sẽ có tính electrophilic (ái điện tử). Các
olefin này dễ xảy ra phản ứng thế thông qua hệ liên hợp khi có các nhóm
rời đi tốt ở vị trí β.
8) Xác định sản phẩm của phản ứng sau:

9) Hoàn thành sơ đồ tổng hợp thuốc chống loét Tagamet sau bằng cách
xác định công thức cấu tạo các chất U-W.

154 | Bản quyền thuộc về Tạp chí Olympiad Hóa học KEM
Bài 6
Nhiều phản ứng hữu cơ có sự tạo thành các tiểu phân trung gian như
carbocation, carbanion, gốc carbon tự do, carbene, … Cấu trúc và độ bền
của các tiểu phân trung gian là những yếu tố quyết định cơ chế phản ứng.
Ví dụ, trong một phản ứng có sự hình thành của carbocation thì độ bền
của carbocation sẽ quyết định tốc độ phản ứng.
Định đề Hammond là công cụ quan trọng để nghiên cứu về trạng thái
chuyển tiếp. Định đề này chỉ ra rằng trạng thái chuyển tiếp có cấu trúc gần
giống với hệ có năng lượng tự do gần nó hơn.
1) Sắp xếp các olefin sau theo thứ tự đúng về tốc độ phản ứng cộng HI.

Chuyển vị carbocation là phản ứng khá phổ biến. Nhóm bị chuyển vị


thường là nhóm giàu electron. Do phản ứng chuyển vị nên thi thoảng vẫn
có một sản phẩm khác so với dự kiến được tạo thành.
Cho hợp chất A phản ứng với một Bronsted acid thu được sản phẩm C.
Trong phản ứng này, B là tiểu phân trung gian tạo ra qua quá trình chuyển
vị.

2) Xác định B, C.
Cho hợp chất D phản ứng với một lượng nhỏ clo dưới tác dụng của tia tử
ngoại (UV) ở diều kiện thường, tạo ra một hỗn hợp sản phẩm.

155 | Bản quyền thuộc về Tạp chí Olympiad Hóa học KEM
3) Xác định sản phẩm chính của phản ứng.
Cho các hợp chất sau:

4) Hãy cho biết những chất nào


a) có tính thơm?
b) phản thơm?
c) không thơm?
d) không thơm nhưng có cấu trúc cộng hưởng có tính thơm?
e) không thơm nhưng có base liên hợp có tính thơm?
f) không thơm, có pKa xấp xỉ -3.8.
Các hợp chất thơm rất khó bị khử. Tuy nhiên, nhà hóa học người Úc,
A.J.Birch, đã phát triển thành công một phương pháp để khử các hợp chất
thơm thành diene không liên hợp bằng phản ứng với Li/K/Na trong
ammonia lỏng, có mặt alcohol. Ví dụ, có thể khử benzene thành 1,4-
cyclohexadiene. Phương pháp này được gọi là phản ứng khử Birch. Trong
phản ứng, nguyên tử kim loại nhường 1 electron cho vòng thơm để tạo
thành anion gốc (tiểu phân trung gian dạng anion có 1 electron chưa ghép

156 | Bản quyền thuộc về Tạp chí Olympiad Hóa học KEM
cặp). Trong tiểu phân trung gian này, vị trí tương đối của anion và gốc là
1,4.
Năm 1944, Birch đã thực hiện phản ứng khử 3-methylanisole (3-
methylmethoxybenzene), thu được sản phẩm F. Đun nóng F với acid vô
cơ loãng, thu được chất G (C 7H10O).
5) Xác định các công thức cấu tạo có thể có của F.
6) Xác định các công thức cấu tạo có thể có của G.
Chất G thu được trong thực tế không phải là hợp chất quang hoạt. Khi
phản ứng với Br 2, G tạo ra H. Đun nóng H với KOH trong alcohol, thu được
I (C 7H9BrO).
7) Xác định G, H, I.
E.Vogel nổi tiếng vì những nghiên cứu về các annulene có cầu nối.
Annulene là các hợp chất vòng lớn, chứa các hệ liên hợp nối tiếp.
Annulene O được tổng hợp từ naphthalene (J) theo chuỗi phản ứng dưới
đây. Biết rằng:
- Hợp chất K cộng hợp được 3 đương lượng bromine;
- Carbene là tiểu phân trung gian rất hoạt động, phản ứng được với liên
kết đôi carbon-carbon tạo thành vòng cyclopropane.
8) Xác định công thức cấu tạo các chất còn thiếu trong chuỗi phản ứng.

157 | Bản quyền thuộc về Tạp chí Olympiad Hóa học KEM
Hợp chất carbonyl với nguyên tử hydrogen có tính acid có thể xảy ra hiện
tượng hỗ biến hay còn gọi là tautome hóa và có thể tồn tại ở dạng enol.
Base liên hợp của enol được gọi là enolate.
9)
i. Viết công thức cấu tạo của enol bền nhất và enolate tương ứng
của hợp chất sau:

ii. Xác định sản phẩm chính trong các phản ứng sau:

158 | Bản quyền thuộc về Tạp chí Olympiad Hóa học KEM
Aza-enolate là các hợp chất chứa nitrogen có cấu tạo tương tượng
enolate.
10) Xác định công thức cấu tạo của aza-enolate của hợp chất sau:

11) Có thể điều chế hợp chất trên bằng phản ứng ngưng tụ của
A. một aldehyde và một amine bậc II.
B. một ketone và một amine bậc II.
C. một aldehyde và một amine bậc I.
D. một ketonee và một amine bậc I.
Phản ứng Mannich là phản ứng giữa formaldehyde, amine và ketone. Ví
dụ:

12) Xác định công thức cấu tạo của các hợp chất P-S trong sơ đồ tổng hợp
thuốc gây mê cục bộ Tutocaine hydrochloride (S). Biết: giai đoạn đầu
của sơ đồ là phản ứng Mannich.

159 | Bản quyền thuộc về Tạp chí Olympiad Hóa học KEM
160 | Bản quyền thuộc về Tạp chí Olympiad Hóa học KEM
Bài 7
Hợp chất 2,2,4-trimetylpentan (A) được sản xuất với quy mô lớn bằng
phương pháp tổng hợp xúc tác từ C4H8 (X) với C 4H10 (Y). A cũng có thể
được điều chế từ X theo hai bước: thứ nhất, khi có xúc tác axit vô cơ, X
tạo thành Z và Q; thứ hai hydro hóa Q và Z
a) Viết các phương trình phản ứng để minh họa và gọi tên các hợp chất
X, Y, Z, Q theo danh pháp IUPAC
b) Ozon phân Z và Q sẽ tạo thành 4 hợp chất, trong đó có axeton và
fomandehit, viết cơ chế phản ứng

161 | Bản quyền thuộc về Tạp chí Olympiad Hóa học KEM
Bài 8
Đề xuất cơ chế tạo thành (1-bromoethyl)benzene từ phản ứng của
xiclooctatetraen với HBr.

162 | Bản quyền thuộc về Tạp chí Olympiad Hóa học KEM
Bài 9
Diene không liên hợp là chất trung gian quan trọng để tổng hợp các hợp
chất thiên nhiên. Dưới đây là chuỗi phản ứng đơn giản để tổng hợp một
diene như vậy.
1) Xác định công thức cấu tạo các sản phẩm A và B trong chuỗi sau đây:

Chuyển vị di-pi methane là dạng phản ứng quang hóa quan trọng, trong
đó diene không liên hợp chuyển thành cyclopropane có nhóm thế vinyl
như sau:

Zizaene là một terpene thiên nhiên có trong dầu cỏ hương lau (cỏ Vetiver)
và được dùng để tổng hợp α-vetivone (sử dụng trong nhiều loại nước
hóa). Zizaene được tổng hợp từ hợp chất C sau:

2) Xác định công thức cấu tạo tiền chất của C biết rằng C được tạo thành
từ phản ứng chuyển vị di-pi methane.
Chuyển vị vinyl cyclopropane thành cyclopentane (như hình dưới) là một
phương pháp quan trọng dể tổng hợp các hệ vòng carbon 5 cạnh. Các
phản ứng chuyển vị này có thể xảy ra dưới tác dụng nhiệt, quang hóa hoặc
khi có xúc tác. Dưới tác dụng nhiệt và quang hóa, sự chuyển vị diễn ra qua
sự hình gốc kép bền nhất.

163 | Bản quyền thuộc về Tạp chí Olympiad Hóa học KEM
Phản ứng chuyển vị vinyl cyclopropane là giai đoạn chính trong chuỗi tông
hợp Zizaene dưới đây.
3) Xác định công thức cấu tạo các chất D-F.

E phản ứng với LiN(i-Pr)2 (LDA – một base có cấu tạo cồng kềnh) và PhS-
SPh tạo ra hợp chất G. Oxid hóa G bằng NaIO 4 tạo ra hợp chất trung gian
H, và cuối cùng là sản phẩm I.

4) Xác định công thức cấu tạo các chất G, H, I.


5) Xác định công thức cấu tạo của J, K trong chuỗi phản ứng sau:

Hợp chất N là chất trung gian trong chuỗi phản ứng tổng hợp Zizaene. N
được tổng hợp theo cách sau:

6) Xác định các chất L, M, N.

164 | Bản quyền thuộc về Tạp chí Olympiad Hóa học KEM
Biết thành phần nguyên tố trong L là: %C = 60,55; %H = 6,41; %O = 20,95;
còn trong N là: %C = 75,63; %H = 9,97.
Ylide là tiểu phân có điện tích dương trên nguyên tử dị tố và điện tích âm
trên nguyên tử carbon liền kề. Phản ứng Wittig là phản ứng có sự tham
gia của phosphorus ylide.

7) Xác định công thức của O, P và Zizaene.

8) Zizaene có bao nhiêu đồng phân quang học?

165 | Bản quyền thuộc về Tạp chí Olympiad Hóa học KEM
Bài 10
Tương tác giữa các aryl halide và amine là một trong những phản ứng
được nghiên cứu và sử dụng nhiều nhất trong tổng hợp hữu cơ.
1) Xác định sản phẩm hữu cơ của phản ứng sau:

2) Trình bày cơ chế phản ứng trên và cho biết đây là loại cơ chế nào?
3) Xác định chất nào sau đây phản ứng với diethylamine nhanh nhất?
Chất nào phản ứng chậm nhất? Giải thích.

4) Xác định chất nào sau đây phản ứng với diethylamine nhanh hơn?
Giải thích.

5) Tại sao hợp chất A phản ứng với diethylamine chậm hơn nhiều so
với hợp chất B?

166 | Bản quyền thuộc về Tạp chí Olympiad Hóa học KEM
167 | Bản quyền thuộc về Tạp chí Olympiad Hóa học KEM
Bài 11
Phản ứng xảy ra giữa một carbanion và hợp chất carbonyl α,-không no
thông thường là phản ứng cộng Micheal, ví dụ:

Phản ứng phát triển mạch carbon này là một trong những phương pháp
hiệu quả để điều chế các hợp chất vòng.
Một phản ứng theo kiểu “thác nước” (cascade) hay “dây chuyền”
(domino) là chuỗi liên tiếp các phản ứng hữu cơ nội phân tử, diễn ra qua
các trạng thái trung gian hoạt động. Phản ứng này cho phép tổng hợp các
hợp chất đa nhân phức tạp từ một tiền chất mạch hở duy nhất. Chất nền
thường chứa nhiều nhóm chức hoá học và được chuyển hoá cùng một
lúc. Định nghĩa này nhấn mạnh điều kiện tiên quyết là “nội phân tử” để
phân biệt phả nứng này với các loại phản ứng đa tác chất.
Một hợp chất có tên gọi là ester Hagemann tham gia phản ứng dây chuyền
(bao gồm cả phản ứng cộng Micheal) để tạo thành sản phẩm như sau:

Hãy đề nghị cơ chế hợp lí để giải thích sự tạo thành sản phẩm của phản
ứng trên.

168 | Bản quyền thuộc về Tạp chí Olympiad Hóa học KEM
Bài 12
Giải thích cơ chế các phản ứng sau:

169 | Bản quyền thuộc về Tạp chí Olympiad Hóa học KEM
Bài 13
Viết cơ chế của phản ứng:
O
KOH, H2O
N Br o
H2N-CH2-CH2-COOK
40 C
O

170 | Bản quyền thuộc về Tạp chí Olympiad Hóa học KEM
Bài 14
Hoàn thành cơ chế phản ứng sau:

171 | Bản quyền thuộc về Tạp chí Olympiad Hóa học KEM
Bài 15
Bằng cơ chế phản ứng, giải thích sự hình thành sản phẩm trong các sơ đồ
chuyển hóa sau:

172 | Bản quyền thuộc về Tạp chí Olympiad Hóa học KEM
Bài 16
Dùng cơ chế để giải thích quá trình tạo thành sản phẩm trong các phản ứng
sau:

173 | Bản quyền thuộc về Tạp chí Olympiad Hóa học KEM
Bài 17
Enamin có thể được tạo thành khi cho anđehit hoặc xeton phản ứng với
amin bậc hai có xúc tác axit. Xiclohexanon phản ứng với piroliđin tạo ra
enamin H theo sơ đồ sau:
O H
N +
H
+ N H

Đề xuất cơ chế giải thích quá trình tạo thành enamin H.

174 | Bản quyền thuộc về Tạp chí Olympiad Hóa học KEM
Bài 18
Phản ứng sau là một thí dụ của quá trình axyl hóa enamin:
PhCO

CHCl3
Cl-
2 N COCl N +N

Hãy viết cơ chế của phản ứng trên và so sánh với cơ chế phản ứng axyl
hóa amoniac (sự giống nhau và khác nhau giữa hai cơ chế phản ứng).

175 | Bản quyền thuộc về Tạp chí Olympiad Hóa học KEM
Bài 19
Hãy trình bày cơ chế phản ứng của các chuyển hóa sau:
a)
O
1. NaNH2
2
+
2. H3O
O
O

b)
COOCH3
CH2COOCH3 1.CH3ONa
+
COOCH3 2. H3O O

176 | Bản quyền thuộc về Tạp chí Olympiad Hóa học KEM
Bài 20
Cho các quá trình phản ứng sau:
a) Et COPh O Et
Ph COEt
Ph
(1) (2)
AlCl3 /xilen AlCl3 /xilen
o o
t , 48h t , 4h
H = 35% H = 90%

b)
O Br H
KOH/EtOH, t
o O

Br
O OH
O

Dùng cơ chế để giải thích sự tạo thành sản phẩm trong các quá trình phản
ứng trên. Giải thích rõ tại sao có sự khác biệt giữa thời gian và hiệu suất
của phản ứng (1) và (2) (trong (a)), biết nhiệt độ thực hiện hai phản ứng
là như nhau.

177 | Bản quyền thuộc về Tạp chí Olympiad Hóa học KEM
Bài 21
Monastrol được dùng làm thuốc ức chế sự phân bào của các tế bào ung thư.
Trong công nghiệp, monastrol được tổng hợp bằng cách sử dụng phản ứng đa
tác nhân theo sơ đồ hình trên. Hãy đề xuất cơ chế để giải thích sự tạo thành
sản phẩm của phản ứng.

178 | Bản quyền thuộc về Tạp chí Olympiad Hóa học KEM
Bài 22
Phản ứng đa tác nhân là phương pháp tiên tiến để tổng hợp các dị vòng
thơm.
a) Dị vòng 1,4-đihiđro piriđin tạo thành qua các giai đoạn ngưng tụ
anđol, tạo imin, cộng Michael và ngưng tụ aza-anđol theo sơ đồ như
hình dưới. Đề xuất cơ chế của phản ứng.
b) Hợp chất đa dị vòng chứa pyrazin hình thành theo sơ đồ dưới. Đề xuất
cơ chế phản ứng.

179 | Bản quyền thuộc về Tạp chí Olympiad Hóa học KEM
Bài 23
Phản ứng đa tác nhân là một phương pháp để tổng hợp các dị vòng. Dị
vòng X được điều chế theo phản ứng ở hình dưới. Đề xuất cơ chế để giải
thích sự tạo thành X.

180 | Bản quyền thuộc về Tạp chí Olympiad Hóa học KEM
Bài 24
Đề xuất cơ chế để giải thích sự tạo thành sản phẩm trong mỗi phản ứng
sau:
a)

b)

181 | Bản quyền thuộc về Tạp chí Olympiad Hóa học KEM
Bài 25
Đề xuất cơ chế phản ứng chuyển vị bởi nhiệt từ A1 tạo thành adamantan.

182 | Bản quyền thuộc về Tạp chí Olympiad Hóa học KEM
Bài 26
Đề nghị cơ chế phản ứng tạo thành (các) sản phẩm sau:
a)

b)

183 | Bản quyền thuộc về Tạp chí Olympiad Hóa học KEM
Bài 27
Khi nhiệt phân các hợp chất (A), (B), (C), (D) người ta thu được các sản
phẩm khác nhau. Hãy viết công thức và tên sản phẩm; giải thích (dùng
mũi tên cong) vì sao có sự tạo thành các sản phẩm đó.
1) (A) CH3CH2COOCH2CH2CH2CH3
2) (B) CH3[CH2]5CH(OH)CH2CH=CH[CH2]7COOH
3) (C) (CH3)3CCH(CH3)OCSSCH3
4) (D) CH3[CH2]3C(OH)(CH3)CH2CH=CH[CH2]3COOCH(CH3)[CH2]3CH3

184 | Bản quyền thuộc về Tạp chí Olympiad Hóa học KEM
Bài 28
Dùng mũi tên cong chỉ rõ cơ chế chuyển 7-đehiđrocholesterol (I) thành
vitamin D3 (II) và cho biết cấu dạng bền của nó. (R: -CH(CH3)-(CH2)3-
CH(CH3)2)
R R

(I) (II)
HO HO

185 | Bản quyền thuộc về Tạp chí Olympiad Hóa học KEM
Bài 29
Xitral (C 10H16O) là một monotecpen-anđehit có trong tinh dầu chanh. Oxi
hóa xitral bằng KMnO 4 thu được axit oxalic, axeton và axit levulinic (hay
axit 4-oxopentanoic). Từ xitral người ta điều chế β-ionon để điều chế
vitamin A.
a) Xác định cấu tạo và viết tên hệ thống của xitral.
b) Viết sơ đồ các phản ứng chuyển hóa xitral thành β-ionon
β-ionon sinh ra có lẫn một lượng đáng kể chất đồng phân cấu tạo là α-
ionon, không thể tách ra bằng cách chưng cất.
c) Trình bày cơ chế tạo thành α-ionon cùng với β-ionon Nêu phương
pháp tách riêng hai đồng phân đó.
d) Từ β-ionon, axetilen, hợp chất dưới đây và các hoá chất cần thiết, hãy
điều chế vitamin A.

O
OCOCH3

186 | Bản quyền thuộc về Tạp chí Olympiad Hóa học KEM
Bài 30
Ete crao và criptan là những hợp chất vòng có nhiều ứng dụng thực tiễn.
Ete crao có khả năng tạo phức chọn lọc với kim loại kiềm. Ete [18]crao-6
(A) sinh ra trong phản ứng Williamson thông thường (1) với hiệu suất (hs)
rất thấp:

Trái lại, có thể điều chế A với hiệu suất rất cao theo phản ứng (2):

Giải thích sự khác nhau về hiệu suất của hai phản ứng. Trình bày cơ chế
phản ứng (2).

187 | Bản quyền thuộc về Tạp chí Olympiad Hóa học KEM
Bài 31
Fanezol (tách được từ hoa linh lan) và các đồng phân lập thể của nó
(C 15H26O, tất cả kí hiệu là Pi với i: 1, 2, 3, …) khi bị ozon phân chế hóa khử
thì đều cho axeton, 2-hiđroxyetanal và 4-oxopentanal. Khi Pi bị đun nóng
với axit thì đều tạo ra hỗn hợp các chất có công thức C 15H24 là sản phẩm
chính của mỗi phản ứng, gọi chung là Qi với i: 1, 2, 3, …
1) Hãy vẽ công thức cấu trúc các hợp chất Pi.
2) Từ mỗi hợp chất Pi đã tạo ra những hợp chất Qi nào? Giải thích bằng
cơ chế phản ứng.

188 | Bản quyền thuộc về Tạp chí Olympiad Hóa học KEM
Bài 32
1) Hãy cho biết sản phẩm tạo thành khi:
- Đun nóng pyridin với hỗn hợp axit nitric và axit sunfuric.
- Đun nóng 2,4,6-trimetylpyridin với axit nitric đặc.
2) Ba hợp chất dị vòng 5 cạnh 2 dị tử thường gặp là imidazol, oxazol và
thiazol đều là những tiền chất được sử dụng trong tổng hợp các hợp chất
có hoạt tính sinh học.
a) Cho imidazol phản ứng với photgen thu được 1,1’-carbonyldiimidazol
(C 7H6N4O, ký hiệu CDI). Hãy đề xuất cơ chế phản ứng đó.
b) Chất CDI được sử dụng để hoạt hóa nhóm carboxyl trong tổng hợp
peptit do tạo thành hợp chất trung gian là 1-axylimidazol hoạt động tương
tự axyl halogenua. Hãy viết phương trình phản ứng tổng hợp dipeptit Ala
– Gly từ alanin và glyxin, có sử dụng CDI.

189 | Bản quyền thuộc về Tạp chí Olympiad Hóa học KEM
Bài 33
Hãy đề xuất cơ chế để giải thích quá trình tạo ra sản phẩm của các phản
ứng sau:

190 | Bản quyền thuộc về Tạp chí Olympiad Hóa học KEM
Bài 34
Đề nghị cơ chế để giải thích sự hình thành sản phẩm của các phản ứng
sau:

191 | Bản quyền thuộc về Tạp chí Olympiad Hóa học KEM
Bài 35
Đun terpenoid 3,7-dimetylocta-2,6-dienol (geraniol) với etyl vinyl ete (có
xúc tác H+ ) thu được andehit mạch hở F (C 12H20O). Xác định công thức cấu
tạo của F và đề xuất cơ chế phản ứng tạo thành F.

192 | Bản quyền thuộc về Tạp chí Olympiad Hóa học KEM
Bài 36
Phản ứng domino thường xảy ra êm dịu trải qua nhiều giai đoạn trung
gian, thân thiện với môi trường và có chọn lọc lập thể cao. Đề xuất cơ chế
phản ứng domino chuyển hóa N,N’-dimetylbarbituric theo sơ đồ:

193 | Bản quyền thuộc về Tạp chí Olympiad Hóa học KEM
Bài 37
Đề xuất cơ chế giải thích sự hình thành sản phẩm của các phản ứng sau,
biết trong các phản ứng đều hình thành trung gian xeten và đều trải qua
giai đoạn chuyển vị [3.3].
a)

b)

194 | Bản quyền thuộc về Tạp chí Olympiad Hóa học KEM
Bài 38
Đề nghị cơ chế cho các phản ứng sau:

195 | Bản quyền thuộc về Tạp chí Olympiad Hóa học KEM
Bài 39
Hãy cho biết trung gian M và giải thích sự tạo thành P1, P2, P3 bằng cơ chế
phản ứng:

196 | Bản quyền thuộc về Tạp chí Olympiad Hóa học KEM
Chuyên đề 41: Xác định sản phẩm phản ứng

Bài 1
Đọc và trả lời các câu hỏi từ (109)-(123).
Thường thì, khi biểu diễn công thức cấu tạo, các nguyên tử carbon và
hydrogen liên kết với carbon thường bị lược bỏ - như cách biểu diễn trong
hình dưới đây - trừ khi chúng có đặc trưng cụ thể. Ngoài ra, các kí hiệu R
và E trong công thức cấu tạo là chỉ các nguyên tử hoặc nhóm nguyên tử.
Các chỉ dẫn cũng được thêm vào khi cần thiết.

Các phân tử hữu cơ có tên là benzene, trong đó 6 nguyên tử nối với nhau
thành vòng và tạo thành một lục giác đều, là phần cấu trúc quan trọng có
trong bộ khung của nhiều hợp chất hữu cơ. Trong bài tập này, chúng ta
tập trung vào việc làm thế nào để tạo nên lục giác đều này (gọi là vòng
benzene) bởi phương pháp hóa học, và chúng ta sẽ tìm hiểu cách xác định
các hợp chất hữu cơ bằng phương pháp khối phổ.
Giáo trình phổ thông cho biết các phân tử hữu cơ có 2 nguyên tử carbon
nối với nhau bởi liên kết ba, gọi là acetylene, sẽ tam hợp hóa [trimer hóa]
tạo thành vòng benzene (hình 1). Từ khi phản ứng được nhà hóa học
người Pháp M. P. E. Berthelot phát hiện vào năm 1866, nhiều xúc tác (chất
thúc đẩy tiến trình [tốc độ] phản ứng) đã được phát triển để thực hiện
phản ứng hiệu quả và chọn lọc. Đặc biệt, nhiều xúc tác phát triển trên cơ
sở phức kim loại đã được W. J. Reppe báo cáo vào năm 1948.

Hình 1
Ngoài acetylene đơn giản, các phân tử hữu cơ với liên kết ba (như
propyne, …) cũng có cách thức phản ứng tương tự, nhưng nhiều hợp chất
sẽ được tạo thành tùy thuộc vào định hướng của các phân tử phản ứng
(định hướng R trong hình 2). Ví dụ, khi một phân tử có liên kết ba như

197 | Bản quyền thuộc về Tạp chí Olympiad Hóa học KEM
trong hình 2 phản ứng, sử dụng xúc tác chứa molybdenum (Mo), thì có 2
loại phân tử được tạo thành.

Hình 2
Câu hỏi ア: Trong phản ứng ở hình 2, khi định hướng của phân tử khi
phản ứng không bị ảnh hưởng bởi R thì tỉ lệ tối giản của sản phẩm phản
ứng là: I : II = (109) : (110)
Trong hình 3, khi các kiểu phân tử khác nhau phản ứng đồng thời thì cũng
có nhiều phân tử được tạo thành.

Hình 3
Câu hỏi イ: Khi 2 loại phân tử phản ứng đồng thời, như trong hình 3, thì
số loại phân tử có thể được tạo thành là (111), bao gồm cả 2 kiểu đã liệt
kê trong hình.
Câu hỏi ウ: Biết rằng trong phản ứng ở hình 3, nếu sử dụng một xúc tác
nhất định chứa iridiium (Ir), thì có thể tổng hợp chọn lọc với chỉ duy nhất
một hợp chất được tạo thành. Khối lượng phân tử của hợp chất này là
370. Hãy chọn trong số các chất (từ ①〜⑧) dưới đây cấu trúc của hợp
chất này. (112)

198 | Bản quyền thuộc về Tạp chí Olympiad Hóa học KEM
Phản ứng này cũng có thể được áp dụng với các chất có đồng thời 3 liên
kết ba (triyne) trong phân tử.
Câu hỏi エ: Trong số các phương án ①〜⑥, hãy chọn ra sản phẩm có
thể được tạo thành khi các liên kết 3 của hợp chất (triyne) sau đây phản
ứng với nhau. (113)

Phản ứng này thường được sử dụng để tổng hợp


các hợp chất thiên nhiên. Ví dụ, bộ khung của hợp
chất thiên nhiên alcyopterosin E có thể được tạo
thành bởi phương pháp này.
Câu hỏi オ: Tôi muốn tổng hợp triyne phù hợp để
điều chế alcyopterosin E bởi phản ứng ester hóa. Hãy chọn trong số các
phương án dưới đây, carboxylic acid và alcohol có thể sử dụng. Chú ý rằng
trong phản ứng ester hóa thì 1 phân tử nước được tách từ -COOH của
carboxylic và -OH của alcohol, tạo thành liên kết ester -COO-.
Carboxylic acid (114)

199 | Bản quyền thuộc về Tạp chí Olympiad Hóa học KEM
Alcohol (115)

Đến lúc này thì tôi đã thấy một phương pháp để tạo nên benzene từ
acetylene. Liệu bạn có thể tạo nên benzene từ alkane không?
Một phương pháp điều chế benzene từ nguyên liệu đầu hexane (C 6H14)
sử dụng hệ xúc tác chứa iridium (Ir) đã được báo cáo. Trong phản ứng
này, nhiều phân tử hydrogen đã bị tách loại khỏi hexane để tạo thành
benzene, nhưng một alkene (3,3-dimethyl-1-butene) đã được thêm vào
để đóng vai trò chất nhận hydrogen (hình 4).

Hình 4
Câu hỏi カ: Khi 1 phân tử hexane có thể tạo thành 1 phân tử thì có bao
nhiêu phân tử 3,3-dimethyl-1-butene đã phản ứng? (116)
Phản ứng này được cho là diễn ra qua các hợp chất trung gian A đến D
(mỗi trung gian này là hydrocarbon).

Hình 5 biểu diễn lượng hexane và sản phẩm quan sát thấy trong phản ứng
trên trục hoành (tỉ lệ lượng chất khi lấy lượng hexane ở thời điểm ban
đầu là 100) theo thời gian phản ứng - lấy từ trục tung. Các đồ thị (a)-(c) là
benzene hoặc trung gian A hoặc trung gian B, còn các đồ thị được biểu
diễn bởi các dấu ♦ và x lần lượt là hexane và các trung gian khác.

200 | Bản quyền thuộc về Tạp chí Olympiad Hóa học KEM
Hình 5
Chú thích: 各物質の物質量の相対値 = giá trị lượng chất tương đối (%)
của mỗi chất; 反応時間 (h) = thời gian phản ứng (giờ)
Câu hỏi キ: Hãy cho biết các đồ thị (a)-(c) tương ứng với những chất nào
trong số benzene, trung gian A và trung gian B. Chọn ra phương án phù
hợp dưới đây. (117)

Chú thích: ベンゼン = benzene; 中間体 A = trung gian A; 中間体 B =


trung gian B
Câu hỏi ク: Phản ứng nào sau đây tạo thành cấu trúc vòng? (118)
① từ hexane thành trung gian A
② từ trung gian A thành trung B
③ từ trung gian B thành trung C
④ từ trung gian C thành trung D

201 | Bản quyền thuộc về Tạp chí Olympiad Hóa học KEM
⑤ từ trung gian D thành trung benzene
Khi thay đổi chất đầu từ hexane thành octane (C 8H18) thì có 2 loại sản
phẩm được tạo thành như trong phương trình (1) và tỉ lệ sản phẩm là o-
xylene:ethylbenzene = 87:13.

Mặt khác, khi chất đầu là decane (C 10H22) thì có 3 loại sản phẩm được tạo
thành như trong phương trình (2) và tỉ lệ sản phẩm là 1-methyl-2-
propylbenzene: 1,2-diethylbenzene: butylbenzene = 86:10:4.

Dựa vào phản ứng của octane và decane trong phương trình (1) và (2) để
trả lời câu hỏi dưới đây.
Câu hỏi ケ:
(a) Nếu chất đầu là dodecane (C 12H26) thì có bao nhiêu loại sản phẩm
được tạo thành? (119)
(b) Xác định các nhóm thế (120) và (121) của cấu trúc được cho là sản
phẩm chính.

(c) Xác định các nhóm thế (120) và (121) của cấu trúc được cho là tạo
thành ít nhất?

202 | Bản quyền thuộc về Tạp chí Olympiad Hóa học KEM
Bài 2
Tác nhân Grignard là những hợp chất cơ magnesium với công thức tổng
quát RMgX, trong đó R là gốc hữu cơ và X là một halogen. Năm 1912, V.
Grignard đã dành giải Nob3el Hóa học bởi những khám phá về loại hợp
chất này. Chúng được tạo thành qua phản ứng:
R-X + Mg 𝑑𝑢𝑛𝑔 𝑚ô𝑖 → R-MgX
Trong đó, R kí hiệu cho các nhóm aryl, alkyl hoặc vinyl khác nhau.
1) Chọn (các) dung môi thích hợp để điều chế các tác nhân Grignard:
a) CH2Cl2 b) Diethyl ether c) nước d)
acetone
2) Những halogen X nào có thể được sử dụng để tổng hợp các tác nhân
Grignard qua phương trình phản ứng ở trên:
a) F b) Cl c) Br d) I
3) Các tác nhân Grignard tham gia vào rất nhiều loại phản ứng. Nhiều
trong số đó có ý nghĩa thực tiễn lớn. Dưới đây là một số phản ứng chưa
đầy đủ, có sự tham gia của các tác nhân Grignard. Hãy xác định công thức
cấu tạo của các sản phẩm. Chú ý rằng
A, B, C, D và E là các đồng phân.
▪ F và G là các đồng phân.
▪ I và J là các đồng phân.
▪ Các chất K và L có chứa 10 nguyên tử carbon.
▪ Công thức phân tử của M là C 18H18O3.
▪ Hợp chất thơm N có chứa một nhóm methyl và có công thức phân
tử C 9H9N.

203 | Bản quyền thuộc về Tạp chí Olympiad Hóa học KEM
204 | Bản quyền thuộc về Tạp chí Olympiad Hóa học KEM
Bài 3
Một trong những phản ứng hữu cơ quan trọng nhất, alkyl hóa nhân thơm
theo Friedel - Crafts, đã được khám phá vào năm 1877 và ngay lập tức
được ứng dụng thực tế, mặc dù vẫn còn những hạn chế.
1) Viết các sản phẩm của phản ứng (1) giữa C 6H6 và CH3Cl theo tỉ lệ tác
nhân ArH:RX = 4:1 (sản phẩm A) và 1:4 (phổ 1H NMR của sản phẩm
chính B chứa 2 tín hiệu với tỉ lệ cường độ 6:1).
Nếu đun nóng một hỗn hợp C 6H6 và CH3Cl, tỉ lệ mol 1:2, khi không có AlCl3
ở nhiệt độ vừa phải trong thời gian dài, thì tạo ra hỗn hợp cân bằng của
các đồng phân dimethylbenzene (xylene).
2) Tính thành phần của hỗn hợp ở 50 oC, sử dụng phương trình lnK = -
ΔGo/RT và dữ kiện đã cho. Xác định đồng phân bền nhiệt động nhất.
о-xylene m-xylene p-xylene
ΔfH, kJ/mol -24.4 -25.42 -24.34
S, kJ/(mol∙K) 246.0 252.2 247.4
Alkyl hóa benzene với alkyl chloride C1 (AlCl3, 50 oC) tạo thành sản phẩm
D1, còn sản phẩm alkyl hóa với đồng phân alkyl chloride C2 thì ngoài D1
còn tạo thành sản phẩm đồng phân D2.

3) Xác định các chất C1, C2, D1, D2, biết rằng M(D) : M(C 6H6) = 1.54.
4) Những nhược điểm nào của phản ứng alkyl hóa nhân thơm bởi Friedel
- Crafts có thể được rút ra dựa trên cơ sở những phản ứng đã thảo
luận?
Sự phát triển của lĩnh vực hóa học cơ kim đã mở ra những hướng khả thi
trong phản ứng alkyl hóa nhân thơm. Như alkylcyclohexane carboxylic
acid H đã được điều chế theo sơ đồ sau:

205 | Bản quyền thuộc về Tạp chí Olympiad Hóa học KEM
5) Xác định cấu tạo E - H, biết rằng với hợp chất F thì trong vùng không
thơm của phổ có 1 mũi đơn (9H), 2 mũi ba (2H và 3H), 2 mũi đa (2H
và 2H), còn trong vùng thơm thì có 1 mũi đơn, 2 mũi đôi rộng và mũi
đôi của mũi đôi (doublet of doublets).

206 | Bản quyền thuộc về Tạp chí Olympiad Hóa học KEM
Bài 4
Ester là những hợp chất hữu cơ gần gũi với chúng ta dưới dạng dung môi
hữu cơ trong thành phần các hương liệu trái cây. Cũng có nhiều phương
pháp khác nhau đã được biết để tổng hợp chúng, phụ thuộc vào mục đích
và ứng dụng. Phương pháp đơn giản nhất là thêm một lượng nhỏ xúc tác
acid như sulfuric acid vào nguyên liệu carboxylic acid và alcohol rồi đun
nóng (được gọi là “phương pháp tổng hợp ester Fisher”).

Trong phản ứng này, các phân tử nước được tách ra từ carboxylic và
alcohol để tạo thành ester, nhưng phản ứng không thể diễn ra hoàn toàn
theo chiều thuận (về bên phải). Và khi nồng độ của các chất trong hệ (gồm
cả chất đầu và sản phẩm) đạt tới một giá trị nhất định thì sẽ không thay
đổi nữa, và rõ ràng phản ứng không tiếp diễn. VÍ dụ, trong phản ứng trong
đó ethyl acetate được tạo thành từ acetic acid và ethanol như trên thì khi
trộn mỗi 1.00 mol nguyên liệu đầu rồi giữ nhiệt độ ở 76 oC thì khi 0.660
mol ethyl acetate được tạo thành, nồng độ của mỗi cấu tử trong hệ sẽ
không thay đổi.
Trạng thái mà tại đó phản ứng không tiếp diễn được gọi là “trạng thái cân
bằng”. Khái niệm cân bằng có ý nghĩa vô cùng quan trọng trong hóa học
và phương trình dưới đây được dùng để biểu diễn trạng thái cân bằng.
Trong đó, kí hiệu [ ] chỉ nồng độ của mỗi chất, và K là “hằng số cân bằng”.
Do hằng số cân bằng là hằng số của phản ứng ở nhiệt độ nhất định nên
nếu biết hằng số cân bằng thì có thể biết nồng độ của mỗi cấu tử ở nhiệt
độ đó.

1) Tính hằng số cân bằng K ở 76 oC của phản ứng trên (tới chữ số thập
phân thứ 2.)
Do đó, trong phản ứng ester hóa này, các ester không thể được tạo thành
hoàn toàn, và phải thực hiện các biến đổi để làm tăng lượng ester tạo
thành. Ví dụ, trong phản ứng trên, nếu một trong các nguyên liệu thô
được lấy dư thì phản ứng sẽ diễn ra theo chiều làm giảm tác động đó
(chiều thuận), do đó hiệu suất ester sẽ tăng lên. Theo đó, khi một yếu tố

207 | Bản quyền thuộc về Tạp chí Olympiad Hóa học KEM
trong hệ ở trạng thái cân bằng bị thay đổi thì phản ứng sẽ diễn ra theo
hướng chống lại tác động của biến đổi đó. Đây là nội dung chính của
“nguyên lí Le Chatelier”.
2) Nếu cho 1.00 mol acetic acid phản ứng với 3.00 mol ethenol trong
cùng điều kiện như trên thì có bao nhiêu mol acetate được tạo thành?
Chọn giá trị gần nhất trong các số dưới đây.
① 0.60 mol ② 0.70 mol ③ 0.80 mol
④ 0.90 mol ⑤ 1.00 mol
3) Dựa vào nguyên lí Le Chatelier, hãy giải thích cân bằng sẽ thay đổi thế
nào khi nước bị loại khỏi phản ứng bằng cách thêm tác nhân hút nước
vào hỗn hợp phản ứng ester hóa.
Trong trường hợp ester hóa một carboxylic acid có cấu trúc phức tạp,
những vẫn đề như các phản ứng khác ngoài phản ứng ester hóa có thể
diễn ra trong điều kiện phản ứng trên (đun nóng khi có mặt xúc tác acid).
Do đó, một phương pháp đã được phát triển để chuyển carboxylic acid
thành một trạng thái hoạt động hơn (khả năng phản ứng cao hơn) sau đó
cho phản ứng với alcohol để thực hiện phản ứng ester hóa ở nhiệt độ gần
nhiệt độ phòng. Ví dụ, khi cho thionyl chloride (SOCl 2) phản ứng với acetic
acid, phản ứng sau diễn ra tạo thành acetyl chloride (CH 3COCl), trong đó
nhóm hydroxyl của acetic acid đã bị thay thế bởi chlorine. Phản ứng của
acetyl chloride với alcohol khi có mặt trimethylamine ((CH3CH2)3N) nhanh
chóng tạo thành ester ở nhiệt độ phòng.

4) Giải thích vai trò của triethylamine trong phản ứng này.
Trong tất cả các phản ứng ester hóa đã được mô tả cho đến lúc này, nhóm
hydroxyl trên carboxylic acid và hydrogen của alcohol đã bị tách ra và
phần còn lại liên kết với nhau (phản ứng ngưng tụ tách nước). Mặt khác,
một phản ứng ester hóa với ý tưởng hoàn toàn khác đã được phát triển.
Đó là dùng phản ứng với một tác nhân oxide và hóa một tác nhân khử

208 | Bản quyền thuộc về Tạp chí Olympiad Hóa học KEM
trên carboxylic acid và alcohol, mỗi tác nhân nhận 2 nguyên tử hydrogen
và 1 nguyên tử oxygen, và dẫn đến phản ứng ngưng tụ tách nước.
Có thể thấy từ ví dụ được đề cập ở trên, trong một phản ứng tách nước
thông thường, những sáng kiến - như thay đổi điều kiện phản ứng khắc
nghiệt hoặc chuyển thành dẫn xuất hoạt tính cao - là cần thiết, thì trong
phương pháp này, tự thân phản ứng là quá trình kết hợp oxid hóa-khử,
và có thể tạo thành ester dưới những điều kiện hết sức êm dịu. Phản ứng
này là một phương pháp ester hóa được giáo sư Mitsunobu Ooyu ở ĐH
Aoyama Gakuin phát triển, gọi là “phản ứng Mitsunobu”, và nó được sử
dụng bởi các nhà hóa học tổng hợp hữu cơ từ khắp nơi trên thế giới.

Chú thích:
トリフェニルホスフィン = Triphenylphosphine;
トリフェニルホスフィン オキシド = Triphenylphosphine oxide;
ジエチル アゾジカルボキシラート = Diethyl azodicarboxylate;
ジエチル ヒドラジンジカルボキシラート =
Diethylhydrazinedicarboxylate
5) Trong phương trình phản ứng, hãy chỉ ra những nguyên tử bị oxid hóa
và những nguyên tử bị khử.
Bây giờ, hãy nghiên cứu về hóa học thể của các hợp chất hữu cơ một chút.
Khi 1 nguyên tử carbon tạo thành liên kết đơn với 4 nguyên tử khác, nó
có thể tạo thành cấu trúc ba chiều, trong đó nguyên tử carbon được đặt
ở tâm của một tứ diện và các (nhóm) nguyên tử khác được đặt ở 4 đỉnh

209 | Bản quyền thuộc về Tạp chí Olympiad Hóa học KEM
xung quanh. Hãy xét hợp chất C ABDE (trong đó A, B, D, E là chỉ các (nhóm)
nguyên tử khác nhau chứ không phải kí hiệu của bất kì nguyên tố cụ thể
nào). Trong cách biểu diễn cấu trúc này, khi 2 liên kết C-A và C-B được đặt
lên cùng một mặt phẳng (trên giấy), liên kết C-D [nét đậm] là liên kết
hướng lên trên, còn liên kết C-E [nét đứt] là liên kết hướng xuống dưới.
Theo đó, nguyên tử carbon liên kết với 4 (nhóm) nguyên tử khác nhau là
“carbon bất đối” và tạo ra 2 đồng phân quang học. Chúng có mối liên hệ
“ảnh thật” và “ảnh gương” với nhau, không thể chồng khít lên nhau và gọi
là các “đồng phân ảnh gương” (đồng phân quang học).

Chú thích: 鏡像異性体同士 = Đối quang


Như biểu diễn trong hình, A- đến gần mặt dưới của tứ diện C ABDE (1) tạo
thành một liên kết mới với carbon trung tâm và đồng thời. liên kết C-A bị
phân cắt. Hợp chất (2) tạo thành là một đối quang của hợp chất (1). Điều
này sẽ được thấy rõ khi thử lật ngược cấu trúc ba chiều của (2). Khi liên
kết C-A mới được tạo thành thì 3 liên kết còn lại bị nghịch đảo (“sự nghịch
đảo cấu hình”), như khi một chiếc ô lật ngược trong gió mạnh. Thực tế,
trong phản ứng Mitsunobu, khi một alcohol bậc 2, trong đó nhóm
hydroxyl liên kết với một carbon bất đối được sử dụng, thì sẽ quan sát
được hiện tượng sự sắp xếp của các nhóm thế quanh carbon bất đối bị
nghịch đảo khi ester được tạo thành. Mối quan hệ đặc biệt giữa alcohol
ban đầu và sản phẩm ester cũng là nguyên chính chính tại sao phản ứng
Mitsunobu rất hữu ích và được sử dụng rộng rãi.
6) Đồng phân lập thể ester nào được tạo thành bởi phản ứng giữa
alcohol bậc hai có nguyên tử carbon bất đối như hình dưới với acetic
acid trong điều kiện phản ứng Mitsunobu? Xác định cấu trúc ba chiều
của sản phẩm.

210 | Bản quyền thuộc về Tạp chí Olympiad Hóa học KEM
Chú thích: トリフェニルホスフィン= Triphenyl phosphine; ジエチル
アゾジカルボキシラート = diethyl azodicarboxylate
Cuối cùng, hãy xét đến quá trình phân cắt và tạo thành liên kết của các
phân tử trong phản ứng Mitsunobu. Đặc trưng nghịch đảo hóa lập thể của
phản ứng Mitsunobu thực sự đã đưa ra một gợi ý lớn (thậm chí là bằng
chứng) khi xem xét liên kết nào bị phá vỡ, và liên kết nào được tạo thành.
Hãy thử sức với câu hỏi cuối.
7) Trong phản ứng Mitsunobu dưới đây, nghiên tử oxygen nào bị được
coi là bị loại bỏ? Hãy khoanh tròn nguyên tử đó. Giải thích ngắn gọn.

211 | Bản quyền thuộc về Tạp chí Olympiad Hóa học KEM
Bài 5
Bromine phản ứng với buta-1,3-diene trong dung dịch cyclohexane, thu
được hai sản phẩm A và B (không xét đến hóa lập thể). Tỉ lệ của A và B khi
tiến hành phản ứng ở nhiệt độ -15 oC là 62:38, trong khi đó ở 25 oC thì tỉ
lệ A và B là 12:88.
1) Xác định công thức các hợp chất A, B và xác định đâu là sản phẩm bền
nhiệt động, đâu là sản phẩm động học.
2) Ở nhiệt độ phòng, A bị chuyển hóa chậm thành B. Vẽ giản đồ thế năng
của chuyển hóa này và dự đoán cấu trúc của tiểu phân trung gian.
3) Dựa vào các kết quả ở trên, hãy dự đoán các sản phẩm của phản ứng
sau:

212 | Bản quyền thuộc về Tạp chí Olympiad Hóa học KEM
Bài 6
Đọc và trả lời các câu hỏi từ (110)-(133).
Trong lĩnh vực “khám phá thuốc” để chế tạo dược phẩm thì hóa học có
vai trò rất to lớn.
Trước tiên, chúng ta sẽ mô tả một loại thuốc có vòng benzene trong cấu
trúc. Aspirin (acetylsalicylic acid) là loại thuốc được biết đến rộng rãi như
là tác nhân hạ sốt, thuốc giảm nhức đầu và đau răng. Vào thời cổ đại,
người Hi Lạp và Trung Quốc đã biết chữa đau răng bằng cách sử dụng
cành liễu và đến năm 1827, một hợp chất có tên salicin (a) đã được chiết
xuất từ nguồn gốc thực vật này. Salicin bị thủy phân tạo thành salicyl
alcohol và glucose. Salicyl alcohol được oxid hóa để tổng hợp salicylic acid
(b), và trong thập niên 1870, nó bắt đầu được sử dụng làm thuốc giảm
đau chống thấp khớp, tuy nhiên những tác dụng phụ gây ra rối loạn tiêu
hóa mạnh trở thành một vấn đề nghiêm trọng. F. Hoffmann, một nhà hóa
học ở thời kì này, nghĩ rằng tính acid mạnh của salicylic acid là nguyên
nhân gây ra tổn thương dạ dày nên đã tiến hành acetyl hóa nhóm
hydroxyl để tổng hợp acetylsalicylic acid, giúp giảm thiểu các tác dụng
phụ. Năm 1899, thuốc aspirin đã được giới thiệu.
Kể từ đó, aspirin đã được sản xuất bằng con đường tổng hợp hóa học.
Trong điều kiện nhiệt độ và áp suất cao, phenol (c) phản ứng với sodium
hydroxide và (113) để đưa nhóm carboxy vào vị trí ortho rồi trung hòa
bằng sulfuric acid thì thu được salicylic acid. Quá trình acetyl hóa được
thực hiện bởi phản ứng giữa salicylic acid với acetic anhydride, thu được
aspirin. Ngoài aspirin còn thu được ibuprofen và indomethacin, được biết
đến như những tác nhân giảm đau có khả năng ức chế viêm, và cấu trúc
của chúng cũng có chứa các vòng benzene (hình 1).

Hình 1
Chú thích: アスピリン = Aspirin; イブプロフェン = Ibuprofen; インド
メタシン = Indomethacin

213 | Bản quyền thuộc về Tạp chí Olympiad Hóa học KEM
Câu hỏi: Không có kết tủa copper(I) oxide xuất hiện khi cho “salicin” (phần
gạch chân a) vào dung dịch thuốc thử Fehling đun nóng. Xác định cấu trúc
của salicin: (110)

Câu hỏi: Các hợp chất được tạo thành bởi phản ứng của “salicylic acid”
(phần gạch chân b) với sodium hydrogencarbonate được biết đến là có
tác động chống viêm. Hãy chọn phương trình phản ứng phù hợp: (111)

Câu hỏi: Phenol (phần gạch chân c) được sản xuất trong công nghiệp theo
phương pháp cumene. Chọn ra sự kết hợp các nguyên liệu thô phù hợp
nhất cần cho quá trình tổng hợp cumene và cho biết kiểu phản ứng. (112)
Nguyên liệu: a) benzene; b) propane; c) propene; d) acetone
Kiểu phản ứng: a) ngưng tụ; b) phản ứng cộng; c) dehydrogen hóa
Câu hỏi: Chọn tác nhân phù hợp với (113)
① oxygen ② hydrogen ③ carbon monoxide ④ carbon dioxide ⑤
acetic acid ⑥ acetic anhydride ⑦ ethanol ⑧ acetaldehyde
Acetaminophen là thuốc giảm đau hạ sốt nhẹ. Nó thường được sử dụng
ở dạng kết hợp, gọi là “công thức ACE” trong đó (114) và ethenzamide
được thêm vào acetaminophen. (114) đóng vai trò trợ lực cho hiệu quả
của thuốc giảm đau. Acetaminophen được tổng hợp từ phenol. Nitro hóa

214 | Bản quyền thuộc về Tạp chí Olympiad Hóa học KEM
phenol, rồi khử 4-nitrophenol (d) thu được 4-aminophenol, sau đó acetyl
hóa với acetic anhydride (e) thu được acetaminophen. Một phương pháp
tổng hợp khác để sản xuất công nghiệp sử dụng chuyển vị Beckmann (f)
cũng đã được phát triển.

Hình 2
Chú thích: アセトアミノフェ = Acetaminophen; エテンザミド =
Ethenzamide
Câu hỏi: (114) là một loại alkaloid có trong café, trà xanh, trà đen, … Xác
định tên gọi của hợp chất này.
① catechin ② catechol ③ caffeine ④ capsaicin ⑤ chitin ⑥ chitosan
⑦ glucosamine ⑧ collagen ⑨ cholesterol
Câu hỏi: Các nghiên cứu gần đây đã làm sáng tỏ rằng (114) được sinh tổng
hợp từ xanthosine trong thực vật. Xanthosine là nucleoside tạo thành từ
ribose và xanthine, cũng là purine base như adenine và guanine. Xác định
cấu trúc xanthosine. (115)

Hình 3
Câu hỏi: Biết phản ứng trong phần gạch chân d được tiến hành với
hydrogen H2 sử dụng nickel làm xúc tác, hãy xác định bao nhiêu mol H2 sẽ
phản ứng với 1.0 mol 4-nitrophenol. (116)

215 | Bản quyền thuộc về Tạp chí Olympiad Hóa học KEM
① 0.50 mol ② 1.0 mol ③ 1.5 mol ④ 2.0 mol ⑤ 2.5 mol ⑥ 3.0
mol ⑦ 3.5 mol ⑧ 4.0 mol
Kết quả phân tích nguyên tố sản phẩm phụ sinh ra bởi acetaminophen
trong phản ứng ở phần gạch chân e cho thấy có 62 % carbon và 5.7 %
hydrogen. Ngay cả khi thêm bột tẩy trắng vào cũng không có phản ứng
màu xảy ra. Xác định cấu trúc phù hợp của sản phẩm phụ này (117).

Hình 4
Phản ứng chuyển vị Beckmann trong phần gạch chân f là một phản ứng
để điều chế amide từ ketone oxim và cũng được sử dụng trong tổng hợp
ε-caprolactam là nguyên liệu của nylon 6 như trong phản ứng (a) dưới đây
(ベックマン転位 = chuyển vị Beckmann)

Câu hỏi: Xác định nguyên liệu đầu để tổng hợp acetominophen bằng
chuyển vị Beckmann. (118)

Hình 5
Tiếp theo, hãy xem xét chiến lược tổng hợp các hợp chất trong khám phá
thuốc. Ở giai đoạn thiêst kế và tổng hợp các hợp chất tiềm năng cho
những loại thuốc mới, việc phân tích tổng hợp ngược - bằng cách đảo
ngược quy trình phản ứng để tổng hợp hợp chất mục tiêu - là rất quan
trọng để xác định nguyên liệu đầu và các phương pháp tổng hợp. Xét một
phân tích tổng hợp ngược từ hai góc nhìn: (1) chuyển hóa, đưa vào và loại
bỏ nhóm chức; và (2) tạo liên kết carbon-carbon.

216 | Bản quyền thuộc về Tạp chí Olympiad Hóa học KEM
a) Chuyển hóa, đưa vào và loại bỏ nhóm chức
Khi tổng hợp nitrobenzene, có thể tiến hành phản ứng giữa benzene với
hỗn hợp sulfuric acid và nitric acid để nitro hóa. Tuy nhiên, khi tổng hợp
aniline, rất khó để trực tiếp đưa nhóm amino vào vòng benzene, do đó
nó được tổng hợp qua nhiều bước: nitro hóa, sau đó khử hóa.
Trong trường hợp đưa một nhóm thế thứ hai vào benzene một lần thế thì
thế thì nếu nhóm thế thứ nhất là nhóm đẩy (có tính chất nhường electron)
thì nó thường định hướng ortho-para, còn nếu nhóm thế thứ hai là nhóm
hút (có tính chất nhận electron) thì nó định hướng meta. Ví dụ, trong
trường hợp tổng hợp 2,4,6-trinotrobenzoic acid, thay vì nitro hóa trực
tiếp nitrobenzoic acid thì có thể tạo ra sản phẩm cuối bằng cách oxid hóa
nhóm methyl sau khi nitro hóa toluene.
Câu hỏi: Xác định sản phẩm chính A được tạo thành trong chuỗi chuyển
hóa sau: (119)

Hình 6
b) Tạo liên kết carbon-carbon
Các hợp chất thú vị trong khám phá thuốc thường là những phân tử lớn.
Trong trường hợp này, có thể tổng hợp các phân tử lớn bằng cách gắn kết
các phân tử nhỏ bởi các phản ứng tạo liên kết carbon-carbon. Lần này,
chúng ta sẽ giới thiệu 2 phản ứng tập trung vào các nhóm carbonyl.
Trước tiên là phản ứng của hợp chất carbonyl với phosphonium ylide (sản
phẩm dehydrohalogen hóa của muối phosphonium tạo thành từ phản
ứng của triphenylphosphine (C 6H5)3P và alkyl halide với base mạnh, có
nguyên tử phosphorus mang điện tích dương và nguyên tử carbon liên
kết trực tiếp mang điện tích âm). Đây là phản ứng Wittig, trong đó liên
kết C=O bị chuyển thành liên kết C=C.

217 | Bản quyền thuộc về Tạp chí Olympiad Hóa học KEM
Trong phản ứng Wittig, nguyên tử carbon mang điện tích âm của
phosphonium ylide (hình 7a) cộng hợp vào carbon của nhóm carbonyl, và
nguyên tử oxygen mang điện tích âm (của nhóm carbonyl) tạo thành từ
sự cộng hợp này tấn công vào nguyên tử phosphorus tạo thành tiểu phân
trung gian vòng 4 cạnh (hình 7b) kém bền. Sự phân hủy tiểu phân trung
gian này tạo thành alkene.
Trong phản ứng này, khi có một nhóm hút electron như nhóm carbonyl
liên kết với nguyên tử carbon mang điện tích âm của phosphonium ylide
thì điện tích âm của nguyên tử carbon được giải tỏa (bền hóa), do đó nó
được gọi là một ylide bền, còn ngược lại thì gọi là ylide kém bền. Tùy thuộc
vào ylide bền hay kém bền mà sản phẩm chính là alkene dạng cis hay
trans.

Hình 7
Phản ứng Wittig chọn lọc đồng phân trans bởi ylide bền diễn ra kém bởi
tính bền của nó. Kết quả của nhiều nghiên cứu nhằm để cải thiện là phản
ứng Horner-Wadsworth-Emmons sử dụng phosphate ester thay vì muối
phosphonium, và nó được sử dụng rộng rãi để tổng hợp các trans alkene.
Trong phản ứng này, một ester không no α, β (ester trong đó liên kết đôi
C=C liên hợp với liên kết đôi C=O) đã được tổng hợp (hình 8).

Hình 8
Thứ hai là phản ứng Michael, là phản ứng cộng liên hợp carbanion vào
hợp chất carbonyl không no α, β. Ví dụ, xét phản ứng của ethyl
acetoacetate với 3-buten-2-one. Ethyl acetoacetate bị deproton hóa khi

218 | Bản quyền thuộc về Tạp chí Olympiad Hóa học KEM
có mặt base tạo thành carbanion có tính nucleophile được bền hóa bởi
một nhóm carbonyl hút electron (hình 9).

Hình 9
Mặt khác, β carbon (nguyên tử carbon thứ hai so với nhóm carbonyl) của
3-butene-2-one có điện tích dương riêng phần bởi nhóm carbonyl và liên
kết đôi C=C liên hợp tạo thành các cấu trúc cộng hưởng như sau (hình
10):

Hình 10
Nguyên tử carbon mang điện tích âm của carbanion tạo thành từ ethyl
acetoacetat được cộng hợp vào nguyên tử β carbon dương điện của 3-
buten-2-one và anion mới tạo thành có điện tích âm trên nguyên tử
oxygen. Anion này được bền hóa cộng hưởng như trong hình 11 và sau
khi tước proton từ dung môi và ethyl acetoacetate, thì sản phẩm cuối
được tạo thành. Phản ứng này được phát triển bởi nhiều nhà nghiên cứu
và đã được mở rộng cho các phản ứng trong đó tác nhân nucleophile
được cộng hợp vào liên kết không no thiếu electron có các nhóm thế hút
electron khác ngoài carbonyl.

Hình 11

219 | Bản quyền thuộc về Tạp chí Olympiad Hóa học KEM
Chú thích: 3-ブテン-2-オン = 3-butene-2-one; アセト酢酸エチル =
Ethyl acetoacetate
Câu hỏi: Để tổng hợp β-carotene cần sử dụng β-ionylideneacetaldehyde
và một hợp chất khác. Xác định hợp chất còn lại này. (120)

Hình 12a
Chú thích: β-カロテン = β-carotene; β-イオニリデンアセトアルデヒ
ド = β-ionylideneacetaldehyde

Hình 12b
Câu hỏi: Bombycol, một pheromone của loài tằm, có thể được tổng hợp
bởi 2 phản ứng Wittig, sau đó khử hóa ester bởi tác nhân khử mạnh để
tạo thành alcohol bậc một. Xác định cấu trúc của bombycol. (121)

220 | Bản quyền thuộc về Tạp chí Olympiad Hóa học KEM
Hình 13
Chú thích: 還 元剤 = tác nhân khử; ボンビコール = Bombycol
Câu hỏi: Xác định sản phẩm của chuỗi phản ứng sau. (122)

Hình 14
Trong phần cuối của chiến lược tổng hợp, chúng ta sẽ xem xét cách thức
các chiến lược tổng hợp trong khám phá thuốc đã được mô tả được dùng
trong tổng hợp thuốc chữa cúm Tamiflu.
Tamiflu là thuốc chữa cúm có chứa thành phần hoạt tính là oseltamivir
phosphate, được sử dụng trong điều trị nhiễm virus cúm A và B vào năm

221 | Bản quyền thuộc về Tạp chí Olympiad Hóa học KEM
2001. Ban đầu, các nguyên liệu của phản ứng tổng hợp chỉ được lấy từ
các sản phẩm nguồn gốc thiên nhiên, dẫn đến một vấn đề về sự ổn định
của nguồn cung. Nhiều nhà nghiên cứu đã nỗ lực thực hiện các tổng hợp
toàn phần không cần đến sự trợ giúp của các quá trình sinh học, và nhiều
quy trình tổng hợp toàn phần đã được công bố từ năm 2006, và một quy
trình được công bố vào năm 2009 là tổng hợp toàn phần của Yujiro
Hayashi và cộng sự (hình 15).
Trong quy trình này, các nguyên liệu đầu B và C tham gia vào phản ứng
cộng Michael, rồi phản ứng tiếp với hợp chất D theo phản ứng (1) cộng
Michael; (2) Horner-Wadsworth-Emmons tạo thành hợp chất E. Hợp chất
mục tiêu oseltamivir được tổng hợp qua 7 giai đoạn như là đưa vào nhóm
bảo vệ, chuyển hóa nhóm chức và loại nhóm bảo vệ.

Hình 15
Câu hỏi: Xác định các chất đầu B, C. (123)

Hình 16

222 | Bản quyền thuộc về Tạp chí Olympiad Hóa học KEM
Độ tan trong nước của các hợp chất có phân tử khối thấp có thể ảnh
hưởng đến hiệu quả dược và độ an toàn của thuốc. Độ tan trong nước
không chỉ bị ảnh hưởng bởi ái lực với nước, là dung môi của hợp chất, mà
còn bởi tính dễ vỡ. Do đó, cũng có một số nghiên cứu để thay đổi tính dễ
vỡ nhằm làm tăng độ tan trong nước.
Các tinh thể có xu hướng sụp đổ khi các phân tử cấu tạo thành trở thành
không phẳng hoặc bất đối xứng. Ví dụ, trong hợp chất F như trong hình
17, một nhóm hydroxy gắn với vòng benzene, nhưng trong hợp chất G thì
vị trí của nhóm hydroxy đã thay đổi, do đó (124) diễn ra, tinh thể có khả
năng sụp đổ. Ngoài ra, khi một nhóm chức được đưa vào vị trí ortho của
nhóm phenyl của hợp chất G thì (125) xuất hiện trong tương quan về vị
trí của hai vòng benzene do tương tác với nhóm thế được đưa vào. Chính
vì vậy tinh thể nhiều khả năng sẽ sụp đổ. Tinh thể càng dễ sụp đổ thì càng
dễ tan trong nước. Tính dễ vỡ cũng ảnh hưởng đến điểm nóng chảy. Ví
dụ, điểm nóng chảy của các hợp chất F và G trong hình 17 thì (126).

Hình 17
Câu hỏi: Xác định các hiện tượng (124), (125).
① Giải tỏa ② Bất phẳng hóa ③ Bất đối xứng hóa ④ Không chia sẻ
⑤ Không phân tử hóa
Câu hỏi: Xác định cụm từ thích hợp cho (126).
① F có điểm nóng chảy cao hơn. ② G có điểm nóng chảy cao hơn. ③
Điểm nóng chảy của F và G bằng nhau. ④ Không thể đánh giá chất nào
có điểm nóng chảy cao hơn.
Tiếp theo, hãy xét các nguyên tử carbon của methane, ethylene và
acetylene. Về mặt cấu trúc, tất cả các liên kết từ các nguyên tử carbon
trong phân tử ethylene và acetylene đều thuộc cùng một mặt phẳng. Tuy
nhiên, với các nguyên tử carbon của methane, các liên kết (127). Nói cách
khác, có thể thấy rằng sự tồn tại của các nguyên tử carbon lai hóa sp3 như

223 | Bản quyền thuộc về Tạp chí Olympiad Hóa học KEM
carbon trong methane là một chỉ số khi xét đến tính phẳng của phân tử.
Gọi tỉ lệ số nguyên tử carbon sp3 trên tổng số carbon trong phân tử là Fsp3
thì Fsp3 của dimethylpiperidine trong hình 18 là 1.0, trong khi đó Fsp3 của
dimethylpyridine là (128). Do đó dimethylpyridine có tính phẳng cao hơn.

Hình 18
Câu hỏi: Xác định cụm từ phù hợp cho (127)
① Giải tỏa ② Bất phẳng hóa ③ Bất đối xứng hóa ④ Không chia sẻ
⑤ Không phân tử hóa
Câu hỏi: Xác định giá trị (128).
① 1.0 ② 0.34 ③ 0.29 ④ 0.18 ⑤ 0.060
Cuối cùng, chúng tôi giới thiệu các ví dụ về việc các tác dụng phụ đã được
cải thiện bằng cách thiết kế các phân tử thuốc có xem xét đến độ tan.
Viêm mũi và ngứa dị ứng gây ra bởi sự giải phóng quá mức histamine vào
máu do kích thích của các chất dị ứng xâm nhập vào cơ thể, được kích
hoạt bằng cách tạo liên kết với một protein gọi là thụ thể histamine H1.
Thụ thể histamine H1 được phân bố rộng trong cơ thể người và khởi phát
nhiều hiện tượng sinh lí liên quan đến kích thích và tăng cường trí nhớ
trong não.

Hình 19: Các kháng sinh histamine thế hệ đầu tiên.


Các chất trong hình 19 được gọi là các kháng histamine thế hệ đầu tiên
và có (129) tương tự như histamine. Thụ thể histamine H1 liên kết với
chúng thay vì histamine và vô hoạt nó để giảm bớt các chứng dị ứng. Mặt
khác, chúng có tác dụng phụ là gây buồn ngủ mạnh. Điều này là do hợp

224 | Bản quyền thuộc về Tạp chí Olympiad Hóa học KEM
chất này là một chất có (130) với 1 vòng benzene hoặc 1 mạch carbon, nó
đi qua hàng rào máu não và cũng tác động lên thụ thể histamine H1 trong
não. Hàng rào máu não đóng vai trò như một bộ lọc để chọn lọc các chất
đi vào não và có xu hướng cho các chất có khối lượng phân tử thấp như
ethanol, caffeine hoặc có xu hướng đi qua các vật liệu (130) tốt.
Do đó, thế hệ kháng histamine thứ hai (hình 20) đã được phát triển. Đây
là những cấu trúc tương tự như kháng histamine thế hệ đầu tiên, nhóm
chức có (132) như (131) đã được đưa vào, và như dự kiến, sự dịch chuyển
đến não đã ít đi và giảm các tác dụng phụ.

Hình 20: Các kháng histamine thế hệ thứ hai.


Câu hỏi: Xác định các cụm từ thích hợp cho (129)-(132).
① nhóm amino ② nhóm carboxy ③ nhóm hydroxy ④ cầu ether ⑤
nhóm phenyl ⑥ tính ưa nước ⑦ tính kị nước ⑧ hoạt động bề mặt ⑨
sự ăn mòn
Câu hỏi: Histamine không chỉ được hấp thụ từ thực phẩm mà còn bởi
amino acid histidine trong cơ thể chuyển hóa thành bởi decarboxylase.
Xác định cấu trúc của histidine: (133)

Hình 21

225 | Bản quyền thuộc về Tạp chí Olympiad Hóa học KEM
Bài 7
Tuỳ thuộc vào điều khử hoá naphtalen bằng phản ứng Birch có thể thu
được các sản phẩm khác nhau:
a. Khi dùng natri hoặc liti trong NH3 lỏng và đietyl ete ở -780C thu được
98% hợp chất A, 1% hợp chất B và vết của C.
b. Nếu dùng dư Na hoặc Li, sau đó thêm FeCl 3 và nâng nhiệt độ lên -330C
thì hầu như chỉ thu được B.
c. Nếu dùng dư Na hoặc Li không có FeCl 3 và nâng nhiệt độ lên -330C thì
hầu như chỉ thu được C.
d. Khi đun nóng với natri etylat trong etanol A đồng phân hoá thành B.
e. Khử naphtalen hoặc khử chất C bằng Li trong etylamin thu được hỗn
hợp D và E là đồng phân của nhau.
f. Nếu thêm Na vào naphtalen trong amoniac-ete-etanol thu được F là
đồng phân của C.
g. Nếu đun nóng A hoặc B hoặc C với KMnO4/axit đều tạo ra cùng một loại
axit thơm. Oxi hoá D trong điều kiện này tạo ra đixeton vòng G, còn khi
oxi hoá E sinh ra xetoaxit H.
Viết công thức cấu trúc của các chất A, B, C, D, E, F, G, H.

226 | Bản quyền thuộc về Tạp chí Olympiad Hóa học KEM
Bài 8
a) Xác định D, E trong phản ứng dưới đây.
b) Phản ứng này thuộc loại nào?
c) Giải thích tại sao D có hàm lượng cao hơn E.
d) vai trò của dioxan và cho biết.

227 | Bản quyền thuộc về Tạp chí Olympiad Hóa học KEM
Bài 9
Coniine là một alkaloid độc, có thể làm tê liệt hệ hô hấp dẫn tới tử vong.
Chỉ một lượng nhỏ hơn 0.1 gam cũng đủ gây chết người. Năm 399 trước
Công nguyên, Socrates đã bị đầu độc bởi một hỗn hợp chứa coniine. Năm
1881, Hofmann xác định được công thức phân tử coniine là C 8H17N. Khi
methyl hóa hoàn toàn coniine, sản phẩm của phản ứng Hofmann là (4S)-
N,N-dimethyloct-7-en-4-amine.
1) Xác định cấu trúc của (4S)-N,N-dimethyloct-7-en-4-amine và đối
quang của nó.
2) Từ kết quả phản ứng tách Hofmann có thể biết được những đặc trưng
cấu trúc nào của coniine? Dự đoán các cấu trúc có thể có của coniine.

228 | Bản quyền thuộc về Tạp chí Olympiad Hóa học KEM
Bài 10
Tropinone thuộc nhóm alkaloid và là sản phẩm trung gian trong tổng hợp
atropine sulfate. Quá trình tổng hợp chất này là một cột mốc quan trọng
trong lịch sử tổng hợp hữu cơ, mở ra hướng nghiên cứu về phản ứng đa
thành phần. Nhiều dẫn xuất của tropinone có hoạt tính sinh lí tốt nên có
rất nhiều phản ứng tạo dẫn xuất đã được tiến hành. Tiến hành phản ứng
giữa tropinone và benzyl bromide:

Trong môi trường kiềm, các sản phẩm A và B của phản ứng này có thể
chuyển hóa qua lại lẫn nhau. Do đó, chất A hoặc B tinh khiết sẽ trở thành
hỗn hợp của A và B trong kiềm.
Xác định cấu trúc của A, B và các tiểu phân trung gian của mỗi chất trong
chuyển hóa qua lại giữa chúng.

229 | Bản quyền thuộc về Tạp chí Olympiad Hóa học KEM
Bài 11
Viết công thức của sản phẩm tạo thành từ các phản ứng sau:
a. O
b.
OH- O
CHCl2 ? 1. C2H5MgBr (d- )
C=O ?
2. H3O+
O

c. Pent-1-en + NBS, ánh sáng.


d. 1-Brommetyl-2-metylxiclopenten đun nóng trong ancol metylic.

230 | Bản quyền thuộc về Tạp chí Olympiad Hóa học KEM
Bài 12
Có nhiều yếu tố ảnh hưởng đến phản ứng hữu cơ. Ví dụ, các nhóm thế
khác nhau trong chất nền phản ứng sẽ có xu hướng tạo thành các sản
phẩm khác nhau.

1) Khi R = CH3, sản phẩm là A, công thức phân tử C15H12O4. Các nghiên
cứu cho thấy rằng A không chứa nhóm hydroxyl, và phổ 1H NMR
(CDCl3, ppm): δ 1.68 (3H), 2.73-2.88 (2H), 5.57 (1H), 7.72-8.10 (4H).
Gợi ý: các giá trị δ khác nhau tương ứng với các loại hydrogen khác
nhau. Vẽ cấu trúc chất A.
2) Khi R = Ph, sản phẩm là B, công thức phân tử là C 20H14O4. Các nghiên
cứu cho thấy rằng A chứa 1 nhóm hydroxyl, và phổ 1H NMR (CDCl3,
ppm): δ 2.16 (1H), 3.79 (1H), 4.07 (1H), 5.87 (1H), 6.68 (1H), 7.41-7.77
(5H) , 7.82-8.13 (4H). Vẽ cấu trúc chất B và giải thích tại sao B được
tạo thành.
3) Khi R = OEt, sản phẩm là C, công thức phân tử là C 14H10O5. Vẽ cấu
trúc chất C.

231 | Bản quyền thuộc về Tạp chí Olympiad Hóa học KEM
Bài 13
Ngày nay, tổng hợp xanh là mục tiêu của các nhà hóa học hữu cơ, và việc
thay thế các chất oxid hóa chứa kim loại bằng các hợp chất hữu cơ là một
trong những hướng nghiên cứu rất quan trọng.
Nitromethane là một chất oxid hóa hữu cơ êm dịu. Vẽ các cấu trúc cộng
hưởng của nitromethane anion. Xác định tiểu phân trung gian và sản
phẩm của phản ứng sau:

232 | Bản quyền thuộc về Tạp chí Olympiad Hóa học KEM
Bài 14
Phản ứng hydroxy thuỷ ngân hoá cũng có cơ chế tương tự như phản ứng
cộng bromine vào alkene. Dưới đây là hai ví dụ về phản ứng này:

Mặt khác, trong cùng một điều kiện như trên thì hợp chất pent-4-ene-1-
ol tạo thành sản phẩm chính B. Hãy vẽ cấu trúc của các hợp chất A và B.

233 | Bản quyền thuộc về Tạp chí Olympiad Hóa học KEM
Bài 15
1) Một sinh viên tiến hành phản ứng sau với hy vọng có thể bảo vệ được
nhóm hydroxy và amino

Cho biết điều kiện nào phù hợp nhất để tiến hành phản ứng:
a) Phản ứng được tiến hành trong hỗn hợp HCl:EtOH = 1:3.
b) Phản ứng được tiến hành với lượng dư Et3N.
a) Sử dụng lượng nhỏ BF3 và tiến hành phản ứng trong ether khan.
b) Đun hồi lưu hỗn hợp phản ứng trong toluene.
2) Phản ứng giữa aminoethanol và acetic anhydride xảy ra theo sơ đồ
sau:

Khi tiến hành phản ứng trong K2CO3 thu được một sản phẩm không vòng
B. A cũng có thể chuyển thành B dưới tác dụng của K2CO3
a) Xác định B.
b) Cho biết vai trò của HCl trong sự tạo thành A và K2CO3 với sự tạo
thành B.
c) Vì sao A chuyển hóa thành B dưới tác dụng của K2CO3.

234 | Bản quyền thuộc về Tạp chí Olympiad Hóa học KEM
Bài 16
Hoàn thành các phản ứng sau (chỉ viết các sản phẩm chính):
a.
KF/Al2O3
O + CHO A
vi sóng , siªu ©m

b.

N
CH2Cl + B
N

c. trans-2-Bromometylxiclohexan + CH3ONa C
Biết rằng A chứa hệ liên hợp, B có thể được xác định bằng phương pháp
đo độ dẫn điện,

235 | Bản quyền thuộc về Tạp chí Olympiad Hóa học KEM
Bài 17
Cho biết sản phẩm thu được:

236 | Bản quyền thuộc về Tạp chí Olympiad Hóa học KEM
Bài 18
Griseofulvin là một dược chất kháng nấm có thể được tổng hợp từ A và B
dưới sự hiện diện của kali butoxide/butanol bậc 3. Phản ứng xảy ra như
sau:
OCH3 OCH3
O O O CH
O 3

(CH3)3COK/ (CH3)3COH CH3


+ H3COC CCCH CHCH3 O
H3CO O H3CO O
H
Cl Cl
A B 灰黄霉素

(灰黄霉素: Griseofulvin)
1) Xác định cấu hình tuyệt đối của các nguyên tử carbon bất đối xứng
trong phân tử Griseofulvin.
2) Hãy viết tất cả các công thức cấu tạo các đồng phân của B thoả mãn
hai điều kiện: (1) vòng benzene chỉ có hai loại nguyên tử hydrogen
khác nhau, (2) phân tử chỉ có một nhóm chức duy nhất.
3) Hãy cho biết tên của phản ứng tạo thành Griseofulvin từ A và B.

237 | Bản quyền thuộc về Tạp chí Olympiad Hóa học KEM
Bài 19
Hãy vẽ công thức cấu tạo các sản phẩm 1, 2 và 3 trong các chuyển hoá
dưới đây và giải thích ngắn gọn tại sao p-hydroxybenzaldehyde chỉ tạo
thành một sản phẩm, ngược lại m-hydroxybenzaldehyde tạo thành hai
sản phẩm.

CHO
CH3(CH2)11Br, K2CO3

丙酮,回流
OH
1

CHO
CH3(CH2)11Br, K2CO3
+
丙酮,回流
OH

2 3

(丙酮: acetone, 回流: đun hồi lưu)

238 | Bản quyền thuộc về Tạp chí Olympiad Hóa học KEM
Bài 20
Xét phản ứng sau đây:

1) Hãy cho biết vai trò của NH4Cl.


2) Hãy vẽ hai cộng hưởng trung hòa điện tích khác của D.
3) Cho biết nguyên tử N nào trong số 4 nguyên tử N đánh số 1, 2, 3, 4
có tính base mạnh nhất.
4) Cho biết sản phẩm các phản ứng sau:

239 | Bản quyền thuộc về Tạp chí Olympiad Hóa học KEM
Bài 21
1) Hai alkaloid dưới đây có thể chuyển hóa cho nhau qua trung gian A.
Khi đạt cân bằng thì tỉ lệ giữa vòng 5 và vòng 6 là 3/2. Hãy xác định A

2) Cho biết cấu trúc trung gian của phản ứng sau:

240 | Bản quyền thuộc về Tạp chí Olympiad Hóa học KEM
Bài 22
Nhìn chung, amine hóa khử là phản ứng liên quan đến việc đưa nhóm
amino vào hợp chất hữu cơ qua phản ứng khử. Sơ đồ của phản ứng này
được minh họa dưới đây qua phản ứng của benzaldehyde và ethylamine.
Tiểu phân trung gian A có thể được cô lập và có thể được sử dụng mà
không cần qua tinh chế ở giai đoạn tiếp theo.

1) Các ví dụ của phản ứng amine hóa-khử sử dụng các amine và hợp chất
carbonyl khác nhau theo tỉ lệ 1:1 được cho dưới đây. Xác định sản
phẩm cuối của phản ứng. Trong những trường hợp có thể tạo thành
nhiều sản phẩm đồng phân thì hãy biểu diễn cấu trúc tất cả các sản
phẩm có thể có.

Tất cả các phản ứng được đề cập ở trên đều sử dụng amine bậc một. Khi
sử dụng amine bậc hai, thì trong phản ứng amine hóa-khử không thể cô
lập được tiểu phân trung gian bởi nó mang điện tích. Trong những trường
hợp này, phản ứng được tiến hành trong một giai đoạn và sử dụng tác
nhân khử chọn lọc hơn là NaBH(OAc)3. Đặc trưng riêng của tác nhân này
là nó không khử được các hợp chất carbonyl nhưng lại dễ dàng khử cả
hợp chất A và A*.

241 | Bản quyền thuộc về Tạp chí Olympiad Hóa học KEM
2) Dự đoán cấu trúc các amine được dùng để tạo ra các sản phẩm amine
hóa-khử sau đây sử dụng NaBH(OAc)3. Trong trường hợp sản phẩm
được biểu diễn có thể được tạo thành từ nhiều amine bậc hai khác
nhau thì hãy vẽ tất cả các cấu trúc amine có thể. Bỏ qua khả năng xảy
ra phản ứng amine hóa-khử nội phân tử.

Cần lưu ý rằng NaBH(OAc)3 có thể được sử dụng để amine hóa-khử bằng
cách sử dụng cả amine bậc một và bậc hai. Đó là lí do tại sao trong trường
hợp sử dụng 2 đương lượng carbonyl và 1 đương lượng amine bậc một
thì có thể nhận được amine bậc ba trong một giai đoạn:

3) Các ví dụ về phản ứng amine hóa-khử sử dụng các amine và hợp chất
carbonyl khác nhau và lượng dư NaBH(OAc)3 được cho dưới đây. Số
phân tử tác nhân biểu diễn trong sơ đồ tương ứng với tỉ lệ đương
lượng. Xác định sản phẩm cuối. Trong những trường hợp có thể tạo
thành nhiều sản phẩm đồng phân thì hãy biểu diễn cấu trúc tất cả các
sản phẩm có thể có.

242 | Bản quyền thuộc về Tạp chí Olympiad Hóa học KEM
243 | Bản quyền thuộc về Tạp chí Olympiad Hóa học KEM
Chuyên đề 42: Chuỗi phản ứng từ chất đầu đã biết
Lưu ý các bài tập được sắp xếp theo trình tự ngẫu nhiên

Bài 1
Người ta điều chế hydrocarbon C 6H4 theo sơ đồ dưới đây:

Theo ghi nhận của Bergman, khi đun nóng hợp chất C 6H4 trong một ống
kín với dung môi hexane thì thu được benzene, còn khi đun nóng trong
CCl4 thì thấy tạo thành 1,4-dichlorobenzene. Những phản ứng này đều
diễn ra qua sự tạo thành chất trung gian rất hoạt động [X]:

Hợp chất C 12H14O2 trong nước ở 37 oC khi có DNA với chu kì bán huỷ là
11.8 giờ thì thu được sản phẩm là 2,3-dihydroxymethyl-1,2,3,4-
tetrahydronaphthalene, quá trình phản ứng có sự tạo thành một chất
trung gian có bản chất tương tự [X]. Kết quả dẫn đến sự phân cắt mạch
kép của DNA và hợp chất C 12H14O2 có hoạt tính tương tự nhóm kháng sinh
DNA là calicheamicin.
1) Xác định cấu trúc các hợp chất A - D và C 6H4.
2) Đề nghị cấu trúc chất trung gian X trong chuỗi chuyển hoá của
Bergman.
3) Xác định cấu trúc các hợp chất chưa biết trong chuỗi chuyển hoá của
Bergman. Cho biết giai đoạn quyết định tốc độ phản ứng và giai đoạn
cắt mạch kép của ADN.
4) Hãy tính hằng số tốc độ của C 12H14O2 và thời gian để 80% lượng chất
ban đầu tham gia phản ứng.
5) Giải thích về sự giảm đáng kể nhiệt độ của phản ứng giữa C 6H4 (2000C)
và C 12H14O2 (370C) các quá trình chuyển hoá trên.

244 | Bản quyền thuộc về Tạp chí Olympiad Hóa học KEM
Bài 2
Khá nhiều pheromone [kích thích tố] của côn trùng có cấu trúc tương đối
đơn giản. Ví dụ như muscalure [Мускалур] (pheromone của ruồi giấm cái
dùng để thu hút ruồi đực) là (Z)-9-tricosene. Sau khi thiết lập được cấu
trúc của nó, một phương pháp tổng hợp chọn lọc đã được đề xuất:

Ngoài ra thì:

1) Xác định các chất chưa biết, biết rằng C, H, F là các dẫn xuất bromo
hữu cơ. Xác định cấu trúc cấu tạo muscalure.
2) Giải thích kết quả hóa lập thể của phản ứng chuyển hóa G thành
muscalure.
Tuy nhiên, sẽ tốt hơn nếu sử dụng một phương pháp rẻ hơn dựa vào sự
điện phân hỗn hợp đẳng mol các muối sodium của oleic acid (một acid
không no có 18 nguyên tử carbon) và acid L - gọi là phản ứng Kolbe - để
tổng hợp muscalure.

3) Xác định các chất L, M, N biết M(M) > M(N). Xác định công thức cấu
tạo oleic acid.
4) Làm thế nào để tách hỗn hợp các chất tạo thành?
5) Giải thích tại sao phản ứng Kolbe không thể áp dụng để tổng hợp các
hydrocarbon bất đối xứng từ các muối của các carboxylic khác nhau?

245 | Bản quyền thuộc về Tạp chí Olympiad Hóa học KEM
Bài 3
a) Viết tác nhân, điều kiện phản ứng (nếu có) thay cho dấu chấm hỏi (?)
và công thức cấu tạo của các hợp chất hữu cơ A, B, C, D để hoàn thành sơ
đồ phản ứng sau:

b) Bổ sung tác nhân, điều kiện phản ứng, hãy hoàn thành sơ đồ các phản
ứng sau:
(1) (2) (3) (4)
C 6H 6 C8H10 C8H9Br C 8H 8 C6H5CHO
(5)
PhNHNH2.HCl (6)
C6H5CH=CHCOCH=CHC6H5 X (C23H30N2)
NaAc, Na2CO3

Biết rằng phân tử X hấp thụ ánh sáng ở vùng nhìn thấy và có chứa dị vòng
5 cạnh. Viết công thức cấu tạo của X.

246 | Bản quyền thuộc về Tạp chí Olympiad Hóa học KEM
Bài 4
Hợp chất C có công thức phân tử là C 11H12O2 có thể được tổng hợp theo
con đường sau:

Chuỗi tổng hợp trên là các ví dụ có liên quan đế ba phản ứng hữu cơ nổi
tiếng sau đây:
Phản ứng I - Diels - Alder giữa một diene liên hợp với một alkene hoặc
một alkyne trong điều kiện được đun nóng, sẽ tạo thành các dẫn xuất của
cyclohexene, chẳng hạn:

Phản ứng II - cộng đóng vòng [2+2] giữa hai phân tử olefine trong điều
kiện được chiếu sáng thì sẽ tạo thành các dẫn xuất của cyclobutane, chẳng
hạn nếu có vị trí thích hợp trong phân tử thì hai liên kết –C=C– có thể
tham gia đóng vòng [2+2] nội phân tử:

Phản ứng III - khử - ghép mạch (reductive coupling) các ketone với tác
nhân khử là samarium iodide, chẳng hạn hai phân tử ketone sau được
khử - ghép mạch để tạo thành một hợp chất diol:

a) Hãy cho biết công thức cấu tạo của các chất A, B và C, nếu có thể hãy
trình bày rõ cấu trúc dạng không gian ba chiều của chúng.
b) Cho biết những đặc điểm về “hoá học xanh” thể hiện qua chuỗi phản
ứng trên là gì?

247 | Bản quyền thuộc về Tạp chí Olympiad Hóa học KEM
Bài 5
Dưới đây là sơ đồ chuyển hóa của các chất A - J (các giai đoạn trung hòa
có thể bỏ qua). Hãy hoàn thành sơ đồ.

248 | Bản quyền thuộc về Tạp chí Olympiad Hóa học KEM
Bài 6
Vào thập kỉ 1960, phương pháp sau đây để tổng hợp hợp chất E từ
naphthalene đã được phát triển ở Viện Hoá học Hữu cơ, ĐH Cologne
(Đức):

Chất D là hỗn hợp các đồng phân. Phổ 1H NMR của B và E lần lượt chứa 2
và 3 tín hiệu. Một trong các tín hiệu trong phổ 1H NMR của E là ở -0.45
ppm.
1. Xác định công thức cấu tạo của các chất A - E.
2. Xác định số đồng phân chất D tạo thành trong phản ứng C  D. Vẽ
công thức cấu tạo các đồng phân này.
3. Gọi tên E theo danh pháp IUPAC.
4. Xác định vị trí gần đúng của hai nhóm tín hiệu proton khác trong phổ
1 H NMR của chất E với độ chính xác 1 ppm. Tính tỉ lệ cường độ các tín

hiệu này.
5. Đưa ra kết luận về những đặc điểm trong cấu trúc electron của E, cũng
như hoạt tính của nó. Giải thích về việc gán các tín hiệu trong phổ 1H
NMR với các hydrogen tương ứng.

249 | Bản quyền thuộc về Tạp chí Olympiad Hóa học KEM
Bài 7
Hợp chất C thông qua chuỗi chuyển hoá sau, tạo thành hợp chất D.

1) Cho biết tên của hợp chất D.


2) Hãy khoanh tròn nguyên tử O trong E có nguồn gốc từ hợp chất C.
3) Xác định cấu trúc của hợp chất F.

250 | Bản quyền thuộc về Tạp chí Olympiad Hóa học KEM
Bài 8
Năm 1964, Woodward đã đề nghị sử dụng hợp chất A (C 10H10) làm tiền
chất để tổng hợp nên một hợp chất đặc biệt B (C 10H6). Hợp chất A có ba
loại nguyên tử hydrogen khác nhau theo tỉ lệ 6:3:1. Tuy nhiên các nguyên
tử hydrogen trong hợp chất B chỉ thuộc một loại, B có thời gian tồn tại
khoảng 1 microgiây trong vùng tự do ở máy khối phổ và không thể phân
lập được dưới nhiệt độ phòng. Sau ba thập niên, các nhà hoá học cuối
cùng cũng tổng hợp được một dianion thơm [C 10H6]2- từ A.
n-C H MeLi
C10H10 n-BuLi, t-BuOK, 6 14 [C10H6]2- 2K+
Me3SnX
[C10H6]2- 2Li+
A (CH )
3 2NCH 2CH2N(CH 3)2 C2H5OC2H5 CH3OCH2CH2OCH3
+
n-C6H14
- 78 oC C 2Li

Các nguyên tử hydrogen trong hợp chất C có cùng tính chất hoá học (cùng
loại) và có thể được chuyển hoá thành B dưới những điều kiện thích hợp.
Quá trình chuyển hoá từ A thành C được thể hiện trong sơ đồ sau:
1) Xác định công thức cấu tạo hợp chất A, B, C.
2) Hợp chất B có tính thơm hay không? Tại sao?

251 | Bản quyền thuộc về Tạp chí Olympiad Hóa học KEM
Bài 9
Hoàn thành dãy chuyển hoá sau:

(1) (2)

A B C

(光照: ánh sáng)


1) Cho biết điều kiện phản ứng (1), gọi tên loại phản ứng (1), (2).
2) Cho biết hợp chất A có bao nhiêu carbon bậc 1, bậc 2, bậc 3, bậc 4 và
có ít nhất bao nhiêu nguyên tử hydrogen đồng phẳng?
3) Biết D là một đồng phân của B, hãy xác định công thức cấu tạo của D.
4) Hợp chất E là một đồng phân của A, E có chứa carbon lai hoá sp, sp2,
sp3 và không có nhóm methyl trong phân tử. Xác định công thức cấu
tạo có thể có của E.

252 | Bản quyền thuộc về Tạp chí Olympiad Hóa học KEM
Bài 10
Xác định công thức cấu tạo của các hợp chất A - D trong dãy chuyển hoá
sau (không cần quan tâm đến lập thể):
1. Mg/苯 1. LiAlH4
回流 无水醚
A (C14H26O4) B (C12H22O2) C (C12H24O2) D (C12H22Br2)
2. H2O 2. H2O

(苯: benzene, 回流: đun hồi lưu, 无水醚: ete khan)

253 | Bản quyền thuộc về Tạp chí Olympiad Hóa học KEM
Bài 11
Capsaicin là một chất được lấy ra từ ớt, chính chất này tạo ra vị cay khi ăn
ớt. Hợp chất này lần đầu tiên được chiết xuất vào năm 1816 bởi Christian
Friedrich Bucholz. Ông gọi nó là “capsicin” vì nó được chiết ra từ cây thuộc
chi Capsicum. Sau đó vào năm 1876, John Clough Thresh người đã tách
được nó ở dạng gần như tinh khiết mới đặt cho nó một cái tên thông dùng
cho đến ngày nay là “Capsaicin”.
Cảm giác cay mà chúng ta có được là do chất capsaicin kích thích não sản
sinh ra chất endorphin, một chất morphin nội sinh, có đặc tính như những
thuốc giảm đau, đặc biệt có ích cho những bệnh nhân viêm khớp mạn
tính, bệnh đau đầu do thần kinh và các chứng đau do bệnh ung thư.
Có thể điều chế Capsaicin F theo sơ đồ sau:

Xác định cấu trúc của các hợp chất A - F trong chuỗi tổng hợp trên.

254 | Bản quyền thuộc về Tạp chí Olympiad Hóa học KEM
Bài 12
Hoàn thành các chuỗi phản ứng chuyển hóa sau:
O
1. H3C COOH o
O + A t , -H2O
B
ArCHO, AcOH
C , EtOH
H3C COOH piperi®in D
C6H8O4
Mg,ete CO2
A C Kh«ng ph¶n øng
2. H3C-CH=CH2 + Cl2 + CH3OH
CO2 PCl5 Br2
B Mg,ete D E F G
NH3 (3 mol)
NaHCO3, to CH3OH, HCl
J I H
C6H13NO3

255 | Bản quyền thuộc về Tạp chí Olympiad Hóa học KEM
Bài 13
Các dị vòng 5 cạnh đơn giản nhất - furan, pyrrole, thiophne - có những
tính chất hoá học giống nhau. Tuy nhiên, một chất X phản ứng với các dị
vòng này theo những cách khác nhau:

Biết rằng để chuẩn độ 10.00 mL dung dịch chứa 0.49 gam chất X với
100.00 mL nước thì cần 8.70 mL dung dịch NaOH 0.1150 M. Hợp chất C là
1 trong 2 đồng phân hình học có thể có. Chất B có 2 đối quang. Chất F có
2 nguyên tử carbon bất đối.
1. Xác định cấu trúc của X. Kiểm tra lại câu trả lời của bạn bằng các tính
toán (để xác định khối lượng nguyên tử) và dữ kiện phổ IR dưới đây:

2. Vẽ các cấu trúc của A, B, C, D, E, F. Để xác định chất C, hãy sử dụng dữ


liệu phổ 1H NMR dưới đây (400 MHz, không có tín hiệu nào trong
khoảng 0 và 6 ppm; các hằng số ghép cặp J2-3 = 1.15; J2-5 = 3.43; J3-5 =
5.04; J4-6 = 15.50 Hz.)
Vẽ tất cả các đối quang có thể có của F theo công thức chiếu Fischer
và chỉ rõ cấu hình tuyệt đối của các nguyên tử carbon bất đối (R hay
S). Xác định đồng phân F tạo thành trong phản ứng.

256 | Bản quyền thuộc về Tạp chí Olympiad Hóa học KEM
3. Mô tả cơ chế tạo thành A, B, C, D. Tên gọi thông thường của các phản
ứng này (trừ phản ứng tạo thành D) là gì? Trong tổng hợp C nên sử
dụng xúc tác nào? Các phản ứng này thể hiện sự biến đổi của tính chất
nào từ furan qua pyrrole tới thiophene?
4. Vẽ cấu trúc các đồng phân lập thể của C có thể được tạo thành từ
phản ứng của furan với X.

257 | Bản quyền thuộc về Tạp chí Olympiad Hóa học KEM
Bài 14
Aldehyde có thể phản ứng với hydrogen peroxide:

Hướng tổng hợp sau đã được chọn để điều chế một nhóm thuốc mới:

1) Xác định công thức phân tử của A và cấu tạo của B, C, D và acetal G.
2) Phản ứng E → F và F → G thuộc loại phản ứng cơ bản nào?
3) Vẽ các đồng phân quang học của hợp chất G.

258 | Bản quyền thuộc về Tạp chí Olympiad Hóa học KEM
Bài 15
Phản ứng Knoevenagel là một trong những phản ứng ngưng tụ rất hữu
ích và thường dùng trong tổng hợp hữu cơ. Một ví dụ điển hình của phản
ứng này được trình bày dưới đây, cho diethyl malonate ngưng tụ với
benzaldehyde trong xúc tác là hexahydropyridine để tạo thành hợp chất
diethyl 2-benzylidenmalonate

1) Xác định tác nhân nucleophin của phản ứng này.


2) Cho biết ngắn gọn vai trò của hexahydropyridine trong phản ứng này.
3) Từ hợp chất A người ta tiến hành tổng hợp một loại thuốc chống co
giật D (gabapentin), hãy cho biết 2 tác chất ban đầu cần thiết để điểu
chế A.

4) Từ A có thể điều chế D thông qua chuỗi chuyển hoá sau, xác định cấu
trúc các hợp chất B, C và D:

259 | Bản quyền thuộc về Tạp chí Olympiad Hóa học KEM
Bài 16
Dưới đây là hai sơ đồ tổng hợp twistane (T):

1) Hoàn thành sơ đồ phản ứng, viết cấu trúc các chất trung gian A - N.
2) Twistane có phải là hợp chất chiral? Nếu có, chỉ ra trung tâm bất đối
và cấu hình theo danh pháp R/S của cấu trúc twistane trong sơ đồ
tổng hợp.
3) Twistane có thể có bao nhiêu dẫn xuất monochloride?
4) Dưới đây là cấu trúc của một trong số các đồng phân có trong tự nhiên
của twistane - (-)-β-pinene. Chất này có thể có bao nhiêu dẫn xuất
monochloride?

5) Viết công thức cấu tạo đồng phân của twistane mà có thể có 16 dẫn
xuất monochloride (không tính đồng phân cấu hình).

260 | Bản quyền thuộc về Tạp chí Olympiad Hóa học KEM
Bài 17
Phản ứng Vilsmeier-Haack là một trong những phương pháp hiệu quả
nhằm gắn một nhóm fomyl vào một vòng thơm giàu điện tử. Quá trình
này, trước hết là giai đoạn xảy ra phản ứng giữa các tác chất N,N-
đimetylfomanit và POCl 3 để tạo thành tác chất Vismeier-Haack :

Sau đó vòng thơm giàu điện tử sẽ phản ứng với chất Vismeier-Haack vừa
tạo thành, sau đó qua giai đoạn thuỷ phân thì hoàn tất việc gắn nhóm
fomyl vào vòng thơm. Chẳng hạn:

1) Hãy viết công thức cộng hưởng cho tác chất Vilsmeier-Haack.
2) Vẽ cấu trúc các chất trung gian trong chuyển hoá trên, cho biết (1):
thế trên vòng thơm, (2): thế nucleophin nội phân tử, (3): cộng
nucleophin, (4): trao đổi proton, (5): tách loại.

3) Hoàn thành chuỗi phản ứng sau:

261 | Bản quyền thuộc về Tạp chí Olympiad Hóa học KEM
262 | Bản quyền thuộc về Tạp chí Olympiad Hóa học KEM
Bài 18
Vòng hoá Robinson là một phản ứng quan trọng để tổng hợp nhiều hợp
chất thiên nhiên, kháng sinh, steroid, … Phản ứng này gồm các giai đoạn
sau: deproton hoá ketone; cộng Michael của enolate ion tạo thành với
methyl vinyl ketone (hoặc dẫn xuất của nó); ngưng tụ aldol tạo thành dẫn
xuất cyclohexanone tương ứng.

Cho hai phản ứng dưới đây. Hợp chất X là sản phẩm của phản ứng vòng
hoá Robinson. Trái lại, trong phản ứng tạo thành Y có giai đoạn cộng
Michael, nhưng nó không có 2 ngưng tụ mà có 2 nhóm C=O khác nhau.

1) Xác định công thức cấu tạo của X, Y


Hợp chất X được dùng trong tổng hợp toàn phần terpenoid chống ung
thư (±)-widdrol:

2) Xác định công thức cấu tạo A-E.


Phản ứng vòng hoá Robinson cũng được dùng trong tổng hợp toàn phần
isonootkatone, một loại terpenoid khác. Hợp chất đầu trong tổng hợp này
là dibromoalkene F, trong phổ 1H NMR có 2 tín hiệu với cường độ tương
đối 2:3, một trong các sản phẩm ozone phân của nó là acetone.

263 | Bản quyền thuộc về Tạp chí Olympiad Hóa học KEM
3) Xác định công thức cấu tạo của F-M.

264 | Bản quyền thuộc về Tạp chí Olympiad Hóa học KEM
Bài 19
Năm 1869, alizarin đã được chiết xuất từ rễ cây thiên thảo (Rubia
tinctorum), dưới dạng dẫn xuất glycoside. Trước đây, alizarin chỉ được sử
dụng làm thuốc nhuộm, nhưng ngày nay nó còn được ứng dụng trong lĩnh
vực hoá phân tích.
1) Xác định các chất A, B, X trong sơ đồ phản ứng điều chế alizarin:

2) Vào năm 1915, alizarin đã được đề xuất làm tác nhân trắc quang để
xác định nhôm. Tuy nhiên, do ít tan trong nước nên không thể dùng
chính alizarin mà phải thay bằng “alizarin đỏ” – hợp chất này là một
monosulfonic acid, thu được khi sulfonate hoá alizarin. Viết công thức
của alizarin đỏ.

265 | Bản quyền thuộc về Tạp chí Olympiad Hóa học KEM
Bài 20
Cho sản phẩm chính - thu được khi đun nóng urea - phản ứng với NaOH,
tạo thành hợp chất A (là muối trisoidum). A phản ứng với chlorine tạo
thành hợp chất B, công thức phân tử C3N3O3Cl3. B là hóa chất được sản
xuất quy mô lớn, với sản lượng toàn cầu hàng năm trên 400 nghìn tấn.
Riêng ở Trung Quốc, sản lượng mỗi năm trên 50 nghìn tấn. Trong nước, B
có thể tiếp tục tạo thành hypochlorous acid và hợp chất C - được sử dụng
phổ biến để khử trùng bể bơi.
1) Xác định cấu tạo anion của hợp chất A.
2) Xác định cấu tạo của hợp chất B và phương trình phản ứng của nó với
nước.
3) Hợp chất C có một tautomer. Vẽ cấu tạo của C và dạng tautomer
tương ứng.
4) Viết phương trình phản ứng đun nóng urea ở trên.

266 | Bản quyền thuộc về Tạp chí Olympiad Hóa học KEM
Bài 21
Cho sơ đồ phản ứng sau:

Hãy viết các công thức cấu tạo của A, B, C, D1, D2 và E. Biết E là sản phẩm
phụ, có công thức phân tử C 19H22O5

267 | Bản quyền thuộc về Tạp chí Olympiad Hóa học KEM
Bài 22
Viết các tác nhân, điều kiện phản ứng (nếu có) thay cho dấu chấm hỏi (?)
và viết công thức cấu tạo của các hợp chất hữu cơ F, G, H, I, J để hoàn
thành sơ đồ chuyển hóa sau:

268 | Bản quyền thuộc về Tạp chí Olympiad Hóa học KEM
Bài 23
Phản ứng Wittig là một trong những phương pháp nổi tiếng nhất để tạo
thành các liên kết đôi C=C. Tên gọi của phản ứng này được gọi theo nhà
hoá học tiên phong người Đức, người dành giải Nobel Hoá học 1979 -
George Wittig. Ở dạng đơn giản nhất, phản ứng này là tương tác của một
hợp chất carbonyl, ví dụ như benzaldehyde, với hợp chất A. Phản ứng
diễn ra qua một hợp chất trung gian kém bền B, chứa một vòng 4 cạnh,
và sản phẩm tạo thành là (C 6H5CH=CH2) và hợp chất C.

1. Xác định cấu tạo của các chất A - C, biết rằng thành phần nguyên tố
của A là 82.59% C; 6,20% H; 11.21% P.
Hợp chất A có thể được điều chế từ benzene và methane theo sơ đồ sau:

2. Xác định cấu tạo của các chất D - J.


3. Trình bày cơ chế tạo thành E và I.

269 | Bản quyền thuộc về Tạp chí Olympiad Hóa học KEM
Bài 24
Viết công thức cấu tạo của các hợp chất hữu cơ A, B, C, D, E, F, G, H, I, J,
K, M, N để hoàn thành các sơ đồ chuyển hóa sau:
o
NaCN HNO3, CH3COOH 1. NaOH, t
1. PhCHO A B C
+
(C14H12O2) 2. H

PCC, CH2Cl2 H2N-C(CH3)3 LiN[CH(CH3)2]2


2. CH3-CH2-CH2-OH D E F
O H2O, H+
1. CH3 M N
HBr Mg CH3 H2, Pd/C H2O, H+ (C15H20O)
CH2=CH-CHO G H I J K
ete 2. H2O
HO OH

270 | Bản quyền thuộc về Tạp chí Olympiad Hóa học KEM
Bài 25
Nepetalactone là một hợp chất hữu cơ được phân lập từ cây catnip
(Nepeta cataria) vào năm 1941, được sử dụng làm thuốc ngủ, thuốc chống
co giật, thuốc hạ sốt. Dihydronepetalactone có thể thu được bằng cách
hydro hoá nepetalactione, là một chất xua đuổi côn trùng rất hiệu quả.
Khi tiến hành nghiên cứu quá trình tổng hợp và tính chất của
dihidronepetalactone, các phản ứng sau đây được thực hiện:

CHO CH3OH, H+
CrO3
A B
H2SO4, 丙酮,0 oC

HBr 1. LiOH, CH3OH LiAlH4


C D E
过氧化物 2. H+, 无水四氢呋喃

(丙酮: acetone, 过氧化物: peroxide, 无水四氢呋喃: anhydrous


tetrahydrofuran)
Xác định công thức cấu tạo các hợp chất A - E (không cần quan tâm đến
lập thể)

271 | Bản quyền thuộc về Tạp chí Olympiad Hóa học KEM
Bài 26
Viết công thức cấu tạo của các hợp chất hữu cơ A, B, C, D, E, F, G, H, I, J, K
để hoàn thành các sơ đồ chuyển hóa sau:

PPh3
Cl2
1. 1,2-§ iformylbenzen Ph3P A B
(C8H6O2)
o o
CrO3/Py CH2=CH-CH=CH2, 200 C H2N-NH2/C2H5ONa K2Cr2O7/H2SO4,t
2. -Metylnaphtalen C D E F
(C11H10)
o
350 C H2N-OH 1. H2SO4 +G 1.CH3COCl
F G H I J +
K (C16H18O2)
2. LiAlH4 2.H

272 | Bản quyền thuộc về Tạp chí Olympiad Hóa học KEM
Bài 27
Xác định các chất chưa biết trong dãy sau

273 | Bản quyền thuộc về Tạp chí Olympiad Hóa học KEM
Bài 28
1) C được tổng hợp bằng phản ứng dưới đây. Tuy nhiên kết quả lại thu
được một sản phẩm đồng phân

Xác định cấu trúc của D, E.


2) F phản ứng với thionyl chloride trong sự có mặt của hỗn hợp
pyridine/ether lại không thu được sản phẩm chlorine hóa nhưng thu
được hai chất G, H có chứa nối đôi, không có sản phẩm nào khác.

a) Xác định G, H.
b) Về lý thuyết phản ứng này có thể tạo thành một sản phẩm J là
đồng phân của G và H. Cho biết cấu trúc của nó và giải thích tại
sao nó không thể tạo thành.

274 | Bản quyền thuộc về Tạp chí Olympiad Hóa học KEM
Bài 29
Viết công thức cấu tạo của các hợp chất hữu cơ B, C, D, E và cơ chế phản
ứng tạo thành B theo sơ đồ chuyển hóa sau:
O
H3C OH H2N N SH
H3C H dd NaOH, to
O B C + D + E
O

275 | Bản quyền thuộc về Tạp chí Olympiad Hóa học KEM
Bài 30
Hoàn thành các sơ đồ phản ứng dưới đây:

Hãy giải thích sự hình thành hợp chất A trong sơ đồ trên bằng cơ chế phản
ứng.

276 | Bản quyền thuộc về Tạp chí Olympiad Hóa học KEM
Bài 31
Thành phần hoá học trong các loài thực vật và nấm có hoạt tính sinh học
tác động lên cơ thể người thường có những base chứa nitrogen. Các hợp
chất chứa nitrogen này được gọi là alkaloid và là thành phần cơ bản để
tạo nên nhiều loại dược phẩm, thuốc và sản phẩm nông hoá học.
Hiện nay, có nhiều phương pháp để tổng hợp các base chứa nitrogen như
vậy – ví dụ như amino hoá-khử, trong đó hợp chất carbonyl phản ứng với
amine bậc một hoặc bậc hai, sau đó khử hoá hợp chất trung gian.
1) Xác định chất A, B trong chuỗi phản ứng sau:

Phản ứng Mannich (do Carl Ulrich Franz Mannich khám phá năm 1912) là
phương pháp thường dùng để tổng hợp các β-amino carbonyl. Phản ứng
được thực hiện trong môi trường acid, giữa một aldehyde hoặc ketone có
thể enol hoá, với một aldehyde khác và một amine bậc một hoặc hai.

Bộ khung D dưới đây được tìm thấy trong các alkaloid có trong cây bụi, và
là tác nhân gây ra tổn thương gan khi đi vào cơ thể động vật.

2) Xác định chất đầu C (C8H15NO2) có thể dùng để điều chế hợp chất D
qua phản ứng Mannich.
Một trong các phương pháp được dùng để xác định cấu trúc các alkaloid
là thoái phân Hoffman. Alkaloid bị chuyển thành ammonium hydroxidee
bậc bốn, sau đó đun nóng tạo thành olefin và một amine bậc ba.

277 | Bản quyền thuộc về Tạp chí Olympiad Hóa học KEM
3) Xác định công thức sản phẩm nhận được khi tiến hành thoái phân
Hoffman hợp chất sau:

Pethidine (C 15H21NO2) là một chât tổng hợp có cấu trúc tương tự


morphine (một alkaloid tự nhiên) và được dùng làm thuốc giảm đau. Thực
hiện chuỗi phản ứng sau:

4) Xác định công thức cấu tạo của các chất trung gian F, G và pethidine.
Pseudoephedrin, một alkaloid tự nhiên, được dùng làm thuốc chống sung
huyết mũi, khi trải qua thoái phân Hoffman tạo thành H (C 9H10O). Tuy
nhiên, H không làm mất màu nước bromine và không tạo thành dẫn xuất
oxime.
5) Xác định cấu trúc lập thể của chất H.

Indole là một dị vòng nitrogen, được tìm thấy trong nhiều alkaloid – một
trong số đó là indole-3-acetic acid, có khả năng điều tiết tốc độ tăng

278 | Bản quyền thuộc về Tạp chí Olympiad Hóa học KEM
trưởng thực vật. Indole có thể bị proton hoá ở các vị trí khác nhau trên
vòng pyrole.
6) Xác định công thức của cation bền nhất (I), tạo ra từ phản ứng proton
hoá indole.

Indole dễ bị oxid hoá bởi không khí và nhiều tác nhân oxid hoá khác, tạo
thành hợp chất O. Hợp chất O có thể được tổng hợp từ o-xylen theo chuỗi
phản ứng sau:

7) Xác định công thức cấu tạo các chất J - O.


Dime hoá-oxid hoá O sẽ tạo thành P, về mặt lí thuyết thì chất này có 3
đồng phân quang học.
8) Xác định công thức cấu tạo của P.
P bị oxid hoá tạo thành hợp chất Q (C16H10N2O2), là một loại thuốc nhuộm
quan trọng trong công nghiệp dệt.
9) Xác định công thức cấu tạo của Q.
10) Q có thể có bao nhiêu đồng phân quang học?
11) Indirubun R là một đồng phân của Q, là sản phẩm phụ trong quá trình
trao đổi chất ở vi khuẩn. R tạo thành từ phản ứng của Q với isatin. Xác
định công thức cấu tạo của R.

279 | Bản quyền thuộc về Tạp chí Olympiad Hóa học KEM
Imidazole là một dị vòng chứa hai nguyên tử nitrogen.

12) (Những) phát biểu nào sau đây là đúng:


i. Imidazole là base yếu hơn pyridine và là acid mạnh hơn
pyrrole.
ii. Imidazole là base mạnh hơn pyridine và là acid yếu hơn
pyrrole.
iii. Imidazole là base mạnh hơn pyridine và là acid mạnh hơn
pyrrole.
iv. Imidazole là base yếu hơn pyridine và là acid yếu hơn pyrrole.
13) Hướng tấn công của tác nhân electrophile vào hệ dị vòng chịu ảnh
hưởng của các nhóm thế tương tự như với hệ benzene. Xác định công
thức cấu tạo các chất S, T để hoàn thành chuỗi phản ứng sau.

Các sulfur ylide U là những chất có nguyên tử carbon mang điện tích âm
liên kết với nguyên tử lưu huỳnh mang điện dương. Chúng phản ứng với
các hợp chất carbonyl để tạo thành hệ dị vòng đơn giản nhất - oxiran (hay
epoxidee). U được tạo thành khi cho hợp chất chứa sulphonium ion (S
mang điện tích dương) có -H phản ứng với base.

14) Xác định các chất V - X trong sơ đồ tổng hợp thuốc kháng nấm
fluconazole X (C13H12OF2N6).

280 | Bản quyền thuộc về Tạp chí Olympiad Hóa học KEM
281 | Bản quyền thuộc về Tạp chí Olympiad Hóa học KEM
Bài 32
Muscimol (E) là một trong các hợp chất vòng, độc và gây ảo giác, có trong
thành phần nấm tán bay đỏ. Muscimol có thể được tổng hợp từ 3-
chloroprop-1-yne (A) theo sơ đồ dưới đây. Trong giai đoạn đầu tiên,
alkyne bị deproton hoá. Sau đó xảy ra phản ứng thế ở nguyên tử carbon
của nhóm carbonyl, tạo thành hợp chất B (C6H7O2Cl). Từ ester B thu được
amide C (C 4H4O2NCl), sau đó tự diễn ra phản ứng đóng vòng tạo thành
chất D. Phản ứng của D với ammonia trong methanol tạo thành chất E.

a) Trình bày cơ chế phản ứng thế ở nguyên tử carbon của nhóm carbonyl
trong môi trường acid với ví dụ của CH3OH và CH3COCl.
b) Vẽ cấu tạo của các hợp chất A - C và E.
c) Vẽ cấu tạo dạng tautomer của hợp chất D.
Ở giai đoạn cuối của tổng hợp này, các sản phẩm phụ tạo thành là hợp
chất F (M = 211 gam/mol), với 2 nhóm hydroxyl và một lượng nhỏ hợp
chất G (M = 308 gam/mol), với 3 nhóm hydroxyl.
d) Vẽ cấu tạo của các hợp chất F và G.
e) Đề xuất một cách để giảm tỉ lệ các sản phẩm phụ F và G trong tổng
hợp muscimol từ D.

282 | Bản quyền thuộc về Tạp chí Olympiad Hóa học KEM
Bài 33
Pirimidin và dẫn xuất của nó có khá nhiều hoạt tính sinh học như kháng
khuẩn, kháng nấm, chống sốt rét, kháng virus HIV… Sơ đồ dưới đây dùng
để tổng hợp 2-amino-6-aryl-4-(5-hydroxi-4-metylcumarin-6-yl)pirimidin
(chất Z)

Hãy cho biết công thức cấu tạo của các chất T, X, Y và Z trong sơ đồ trên.

283 | Bản quyền thuộc về Tạp chí Olympiad Hóa học KEM
Bài 34
a) Giải Nobel Hóa học năm 2000 được trao cho các
công trình nghiên cứu về polime dẫn điện.
Polithiophen là một trong số polime dẫn điện được
nghiên cứu nhiều do có nhiều tính chất điện hóa thú
vị. Sở dĩ polithiophen có những tính chất thú vị này
là do nó có thể chuyển từ trạng thái khử (hình vẽ)
sang trạng thái oxy hóa bằng quá trình pha tạp (pha tạp điện hóa hoặc
hóa học). Polithiophen có hai trạng thái oxy hóa cơ bản là polaron và
bipolaron. Ở trạng thái polaron, trên ba mắt xích thiophen liên tiếp xuất
hiện một điện tích dương và một gốc tự do. Ở trạng thái bipolaron, trên
ba mắt xích thiophen liên tiếp xuất hiện hai điện tích dương. Hãy trình bày
cấu trúc polaron và bipolaron của polithiophen.
b) Polithiophen có thể được tổng hợp từ thiophen với chất oxy hóa FeCl3.
Hãy trình bày cơ chế của phản ứng này, biết trong quá trình phản ứng xuất
hiện các tiểu phân vừa mang điện tích dương, vừa mang gốc tự do (được
gọi là cation-gốc).
c) Gần đây các hợp chất đa điện ly (polyelectrolyte) được quan tâm nghiên
cứu do có nhiều ứng dụng trong các linh kiện điện hóa. Hợp chất đa điện
ly trên cơ sở polythiophen càng được lưu tâm nghiên cứu do tích hợp
được các tính chất điện hóa của polythiophen. Sơ đồ dưới đây dùng để
tổng hợp một loại hợp chất đa điện ly (chất D). Hãy cho biết công thức
cấu tạo của các chất A, B, C và D trong sơ đồ:

284 | Bản quyền thuộc về Tạp chí Olympiad Hóa học KEM
Bài 35
a) Nguyên tử hydro ở vị trí α so với nhóm carbonyl khá linh động. Dưới
tác dụng của xúc tác bazơ mạnh, hợp chất carbonyl chuyển hóa thành ion
enolat. Viết công thức các sản phẩm enolat tạo thành từ 2-metylpentan-
3-on (A) và so sánh độ bền của chúng.
b) Xác định công thức cấu tạo của các chất B, C, D, E, G, H theo sơ đồ phản
ứng sau:

285 | Bản quyền thuộc về Tạp chí Olympiad Hóa học KEM
Bài 36
Phát hiện gần đây về việc nhiễm trùng virus Zika khi mang thai được cho
là có liên quan tới dị tật đầu nhỏ ở trẻ sơ sinh đã buộc Tổ chức Y tế thế
giới (WHO) ra tuyên bố Tình trạng Khẩn cấp Y tế Công cộng Quốc tế
(PHEIC) 5 vào ngày 1 tháng Hai 2016.
Virus Zika đi vào cơ thể người khi họ bị muỗi Aedes nhiễm bệnh đốt. Hiện
nay (tính tới tháng Hai, 2016) không có một loại vaccine hoặc thuốc điều
trị nào có thể chống lại virus Zika, do đó việc giảm số lượng muỗi và khả
năng bị chúng đốt là những việc làm quan trọng để giảm tỉ lệ nhiễm trùng.

Sử dụng thuốc trừ sâu để xử lí các quần thể muỗi là cách làm hiệu quả để
giảm nhiễm trùng bởi các bệnh dịch lây truyền từ chúng. Một trong các
họ thuốc trừ sâu thường dùng để xử lí quần thể muỗi là pyrethroid, bởi
chúng có tác động tới môi trường thấp hơn các loại thuốc cơ phosphate
và cũng ít độc hại với cơ thể người hơn.
Phần I: Pyrethrin
Các pyrethroid là những hợp chất tổng hợp từ các pyrethrin tồn tại trong
tự nhiên. Các pyrethrin được tạo ra trong hoa Chi Cúc (hay còn gọi là
pyretherum, nghĩa là hoa cúc lá nhỏ) và được dùng để sản xuất thuốc trừ
sâu từ đầu những năm 1900. Dưới đây là cấu trúc của 1 trong 6 pyrethrin
được tạo ra từ hoa Chi Cúc:

5 Public Health Emergency of International Concern

286 | Bản quyền thuộc về Tạp chí Olympiad Hóa học KEM
a) Xác định công thức phân tử của Pyrethrin I.
b)
• Giả sử rằng chỉ có các liên kết C=C phản ứng thì cần bao nhiêu mol
Br2 để phản ứng hết với 500 mg mẫu pyrethrin I?
• Tính thể tích nước bromine 0.0500 M (theo cm 3) cần để phản ứng
hết với 500 mg mẫu Pyrethrin I.
Phần II: Pyrethroid
Các pyrethrin bị phân huỷ nhanh chóng dưới ánh sáng cực tím, vậy nên
vào giữa thế kỉ 20, chúng đã bị thay thế bới các pyrethroid tổng hợp. Mặc
dù các pyrethroid có độc tính thấp với động vật có vú nhưng việc sử dụng
nhiều hơn và bền hơn pyrethrin nên chúng sẽ trở thành mối nguy hại nếu
bị tích luỹ trong chuỗi thức ăn. Kĩ thuật sắc kỉ lỏng khối phổ (LC-MS) được
dùng để xác định chính xác nồng độ các pyrethroid trong thực phẩm.
Trong đó, kĩ thuật sắc kí dùng để tách riêng các hợp chất, còn khối phổ để
xác định và định lượng chúng.
Đường chuẩn sau biểu diễn kích thước các peak (mũi) ion phân tử tại 5
mức nồng độ khác nhau của cypermethrin trong quả việt quất xay
nhuyễn. Phương trình đường thẳng phù hợp nhất với dữ kiện là
Diện tích peak = 44.547 × (Nồng độ cypermethrin / µmol dm –3) + 2.403

287 | Bản quyền thuộc về Tạp chí Olympiad Hóa học KEM
Lượng cypermethrin có thể ăn vào mà không gây nguy hiểm, gọi là MRL6
(mức độ rủi ro tối thiểu), bằng 0.020 mg kg -1 ngày-1. Nghiền 4 quả việt
quất thành 15 cm3 mẫu rồi tiến hành phân tích, quan sát được peak có
diện tích bằng 4.8.
c)
• Tính nồng độ cypermethrin trong mẫu.
• Tính khối lượng cypermethrin trong mẫu.
• Một đứa trẻ tập đi nặng 15 kg có thể ăn bao nhiêu quả việt quất
mỗi ngày mà không vượt quá MRL?
Nhiều pyrethroid, bao gồm cả cypermethrin, có chứa những halogen như
chlorine và bromine, là những chất có thể gây ra nhiễu loạn khi giải khối
phổ. Bromine trong tự nhiên có hai đồng vị bền là 79Br và 81Br với hàm
lượng xấp xỉ nhau. Chlorine cũng có hai đồng vị bền là 35Cl và 37Cl với hàm
lượng lần lượt là 75 % và 25 %.
d) Các đồng vị 79Br và 81Br có thể kết hợp với nhau tạo thành những phân
tử Br2 nào?
e) Phổ khối nào sau đây phù hợp với Br 2?

f)
• Các peak xuất hiện trong khối phổ của Cl2 tương đương với những
giá trị m/z nào?
• Xác định cường độ tương đối của các peak trong khối phổ Cl2.
g) Với mỗi hợp chất dưới đây, hãy cho biết giá trị m/z của tất cả các ion
phân tử và cường độ tương đối của chúng.

6 minimum risk level

288 | Bản quyền thuộc về Tạp chí Olympiad Hóa học KEM
Deltamethrin là một trong những loại thuốc trừ sâu phổ biến nhất được
dùng trên toàn thế giới để xử lí các quần thể muỗi. Nồng độ khuyến cáo
của deltamethrin trên mỗi quần thể muỗi là 55 mg m-2.
h) Giả sử rằng toàn bộ deltamethrin sẽ được hấp thụ, tính thể tích dung
dịch deltamethrin 10 % (10 gam/100 cm 3) cần để xử lí một quần thể
muỗi có diện tích 12.5 m2.
Phần III: Chuyển hoá pyrethroid trong động vật có vú
Các pyrethroid có độc tính thấp với động vật có vú do bị thuỷ phân nhanh
các liên kết ester, tạo thành những sản phẩm không độc. Trong phản ứng
thuỷ phân ester, nước tấn công vào nhóm carbonyl, tạo thành các tiểu
phân tứ diện tạm thời, sau đó bị phân huỷ thành carboxylic acid và
alcohol.

i) Xác định các sản phẩm nhận được từ phản ứng thuỷ phân các hợp
chất sau:

Phần IV: Primaquine


Mặc dù virus Zika chỉ mới được phát hiện gần đây, nhưng các hợp chất
hoá học đã được dùng để chống lại các căn bệnh truyền nhiễm do muỗi
từ hàng trăm năm trước. Đa số các hoá chất dùng để điều trị sốt rét là từ

289 | Bản quyền thuộc về Tạp chí Olympiad Hóa học KEM
quinoline. Một trong số đó, primaquine, là loại thuốc duy nhất điều trị
được cả sốt rét tái phát và cấp tính.

Trong quy trình tổng hợp primaquine, tất cả các nguyên tử carbon trong
sản phẩm mong muốn đều có nguồn gốc từ các tác nhân U và V.

Nhóm NH2 trong tác nhân U là một nucleophile tốt, còn tác nhân V là một
electrophile tốt. Tuy nhiên, nỗ lực để tổng hợp primaquine bởi phản ứng
thế nucleophile giữa các tác nhân này rất khó khăn do có nhiều sản phẩm
có thể được tạo thành. Từ 1 phân tử tác nhân V, và phản ứng với U chỉ sử
dụng duy nhất nguyên tử nitrogen trong nhóm NH2 làm tâm nucleophile,
có thể tạo ra tới 4 sản phẩm: 2 trong số đó là đồng phân cấu tạo, 2 chất
còn lại có công thức phân tử khác.
j) Xác định công thức cấu tạo của 4 sản phẩm đó.
k) Chỉ có 1 trong 4 sản phẩm là hợp chất sử dụng được cho tổng hợp
primaquine.
• Xác định chất đó.
• Đề xuất tác nhân để xử lí sản phẩm này để thu được primaquine.
Thực tế thì sản phẩm hữu ích trên chiếm một lượng nhỏ trong hỗn hợp
phản ứng giữa U và V, do vậy quy trình tổng hợp primaquine phải tiến
hành theo 7 giai đoạn như sau:

290 | Bản quyền thuộc về Tạp chí Olympiad Hóa học KEM
l) Xác định công thức cấu tạo của các anion W-, hợp chất X, Y và sản
phẩm phụ Z.

291 | Bản quyền thuộc về Tạp chí Olympiad Hóa học KEM
Bài 37
Permethrin là một loại thuốc trừ sâu có hiệu quả cao mà lại ít độc tính, có
thể được điều chế theo chuỗi chuyển hoá sau:
OH CH3 OH CH3
H3 C SO3H
AlCl3
A (C4H8) + B (C2HOCl3) Cl3CCHCH2C=CH2 Cl3CCHCH=CCH3

C D
Cl
CH3 COOCH2CH3
CH3C(OC2H5)3 CH3CH2ONa
Cl2C=CHCHCCH2COOCH2CH3 Cl
C2H5OH
Cl CH3

E
F
Cl Cl
COONa COOCH2
NaOH H + N(CH2CH3)3 + NaCl
Cl Cl
C2H5OH O
CH3

G I

1) Biết rằng hợp chất A có thể làm mất màu brom/CCl 4 và không có đồng
phân hình học. Hãy xác định công thức cấu tạo của A và B.
2) Gọi tên theo danh pháp của E và tên các nhóm chức trong I.
3) Quá trình chuyển hoá từ E sang F trải qua mấy giai đoạn? Cho biết mỗi
giai đoạn đó thuộc loại phản ứng nào.
4) Trong phản ứng chuyển hoá hợp chất E thành F, có thể sử dụng tác
chất NaOH/C 2H5OH thay cho C 2H5ONa/C 2H5OH được không? Tại sao?
5) Hợp chất G và H phản ứng với nhau tạo thành hợp chất I, N(CH2CH3)3
và NaCl. Hãy suy ra công thức cấu tạo của H. Cho biết phân tử H có ít
nhất bao nhiêu nguyên tử đồng phẳng với oxygen.
6) Hợp chất J có ít hơn 2 nguyên tử hydrogen so với F, chúng thuộc ba
loại khác nhau theo tỉ lệ 9:2:1. Hãy cho biết các công thức cấu tạo có
thể có của hợp chất J.

292 | Bản quyền thuộc về Tạp chí Olympiad Hóa học KEM
Bài 38
Anlylmagie bromua (A) phản ứng với acrolein tạo thành chất B, sau khi
thuỷ phân B sẽ được sản phẩm C duy nhất. Đun nóng C nhận được chất
D. Cho D phản ứng
với C 6H5Li thu được sản phẩm E. Đun nóng E khi có vết iot thì được F có
công thức C 12H14.
1. Hoàn thành sơ đồ dãy phản ứng trên (viết công thức cấu trúc của các
chất hữu cơ từ C đến F).
2. Ghi kí hiệu cơ chế các giai đoạn của phản ứng dưới các mũi tên trong
sơ đồ, trừ giai đoạn tạo thành F.
3. Cho biết cấu hình của F.

293 | Bản quyền thuộc về Tạp chí Olympiad Hóa học KEM
Bài 39
Ciprofloxacin là thuốc kháng sinh bán tổng hợp, có phổ kháng khuẩn rộng,
thuộc nhóm quinolone, còn được gọi là các chất ức chế ADN girase. Do
ức chế enzim ADN girase nên thuốc ngăn cản sự sao chép của nhiễm sắc
thể khiến cho vi khuẩn không sinh sản được nhanh chóng. Ciprofloxacin
có tác dụng tốt với các vi khuẩn kháng lại kháng sinh thuộc các nhóm khác
(aminoglycoside, cephalosporin, tetracyclin, penicillin…) và được coi là
một trong những thuốc có tác dụng mạnh nhất trong nhóm
flouroquinolon. Nó có thể được tổng hợp thông qua chuỗi phản ứng dưới
đây:

1) Xác định cấu trúc của các hợp chất A, B, D, E, F.


2) Tại sao khi chuyển hoá A thành B người ta phải sử dụng NaH, trong
khi đó chuyển hoá B thành C lại dùng NaOC 2H5 ?
3) Phản ứng chuyển hoá từ C thành D gồm hai giai đoạn, hãy cho biết
và giải thích ngắn gọn hai giai đoạn đó.

294 | Bản quyền thuộc về Tạp chí Olympiad Hóa học KEM
Bài 40
Thành phần chính của dầu thông là α-pinen (2,6,6-trimetylbixiclo[3.1.1]
hept-2-en).
1) Cho α-pinen tác dụng với axit HCl được hợp chất A, sau đó cho A tác
dụng với KOH/ancol thu được hợp chất camphen (B). Viết cơ chế phản
ứng chuyển hóa α-pinen thành A và A thành B. Trong môi trường axit,
B quang hoạt chuyển hóa thành B raxemic. Giải thích hiện tượng này.
Viết sơ đồ các phản ứng tổng hợp B từ xiclopentađien và acrolein cùng
các hóa chất cần thiết khác, biết rằng một trong số các sản phẩm trung
gian của quá trình tổng hợp là một enol axetat C.

2) Khi cho α-pinen tác dụng với axit vô cơ mạnh thì xảy ra quá trình đồng
phân hóa tạo thành 4 tecpen-hiđrocacbon (D1, D2, D3, D4) và quá trình
hiđrat hóa sinh ra 4 tecpen-ancol (E1, E2, E3, E4). Hãy vẽ công thức của
8 sản phẩm và trình bày cơ chế tạo thành các hợp chất này. Biết rằng
8 sản phẩm đều có bộ khung monoxiclic kiểu p-mentan hoặc bộ khung
bixiclic kiểu bixiclo[2.2.1]heptan; từ mỗi cacbocation trung gian sinh
ra một tecpen-hiđrocacbon và một tecpen-ancol.

295 | Bản quyền thuộc về Tạp chí Olympiad Hóa học KEM
Bài 41
Hãy hoàn thành sơ đồ phản ứng sau, giải thích sự hình thành X5 và X6:
Br2 NaI Mg X4 (C16H12) t o HCl
o-Xilen o X1 X2 X3 X5 X6 (C16H13Cl)
t - ,,
X7 (C16H12)

-
(C8H 6Br 4) (C8H 6Br 2) (C8H 6) (khong mat' mau nuoc brom)

296 | Bản quyền thuộc về Tạp chí Olympiad Hóa học KEM
Bài 42
Dùng công thức cấu tạo hãy hoàn thành sơ đồ phản ứng sau và giải thích
sự hình thành V1 và V2.
O O
150 oC O O O
U+ O O V1, V2 C14 H12O 6 (W1, W2)

O O

297 | Bản quyền thuộc về Tạp chí Olympiad Hóa học KEM
Bài 43
Nhân indole xuất hiện rất phổ biến trong các hợp chất thiên nhiên. Để
tổng hợp cấu trúc vòng indole này, một trong những phương pháp được
sử dụng rộng rãi nhất đó là tổng hợp Fischer. Tổng hợp này dùng nguyên
liệu là aldehyde, ketone, ketoacid, ester hoặc là các nhân thơm có nhóm
thế đion cho tác dụng với hydrazone xúc tác ZnCl 2 trong môi trường có
poliphosphoric acid hoặc Lewis acid như BF3 và đun nóng. Ví dụ:

Melatonin là một dẫn xuất indole có hoạt tính sinh học, có thể được điều
chế từ nguyên liệu ban đầu là p-anisidine theo sơ đồ tổng hợp sau:

Viết cấu trúc hoặc điều kiện phản ứng cần thiết cho các chữ kí tự A, B, C,
D, E, F, G, H và I.

298 | Bản quyền thuộc về Tạp chí Olympiad Hóa học KEM
Bài 44
N-metyl-N’-nitro-N-nitrosoguanidin (MNNG) là một hóa chất thương mại,
được tổng hợp và sử dụng theo sơ đồ sau:

a) Xác định công thức cấu tạo của các chất A, B, C, D, E, F và MNNG.
b) Đề nghị cơ chế phản ứng để giải thích sự tạo thành các chất A, B, C và
D.
c) Ở nhiệt độ phòng E là chất rắn, lỏng hay khí, bền hay không bền, tan tốt
trong nước hay dietyl ete? Giải thích.
d) Vì sao MNNG là hóa chất thương mại còn E thì không?

299 | Bản quyền thuộc về Tạp chí Olympiad Hóa học KEM
Bài 45
Muốn thực hiện phản ứng đồng phân hóa cis-trans, người ta có thể dùng
tác nhân quang hóa hoặc tác nhân hóa học. Sơ đồ dưới đây sử dụng phản
ứng đồng phân hóa dùng tác nhân hóa học.

a) Hoàn thành sơ đồ phản ứng và cho biết cấu trúc các hợp chất G, H, I, K
và L.
b) Giải thích tiến trình lập thể của phản ứng tạo thành K từ H.

300 | Bản quyền thuộc về Tạp chí Olympiad Hóa học KEM
Bài 46
Xác định công thức cấu tạo của các hợp chất G, H, I, J, K, L, M, N1 và N2
trong sơ đồ phản ứng:

Biết L không có tính quang hoạt; N1 và N2 có cùng công thức phân tử


C 7H12O4 nhưng chỉ N1 có tính quang hoạt.

301 | Bản quyền thuộc về Tạp chí Olympiad Hóa học KEM
Bài 47
Một hợp chất có cấu trúc không gian đặc biệt cồng kềnh là 2,3,4,5,6-
phentaphenylbenzaldehyde – nguyên liệu phát huỳnh quang đỏ mới này
là một chất trung gian quan trọng trong việc tổng hợp nhiều hợp chất
khác phức tạp hơn. Nó đã được tổng hợp theo chuỗi phản ứng sau:

Hợp chất A được điều chế nhờ phản ứng:

Một nhà hoá học sau khi nghiên cứu các tài liệu thích hợp, và từ nguyên
liệu có sẵn là cinnamic aldehyde (F), đã đề nghị một con đường tổng hợp
2,3,4,5,6-pentabenzealdehyde theo sơ đồ sau:

Trong các phương trình trên p-TsOH, DMSO và TCQ tương ứng là p-
toluenesulfonic acid, dimethyl sulfoxide và chloranil.
1) Viết công thức phân tử của A.
2) Vẽ cấu trúc của các hợp chất A, C, E, G, H, I, J trong chuỗi tổng hợp
trên.
3) Gọi tên hợp chất A và E.
4) Cho biết phản ứng tổng hợp C và A thuộc loại phản ứng gì?
5) Hãy cho biết mục đích của việc chuyển hoá F thành G ? Tại sao không
sử dụng HCl khô thay vì p-TsOH)?

302 | Bản quyền thuộc về Tạp chí Olympiad Hóa học KEM
Bài 48
Dưới đây là sơ đồ tổng hợp một hợp chất chống co giật E:

1) Xác định công thức cấu tạo các hợp chất A, B, C, D, E.


2) Hãy cho biết các đồng phân lập thể của E bằng việc dùng các kí hiệu
trước chữ E để cho biết cấu hình lập thể của từng đồng phân, giữa hai
đồng phân là đối quang của nhau thì dùng dấu gạch đứng.
3) Hãy vẽ một công thức cấu tạo của E và xác định rõ cấu hình tuyệt đối
tại các nguyên tử carbon bất đối xứng.

303 | Bản quyền thuộc về Tạp chí Olympiad Hóa học KEM
Bài 49
Để tổng hợp axit permetrinic (E), là một sản phẩm lí thú trong hóa học về
thuốc trừ sâu hại trong nông nghiệp, người ta thực hiện các phản ứng
theo sơ đồ sau:
H+ CH3C(OEt)3
a. 2-Metylbut-3-en-2-ol 3-Metylbut-2-en-1-ol A (C9H16O2)
(-EtOH)

Viết công thức cấu tạo của A và trình bày cơ chế của hai giai đoạn phản
ứng.

b.
A to CCl4 tBuONa KOH
B C D
3,3 FeCl3 (C6H6) EtOH Cl
HOOC
E Cl

Viết công thức cấu tạo của B, C, D và trình bày cơ chế phản ứng B → C và
C → D.
2. Hãy điều chế axit trans-crysanthemic (hình dưới) từ B (trong sơ đồ b. ở
trên) và các hóa chất tuỳ chọn.

HO

304 | Bản quyền thuộc về Tạp chí Olympiad Hóa học KEM
Bài 50
Khi cho 7-cacboximetoxi-6-hydroxi-3-sunfoquinolin (7-HOOCCH2O-6-HO-
3-HO3SC 9H4N, ký hiệu là M) phản ứng với dimetyl sunfat trong môi trường
kiềm, sau đó trung hòa thu được sản phẩm metyl hóa ở nguyên tử nitơ
của vòng quinolin (C 12H11NO7S, ký hiệu là N). Cho N phản ứng với dung
dịch nước của ankylamin R-NH2 ở nhiệt độ 90oC người ta thu được hợp
chất O mà nhóm cacboximetoxi của N bị thế thành nhóm R-NH. Trong
điều kiện phản ứng như vậy, nhóm cacboximetoxi ở hợp chất M không bị
thế.
a) Dùng công thức cấu tạo viết sơ đồ phản ứng tạo thành N. Đề nghị cơ
chế phản ứng để giải thích vì sao nguyên tử nitơ của vòng quinolin bị metyl
hóa còn nhóm HO-phenol không bị metyl hóa.
b) Sự thế nhóm cacboximetoxi của N thành nhóm R-NH là phản ứng thế
bình thường hay không bình thường? Giải thích
c) Đề nghị cơ chế phản ứng để giải thích sự tạo thành O từ N.

305 | Bản quyền thuộc về Tạp chí Olympiad Hóa học KEM
Bài 51
β – Lactam F được tổng hợp theo sơ đồ dưới đây:

a) Xác định công thức cấu tạo của các hợp chất A, B, D, E và F trong sơ đồ
trên.
b) Đề nghị cơ chế của phản ứng tạo thành F.

306 | Bản quyền thuộc về Tạp chí Olympiad Hóa học KEM
Bài 52
Xác định các chất trung gian và sản phẩm (A - E) trong chuỗi phản ứng
sau:

307 | Bản quyền thuộc về Tạp chí Olympiad Hóa học KEM
Bài 53
Caffein là một ancaloit thuộc nhóm purin, có tác dụng kích thích hệ thần
kinh và hệ tuần hoàn. Caffein có nhiều trong cà phê và chè. Trong công
nghiệp caffein được tổng hợp theo sơ đồ phản ứng:

Xác định công thức cấu tạo của các hợp chất P1, P2, P3, P4, P5, P6 trong
sơ đồ trên.

308 | Bản quyền thuộc về Tạp chí Olympiad Hóa học KEM
Bài 53
Vitamin A (còn gọi là retinol) là dẫn xuất của một polien mà các nối đôi ở
mạch chính có cấu hình E.

a) Từ β – ionon (Y) người ta có thể tổng hợp ra vitamin A. Chất Y được


tổng hợp từ 5-clopentan-2-on theo sơ đồ phản ứng sau:

Xác định công thức các chất Q1, Q2, Q3 và Q4 trong sơ đồ trên.
b) Từ Y và các hóa chất LDA, MeMgBr, DIBAL-H, MnO2, NaBH4, (EtO)2PO –
CH2 – CN, dung dịch axit loãng và các dung môi cần thiết, hãy đề nghị
sơ đồ tổng hợp vitamin A.
c) Hãy đề nghị cơ chế phản ứng giữa Q3 và (EtO)2PO – CH2 – CN (xúc tác
LDA).

309 | Bản quyền thuộc về Tạp chí Olympiad Hóa học KEM
Bài 54
Hợp chất B là -ionon, một trong những chất trung gian quan trọng để
tổng hợp nên vitamin A

Sơ đồ sau đây trình bày một phương pháp để tổng hợp -ionon:

1) Hãy tính độ bất bão hoà của vitamin A.


2) Hợp chất thơm C là một đồng phân của -ionon, C được tạo thành
bởi phản ứng hydrogen phân có xúc tác hợp chất thơm D. Phổ 1H NMR
của D chỉ cho một mũi đơn. Hãy vẽ cấu trúc của hợp chất C.
3) Hãy vẽ cấu tạo của chất trung gian B.
4) Sơ đồ trên nhằm tổng hợp chất trung gian B từ nguyên liệu ban đầu
là -ionon. Hãy cho biết các phản ứng (2), (3), (4), (6) thuộc loại phản
ứng nào (thế electrophile, thế nucleophile, thế gốc tự do, hydrogen
hoá…)

310 | Bản quyền thuộc về Tạp chí Olympiad Hóa học KEM
Bài 55
Hợp chất H tham gia vào thành phần của một số alkaloid. Chất H được
tổng hợp theo sơ đồ sau:

Vẽ công thức các hợp chất từ A đến H.

311 | Bản quyền thuộc về Tạp chí Olympiad Hóa học KEM
Bài 56
Các dẫn xuất của hợp chất C được sử dụng làm chất nhạy sáng trong pin
mặt trời màng mỏng. Hợp chất C được tổng hợp theo sơ đồ sau

Vẽ công thức cấu tạo của các hợp chất từ A đến C.

312 | Bản quyền thuộc về Tạp chí Olympiad Hóa học KEM
Bài 57
Hợp chất D được dùng làm thuốc giảm đau và được tổng hợp theo sơ đồ
sau:

Vẽ công thức cấu tạo của các chất từ A đến C Cho biết PBr3 là tác nhân
khử.

313 | Bản quyền thuộc về Tạp chí Olympiad Hóa học KEM
Bài 58
Xác định cấu trúc các chất trong sơ đồ sau:

314 | Bản quyền thuộc về Tạp chí Olympiad Hóa học KEM
Bài 59
Hợp chất F có tác dụng ức chế sự phát triển của tế bào ung thư và được
tổng hợp theo sơ đồ dưới đây.

Vẽ công thức cấu tạo của các chất từ A đến E.

315 | Bản quyền thuộc về Tạp chí Olympiad Hóa học KEM
Bài 60
Vẽ công thức cấu tạo của các hợp chất từ A đến C trong sơ đồ tổng hợp
thuốc zenarestat (C) sau đây:

316 | Bản quyền thuộc về Tạp chí Olympiad Hóa học KEM
Bài 61
Vẽ công thức cấu tạo của các chất từ A đến D trong sơ đồ chuyển hóa sau:

317 | Bản quyền thuộc về Tạp chí Olympiad Hóa học KEM
Bài 62
Xác định cấu trúc của các hợp chất A – D trong chuỗi phản ứng sau:

Gợi ý: Tổng hợp loại thuốc này có liên quan đến phản ứng sau đây:

318 | Bản quyền thuộc về Tạp chí Olympiad Hóa học KEM
Bài 63
Vẽ công thức cấu tạo của các hợp chất từ A đến C trong sơ đồ tổng hợp
sau:

319 | Bản quyền thuộc về Tạp chí Olympiad Hóa học KEM
Bài 64
Vẽ công thức cấu tạo của các chất từ A đến E trong dãy chuyển hóa sau.
Biết rằng chất đầu (X) hỗ biến trước khi phản ứng chuyển thành A.

320 | Bản quyền thuộc về Tạp chí Olympiad Hóa học KEM
Bài 65
F là hợp chất trung gian trong tổng hợp toàn phần ankaloid strychnin. Vẽ
công thức cấu tạo của các hợp chất từ A đến F trong sơ đồ sau:

321 | Bản quyền thuộc về Tạp chí Olympiad Hóa học KEM
Bài 66
Vẽ công thức cấu tạo của các hợp chất A, B trong dãy chuyển hóa sau:

322 | Bản quyền thuộc về Tạp chí Olympiad Hóa học KEM
Bài 67
Cho biết cấu trúc của các hợp chất từ E – I trong chuỗi phản ứng sau:

323 | Bản quyền thuộc về Tạp chí Olympiad Hóa học KEM
Bài 68
Hợp chất D là thuốc nổ thế hệ mới, được tổng hợp theo sơ đồ sau:

Vẽ công thức cấu tạo các hợp chất từ A đến D.

324 | Bản quyền thuộc về Tạp chí Olympiad Hóa học KEM
Bài 69
Hoàn thành sơ đồ tổng hợp Riboflavin (D) là thành phần cofactor trong
các phân tử sinh học sau đây:

325 | Bản quyền thuộc về Tạp chí Olympiad Hóa học KEM
Bài 70
Alloxan là chất oxy hóa mạnh. Khử alloxan thu được axit dialuric (I). Phản
ứng giữa alloxan và axit dialuric tạo ra alloxanthin. Mặt khác phản ứng
giữa alloxan và axit dialuric khi có mặt NH3 tạo thành chất nhuộm màu
tím murexide (K).

Vẽ công thức cấu tạo của các hợp chất từ I đến K.

326 | Bản quyền thuộc về Tạp chí Olympiad Hóa học KEM
Bài 71
Alloxan (C) cũng được điều chế theo sơ đồ sau:

Vẽ công thức cấu tạo của các hợp chất H và G.

327 | Bản quyền thuộc về Tạp chí Olympiad Hóa học KEM
Bài 72
Alloxan là chất ức chế quá trình tổng hợp insulin trong tế bào beta, gây ra
bệnh tiểu đường. Alloxan được tổng hợp theo sơ đồ sau:

Vẽ công thức cấu tạo của các hợp chất từ A đến D.

328 | Bản quyền thuộc về Tạp chí Olympiad Hóa học KEM
Bài 73
Xác định cấu trúc của các chất A, B, C, D và E trong sơ đồ chuyển hóa
terpenoit sau:

329 | Bản quyền thuộc về Tạp chí Olympiad Hóa học KEM
Bài 74
Gừng gió và thanh hao hoa vàng là hai cây thuốc Việt Nam. Từ gừng gió
người ta tách được Zerumbon [(2E, 6E, 10E)-2,6,9,9-
tetrametylxicloundeca-2,6,10-trien-1-on]. Từ thanh hao hoa vàng người
ta tách được Artemisinin. Zerumbon và artemisinin là hai tecpenoit đều
có hoạt tính chống ung thư. Thuốc lai X có cấu trúc chứa hai hợp phần
Zerumbon và Artemisinin có hoạt tính ưu việt hơn so với các thuốc đơn.
Quy trình tổng hợp thuốc lai X theo ba giai đoạn:
- Giai đoạn biến tính Artemisinin:

- Giai đoạn biến tính Zerumbon: Zerumbon phản ứng với m-CPBA trong
dung môi CH2Cl2 thu được chất N1. Cho N1 phản ứng với NH2OH.HCl
trong dung môi etanol thu được hỗn hợp gồm chất N2 và chất N3 (là
đồng phân hình học của nhau). Xử lý hỗn hợp trên với ZnCl2 trong dung
môi CH3CN và môi trường khí trơ. Loại bỏ dung môi, xử lý và tách sản
phẩm bằng sắc ký cột thu được hai chất N4 và N5 là đồng phân tỉ lệ
1:4 có công thức phân tử C15H23NO2.
- Giai đoạn tạo thuốc lai X: Cho N5 phản ứng với M2 (xúc tác bazơ) trong
dung môi CH3CN thu được thuốc lai X có công thức phân tử C32H49NO7.
Xác định công thức cấu tạo của các chất M1, M2, N1, N4, N5 và công thức
lập thể của N2 và N3.

330 | Bản quyền thuộc về Tạp chí Olympiad Hóa học KEM
Bài 75
Artemisinin, còn được gọi là Thanh hao tố (tiếng Trung: 青蒿素) và các
dẫn xuất của nó là một nhóm các loại thuốc có tác dụng chống lại bệnh
sốt rét Plasmodium falciparum. Phương pháp điều trị có chứa một dẫn
xuất của artemisinin (liệu pháp tổ hợp artemisinin, ACT) hiện đang là
phương pháp điều trị chuẩn trên toàn thế giới đối với P. falciparum.
Artemisinin được phân lập từ Artemisia annua - tức cây thanh hao hoa
vàng, một loại thảo dược được sử dụng trong y học cổ truyền Trung Quốc.
Ở trung quốc artemisinin còn được biết đến với tên gọi qinghao su.
Dưới đây là một sơ đồng tổng hợp qinghao su được công bố năm 1983

1) Có bao nhiêu nguyên tử carbon bất đối trong hợp chất qinghao su ?
Xác định cấu hình tuyệt đối tại các vị trí C2, C3, C4 và C5.
2) Hãy giải thích quá trình chuyển hoá từ D sang E trong sơ đồ trên.
3) Xác định cấu trúc các chất trung gian B, F và G.
4) Gọi tên theo danh pháp hợp chất trung gian A.

331 | Bản quyền thuộc về Tạp chí Olympiad Hóa học KEM
Bài 76
Artemisinin (Z) được nhà dược lí học người Trung Quốc Tu Yu được phát
hiện vào năm 1971 khi tìm kiếm các loại thuốc sốt rét trong chiến tranh
Việt Nam. Dựa vào công thức được mô tả trong một cuốn sách, viết vào
năm 340 Sau Công nguyên, bà đã tìm ra chất Z từ cây ngải tây 1 năm tuổi.
Năm 1973, bà thu được dihydroartemisinin (A), có hoạt tính chống sốt rét
mạnh hơn Z, và các dẫn xuất có hoạt tính khác. Nhờ những nghiên cứu
này, bà đã dành được giải Nobel Sinh lí học và Y khoa vào năm 2015. Ngày
nay, artemisinin và các dẫn xuất (artemether, artesunate) được dùng để
điều trị sốt rét trên toàn thế giới. Trong những năm gần đây, hoạt tính
chống ung thư của Z đã được phát hiện và đang được thử nghiệm.

артеметер = artemether; артесунат = artesunate


Dưới đây là sơ đồ tổng hợp Z từ artemisinic acid, có hàm lượng trong cây
ngải tây cao hơn nhiều so với artemisinin.

1) Hoàn thành sơ đồ chuyển hóa và xác định công thức các sản phẩm B
- E.
Hợp chất B cũng có thể nhận được từ các hợp chất thiên nhiên sẵn có -
isopulegol và citronellal - có trong các tinh dầu.

332 | Bản quyền thuộc về Tạp chí Olympiad Hóa học KEM
2) Hoàn thành sơ đồ chuyển hóa và xác định công thức các sản phẩm G
- S.

333 | Bản quyền thuộc về Tạp chí Olympiad Hóa học KEM
Bài 77
Cho phlorogluxinol (1,3,5-trihydroxibenzen) phản ứng với cloaxetonitrin
(ClCH2CN) khi có mặt ZnCl2 thu được chất A (C8H8NO3Cl). Sục khí HCl vào
dung dịch chứa chất A rồi đun hồi lưu thu được chất B. Đun hỗn hợp gồm
B, metyl iodua và K2CO3 trong DMF ỏ 80oC. Chiết sản phẩm bằng etyl
axetat rồi cất loại dung môi thu được chất C. Vừa khuấy vừa đun nhẹ ở
60oC trong 30 phút hỗn hợp gồm C và 3,4-dimetoxybenzandehit trong
metanol có cho thêm KOH thu được chất D (C 19H18O6).
a) Xác định công thức cấu tạo các chất A, B, C, D, Đề xuất cơ chế phản ứng
tạo thành A và B.
b) Xử lý D bằng H2O2 thu được chất E (C19H18O7). Khử chất E bằng H2/Pd-
C thu được chất F (C19H20O7). Xử lý chất F bằng K2CO3 thu được
flavonoit G. Xác định công thức cấu tạo các chất E, F và G.

334 | Bản quyền thuộc về Tạp chí Olympiad Hóa học KEM
Bài 78
Hisruten (một tecpen đa vòng) là trung gian trong sinh tổng hợp các hợp
chất thiên nhiên có cấu trúc và hoạt tính sinh học lí thú. Con đường tổng
hợp hóa học Hisruten theo sơ đồ sau:

Vẽ cấu tạo các chất A - G (không cần xét đến yếu tố lập thể).

335 | Bản quyền thuộc về Tạp chí Olympiad Hóa học KEM
Bài 79
Taxol được sử dụng để bào chế thuốc điều trị ung thư, đặc biệt là ung thư
vú. Taxol được tổng hợp theo con đường đi qua chất trung gian L chứa
vòng 6 cạnh. Chất L được tổng hợp theo sơ đồ sau:

Vẽ cấu tạo các chất H, I, K và L (không cần xét đến yếu tố lập thể). Cho
biết: TBSCl: t-butylđimetylsilyl clorua; H 2NNHSO2Ar: 2,4,6-
triisopropylbenzensulfonylhiđrazin.

336 | Bản quyền thuộc về Tạp chí Olympiad Hóa học KEM
Bài 80
Hợp chất dị vòng 2-methyldihydrofuran-3(2H)-one (sản phẩm I) là thành
phần chủ yếu tạo nên hương thơm cho cà phê, người ta đã đề nghị rất
nhiều phương pháp để tổng hợp nó. Dưới đây là một cách tổng hợp hợp
chất này từ nguyên liệu ban đầu là ester ethyl acetoacetate:

Trong bước cuối cùng của chuỗi tổng hợp trên, hợp chất H được tiến hành
chưng cất hơi nước, và cuối cùng người ta thu được sản phẩm I.

Gợi ý: Trong chuỗi tổng hợp trên có liên quan tới phản ứng sau:

1) Hãy xác định công thức cấu tạo của các chất E, F, G, J, K, L, M và N.
2) Cho biết mối liên hệ giữa hợp chất J và K (hoặc J và L), K và L?
3) Sản phẩm I thu được nhờ chưng cất hơi nước, nhưng hợp chất K, L,
M vẫn còn lại trong dung dịch lỏng ban đầu. Hãy giải thích tại sao
chúng không bị lôi cuốn theo hơi nước.

337 | Bản quyền thuộc về Tạp chí Olympiad Hóa học KEM
Bài 81
(-)-Preclavulone được tách ra từ một loài san hô sống ở Thái Bình Dương.
Preclavulone được tổng hợp theo phương pháp Corey như sau:

Vẽ cấu tạo các chất P1 - P10 (không cần xét đến yếu tố lập thể).

338 | Bản quyền thuộc về Tạp chí Olympiad Hóa học KEM
Bài 82
Quy trình tổng hợp chất K5 theo sơ đồ sau:

Cho biết, phản ứng Grubbs (sử dụng xúc tác là phức chất của Ru) là phản
ứng tiên tiến tổng hợp olefin từ hai phân tử olefin có nối đôi đầu mạch,
kèm theo sự giải phóng etilen. Vẽ cấu tạo các chất K1 - K5.

339 | Bản quyền thuộc về Tạp chí Olympiad Hóa học KEM
Bài 83
Viết công thức cấu tạo của các chất A và B trong sơ đồ điều chế nhựa
melamin sau:
NH2
NH3 CH2O
Xianogen clorua A N N B
H2N N NH2
Xianuramit (melamin)

340 | Bản quyền thuộc về Tạp chí Olympiad Hóa học KEM
Bài 84
Phức chất Q có khả năng phát quang và có hoạt tính sinh học. Q được
tổng hợp theo sơ đồ sau:

Vẽ cấu tạo các chất P, Q.

341 | Bản quyền thuộc về Tạp chí Olympiad Hóa học KEM
Bài 85
Cho sơ đồ chuyển hóa:

Viết công thức cấu tạo của các chất X1, X2, X3 và X4.

342 | Bản quyền thuộc về Tạp chí Olympiad Hóa học KEM
Bài 86
Xác định công thức cấu tạo các chất F, G, H, I theo chuỗi chuyển hoá sau:

343 | Bản quyền thuộc về Tạp chí Olympiad Hóa học KEM
Bài 87
Cho sơ đồ chuyển hoá sau:
F
CH3COCl NaH NaOC H
2 5
AlCl 3 C8H5Cl 2FO C11H9Cl 2FO3 HC(OC H ) C14H13Cl 2FO4
C2H5OCOOC2H5 2 5 3
Cl Cl A B C
H N N H
NH2 to H
+
C17H18FN3O3.
C15H14Cl 2FNO3 C15H13ClFNO3 C13H9ClFNO3
D E F G

1. Viết công thức cấu trúc của A, B, C, D, E, F và G.


2. Tại sao dùng NaH chuyển hoá A thành B nhưng lại dùng C 2H5ONa để
chuyển B thành C?
3. Quá trình chuyển hoá C thành D phải qua mấy giai đoạn?

344 | Bản quyền thuộc về Tạp chí Olympiad Hóa học KEM
Bài 88
Xicloocta-1,3,5,7-tetraen (COT) được tổng hợp lần đầu tiên vào năm
1911. Khi tương tác với kali, COT nhận thêm 2 electron theo kiểu cộng 1,4
và trở thành đianion với 2 nguyên tử cacbon mang điện tích âm, kí hiệu
là [COT]2-. Phương pháp nhiễu xạ tia X cho thấy COT có cấu trúc không
phẳng, khoảng cách giữa các nguyên tử cacbon cạnh nhau lần lượt là 1,33
và 1,46 Å.Trong khi đó, đianion [COT] 2- có cấu trúc phẳng, khoảng cách
giữa các nguyên tử cacbon cạnh nhau đều bằng 1,41 Å.
1.Hãy vẽ cấu trúc dạng ghế và cấu trúc dạng thuyền của COT. So sánh độ
bền của hai cấu trúc này và giải thích.
2.Vẽ công thức cấu tạo của [COT]2-. Vì sao nó có cấu trúc phẳng và có các
cạnh dài như nhau?
3. Hãy hoàn thành sơ đồ phản ứng tổng hợp COT từ hợp chất K dưới đây:
O NMe 2 Me2N
N Na/EtOH H2SO4
L M MeI N COT

K P Q R NMe2

345 | Bản quyền thuộc về Tạp chí Olympiad Hóa học KEM
Bài 89
Mặc dù cấu trúc khung của cyclohexane (C 6H12) thường được biểu diễn ở
dạng lúc giác đều nhưng thực chất thì nó là một phân tử tồn tại ở nhiều
dạng hình học khác nhau, gọi là các cấu dạng. Hai trong số các cấu dạng
đó là dạng ghế và thuyền xoắn (twist-boat), được biểu diễn dưới đây. Các
nguyên tử carbon trong vòng 6 cạnh được đánh số 1-6 để biểu diễn mối
liên hệ giữa chúng.

Dạng thuyền là cấu dạng có mức năng lượng thấp nhất của cyclohexane
với tất cả các góc liên kết gần như bằng góc liên kết lí tưởng (đối với
nguyên tử carbon tứ diện). Dạng thuyền xoắn có mức năng lượng cao
hơn.
a) Góc liên kết C-C-C lí tưởng trong dạng ghế của cyclohexane khác bao
nhiêu độ so với góc của lục giác đều.
Phân tử adamantane có thể được biểu diễn trực quan bằng cách thêm 4
nguyên tử carbon vào cấu dạng ghế của cyclohexane như dưới đây. Khi
đó, các vòng 6 cạnh khác được tạo thành. Trong adamantaen, tất cả các
vòng sáu dạng đều bị “khóa” theo cấu dạng ghế.

b) Với mỗi vòng 6 cạnh trong adamantane, hãy viết số thứ tự của 6
nguyên tử carbon trong vòng theo trật tự liên kết với nhau, bắt đầu
từ số nhỏ nhất. Ví dụ -1-2-3-4-5-6-
c) Có bao nhiêu tín hiệu trong phổ 13C NMR của adamantane (nghĩa là có
bao nhiêu loại carbon khác nhau)?
Phân tử twistane có thể được biểu diễn trực quan bằng cách thêm 4
nguyên tử carbon vào cấu dạng thuyền xoắn của cyclohexane như dưới
đây. Khi đó, các vòng 6 cạnh khác được tạo thành. Trong twistane, tất cả

346 | Bản quyền thuộc về Tạp chí Olympiad Hóa học KEM
các vòng sáu dạng đều bị “khóa” theo cấu dạng thuyền xoắn - đây chính
là nguồn gốc tên gọi của nó.

d) Với mỗi vòng 6 cạnh trong twistane, hãy viết số thứ tự của 6 nguyên
tử carbon trong vòng theo trật tự liên kết với nhau, bắt đầu từ số nhỏ
nhất. Ví dụ -1-2-3-4-5-6-
e) Có bao nhiêu tín hiệu trong phổ 13C NMR của twistane (nghĩa là có bao
nhiêu loại carbon khác nhau)?
f) Adamantane và twistane là các đồng phân của nhau. Hãy xác định
công thức phân tử.
Do tất cả các vòng đều ở cấu dạng ghế nên phân tử adamantane rất bền.
Nó được tìm thấy ở dạng thành phần của dầu thô vào năm 1933. Ngược
lại, twistane là một phân tử có sức căng lớn do các vòng đều ở cấu dạng
thuyền xoắn với năng lượng cao. Quy trình tổng hợp chất này phải được
thực hiện đặc biệt cẩn thận và được hoàn thành lần đầu vào năm 1962.
Dưới đây là sơ đồ tổng hợp:

347 | Bản quyền thuộc về Tạp chí Olympiad Hóa học KEM
Không có liên kết đơn C-C nào bị phá vỡ trong quy trình tổng hợp và do
đó những liên kết này có trong chất đầu vẫn được giữ nguyên.
g) Xác định cấu tạo các hợp chất A - H, anion J- và hợp chất K.
h) Trả lời các câu hỏi trắc nghiệm sau (trong Phiếu trả lời):
i. Twistane có bao nhiêu mặt phẳng đối xứng?
ii. Twistane có bao nhiêu trục quay đối xứng?
iii. Twistane có tính trùng ảnh gương không?

348 | Bản quyền thuộc về Tạp chí Olympiad Hóa học KEM
Bài 90
Longifolen là một sesquitecpen có trong thành phần nhựa thông và tinh
dầu của một số cây lá kim, được dùng trong công nghiệp hương liệu, mĩ
phẩm,... Chất Y được sử dụng để tổng hợp longifolen. Từ xiclopentađien,
chất Y được tổng hợp theo sơ đồ sau:
O O
O
o
1. Mg/ete, t OCOOCH2Ph
+HCl 2. CO2 1. Enamin X (C9H15NO) PhCH2OCOCl h
I K M
+ [2+2]
3. H3O 2. H3O+
4. SOCl2 L Y

- Xác định các chất I, K và M.


- Xác định công thức cấu tạo của enamin X, biết X chứa vòng 6 cạnh.

349 | Bản quyền thuộc về Tạp chí Olympiad Hóa học KEM
Bài 91
Adamantan (được đặt theo từ adamos trong tiếng Hy Lạp có nghĩa là
kim cương) được tổng hợp theo sơ đồ:

Hãy cho biết cấu trúc của các chất A, B, C, D.

350 | Bản quyền thuộc về Tạp chí Olympiad Hóa học KEM
Bài 92
Trong tháng 9/2017, Thủ tướng Anh, Theresa May, đã bị ho dữ dội khi
đang phát biểu tại Hội nghị Đảng Bảo thủ. Thuốc ức chế ho
dextromethorphan, được dùng trong các liệu pháp chữa ho với tên gọi
Benylin®, đã giúp ích cho bà. Câu hỏi này là về việc tổng hợp
dextromethorphan. Quy trình tổng hợp có chứa thông tin một số liên kết
mạnh và một số carbocation bền.

Dextromethorphan thường được dùng ở dạng muối hydrobromide


monohydrate.
a) Hãy khoanh tròn nguyên tố trong dextromethorphan bị proton hóa
khi ở dạng muối.
b) Xác định công thức phân tử dextromethorphan và tính khối lượng mol
của muối dextromethorphan hydrobromide monohydrate.
Quy trình tổng hợp dextromethorphan diễn ra theo nhiều bước. Chú ý
rằng trong sơ đồ mô tả quy trình tổng hợp, không phải lúc nào các sản
phẩm phụ cũng được chỉ rõ. Quy trình tổng hợp dextromethorphan bắt
đầu với việc tổng hợp hợp chất F.

351 | Bản quyền thuộc về Tạp chí Olympiad Hóa học KEM
Dịch nghĩa: compound = hợp chất; very board = (dải) rất rộng; sharp =
nhọn; molar mass = khối lượng mol.
c) Vẽ cấu trúc các hợp chất A-E và khí X.
Trong phản ứng tạo thành hợp chất D, cyanoacetic acid có thể bị deproton
hóa 2 lần bởi potassium hydroxide.

d) Vẽ cấu trúc anion Z- và dianion Z2-.


Quy trình tổng hợp tiếp tục với chuyển hóa hợp chất F thành hợp chất J.

e) Chỉ ra số thứ tự của 2 nguyên tử carbon trong hợp chất F liên kết với
nhau để tạo ra G.
f) Vẽ cấu trúc hợp chất H và cation I+ .
Quy trình tổng hợp tiếp tục với chuyển hóa của hợp chất J thành các hợp
chất M1 và M2 trong một giai đoạn. Đun nóng với phosphoric acid trong
nước, hợp chất J trước tiên chuyển thành hợp chất trung gian K và chất
lỏng không màu Y. Hợp chất trung gian K sau đó chuyển thành
carbocation L+ , tiểu phân này bị vòng hóa tạo thành hỗn hợp hai đồng
phân M1 và M2. Xử lí hỗn hợp với M1 và M2 với
trimethylphenylammonium chloride (một tác nhân methyl hóa) thu được
hỗn hợp levomethorphan và dextromethorphan. Cộng hợp D-tartaric acid
vão hỗn hợp này cho phép dextromethorphan mong muốn được tách ra
khỏi levomethorphan. Quá trình này được gọi là phân giải.

352 | Bản quyền thuộc về Tạp chí Olympiad Hóa học KEM
Trong chuyển hóa hợp chất J thành các hợp chất M1 và M2, có thể có
những cặp nguyên tử carbon khác nhau được liên kết với nhau.
g) Xác định tất cả các cặp nguyên tử carbon có thể liên kết với nhau trong
phản ứng này.
h) Vẽ cấu trúc hợp chất trung gian K, carbocation L+ và chất lỏng Y.
i) Xác định cấu trúc các đồng phân M1 và M2, biểu diễn đúng lập thể.
Xác định kiểu đồng phân của M1 và M2.

353 | Bản quyền thuộc về Tạp chí Olympiad Hóa học KEM
Bài 93
Hoàn thành chuỗi chuyển hoá dưới đây, hãy xác định rõ cấu hình lập thể
nếu có:

Biết rằng B là hỗn hợp của hai đồng phân hình học.

354 | Bản quyền thuộc về Tạp chí Olympiad Hóa học KEM
Bài 94
Cho sơ đồ chuyển hóa:

Viết công thức cấu tạo của các chất A, B, C, D và E. Biết B là một -lacton.

355 | Bản quyền thuộc về Tạp chí Olympiad Hóa học KEM
Bài 95
Ertiprotafib được sử dụng làm thuốc kháng tế bào ung thư. Sơ đồ tổng
hợp ertiprotafib như sau:

Xác định công thức cấu tạo của các chất A, B, C và D.

356 | Bản quyền thuộc về Tạp chí Olympiad Hóa học KEM
Bài 96
Bismaleimit tinh thể lỏng (BMI) được sử dụng trong công nghệ điện tử,
hàng không vũ trụ,… Một loại BMI được tổng hợp theo sơ đồ sau:

Biết X8 là

a) Xác định công thức cấu tạo của các chất X1, X2, X3, X4, X5, X6, X7 và BMI.
b) Viết sơ đồ tổng hợp X8 từ X3, octametylen dibromua và các chất cần thiết
khác.

357 | Bản quyền thuộc về Tạp chí Olympiad Hóa học KEM
Bài 97
Verapamile là một loại thuốc ngăn ngừa canxi được sử dụng để điều trị
cao huyết áp và rối loạn nhịp tim. Cấu trúc của Verapamile được trình bày
dưới đây, nó có một trung tâm lập thể.

1. Xác định cấu hình tuyệt đối của trung tâm lập thể này.
Verapamile có thể được tạo thành từ hai thành phần C và M. Sự tổng hợp
C và M được trình bày dưới đây. Trong cả hai quá trình, nguyên liệu ban
đầu là A, 3,4-dimetoxyaxetonitrin.
Sơ đồ phản ứng tổng hợp C là:

2. Vẽ công thức cấu tạo của B, C và một công thức có thể có cho xúc tác
a.
Để tổng hợp thành phần thứ hai (M), A chuyển hóa theo 2 bước tạo thành
methylester E:

H+/H2O b
A D
oder OH-/H2O
E
3. Vẽ công thức cấu tạo của hợp chất D. Vẽ công thức xúc tác b và đề
xuất 2 điều kiện cho phản ứng D tạo ra E.
E chuyển hóa theo 3 bước tạo axit racemic G. G chịu sự phân giải đối
quang để chọn lọc ra duy nhất đối quang H để sử dụng cho các chuyển
hóa tiếp theo.

358 | Bản quyền thuộc về Tạp chí Olympiad Hóa học KEM
4. Viết các công thức có thể có của xúc tác c, d, e, f , g và h.
Cho sơ đồ sau. Lưu ý J và K đều chứa nguyên tố boron.

5. Vẽ công thức cấu tạo của I, J, K, L, và M.


Với sự có mặt của Et3N M và C kết hợp với nhau để tạo nên sản phẩm
mong muốn, Verapamile.
6. Phản ứng giữa M và C tạo ra Verapamile thuộc loại phản ứng nào? Tại
sao phải thêm Et3N?

359 | Bản quyền thuộc về Tạp chí Olympiad Hóa học KEM
Bài 98
Pyrrolizidine-alkaloid và những dẫn xuất không có trong tự nhiên của
chúng đóng vai trò quan trọng trong hóa học hữu cơ và có hoạt tính sinh
học cao. Polyhydroxyl hóa pyrrolizidines mô phỏng cấu trúc của các loại
đường và ức chế các glycosidase khác nhau, một tính chất hữu dụng trong
việc điều trị bệnh tiểu đường, cúm và HIV.
Chúng ta trình bày một quy trình tổng hợp khả dụng cho dihydroxylated
pyrrolizidine alkaloide Platynecine, 1-hydroxy-7-hydroxymethyl-
pyrrolizidine:

1) Xác định cấu hình tuyệt đối của các trung tâm lập thể trong cấu trúc
trên.
Sự tổng hợp racemic Platynecine đồng nghĩa với việc dựng nên hai vòng
năm cạnh có một nguyên từ nitơ chung.
Sơ đồ phản ứng sau thể hiện sự hình thành vòng năm cạnh đầu tiên. Khởi
đầu với methylester của axit 4-iodo-butanoic (=ethyl-4-iodobutanoate) và
dibenzylamin (ký hiệu viết tắt Bn). (Chú ý: Q là một enamine vòng).

2) Xác định loại phản ứng của N với diethyl oxalate.


3) Vẽ một cơ chế khả thi cho bước đầu tiên của phản ứng với diethyl
oxalate.
4) Vẽ công thức cấu tạo của các hợp chất N, O, P, và Q.
Vòng năm cạnh thứ hai được tạo thành theo sơ đồ sau dẫn đến sự hình
thành sản phẩm mong muốn platynecine.

360 | Bản quyền thuộc về Tạp chí Olympiad Hóa học KEM
Chú ý: R là hợp chất có 1 vòng, T là hợp chất có 3 vòng, [S] không được
tách ra.
5) Vẽ công thức không gian của R, S, và T và đề xuất một tác nhân i.
6) Vẽ sản phẩm tạo thành trong phản ứng của R với bazơ NaH. Sử dụng
mũi tên cong để biểu thị các hướng tấn công dẫn đến sự hình thành
hợp chất có hai vòng.

361 | Bản quyền thuộc về Tạp chí Olympiad Hóa học KEM
Bài 99
Norfloxacine là một kháng sinh tổng hợp thể hiện hoạt động in vitro cao
chống lại vi khuẩn hiếu khí gram âm và gram dương. Norfloxacine thuộc
nhóm floroquinolon.

Các hợp chất thuộc nhóm kháng sinh quinolone luôn có nhóm axit
carboxyl ở vị trí 3 như cấu trúc dưới đây. Cách tổng hợp ngược cho thấy
nó được tạo thành từ aniline và một dẫn xuất của malonic ester.

Dẫn xuất của malonic ester thường kém bền; enolether của hợp chất này
(U) được sử dụng thay thế trong quá trình tổng hợp. Sự tổng hợp
Norfloxacine có thể được biểu diễn theo các bước trong sơ đồ sau:

1) Vẽ công thức cấu tạo của các hợp chất W, X, Y, và Z.


2) Phản ứng chuyển hóa V thành W thuộc loại nào?
U có thể được điều chế dễ dàng từ diethylmalonate và ethyl
orthoformate.
3) Viết và cân bằng phương trình cho phản ứng trên.

362 | Bản quyền thuộc về Tạp chí Olympiad Hóa học KEM
Bài 100
Vào năm 1967, bài hát nổi tiếng “Lucy in the Sky with Diamonds” đã được
phát hành bởi nhóm nhạc huyển thoại the Beatles. Cũng không mất nhiều
thời gian để các thanh niên thời đó nhận ra rằng tên bài hát viết tắt cho
chữ LSD. Chất này, là một trong những hợp chất hoá học được tổng hợp
rất sớm vào những năm 1938 bởi nhà hoá học người Thuỵ Sĩ Albert
Hofmann và cho tới tận thập niên 80 của thế kỉ trước, nó vẫn được sử
dụng làm thuốc an thần hoặc dùng cho mục đích nghiên cứu.
Và bây giờ, bạn sẽ có cơ hội để tổng hợp (dĩ nhiên trên giấy!) hợp chất có
hoạt tính sinh hoạt rất cao này. Chất này có khung sườn carbon gồm 4
vòng. Nếu ai đó cố gắng tiếp cận tổng hợp này từ những nguyên liệu ban
đầu rất đơn giản, thì một cấu trúc như thế sẽ là một thử thách khá khó,
do đó ở đây, bạn sẽ bắt đầu tổng hợp này từ hợp chất bicyclic indole-3-
propionic acid.

363 | Bản quyền thuộc về Tạp chí Olympiad Hóa học KEM
Gợi ý: Hợp chất 1 có ba vòng, hợp chất 7 có 4 vòng, hợp chất 9 có một
tâm lập thể.
1) Hãy vẽ cấu trúc của tất cả các hợp chất từ 1-11 (lưu ý về các đồng
phân lập thể nếu có).
2) Đề nghị cơ chế cho phản ứng 8 + TsCH2NC tạo thành 9.
Theo quan điểm sinh hoá, lysergic acid diethylamideliên kết với thụ thể
serotonin trong cơ thể, nhưng hiệu ứng sinh ảo giác kéo dài từ 6 đến 8
tiếng, có nghĩa là sự chuyển hoá của nó xảy ra rất nhanh. Các nghiên cứu
chỉ ra rằng chuyển hoá trong các cơ quan trong cơ thể người diễn tiến
thông qua trung gian O-H-LSD. Cơ chế tạo thành O-H-LSD được đề nghị
như sau, nhưng một số thông tin còn thiếu:

364 | Bản quyền thuộc về Tạp chí Olympiad Hóa học KEM
3) Xác định cấu trúc của X và Y, đồng thời đề nghị enzyme (ENZ) cho giai
đoạn cuối cùng. Biết rằng ENZ thuộc nhóm các enzyme
oxidoreductases. Lưu ý rằng không cần nêu đích danh enzyme (chẳng
hạn câu trả lời là deaminase hay oxidase là chấp nhận được)
Các nhà sinh hoá đã chỉ ra rằng LSD và tiền chất của nó ergotamine ERG
liên kết cùng thụ thể serotonin với vị trí hoạt động của nó.
4) Dựa trên dữ kiện rằng LSD và ERG liên kết cùng một vị trí thụ thể như
nhau, hãy chỉ ra phần mang hoạt tính sinh lý trong mỗi phân tử
5) Hãy đặt phân tử LSD vào vị trí hoạt hoá của thụ thể serotonin và biểu
diễn các tương tác phân tử.

365 | Bản quyền thuộc về Tạp chí Olympiad Hóa học KEM
366 | Bản quyền thuộc về Tạp chí Olympiad Hóa học KEM
Bài 100
Carbofuran là một trong những thuốc trừ sâu có độc tính cao nhất, và vì
thế nó đã bị cấm sử dụng ở Canada và Châu Âu. Tuy nhiên, loại thuốc trừ
sâu này vẫn được dùng rộng rãi ở các quốc gia khác để kiểm soát lượng
côn trùng trong khoai tây, bí, hoa hướng dương.
1) Carbofuran có thể được sản xuất rất dễ dàng từ các chất hoá học đơn
giản, nhưng nhược điểm của các quá trình này là giá thành sản xuất
và độc tính của các chất hoá học được sử dụng. Mặc dù vậy. trong
thập niên 1980, nhiều phương pháp thay thế khác để điều chế
carbofuran đã ra đời, chúng tương đối an toàn và giá thành nguyên
liệu ban đầu rẻ hơn (acetone trong trường hợp này). Sơ đồ tổng hợp
carbofuran, một hợp chất bicyclic được minh hoạ như hình sau đây.
Vẽ cấu trúc của các hợp chất A và B và cơ chế của phản ứng từ A sang
B.
2) Vẽ cấu trúc của hợp chất C và D (D là một hỗn hợp các đồng phân). Vẽ
tất cả các đồng phân có thể có của D, có thể được tạo thành từ C →
D, đánh dấu tất cả các tâm lập thể bằng dấu * và xác định rõ cấu hình
của chúng (R/S)
3) Vẽ cấu trúc của hợp chất E. Biết rằng phản ứng D → E được tiến hành
nhờ sử dụng một hợp chất đối quang tinh khiết D’ (là một trong các
đồng phân lập thể của D) với tất cả các tâm lập thể đều có cấu hình R.
Vẽ (các) đồng phân lập thể được tổng hợp thông qua phản ứng D →
E.
4) Vẽ cấu trúc các hợp chất F, G, H và I
5) Đề nghị cơ chế cho phản ứng từ G → H

367 | Bản quyền thuộc về Tạp chí Olympiad Hóa học KEM
Khi đun nóng hợp chất E với Pd/alumina trong môi trường khí nitrogen
thì tạo thành chất trung gian I. Chất này nếu tiếp tục được đun nóng trong
cùng điều kiện sẽ tạo thành một hợp chất J bền hơn và sau đó thu được
hợp chất bicyclic K. Những chất trung gian này không được phân lập trong
phản ứng nhưng được đun nóng để cuối cùng tạo thành hợp chất L.
6) Vẽ cấu trúc của các hợp chất J, K và L.
7) Hãy viết tên theo danh pháp hệ thống của hợp chất J sử dụng danh
pháp IUPAC.
8) Trong phản ứng tạo chuyển hoá J → K, một vòng mới được tạo thành.
Hãy cho biết tên của vòng này?
A. oxyfurane

368 | Bản quyền thuộc về Tạp chí Olympiad Hóa học KEM
B. dehydrofurane
C. benzofurane
D. dihydrofurane
E. ketofurane
9) Giai đoạn cuối cùng cho ta sản phẩm mong muốn, carbofuran. Hãy vẽ
cấu trúc của carbofuran này.

369 | Bản quyền thuộc về Tạp chí Olympiad Hóa học KEM
Bài 101
Từ lâu, thuốc Ritalin® dã được sử dụng để điều trị chứng rối loạn tăng
động giảm chú ý (thường gọi là ADHD). Gần dây, đã có thông tin về việc
về việc các sinh viên sử dụng dược chất này làm thuốc kích thích khi ôn
thi. Dưới đây là công thức cấu tạo của Ritalin, trong đó R là một gốc
hydrocarbon.

Ritalin được tổng hợp theo sơ đồ sau. Trong sơ đồ, một vài tần số đặc
trưng cho dao động liên kết trong phổ IR được ghi kèm với các hợp chất
trung gian.

Sơ đồ tổng hợp đi từ phản ứng của benzyl chloride với sodium cyanide,
tạo thành hợp chất A.
a) Xác định công thức cấu tạo của A.
Deproton hóa (tách H+ ) chất A, thu được anion B-.
b) Xác định công thức cấu tạo của B-.

370 | Bản quyền thuộc về Tạp chí Olympiad Hóa học KEM
c) Xác định công thức cấu tạo các chất C, D, E, F. Không cần xác định gốc
hydrocarbon.
Trong thuốc viên, Ritalin tồn tại ở dạng muối hydrochloride (muối HCl).
d) Khoanh tròn nguyên tử bị proton hóa để tạo thành muối
hydrochloride.
e)
i. Trong mỗi viên thuốc chứa 10,00 mg muối hydrochloride tương
đương với 8,647 mg Ritalic. Sử dụng thông tin này để xác định khối
lượng mol của Ritalic.
ii. Xác định công thức gốc hydrocarbon R.
f) Xác định các nhóm chức tương ứng với các giá trị tần số dao động
trong phổ IR của các chất trong sơ đồ. Dùng kí hiệu mũi tên để chỉ rõ
liên kết dao động.

Đồng phân quang học của một phân tử là các đồng phân có trật tự sắp
xếp các nguyên tử giống nhau nhưng khác nhau về cấu trúc không gian ba
chiều. Dược tính của một loại thuốc phụ thuộc vào cấu trúc không gian
của phân tử.
Sơ đồ tổng hợp trên có thể tạo ra hỗn hợp 4 đồng phân của Ritalin như
dưới đây. Một số đồng phân có hoạt tính cao hơn các đồng phân khác.

g) Hãy cho biết những đồng phân nào là đối quang của nhau.
Một số đồng phân lập thể có hoạt tính yếu có thể được chuyển thành
dạng có hoạt tính mạnh hơn bằng cách deproton hóa Ritalin bằng alkoxide
base OR-, tạo ra anion G-. Sau đó, proton hóa lại để tạo ra đồng phân lập
thể khác.

371 | Bản quyền thuộc về Tạp chí Olympiad Hóa học KEM
h) Xác định công thức cấu tạo anion G-.
i) Các đồng phân nào có thể chuyển hóa lẫn nhau qua trạng thái trung
gian anion G-.
Cặp đồng Có thể chuyển Không thể chuyển
phân hóa qua anion G- hóa qua anion G-
1 và 2
1 và 3
1 và 4
2 và 3
2 và 4
3 và 4

372 | Bản quyền thuộc về Tạp chí Olympiad Hóa học KEM
Bài 102
Có rất nhiều hợp chất chứa nitrogen được tìm thấy trong thế giới động-
thực vật, điển hình là: alkaloid, nucleic acid, vitamin, … và các hợp chất
này đều có ảnh hưởng sinh lý.
Trung tâm lập thể có thể là nguyên tử hoặc nhóm nguyên tử, trong đó sự
thay đổi vị trí của 2 nhóm bất kì tạo thành một đồng phân lập thể khác.
Các hợp chất chứa nitrogen cũng có thể có cấu trúc bất đối và có tính
quang hoạt.
Cho ba hợp chất: (I): không quang hoạt; (II): quang hoạt nhưng không tách
riêng các đối quang được; (III): quang hoạt và có thể tách riêng các đối
quang.

1) Hãy cho biết I, II, III ứng với những hợp chất nào ở trên.
Alkaloid là nhóm hợp chất chứa nitrogen có tính base, có nguồn gốc thực
vật. (+)-Muscarine là một alkaloid có độc tính, được tìm thấy trong một
số loại nấm.

2) Vẽ và xác định cấu hình tuyệt đối của đối quang (+)-Muscarine.
Trong thuốc phiện có nhiều alkaloid, một trong số đó là morphine - có thể
dùng làm thuốc giảm đau.

373 | Bản quyền thuộc về Tạp chí Olympiad Hóa học KEM
3) Trong morphine có bao nhiêu tâm bất đối?
Phản ứng tách Hoffmann là một phương pháp để xác định cấu trúc
alkaloid. Phản ứng này gồm giai đoạn tạo ra muối ammonium bậc IV rồi
tách nitrogen để tạo thành alkene. Phản ứng tổng quát được mô tả như
sau:

Một dẫn xuất diacetyl của morphine là heroin, cũng được dùng làm thuốc
gây mê. Thực hiện phản ứng tách Hoffmann với heroin, thu được sản
phẩm A.
4) Tính số mol methyl iodide cần để tách hoàn toàn nitrogen khỏi heroin.
5) Xác định công thức cấu tạo của A.
Cho A phản ứng với bromine dư.
6) Mỗi phân tử A phản ứng được với bao nhiêu phân tử bromine?
Một nhóm alkaloid chứa nitrogen thú vị khác là Indole alkaloid - trong
phân tử có chứa vòng indole.

Indole và dẫn xuất của nó được tổng hợp bằng phương pháp Fischer
Indole, trong đó có một giai đoạn thú vị là sự chuyển vị (với xúc tác acid)
của arylhydrazone như sau:

374 | Bản quyền thuộc về Tạp chí Olympiad Hóa học KEM
Dẫn xuất Indole sau cũng được tổng hợp bằng phương pháp Fischer, đi
từ hợp chất carbonyl B và dẫn xuất thế phenyl hydrazine C.

7) Xác định công thức cấu tạo của B, C.


Muối arene diazonium là các hợp chất trung gian quan trọng, có thể bị
khử trực tiếp thành aryl hydrazine hoặc được dùng để tổng hợp các aryl
hydrazone. Các muối này cũng được dùng để tổng hợp thuốc nhuộm azo,
trong trường hợp này chúng đóng vai trò tác nhân electrophile.
8) Vẽ các cấu trúc cộng hưởng quan trọng nhất của benzene diazonium
ion.
Muối arene diazonium ghép cặp được với các hợp chất béo có chứa
nguyên tử carbon có tính acid (hợp chất chứa nhóm methylene hoạt
động) tạo thành dẫn xuất azo. Hợp chất azo bị hỗ biến tạo thành dẫn xuất
hydrazo. Nếu một hợp chất không thể hỗ biến thì nó sẽ loại bỏ (chuyển
vị) một nhóm chức phù hợp ở vị trí tạo nối đôi để cho phép xảy ra sự hỗ
biến).
9) Hoàn thành phương trình phản ứng sau:

375 | Bản quyền thuộc về Tạp chí Olympiad Hóa học KEM
Alkaloid J là cofactor (nhóm ngoại) oxid hóa-khử của enzyme alcohol
dehydrogenase (tách hydrogen khỏi alcohol) và đóng vai trò quan trọng
trong việc truyền tín hiệu và giao lưu thông tin tế bào.

I là hợp chất trung gian quan trọng trong quá trình tổng hợp J. Quá trình
tổng hợp này có sự hình thành một hydrazone.
10) Xác định các chất còn thiếu trong sơ đồ phản ứng sau.

376 | Bản quyền thuộc về Tạp chí Olympiad Hóa học KEM
Từ hợp chất I, có thể tổng hợp K là tiền chất của J.

11) Xác định nguyên tử có tính base mạnh nhất của K.


Nhiều bằng chứng cho thấy các hợp chất thiên nhiên chứa vòng imidazole
có hoạt tính sinh lí. Do vậy, đã có nhiều hợp chất như vậy được tổng hợp
nhân tạo. Một trong số đó là dược chất Pentostatin - dùng làm hoạt chất
chống virus và kháng u. Hợp chất O (C 12H10N4O5) là hợp chất trung gian
quan trọng trong quá trình tổng hợp Pentostatin.

377 | Bản quyền thuộc về Tạp chí Olympiad Hóa học KEM
12) Xác định công thức cấu tạo các chất còn thiếu trong sơ đồ phản ứng
sau.

378 | Bản quyền thuộc về Tạp chí Olympiad Hóa học KEM
Bài 103
Khi đi qua tiệm làm bánh, bạn có thể sẽ dừng bước bởi hương vị thơm
ngon của những chiếc bánh mì mới ra lò. Một người hùng trong tiểu
thuyết đã từng nói trong tình huống tương tự: “Nếu cậu nói với tôi rằng
nó không hoàn hảo, thì cậu là kẻ thù của tôi mãi mãi.” Thành phần tạo
nên mùi hương chủ đạo của bánh mì được phát hiện vào năm 1969, đó là
hợp chất X, tồn tại trong một cân bằng với dạng tautomer (hỗ biến) Y của
nó theo tỉ lệ 2:1. Đáng tiếc, cả hai dạng đều không bền, và sau vài giờ thì
bánh mì không còn mùi vị ngon lành như vậy nữa.
Năm 1993, hỗn hợp tautomer của X và Y đã được tổng hợp từ piperidine
theo chuỗi phản ứng trong Sơ đồ 1. Đáng chú ý là ban đầu tỉ lệ của X và Y
là 1:4, rồi tỉ lệ này thay đổi dần dần thành một cân bằng.
Sơ đồ 1.

Hợp chất B đặc trưng bởi 1 trục đối xứng bậc ba (nghĩa là sự quay một
góc 120o tạo thành một phân tử giống hệt chất đầu), tồn tại trong một
cân bằng với đồng phân dia C của nó. Phản ứng chuyển hóa qua lại giữa
hai dạng này diễn ra qua một tiểu phân trung gian A, cũng là tiểu phân
trung gian trong phản ứng tạo thành B và C, cũng như sự chuyển hóa
chúng thành D. Các hợp chất A, B, C có thành phần nguyên tố giống nhau:
C = 72.24%, H = 10.91%, N = 16.85%.
1) Xác định công thức cấu tạo các hợp chất A-E, X, Y.
Xử lí hợp chất E với phức CH3Li.LiBr trong (C 2H5)2O ở 0 oC không tạo thành
các sản phẩm mong muốn X, Y. Thay vào đó, ban đầu tạo thành kết tủa
vàng F. Khi xử lí với nước, kết tủa này chuyển thành hỗn hợp của chất C
và dạng tautomer G của nó.
2) Xác định công thức cấu tạo của các hợp chất F và G.
Một hướng khác để tổng hợp chất X là sử dụng dẫn xuất H của pipecolinic
acid theo Sơ đồ 2 dưới đây.

379 | Bản quyền thuộc về Tạp chí Olympiad Hóa học KEM
Sơ đồ 2.

3) Xác định công thức cấu tạo của các chất I và J.

380 | Bản quyền thuộc về Tạp chí Olympiad Hóa học KEM
Bài 104
Ở Azerbaijan, nơi rất nổi tiếng vì những loại rau quả, thì quả lựu được gọi
là “vua trái cây” . Với nhiều tôn giáo, quả lựu còn được tôn vinh như là
“trái cây Thiên đường”, biểu tượng của công bằng, sung túc, hi vọng về
một cuộc sống trường tồn.
Năm 1878, alkaloid pelletierine đã được cô lập từ vỏ cây lựu
(Punica granatum L., Lythraceae). Alkaloid này được sử
dụng trong dân gian làm thuốc trị mụn. Ban đầu, pelletierine
bị nhầm với XW (3-(piperidin-2-yl)propanal). Nhưng ngày
nay, giới khoa học đã xác nhận rằng pelletierine tự nhiên là
(S)-1-(piperidin-2-yl)propan-2-one (XS).
1) Xác định công thức cấu tạo của XW và XS. Chất thứ hai phải đúng hóa
lập thể.
Quy trình tổng hợp pelletierine nhân tạo (XS) dựa trên sự chuyển hóa
nortropanol A đã được xây dựng gần đây.

2) Xác định công thức cấu tạo của các chất B-G, kèm theo hóa lập thể.
3) Nortropanol A đã được dùng trong phản ứng này dưới dạng đối quang
tinh khiết. Hợp chất A có thể tồn tại bao nhiêu đồng phân lập thể (gồm
cả A và bỏ qua tính quang hoạt của nitrogen)?
Đối quang của XS đã được tổng hợp từ tert-butanesulfinamide (H):

381 | Bản quyền thuộc về Tạp chí Olympiad Hóa học KEM
4) Xác định công thức cấu tạo của các chất I-L, kèm theo hóa lập thể.

382 | Bản quyền thuộc về Tạp chí Olympiad Hóa học KEM
Bài 105
Công ty Áo Sandoz GmbH với nhà máy ở Tirol là một trong những nhà sản
xuất kháng sinh, và penicillin nói riêng, lớn nhất ở phương Tây. Công ty
chiếm khoảng 2/3 thị phần trên toàn thế giới của penicillin V, một dạng
acid penicllin bền được phát hiện bởi hai nhà khoa học từ Kundl vào năm
1951.
Penicillin V, cùng với ba loại kháng sinh khác, đã góp một phần nhỏ vào
những hiểu biết về các hướng tổng hợp trong hoá hữu cơ.
A. Prontosil

Gợi ý: Trong khối phổ, B có các tín hiệu ở m/z = 233 và m/z = 235 với tỉ lệ
3:1.
1) Xác định công thức cấu tạo của các hợp chất B, C, D, E, F và benzene-
1,3-diamine.
2) Vì sao giai đoạn E → F không thể xảy ra nếu dùng benzene thay cho
benzene-1,3-diamine.
3) F có loại đồng phân lập thể nào?

383 | Bản quyền thuộc về Tạp chí Olympiad Hóa học KEM
B. Chloramphenicol

4) Xác định công thức cấu tạo của các hợp chất A, B và cấu trúc không
gian của các hợp chất C, E.
5) Xác định công thức cấu tạo các tiểu phân hoạt động trong các phản
ứng A → B và D → E. Cho biết tên gọi các cơ chế tương ứng.
6) Xác định tên gọi hợp chất C theo danh pháp IUPAC.
C. Trimethoprim

Gợi ý:

384 | Bản quyền thuộc về Tạp chí Olympiad Hóa học KEM
● Tên gọi chất B là 3,4,5-trihydroxybenzenecarboxylic acid methyl
ester.
● F là hỗn hợp đồng phân.
7) Xác định công thức cấu tạo các chất B, C, D, E và công thức phân tử
của X.
Guanidine (tác nhân chuyển E → F) có tính base tương đối mạnh, pKB =
0.35.
8) Tính chất này gây ra bởi hiệu ứng nào trong cấu trúc guanidine được
proton hoá? Vẽ ít nhất hai cấu trúc để biểu diễn hiệu ứng này.
D. Penicillin V

9) Xác định cấu trúc không gian của các hợp chất A, B.
10) Trình bày cơ chế phản ứng A → B. Chỉ vẽ những phần của phân tử
tham gia phản ứng, phần còn lại thì kí hiệu là R.
11) Nhóm chức tạo thành trong phản ứng A → B là gì?

385 | Bản quyền thuộc về Tạp chí Olympiad Hóa học KEM
Bài 106
Artemisinin là một loại thuốc có tác dụng nhanh nhất trong tất cả các loại
thuốc trị bệnh sốt rét. Nó được tìm ra vào năm 1970 bởi Tu Youyou, một
nhà khoa học người Trung Quốc, chính nhờ phát hiện này mà bà được
trao giải thưởng Nobel vào năm 2015, bà cũng chính là người Trung Quốc
đầu tiên nhận được giải thưởng danh giá này. Ban đầu, artemisinin được
chiết xuất từ một loài thảo dược có tên gọi là Artemisa Annua. Tuy nhiên
vì số lượng loài này rất ít nên vào năm 1992 Avery và các cộng sử đã công
bố một con đường tổng hợp Artemisinin như hình dưới đây:

Biết rằng:
● Công thức phân tử của các hợp chất lần lượt là B (C 13H16O2S),
D (C 16H28O3), G (C 22H40O4Si).

386 | Bản quyền thuộc về Tạp chí Olympiad Hóa học KEM
● Tất cả các bước từ B đến I đều có tính chọn lọc lập thể (riêng
phản ứng tạo thành A và B thì không), vì thế hãy luu ý vẽ các
đồng phân đối quang chính của tất cả các chất trung gian từ C
đến I.
● LDA và LDEA là những base cồng kềnh.
● Chuyển hoá G thành H là phản ứng chuyển vị Claisen.
a) Hãy vẽ cấu trúc các hợp chất từ A đến E, lưu ý đến hoá học lập thể của
các hợp chất C, D và E.
b) Đề nghị cơ chế cho chuyển hoá từ A tạo thành B.
c) Đề nghị cơ chế cho sự chuyển hoá từ E tạo thành F. Lưu ý rằng, DMF
không phải là dung môi của phản ứng và không có sự racemic hoá diễn
ra.
d) Động lực chính thúc đẩy phản ứng từ E đến F là gì ?
e) Vẽ cấu trúc của các hợp chất G và H, lưu ý đến hoá học lập thể.
f) Đề nghị cơ chế cho sự chuyển hoá từ G tạo thành H.
g) Xác định cấu hình tuyệt đối (R hay S) của tất cả các tâm lập thể của
hợp chất G và I.

387 | Bản quyền thuộc về Tạp chí Olympiad Hóa học KEM
Bài 107
Artemisinin là một dược chất được chiết xuất bởi các nhà khoa học Trung
Quốc vào năm 1972 từ cây thanh hao hoa vàng (Artemisia annua) và cấu
trúc của nó đã được khẳng định lại vào năm 1976.
Theo một bài báo đăng đăng trên tạp chí Nature của Anh vào tháng 8 năm
2003, artemisinin tác dụng trực tiếp lên màng tế bào tế bào ký sinh trùng
sốt rét và có tính kháng khuẩn. Vì thời gian bán huỹ ngắn nên ký sinh trùng
sốt rét không có khả năng kháng thuốc. Thêm vào đó, arteminisin không
có các tác dụng phụ của thuốc chống sốt rét quinine. Năm 2004 Artermisin
đã được trao giải thưởng cao quý ở Thái Lan để ghi nhận những hiệu quả
mang lại của thuốc này. Cấu trúc của Artemisin được cho trong hình dưới
đây:

a) Cho biết các nhóm chức có trong phân tử artemisinin.


b) Phân tử artemisinin có bao nhiêu nguyên tử carbon bất đối xứng?

388 | Bản quyền thuộc về Tạp chí Olympiad Hóa học KEM
c) Tổng hợp toàn phần lần đầu tiên artemisini được hoàn thành bởi các
nhà hoá học từ Trung Quốc, được thể hiện theo sơ đồ dưới đây. Hãy
hoàn thành các tác chất a, b, c, d, e, f, g, h, i, j còn thiếu.

389 | Bản quyền thuộc về Tạp chí Olympiad Hóa học KEM
Bài 108
Vào năm 2015, dược chất Addyi (có tên Hóa học là Flibanserin) được công
bố là loại thuốc đầu tiên được chấp thuận trong điều trị rối loạn (giảm
ham muốn) tình dục của nữ giới. Dưới đây là công thức cấu tạo của Adyyi:

Hai tiền chất của Flibanserin, A và C, được điều chế theo các sơ đồ sau:

a) Xác định công thức thực nghiệm của A.


b) Xác định công thức cấu tạo của A, B, C.
Phần còn lại của quá trình tổng hợp được bắt đầu với chất Z như sau:

c) Xác định công thức cấu tạo các hợp chất D, E, F, G và sản phẩm phụ H.

390 | Bản quyền thuộc về Tạp chí Olympiad Hóa học KEM
Khi mới được tổng hợp, Flibanserin tồn tại ở dạng muối monoprotic mang
1 điện dương.
d) Hãy khoanh tròn nguyên tử bị proton hóa trong cấu trúc Flibanserin,

Hợp chất Z được tổng hợp từ 1,2-diaminobenzene và ethyl acetoacetate


theo sơ đồ sau:

e) Xác định công thức cấu tạo X, Y.

391 | Bản quyền thuộc về Tạp chí Olympiad Hóa học KEM
Bài 109
Câu hỏi này liên quan đến quy trình tổng hợp dolasetron mesylate, Z, một
loại thuốc được bán ra với tên thương mại Anzemet, dùng để chữa chứng
buồn nôn và ói mửa sau phẫu thuật.

Quy trình tổng hợp được cho dưới đây:

Giai đoạn đầu tiên tạo thành hợp chất A chỉ chứa C, H, O. Hợp chất G
không quang hoạt và có thể được điều chế trực tiếp từ D bằng phản ứng
với ozone, sau đó khử; hoặc qua các đồng phân lập thể E1 và E2 với OsO4
– hoặc qua các đồng phân lập thể F1 và F2 – rồi khử hoá với peracid như
trong hình.

392 | Bản quyền thuộc về Tạp chí Olympiad Hóa học KEM
1) Xác định công thức thực nghiệm của G từ dữ kiện % cho ở trên.
2) Xác định công thức cấu tạo của A, B, C, D, E1, E2, F1, F2 và G.
Hợp chất G được dùng cho giai đoạn tiếp theo của quy trình tổng hợp,
trong môi trường đệm, tạo thành H (là hỗn hợp của 2 đồng phân dia
không quang hoạt). Khử hoá H bằng NaBH4 tạo thành alcohol I (là hỗn hợp
của 4 đồng phân dia không quang hoạt). I phản ứng với dihydropyran
được acid hoá, tạo thành J (là hỗn hợp của nu đồng phân hơn nữa). Sau
đó, J phản ứng với t-butoxide base, rồi đun hồi lưu với acid, sau đó chiết
trong môi trường base yếu, tạo thành K, là hỗn hợp của 2 đồng phân dia:
K1 (sản phẩm chính) và K2 (sản phẩm phụ). Các chất này có thể tách riêng,
và K1 được dùng trong giai đoạn cuối của quy trình tổng hợp.

3)
a) Xác định cấu trúc của H, I, J. Không cần biểu diễn các đồng phân
dia được tạo thành khác.
b) Xác định cấu trúc của các đồng phân dia K1 và K2.
Trong giai đoạn cuối của quy trình tổng hợp, L và M phản ứng với nhau
tạo thành hợp chất trung gian N. Sau đó N phản ứng với K1, sau đó chiết,
thu được amine trung hoà, chất này tạo thành sản phẩm cuối khi được
proton hoá với CH3SO3H.

393 | Bản quyền thuộc về Tạp chí Olympiad Hóa học KEM
4) Xác định cấu trúc của N.

394 | Bản quyền thuộc về Tạp chí Olympiad Hóa học KEM
Bài 110
Mặc dù hiện nay vẫn chưa thể chữa dứt điểm bệnh Alzheimer nhưng đã
có sẵn nhiều loại thuốc để kiểm soát rối loạn thoái hoá thần kinh. Trong
số đó có các chất ức chế acetylcholinesterase, mà galantamine (1) là một
ví dụ điển hình. Phân tử này có thể được phân lập từ cây giọt tuyết
Caucasian, một loại thực vật bản địa ở Georgia; tuy nhiên, cần phải đi theo
hướng tổng hợp thì mới có lượng chất lớn để phục vụ cho điều trị. Dưới
đây là một hướng tổng hợp công nghiệp galantamine:

Một số lưu ý trong quy trình tổng hợp này:


● Phổ 1H NMR cho thấy A có hai proton thơm ở vị trí para.
● C không bền trong các điều kiện có nước, do vậy nó không được
phân lập mà thay vào đó sẽ phản ứng ngay với NaBH4 để chuyển
thành D.

395 | Bản quyền thuộc về Tạp chí Olympiad Hóa học KEM
1) Đề xuất cấu trúc của A, B, C, D, F và G. Không phản ứng nào, ngoại trừ
chuyển hoá cuối cùng với L-selectride, có chọn lọc lập thể. Tuy nhiên,
trong câu trả lời không cần xác định hoá lập thể.
2) Xác định công thức có thể có của tác nhân X, để chuyển hợp chất D
thành E.
Độ quay quang của chất thu được bằng phương pháp giải quang là -
400o cm2 g -1, trong khi đó giá trị của hợp chất tinh khiết quang học là
-415o cm2 g-1 (đo trong cùng điều kiện). Giả sử rằng tạp chất có tính
quang hoạt duy nhất là đối quang khác.
Một cách để xác định tạp chất quang hoạt là dùng độ tinh khiết quang
(ee), được định nghĩa là chênh lệch phần trăm các đối quang trong
hỗn hợp. Ví dụ trong hỗn hợp có 70 % R và 30 % thì ee là 40 %.
3)
a) Tính độ tinh khiết quang của hợp chất được phân giải khi tổng hợp
theo hướng công nghiệp trên.
L-selectride là tác nhân thương mại, có thể thực hiện phản ứng cuối theo
hướng chọn lọc lập thể.
b) Gắn các tâm lập thể trong (-)-1 với kí hiệu cấu hình R hoặc S.

c) Xác định công thức của một tác nhân cũng thực hiện phản ứng
giống L-selectride, chuyển H thành 1. Không cần quan tâm để hoá
lập thể.
Dưới đây là một hướng thay thế để tổng hợp galatamine, với vòng 7 cạnh
là vòng cuối cùng được tạo thành.

396 | Bản quyền thuộc về Tạp chí Olympiad Hóa học KEM
4)
a) Xác định công thức hợp chất Y để tiến hành giai đoạn đầu tiên.
b) Đề xuất cấu trúc của J và K.

397 | Bản quyền thuộc về Tạp chí Olympiad Hóa học KEM
Bài 111
Bài tập này liên quan đến các dẫn xuất của tropane, còn được gọi là các
tropane alkaloid. Tropane là amine bicyclic (hai vòng), có danh pháp là 8-
methyl-8-aza-bicyclo[3.2.1]octane. Những hợp chất này được tạo ra bởi
các loài thực vật, đặc biệt là Solanaceae (họ Cà), để chống lại những kẻ
thù tự nhiên. Tropane alkaloid có hoạt tính dược lí đa dạng, do vậy những
nghiên cứu để tổng hợp nhóm hợp chất này đã được tiến hành trong
khoảng 100 năm nay.
Phần đầu tiên của bài tập này liên quan đến tổng hợp atrophine, độc dược
của cây cà độc. Phần thứ hai là về ferruginine, một chất chủ vận thụ thể
nicotinic acetylcholine.
A. Tổng hợp atropine

Hãy sử dụng các gợi ý sau:


● Hợp chất E là ethyl-2-bromo-2-phenylethaneoat.
● A và B là các đồng phân lập thể.
1) Xác định công thức lập thể của các hợp chất A, B, H và công thức cấu
tạo của C, D, F, G, I và công thức phân tử của X.

398 | Bản quyền thuộc về Tạp chí Olympiad Hóa học KEM
2) Viết cơ chế phản ứng chuyển F thành G. Cho biết tên gọi của loại phản
ứng này.
3) Cho biết tên gọi của loại đồng phân lập thể tồn tại trong A, B.
4) Vẽ các mũi tên chỉ rõ sự tạo thành C từ chất đầu. Cho biết tên gọi của
loại phản ứng đã xảy ra.
B. Tổng hợp ferruginine

Sử dụng các gợi ý sau:


● Danh pháp IUPAC của hợp chất A là 2-methylcyclopentanone.
● Phản ứng C → D tạo ra một tiểu phân trung gian.
● Nhóm chức methyl ester vẫn giữ nguyên trong phản ứng F → G.
● Một nhóm bảo vệ bị chuyển thành một nhóm khác trong giai đoạn G
 hợp chất bicyclic.

399 | Bản quyền thuộc về Tạp chí Olympiad Hóa học KEM
● Không có phản ứng nào trong các vòng ở giai đoạn H  I, phân tử X bị
thay thế có khối lượng mol 30,03 gam/mol.
● Các kí hiệu: Bn = benzyl, Cbz = benzyl (benzyloxycarbonyl), Mes =
mesyl = methanesulfonat, TMSI = Trimethylsilyliodide, NaHDMS =
[(CH3)3Si]2NNa
5) Xác định công thức lập thể của A, B, H và công thức cấu tạo của C, D,
E, F, G, I, J, K, L, X.

400 | Bản quyền thuộc về Tạp chí Olympiad Hóa học KEM
Bài 112
Trong một thập kỉ trở lại đây, các
macrolide vòng 10 cạnh đã thu hút được
nhiều sự quan tâm. Một trong các
macrolide như vậy là Stagonolide D,
năm 2007 đã được phân lập từ nấm
Stagnospora Cirsii. Stagonolide D tạo ra
các tổn thương hoại tử trên lá và thể
hiện hoạt tính diệt cỏ. Trong bài này,
chúng ta sẽ thảo luận về quy trình tổng
hợp toàn phần Stagonolide D.
Để tạo ra Stagonolide D, trước tiên chúng ta cần tạo ra mảnh X, quy trình
tổng hợp được cho dưới đây:

Mảnh tạo thành được dùng tiếp để tổng hợp Stagonolide D theo sơ đồ
dưới đây, và giai đoạn then chốt là phản ứng đóng vòng trao đổi (Ring
Closing Metathesis).

401 | Bản quyền thuộc về Tạp chí Olympiad Hóa học KEM
Các tác nhân và kí hiệu:

1) Xác định công thức cấu tạo các chất A-N. Biểu diễn hóa lập thể phù
hợp, nếu cần thiết.
2) Đề nghị cơ chế cho chuyển hóa tạo thành K.

3) Đề nghị cơ chế cho chuyển hóa F  G.

402 | Bản quyền thuộc về Tạp chí Olympiad Hóa học KEM
Bài 113
Phân chim biển và dơi từng là một hạng mục xuất khẩu lớn từ thuộc địa
châu Mỹ đến châu Âu vào thế kỉ 19, để làm phân bón và chiết xuất uric
acid – được dùng trong sản xuất nhiều hoá chất thương mại, gồm cả
murexide, một loại thuốc nhuộm rất có giá trị. Uric acid là sản phẩm
chuyển hoá phụ của các protein và nucleic acid. Vào năm 1864, khi đang
nghiên cứu về tính chất của uric acid, nhà Hoá học Đức Adolf von Baeyer
đã khám phá ra barbituric acid C. Các dẫn xuất của hợp chất này có tác
động rất mạnh trong việc gây buồn ngủ ở động vật. Về sau, nhóm hợp
chất này trở thành loại thuốc rất hiệu quả trong việc chữa rối loạn tâm lí.
Một phương pháp đơn giản để điều chế C là ngưng tụ (trong POCl 3) hợp
chất A (C 3H4O3, nguồn gốc từ nước táo) và hợp chất B (CH4N2O, có nhiều
trong phân động vật).
1. Xác định công thức cấu tạo A, B.

Thiopental, thường được gọi là “huyết thanh sự thật” - do nó có thể khiến


con người nói nhiều hơn (thường tiết lộ sự thật) - đã được phát hiện từ
rất sớm. Nó có cấu trúc vòng tương tự như C. Để tổng hợp E, trước tiên
cho 2-ethylpropanedioic acid diethyl ester phản ứng với 2-bromopentane
trong sodium ethoxidee, tạo thành sản phẩm D (C14H26O4). D tạo thành ở
dạng hỗn hợp racemic, được ngưng tụ tiếp với thiourea tạo thành
thiopental E (C 11H18N2O2S). Đồng phân R của thiopental được dùng làm
thuốc chống trầm cảm, còn đồng phân S là nguyên nhân gây trầm cảm
thần kinh.
2. Xác định công thức cấu tạo của D và hai đồng phân E, chỉ rõ cấu hình
R/S.
3. Hợp chất C có tính acid khá mạnh (pKa = 4.01). Xác định công thức
các dạng tautomer (hỗ biến) bền nhất ứng với base liên hợp của C.
Hợp chất F (C4H2N2O4) là một trong những hợp chất hữu cơ tổng hợp lâu
đời nhất, được Justus von Liebig và Friedrich Wöhler khám phá vào năm
1838. F được dùng để sản xuất thuốc nhuộm murexide. Khi hấp thụ vào

403 | Bản quyền thuộc về Tạp chí Olympiad Hóa học KEM
cơ thể, nó tích tuỵ trong tuyến tuỵ, phá huỷ các tế bào sản xuất insulin và
gây ra bệnh đái tháo đường loại II. Do vậy, F thường được tiêm vào chuột
thí nghiệm để nghiên cứu bệnh tiểu đường.
4. Điều chế F bằng cách oxid hoá sản phẩm G của phản ứng ngưng tụ
giữa benzaldehyde và C khi có xúc tác piperidine (một loại base). Xác
định công thức cấu tạo của F và G.
CrO ,t0
⎯⎯⎯⎯→
piperidine
−H O
⎯⎯⎯⎯⎯ 3
−C H COOH
⎯→
Benzaldehyde + C 2 G (C 11H8N2O3) 6 5 F
(C 4H2N2O4)
5. Phản ứng ngưng tụ tạo ra G ở trên xảy ra nhanh, điều này chứng tỏ
rằng pKa của acid liên hợp của piperidine là
A. < 4.01
B. > 4.01
C. = 4.01
Trung dung dịch trung tính, F tồn tại ở dạng ketal H. Trong môi trường
acid, H chuyển trở lại thành F. Khử hoá F với hydrogen sulphide, thu được
I – chất này phản ứng với F dư tạo thành hemiketal J. Hợp chất J phản ứng
với NH3, tạo ra thuốc nhuộm murexide.

6. Xác định công thức các chất H - J.


Năm 1993, Paul György và Richard Kuhn đã cô lập được một sắc tố vàng,
có thể chuyển thành một vitamin thiết yếu và chữa được nhiều loại bệnh.
Sau này, sắc tố này được xác định là hợp chất K (C17H20N4O6) với một
nhánh tạo ra từ D-aldopentose (L). Trong L, nguyên tử carbon C3 có cấu
hình R.

404 | Bản quyền thuộc về Tạp chí Olympiad Hóa học KEM
7. L (C 5H10O5) bị khử bởi sodium borohydride (NaBH4) tạo thành một
alditol không quang hoạt. Vẽ công thức chiếu Fischer của L, biểu diễn
đúng cấu hình của nguyên tử carbon quang hoạt.
8. Dưới đây là một chuỗi tổng hợp chất K:

(NaCNBH3 là một tác nhân khử)


a) Xác định công thức cấu tạo M, N, O.
b) Chất K (C17H20N4O6) tạo thành từ phản ứng ngưng tụ của O và F,
có cấu trúc vòng như sau. Dựa vào cấu trúc này, xác định công
thức cấu tạo của K.

Những nghiên cứu về sự thoái phân K đóng vai trò quan trọng trong việc
xác định cấu trúc của nó. Thuỷ phân K bằng NaOH 1 M ở 100 oC thì xảy ra
sự mất 1 phân tử B, tạo ra P (C 16H19N2NaO7). Tiếp tục thuỷ phân P bởi
NaOH 5 M ở 100 oC thu được Q (C 13H22N2O4).
9. Xác định công thức của P và Q.

405 | Bản quyền thuộc về Tạp chí Olympiad Hóa học KEM
Bài 114
OLED là công nghệ cạnh tranh với màn hình LCD truyền thống. Màu sắc
của các ánh sáng phát ra từ màn hình công nghệ OLED phụ thuộc vào kim
loại lân quang và các phối tử của nó. Một trong những kim loại tiềm năng
trong OLED là M. Dẫn xuất halogen A màu xanh lục thẫm, chứa 64.4 %
kim loại M về khối lượng. A phản ứng với 2-phenylpyridine (D) tạo thành
một phức đa nhân B (1072 gam/mol), trong đó 2 nhân được nối với nhau
bởi 2 phối tử, nghĩa là các liên kết dạng cầu. Khi xử lí phức chất này trong
acetylacetone thu được hợp chất C có màu xanh lục. Tất cả các phản ứng
ở trên diễn ra mà không có sự thay đổi số oxid hoá của kim loại M. Số phối
trí của M trong hợp chất B và C đều bằng 6. Giữa kim loại M và vòng
benzene có một liên kết cộng hoá trị.

a) Hợp chất C có bao nhiêu đồng phân?


b) Xác định kim loại M.
c) Vẽ cấu tạo của một trong các đồng phân của B.
Để thay đổi màu sắc của OLED, chỉ cần thay đổi phối tử hữu cơ của kim
loại. Cho hợp chất A phản ứng với chất E, thu được phức chất G (1296
gam/mol) dạng dimer. Phức chất này phản ứng acetylacetone tạo thành
hợp chất I màu vàng. Phản ứng của A với Y tạo thành phức chất H (1296
gam/mol) dạng dimer. Phức chất này phản ứng với acetylacetone tạo
thành hợp chất J màu đỏ. Trong các hợp chất E và F đều có 3 vòng.
d) Vẽ cấu tạo các hợp chất E, F và một trong các đồng phân cấu tạo của
mỗi chất I, J.

406 | Bản quyền thuộc về Tạp chí Olympiad Hóa học KEM
Bài 115
Alkaloid là nhóm sản phẩm thiên nhiên chứa nitrogen. Sự phức tạp trong
cấu trúc cùng hoạt tính sinh học rất mạnh của chúng đã thu hút được
nhiều sự quan tâm. Hai ví dụ tiêu biểu về alkaloid, sauristolactam và
pancratistatin, là trọng tâm của các câu hỏi sau đây.
Phần A
Sauristolactam có hoạt tính cytotoxicity (gây độc tế bào) cực mạnh với
nhiều dòng tế bào ung thư. Nó có thể được điều chế từ chuỗi tổng hợp
dưới đây. (Phổ 1H NMR được ghi trong CDCl3 ở 300 MHz.)
A1) Vẽ cấu tạo các hợp chất A-G.
Ghi chú: A: sản phẩm có hai vòng thơm: một vòng có một nhóm thế, vòng
còn lại có 4 nhóm thế với 2 mũi đơn trong phổ 1H NMR.
B: Hấp thụ hồng ngoại mạnh trong khoảng 1725-1700 cm-1 và hấp thụ
hồng ngoại rộng trong khoảng 330-2500 cm-1.
C: Hấp thụ hồng ngoại mạnh trong vùng 1750-1735 cm-1.
D: Ngoài vùng thơm, còn có các tín hiệu 1H NMR trong vùng 0-6 ppm; 3,87
(s, 3H); 3,84 (s, 3H); 2,63 (s, 3H); 2,31 (s, 3H).
E: Các tín hiệu 1H NMR của toàn phân tử: 7,59 (s, 1H); 3,88 (s, 3H); 3,87
(s, 3H); 2,68 (s, 3H); 2,35 (s, 3H).
F: Các tín hiệu 1H NMR của toàn phân tử: 7,74 (s, 1H), 5,19 (s, 2H); 3,93 (s,
3H); 3,91 (s, 3H); 2,36 (s, 3H).
G: Các tín hiệu 1H NMR của toàn phân tử: 7,40 (s, 1H); 4,22 (s, 2H); 3,98
(s, 3H); 3,19 (s, 3H); và một proton có thể trao đổi với D2O.

407 | Bản quyền thuộc về Tạp chí Olympiad Hóa học KEM
Phần B
Pancratistatin, được phân lập từ một loại thực vật bản địa của Hawaii,
hoa loa kèn nhện, có hoạt tính ức chế sự phát triển của tế bào ung thư và
kháng virus rất mạnh, đã được thử nghiệm ở quy mô in vitro (trong ống
nghiệm), lẫn in vivo (trên người và động vật).

408 | Bản quyền thuộc về Tạp chí Olympiad Hóa học KEM
Pancratistatin có thể được tổng hợp qua các hợp chất trung gian X1 và
X2. Quy trình tổng hợp các chất trung gian này được cho trong các giản
đồ sau:
B1) Vẽ cấu trúc của A, B.

B2) Chất trung gian X1 tinh khiết quang học với cấu trúc lập thể cho dưới
đây) được đánh dấu đồng vị bằng deuterium. Đề xuất cấu dạng ghế 3
chiều của hợp chất E và cấu trúc của hợp chất F, chỉ rõ hóa lập thể. Hãy
cho biết trong chất trung gian X2 thì Y là proton hay deuterium?

409 | Bản quyền thuộc về Tạp chí Olympiad Hóa học KEM
410 | Bản quyền thuộc về Tạp chí Olympiad Hóa học KEM
Bài 116
Streptonigrin là một kháng sinh kháng u được Rao và Cullen phân lập từ
Streptomyces flocculus vào năm 1959. Cấu trúc của nó đã được Rao,
Biemann và Woodward thiết lập vào năm 1963 bằng phương pháp phân
tích quang phổ và thoái phân hóa học. Do cấu trúc phức tạp liên quan đến
những hoạt tinh sinh học, nó đã mang tới thách thức lớn cho các nhà tổng
hợp hữu cơ và nhiều quy trình tổng hợp chất này đã được báo cáo. Chiến
lược hiệu quả nhất là dựa vào phân tích tổng hợp ngược được trình bày
trong Sơ đồ 1, gồm 2 giai đoạn ghép cặp chéo giữa các mảnh cấu trúc 1,
2 và 3. Cấu trúc 2 cần phải được thay thế từ 5 - chất này ban đầu được
chuyển thành 4 bởi phản ứng hoán vị đóng vòng (Ring-Closing Metathesis,
RCM), sử dụng xúc tác Hoveyda-Grubbs thế hệ thứ hai (GH II).

Sơ đồ 1: Phân tích tổng hợp ngược Streptonigrin. (acoplamento cruzado = ghép cặp chéo)

Quy trình tổng hợp các mảnh 1 và 2 được cho trong Sơ đồ 2 và 3.

411 | Bản quyền thuộc về Tạp chí Olympiad Hóa học KEM
Sơ đồ 2: Quy trình tổng hợp mảnh có nhân pyridine.

Sơ đồ 3: Quy trình tổng hợp mạnh có nhân quinoline.

1) Xác định cấu trúc các sản phẩm và tác nhân A-J.
2) Trong chuyển hóa từ A đến B, tác nhân nào khác có thể được sử dụng
thay cho (COCl)2?

412 | Bản quyền thuộc về Tạp chí Olympiad Hóa học KEM
3) Xác định cấu tạo hợp chất trung gian nhận được bằng cách xử lí 6 với
MeONH2·HCl và hợp chất nhận được từ phản ứng của crotyl bromide với
kẽm.
4) Với hợp chất H, xác định cấu trúc hai đồng phân và trình bày cơ chế
chuyển hóa chúng thành 13. Trong trường hợp này, chỉ cần biểu diễn cơ
chế của một trong các đồng phân.
Kí hiệu viết tắt
Tf2O - anidrido tríflico
HFIPA - hexafluoroisopropanol
DTBMP - 2,6-di-tert-butil-4-metilpiridina
NBS - N-bromossuccinimida

413 | Bản quyền thuộc về Tạp chí Olympiad Hóa học KEM
Bài 117
A. Trùng hợp
Các polylactides, viết gọn là PLA, là những polymer có thể được tạo thành
bởi ROP (ring opening polymerization - phản ứng trùng hợp mở vòng).
Các polylactide đã được mô tả lần đầu tiên vào năm 1845 bởi Théophile-
Jules Pelouze, người đã quan sát được sự tạo thành oligo- và polymer
trong quá trình cô lập lactic acid. Các loại nhựa nhiệt dẻo được chế tạo từ
PLA ngày càng được ưa thích bởi khả năng phân hủy sinh học của chúng.
Ví dụ, màng phủ PLA đang cạnh tranh mạnh mẽ với màng phủ PE truyền
thống trong lĩnh vực nông nghiệp và làm vườn. Một quá trình sản xuất
PLA từ lactide đã được phát triển từ năm 1932 và được DuPont chuyển
thành giải pháp sáng chế vào năm 1954.

1) Vẽ công thức cấu dạng của tất cả các đồng phân lập thể có thể có của
lactide và xác định cấu hình tuyệt đối của tất cả các tâm bất đối.
2) Vẽ cơ chế cho các quá trình khơi mào (1), phát triển mạch (2, với n(LA)
= 1) và tắt mạch (3) của phản ứng trùng hợp mở vòng lactide (LA). Nhớ
chú ý vào hóa lập thể.

3) Phản ứng trùng hợp này dẫn đến sự tạo thành


A. Một polyamide.
B. Một polyester.
C. Một polyanhydride.
D. Một polycarboxylic acid.
4) Gọi tên IUPAC (các) sản phẩm thủy phân acid của lactide LA, kèm theo
các mô tả lập thể.

414 | Bản quyền thuộc về Tạp chí Olympiad Hóa học KEM
B. Tổng hợp dẫn xuất caffeic acid
Phần này liên quan đến tổng hợp một dẫn xuất caffeic acid I, đã được
nghiên cứu cùng với 20 ester khác của caffeic acid về tính chất chống lại
độc tính gây ngộ độc tế bào bởi H2O2.

Để hoàn thành bài tập này, có một số lưu ý sau:


- Chất A có tên là 3,4-dihydroxybenzene carbaldehyde.
- E và E’ là đồng phân quang học.
- F và F’ là đồng phân quang học.
- Chất F có thể chuyển thành F’ bởi phản ứng đồng phân hóa.

415 | Bản quyền thuộc về Tạp chí Olympiad Hóa học KEM
- F và F’ có thành phần nguyên tố (w/w) như sau: 60.00% C, 4.48%
H, 35.52% O.
- Các kí hiệu sau được sử dụng:
Ph = phenyl,
Bn = benzyl (phenylmethyl),
MOMCl = CH3OCH2Cl,
PDC = pyridinium dichromate.
5) Xác định công thức phân tử F và F’.
6) Vẽ cấu trúc (có xét đến hóa lập thể) của các chất A, B, D, E, E', F, F', G,
H, I, và Z và xác định công thức phân tử của X, Y (ghi vào ô trống trong
sơ đồ). Kí hiệu các nhóm chức benzyl, phenyl, MOM là Bn, Ph, MOM.
7) Xác định cơ chế phản ứng P(OBn)3 + Z tạo thành C (giai đoạn thứ nhất).
8) Chỉ rõ mối quan hệ lập thể của E và E’, F và F’.
9) Đề xuất cơ chế phản ứng đồng phân hóa F thành F’ và chỉ rõ điều kiện
phản ứng.
10) Xác định vai trò của NaOEt ở bước đầu tiên tạo thành F từ B và C. Vẽ
công thức cấu tạo của tiểu phân trung gian hoạt động. Cơ chế phản
ứng này thuộc loại nào? Trong trường hợp này phản ứng có tên là gì?
11) Xác định cơ chế tổng quát chi tiết nhất có thể (sử dụng kí hiệu R cho
phần gốc) của phản ứng tạo thành E hoặc E’ từ D và ethanal. Cho biết
tên phản ứng.
12) Cho biết tên phản ứng tạo thành D từ A. Cơ chế phản ứng thuộc loại
nào?

416 | Bản quyền thuộc về Tạp chí Olympiad Hóa học KEM
C. Tổng hợp colchicine của Woodward
Colchicine, alkaloid chính trong cây bả chó
(Colchicum autumnale), là một trong những
sản phẩm tự nhiên nổi bật nhất. Ngoài việc là
loài thực vật có độc tố mạnh, nó còn có tiềm
năng dược phẩm to lớn bởi khả năng ức chế sự
phân bào có tơ.
Những quy trình tổng hợp đầu tiên ở thập niên
1950 đã đặt ra những cột mốc lớn trong tổng
hợp hợp chất thiên nhiên. Chỉ có một số ít hợp
chất mục tiêu được tổng hợp theo nhiều
phương pháp đa dạng như colchicine. Chuỗi
phản ứng trong bài này đưa chúng ta quay trở
lại với nhà hóa học hữu cơ nổi tiếng Woodward
(1963). Điểm đặc biệt của quy trình tổng hợp
này là chiến lược đưa vào rất sớm nhóm chức nitrogen ở C-7. [1]
Woodward đã dành được giải Nobel Hóa học năm 1965 cho những thành
tựu của mình.
[1] Graening, T. and Schmalz, H. (2004), Colchicin‐Totalsynthesen im
Vergleich: Eine Reise durch 50 Jahre Organische Synthesechemie.
Angewandte Chemie, 116: 3292-3318.
doi:10.1002/ange.200300615
Một số gợi ý:
- Chất A là methyl (E) -3-aminobut-2-enoate.
- N2H4/H2O2/Cu2+ : tác nhân khử đặc biệt, không khử các liên kết đôi
của vòng thơm.
- PCC: pyridinium dichromate.
13) Vẽ cấu trúc của A-R và công thức phân tử của W, X, Y. Với một số chất,
một phần cấu trúc đã được chỉ rõ, bạn chỉ cần bổ sung thêm.
14) Đề xuất tác nhân Z1 và Z2 cho chuyển hóa I thành K.
15) Cho biết tên gọi phản ứng của giai đoạn chuyển H thành I (giai đoạn
vòng hóa). Cơ chế phản ứng thuộc loại nào?
16) Xác định (các) dạng thủ tính (chiral) của colchicine.
17) Xác định (các) mô tả lập thể của colchicine.

417 | Bản quyền thuộc về Tạp chí Olympiad Hóa học KEM
418 | Bản quyền thuộc về Tạp chí Olympiad Hóa học KEM
Bài 118
Strychnine là một alkaloid kết tinh có vị đắng,
rất độc, được dùng làm thuốc trừ sâu, đặc biệt
là là để tiêu diệt các động vật có xương sống
nhỏ như chim và động vật gặm nhấm. Hóa
chất này được thu thập tự nhiên từ cây
Strychnos nux-vomica. Trong hóa học hữu cơ,
đối quang tinh khiết của strychnine được sử
dụng để phân giải các phân tử hữu cơ. Do tính
phức tạp, hiệu ứng sinh lí và nhu cầu cao, các
nhà tổng hợp hữu cơ đã tìm kiếm nhiều hướng khác nhau để tổng hợp
toàn phần hợp chất này trong nhiều năm. Trong bài này, bạn sẽ được biết
đến một trong những con đường tổng hợp strychnine ngắn nhất.

1) Vẽ cấu trúc hợp chất X, bao gồm thông tin hóa lập thể phù hợp. Xác
định cấu hình (các) tâm bất đối, sử dụng hệ danh pháp R/S.

419 | Bản quyền thuộc về Tạp chí Olympiad Hóa học KEM
2) Vẽ cấu trúc hợp chất B. Trong phản ứng từ A thành B, tác nhân DCC
được chuyển thành hợp chất khác. Vẽ cấu trúc của nó.
3) Viết cơ chế tạo thành hợp chất D. Trong phản ứng này, vai trò của BF3
là để tạo càng với nhóm carbonyl của aldehyde không no, nhưng bạn
không cần biểu diễn nó trong cơ chế. Nhớ rằng trong quá trình này có
xảy ra chuyển vị [1,5] hydride. 2 nguyên tử carbon được đánh số trong
cấu trúc C và D chỉ ra vị trí các nguyên tử trong cả 2 cấu trúc (trước và
sau phản ứng).
4) Sau giai đoạn đầu tiên của phản ứng tách methyl của amine (từ D
thành E), hợp chất trung gian D1 được tạo thành, sau khi xử lí với dung
dịch nước Na2CO3 tạo thành hợp chất E. Đề xuất cơ chế phù hợp của
chuyển hóa từ D thành D1. Xác định D1. Nhớ rằng chloromethane là
sản phẩm phụ của giai đoạn này.
5) (Các) tác nhân Y nào là cần thiết để thủy phân acetal?
6) Vẽ cấu trúc của H và I, chỉ rõ hóa lập thể.
7) Phản ứng H thành I được gọi là Horner-Wadsworth-Emmons. Vẽ tất
cả các cấu trúc cộng hưởng có thể có của hợp chất trung gian có tính
nucleophile chính, được tạo ra tức thì (in situ - trực tiếp trong hỗn hợp
phản ứng) sẽ phản ứng với hợp chất H.
8) Trong phản ứng từ H thành I, thực tế có hai sản phẩm được tạo thành.
Một trong số chúng là sản phẩm mục tiêu I, còn lại là sản phẩm phụ
I*. Vẽ cấu trúc I*.
9) Vẽ cấu trúc của hợp chất về mặt lí thuyết sẽ thu được từ I nếu chúng
ta sử dụng LiAlH4 thay vì hệ khử hóa DIBAL-H/NaBH4.
10) Xác định hóa lập thể của nguyên tử carbon được đánh số 3 trong hợp
chất J sử dụng hệ danh pháp R/S.
11) Strychnine thể hiện tính chất nào: a) tính acid; b) tính base; c) lưỡng
tính; d) không.

420 | Bản quyền thuộc về Tạp chí Olympiad Hóa học KEM
Bài 119
Tổng hợp các sản phẩm tự nhiên là một phần quan trọng của hóa hữu cơ.
Các sản phẩm này thường thể hiện hoạt tính sinh học cao và có những
tính chất đặc biệt thú vị xét trên quan điểm dược học. Tuy nhiên, việc
tổng hợp các phân tử này không đơn giản vì chúng có khuynh hướng tồn
tại nhiều cấu trúc và đa dạng về các nhóm chức (chính vậy đã tạo nên các
loại thuốc tiềm năng). Một trong những hợp chất này là Erythronolide B,
tiền chất hóa sinh của kháng sinh Erythromycin. Nhiều nhà hóa hữu cơ
nổi tiếng, như Woodward, Evans, Carreira, Hoffmann và Danishefsky đã
thành công trong việc tổng hợp chất này. Trong bài tập này, chúng ta sẽ
thảo luận về quy trình tổng hợp Erythronolide B của Elias Corey. Quy trình
tổng hợp hoàn chỉnh không được dẫn ra ở đây, mà sẽ là phần đầu tiên,
được Corey gọi là Mảnh 11.

Trong tổng hợp này, các hợp chất A, B, C là những hợp chất meso không
thơm. Chất D, mặc dù ban đầu được tạo thành ở dạng hỗn hợp racemic,
được tinh chế thành đối quang tinh khiết. Chuyển hóa F thành G dẫn tới
sự nghịch đảo cấu hình ở tâm phản ứng, nhưng chuyển hóa G thành H
diễn ra mà vẫn bảo toàn hóa lập thể ở tâm phản ứng.

421 | Bản quyền thuộc về Tạp chí Olympiad Hóa học KEM
1) Xác định các cấu trúc các hợp chất A-M. Với các hợp chất D-M, hãy
biểu diễn chính xác hóa lập thể (không cần chỉ ra R/S).
2) Đề xuất cơ chế chuyển hóa M thành Mảnh 11.

422 | Bản quyền thuộc về Tạp chí Olympiad Hóa học KEM
Bài 120
Tetracycline là một nhóm thuốc kháng sinh được sử dụng phổ biến cho
các bệnh nhiễm trùng đường hô hấp và bỏng. Cấu trúc của một trong
những đại diện của nhóm chất này - doxycycline - được cho dưới đây:

Dưới đây là sơ đồ tổng hợp một đại diện khác của nhóm này.

Tác nhân 1) ở bước chuyển H → I là piperidine.


1) Xác định cấu trúc các hợp chất A - O (có xét đến hóa lập thể).
2) Đề xuất cơ chế chuyển J → K.

423 | Bản quyền thuộc về Tạp chí Olympiad Hóa học KEM
Bài 121
Thân rễ gừng (Zingiber officinale) được sử dụng ở nhiều quốc gia làm phụ
gia nấu ăn và các sản phẩm mứt kẹo, như một thứ gia vị cho sushi và các
món cuốn, hoặc trong một số loại trà, bia, … Vị cay của gừng là do gingerol
(X). Khi làm khô gừng trong không khí, gingerol chuyển thành chất Y. Khi
xử lí gừng với nhiệt thì gingerol tạo thành zingerone Z. Năm 1975, các nhà
khoa học Nhật Bản đã đề xuất một quy trình tổng hợp cả 3 hợp chất trên
từ vanillin [ванилин], thành phần chính của tinh dầu vanilla. Sơ đồ tổng
hợp racemic X, Y, Z được mô tả dưới đây:

1) Xác định cấu trúc các hợp chất chưa biết trong sơ đồ.
Năm 1992, các nhà hóa học Mĩ đã đề xuất một cách mới để tổng hợp Y và
Z từ nguồn nguyên liệu thương mại sẵn có khác: 4-hydroxy-3-methoxy
cinnamic acid.

2) Xác định cấu trúc các hợp chất B-D.


Gừng nổi bật giữa các loại gia vị khác không chỉ bởi vị của nó mà còn là
bởi mùi hương. Mùi gừng gây ra bởi thành phần chính của các tinh dầu
của nó: zingiberene (W). Suốt một thời gian dài, cấu trúc chất này không
được xác định rõ. Phải đến năm 1953, các nhà khoa học mới thiết lập
được cấu trúc sau khi tổng hợp được racemate của nó:

424 | Bản quyền thuộc về Tạp chí Olympiad Hóa học KEM
3) Xác định cấu trúc các hợp chất E-H và W, biết rằng F và G là các đồng
phân và W chỉ chứa duy nhất 1 vòng; W tạo thành với dimethyl ether
của acetylene dicarboxylic acid (MeO 2C-C≡C-CO2Me) một sản phẩm
cộng bị phân hủy khi đun nóng tạo thành 4-methylphthalic acid và 3,7-
dimethyloct-1,6-diene.

425 | Bản quyền thuộc về Tạp chí Olympiad Hóa học KEM
Bài 122
Năm 1973, Fisher và Wilkinson nhận được giải Nobel Hóa học vì đã thiết
lập được cấu trúc của ferrocene [ферроцен] ((C 5H5)2X), ví dụ đầu tiên về
các hợp chất “sandwich”. Điều thú vị là chất này được vô tình tổng hợp
lần đầu tiên bởi Pauson và Kealy từ phản ứng giữa chất C và chloride kim
loại X. Mục tiêu của các nhà nghiên cứu là tổng hợp hydrocarbon D, có
hàm lượng carbon trong đó là 93.71 %. Hợp chất C có thể được tổng hợp
từ một chất lỏng B có nhiệt độ sôi thấp, được điều chế trực tiếp bằng cách
nhiệt phân hydrocarbon A (hàm lượng các nguyên tố trong A và B giống
nhau).

1) Xác định cấu trúc các hợp chất A-D và kim loại X.
2) Thiết lập công thức chloride XCln được Pauson và Kealy sử dụng. Viết
phương trình phản ứng của nó với C, biết rằng trong phản ứng này,
cùng với ferrocene thì còn thu được một hydrocarbon có hàm lượng
carbon lớn hơn trong B nhưng bé hơn trong D.
Ferrocene đặc biệt bền, bị thăng hoa không phân hủy thành chất màu
vàng cam có những tính chất tương tự benzene. Ví dụ, khi phản ứng vơi
1 đương lượng acethyl chloride khi có mặt chloride kim loại Y thì thu được
chất E. Nếu chloride kim loại Z (hàm lượng Z là 31.04 %) phản ứng với E
và tác nhân khử F (chứa 71.09 % Y về khối lượng), được sử dụng rộng rãi
trọng hóa hữu cơ, thì có thể thu được các sản phẩm khác nhau tùy thuộc
vào điều kiện phản ứng. Ở nhiệt độ cao, một hỗn hợp đồng phân dia G và
H được tạo thành với hàm lượng X là 26.34 % và ở nhiệt độ phòng thì tạo
thành hỗn hợp đồng phân dia I và J có nhiều hơn 1 nguyên tố (so với G và
H) có hàm lượng tương ứng là 6.98 %.

426 | Bản quyền thuộc về Tạp chí Olympiad Hóa học KEM
3) Xác định cấu trúc các hợp chất E-J và các chloride YCln và ZCln.
4) Đồng phân nào trong số G và H chiếm ưu thế? Tại sao?

427 | Bản quyền thuộc về Tạp chí Olympiad Hóa học KEM
Bài 123
Caryophyllene (C 15H24) là hợp chất thiên nhiên chứa 1 liên kết đôi ở dạng
trans, là nguyên nhân gây mùi chính của cây tử đinh hương. Cấu trúc của
caryophyllene và các dẫn xuất có thể được xác định từ các phản ứng sau:
石竹烯 = caryophyllene

Biết rằng, hỗn hợp đồng phân caryophyllen-isocaryophyllen tạo thành các
sản phẩm A và B ở phản ứng 1 và 2 tương ứng. Sản phẩm C và một đồng
phân của nó được tạo thành sau phản ứng 3. Sản phẩm D được tạo thành
từ phản ứng 4.
- Xác định công thức cấu tạo của A, C và đồng phân của hợp chất C
(không quantâm đến lập thể).
- Xác định công thức cấu tạo của caryophyllen và isocaryophyllen.
- Chỉ ra sự khác nhau về cấu trúc của caryophyllen và
isocaryophyllen

428 | Bản quyền thuộc về Tạp chí Olympiad Hóa học KEM
Bài 124
-caryophyllene (3) là sesquiterpene thiên nhiên có trong cây đinh hương
và một số loài thực vật truyền thống của Czech and Slovakia như cây hoa
bia hoặc cây bồ đề nhỏ. Tổng hợp -caryophyllene đi từ một đối quang
tinh khiết của dienone A. Phản ứng của A với silyl ketene acetal 1, sau đó
là phản ứng khử tiểu phân trung gian và work-up tạo thành ketone 2. Chất
trung gian này sau đó phản ứng với tosyl chloride tạo thành B. Phản ứng
vòng hóa xúc tác base của hợp chất này tạo thành C. Cuối cùng, phản ứng
của C với ylide D tạo thành -caryophyllene.

1) Vẽ cấu trúc các hợp chất A-D, chỉ rõ hóa lập thể. Gợi ý: Trong chuyển
hóa A thành 2, silyl ketene acetal đóng vai trò như một nucleophile.
Một trong các liên kết đôi trong 2 cũng như trong 3 có cấu hình trans và
độ bền của vòng đạt được nhờ kích thước lớn của nó. Trans-cyclooctene
(4) là vòng nhỏ nhất có thể chứa một liên kết đôi trans. Nó có thể được
điều chế theo sơ đồ sau:

2) Vẽ cấu trúc tác nhân E, các chất trung gian F, G, chỉ rõ hóa lập thể.
3) Vẽ cấu trúc đối quang của cycloalkene 4.

429 | Bản quyền thuộc về Tạp chí Olympiad Hóa học KEM
Hai liên kết đôi trong β-caryophyllene thể hiện hoạt tính khác nhau: liên
kết đôi trong vòng (endocylic) có hoạt tính lớn hơn liên kết đôi còn lại
(exocyclic) do sức căng vòng.

4) Vẽ các cấu trúc của các hợp chất Ha + Hb, I và Ja + Jb, chỉ rõ hóa lập
thể. Gợi ý: Ha + Hb và Ja + Jb là các cặp đồng phân dia.
Điều thú vị là hoạt tính của các liên kết đôi thể hiện ngược lại khi isocaryo
phyllene (5) được sử dụng thay vì -caryophyllene (3).

5) Vẽ các cấu trúc của các hợp chất Ka và Kb. Gợi ý: Ka + Kb là một cặp
đồng phân dia.
Các hợp chất đánh dấu đồng vị là những công cụ vô giá để khảo sát cơ chế
phản ứng, xác định cấu trúc và nghiên cứu phổ khối lượng hoặc phổ NMR.
Hãy xem xét quy trình tổng hợp một số hợp chất tương đồng -
caryophyllene được đánh dấu.

430 | Bản quyền thuộc về Tạp chí Olympiad Hóa học KEM
6) Vẽ cấu trúc các hợp chất L, M.
-caryophyllene (3) bị vòng hóa bởi xúc tác acid, tạo thành hỗn hợp sản
phẩm phức tạp. Trong số chúng, các cặp đồng phân dia Na + Nb và 7a +
7b chiếm hàm lượng lớn nhất. Phản ứng bắt đầu với sự proton hóa liên
kết đôi bên trong hoạt động hơn, tạo thành cation O. Chất này vòng hóa
mà không bẻ gãy liên kết đơn carbon-carbon tạo thành các cation ba
vòng, là đồng phân dia Pa + Pb, các tiểu phân này bị hydrate hóa tạo thành
các alcohol mục tiêu Na + Nb. Ngoài ra, các cation Pa + Pb bị chuyển vị với
sự phân cắt một liên kết đôi carbon-carbon tạo thành các cation Qa + Qb,
các tiểu phân này deproton hóa tạo thành các hợp chất 7a + 7b.

7) Vẽ cấu trúc 3 tiểu phân trung gian Q, Pa, Qa dẫn đến sự tạo thành
đồng phân dia 7a.
8) Vẽ cấu trúc các đồng phân dia Na + Nb.

431 | Bản quyền thuộc về Tạp chí Olympiad Hóa học KEM
Bài 125
Năm 1967, C. J. Pedersen đã công bố một bài báo về tổng hợp ether
vương miện (Crown ether) tạo phức chọn lọc với các ion kim loại kiềm,
mở ra một lãnh vực mới đó là hoá học siêu phân tử.
Gần đây các nhà hoá học đã tổng hợp được một siêu phân tử có cấu trúc
rất đặc biệt, ý tưởng thiết kế dựa vào khả năng tự nhận biết và tự kết nối
của các phân tử. Ba hợp chất vòng sẽ gắn vào một phân tử có khung dạng
chữ Y, sau đó mạch nhánh của ba vòng này sẽ tham gia phản ứng tạo
thành một vòng thứ tư theo minh hoạ sau đây:

1) Biết rằng cấu trúc của hợp chất A và B được cho như sau, hãy cho biết
vai trò của sự “tự nhận biết” và “tự kết nối” thể hiện như thế nào?

2) Trong trường hợp sử dụng catechol và 1,8-dichloro-3,6-dioxooctane


để tổng hợp ether vương miện A bằng phản ứng vòng hoá [2+2], hai
phương pháp đã được sử dụng để làm tăng hiệu suất sản phẩm, chẳng
hạn là pha loãng và them vào ion kim loại. giải thích tại sao thêm kim

432 | Bản quyền thuộc về Tạp chí Olympiad Hóa học KEM
loại lại làm tăng hiệu suất sản phẩm, ví dụ Na+ , K+ , Rb+ , Cs+ . Ion nào sẽ
làm tăng hiệu suất cao nhất? Tại sao?
3) K là hợp chất ban đầu để tổng hợp nên A. Dưới đây là sơ đổ tổng hợp
hợp chất K từ E. Xác định công thức cấu tạo của hợp chất F - K.

4) Hợp chất K cũng có thể được tổng hợp theo con đường dưới đây.
Hoàn thành sơ đồ tổng hợp K từ hợp chất L bằng cách xác định các
sản phẩm còn thiếu L, M, N, O.

433 | Bản quyền thuộc về Tạp chí Olympiad Hóa học KEM
Bài 126
Hoàn thành chuỗi phản ứng sau để tổng hợp một hợp chất huỳnh quang
FL5 (C 32H21N2O7Cl):

434 | Bản quyền thuộc về Tạp chí Olympiad Hóa học KEM
Bài 127
Trong những năm gần đây, hợp chất hydrocarbon thơm đa vòng không
phẳng (PAH) trở thành một trong những lĩnh vực được nghiên cứu khá
nhiều vì những ứng dụng rất có giá trị của chúng trong các vật liệu quang
điện. Corannulene và sumanene là hai trong số các đại diện tiêu biểu,
chúng là các đơn vị cấu trúc trên bề mặt của fullereme C60, đồng thời
cũng là những mảnh cấu trúc của ống nano carbon,

Dưới đây là sơ đồ để tổng hợp một hợp chất như thế, hãy cho biết cấu
trúc các tác chất còn thiếu

435 | Bản quyền thuộc về Tạp chí Olympiad Hóa học KEM
Bài 128
Giải Nobel hoá học năm 2005 được trao cho ba nhà hoá học Chauvin,
Grubbs và Schrock để vinh danh những cống hiến của họ cho nghiên cứu
về phản ứng hoán vị. Phản ứng này được sử dụng rất rộng rãi trong tổng
hợp hữu cơ và ngành công nghiệp hoá học nhất là dược phẩm và nhựa
dẻo. Gần đây, loại người ta đã ứng dụng nó để tổng hợp một hợp chất
thiên nhiên có tên gọi (+)-Angelmarin.
(+)-Angelmarin là một thành phần có hoạt tính sinh học được chiết xuất
từ các thảo mộc ở Trung Quốc. Vào năm 1971, nhóm nghiên cứu của
Franke đã công bố tổng hợp được chất trung gian quan trọng E (xem sơ
đồ 1). Gần đây, nhóm của Coster đã thay thế tác chất mắc tiền 3-chloro-
3-methylbut-1-yne của nhóm Franke bằng một hợp chất allyl rẻ tiền hơn,
mục tiêu tổng hợp được chất trung gian G, sau đó tiến hành phản ứng
Grubbs để tạo thành D, cuối cùng qua một chuỗi phản ứng 4 bước có chọn
lọc đối quang để thu được (+)-Angelmarin.
Sơ đồ 1:

Sơ đồ 2:

1) Xác định công thức cấu tạo các chất trung gian B, C và F.
2) Hoàn thành các tác chất còn thiếu (dấu ?) ở trong hai sơ đồ trên.
3) Giải thích quá trình chuyển hoá từ D sang E trong sơ đồ 1.

436 | Bản quyền thuộc về Tạp chí Olympiad Hóa học KEM
4) Trong sơ đồ 2, bước chuyển hoá từ F sang G còn tạo ra một sản phẩm
phụ là đồng phân của G. Hãy cho biết công thức cấu tạo của chất đó
5) Hợp chất 3-chloro-3-methylbut-1-yne được sử dụng làm nguyên liệu
ban đầu trong sơ đồ 1. Từ các chất hữu cơ đơn giản hãy tổng hợp chất
này (không quá ba giai đoạn).

437 | Bản quyền thuộc về Tạp chí Olympiad Hóa học KEM
Bài 129
Acutumine là một loại alkaloid azolethane được phân lập lần đầu từ các
thảo dược ở Trung Quốc bởi hai nhà hoá học Goto và Sudzuki năm 1929.
Hợp chất này có rất nhiều hoạt tính sinh lý quan trọng, tuy nhiên việc tổng
hợp nó vẫn là một vấn đề khó và hiện đang thu hút sự quan tâm của rất
nhiều nhà khoa học. Trong quá trình nghiên cứu tổng hợp acutumine,
chuỗi phản ứng sau đây đã được tiến hành:

1) Xác định cấu trúc của các sản phẩm A, B, C và điều kiện phản ứng D.
Hợp chất (5) khi được đun nóng trong tert-butyl ancol thì tạo thành sản
phẩm cuối cùng (6) qua 3 trung gian E, F, G. Biết rằng E và G chứa 3 nhóm
carbonyl, trong khi đó F chỉ chứa 2 nhóm carbonyl.

438 | Bản quyền thuộc về Tạp chí Olympiad Hóa học KEM
2) Xác định cấu trúc của E, F, G.
3) Đề nghị cơ chế tạo thành sản phẩm (6) từ G bằng mũi tên cong.
4) Khi cho hợp chất (1) tác dụng với tác chất Grignard (7), sau đó xử lý
với CH3I và CH3COOH thì thu được sản phẩm cuối cùng (8) có công
thức như hình vẽ sau:

Hãy viết quá trình phản ứng từ J đến (8).

439 | Bản quyền thuộc về Tạp chí Olympiad Hóa học KEM
Bài 130
Trong hoá trị liệu bệnh AIDS, các thuốc ức chế men dịch mã ngược HIV
protease thường được sử dụng, tuy nhiên tỉ lệ kháng thuốc khá cao. Trong
những năm gần đây, các nhà khoa học đã thiết kế và tổng hợp một loại
ức chế enzyme HIV dựa trên sự gắn kết với biotetrahydrofuran ligand.
Hợp chất trung gian quan trọng (4) trong tổng hợp này có thể được điều
chế theo hai con đường:
Con đường 1:

Phản ứng từ D tạo thành E và F là phản ứng hoán vị olefin (giảnh giải Nobel
Hóa học năm 2005). Dưới đây là một ví dụ của phản ứng này (phức chất
Ru là đóng vai trò là chất xúc tác):

Con đường 2:

440 | Bản quyền thuộc về Tạp chí Olympiad Hóa học KEM
1) Xác định cấu tạo của các hợp chất B, D, E, F, G (không cần quan tâm
đến hoá học lập thể) và điều kiện phản ứng A, C.
2) Xác định cấu tạo của các hợp chất H, J, M, N (không cần quan tâm đến
hoá học lập thể) và điều kiện phản ứng I, K, L.
Một đồng phân lập thể của hợp chất (4) là (4-1) khi tiến hành oxi hoá với
điều kiện thích hợp thì thu được Q, sau đó khử với NaBH4/EtOH thì thu
được sản phẩm (4-2)

3) Vẽ công thức lập thể của hợp chất Q và (4-2).

441 | Bản quyền thuộc về Tạp chí Olympiad Hóa học KEM
Bài 131
Các hợp chất A, B có thể được sử dụng trong quy trình tổng hợp nhiều
hợp chất có hoạt tính sinh học. Một trong những phương pháp truyền
thống nhất để tổng hợp A ở qui mô phòng thí nghiệm được trình bày
trong sở đồ sau:

Khi đun hồi lưu B với dung dịch formaldehyde khi có mặt p-
toluenesulfonic acid, tiếp theo đó sục oxygen qua dung dịch trong vài
tiếng thì thu được một chất H có công thức phân tử C20H14N4. Phổ 1H NMR
của chất này chứa 3 tín hiệu với tỉ lệ cường độ là 4:2:1.
Thu được chất J khi xử lí liên tiếp chất A với POCl3+DMF, sau đó là TiCl4 +
Zn (phản ứng McMurry), rồi xử lí với oxygen. Nó có cùng công thức phân
tử với H, nhưng phổ 1H NMR thì khác, có 4 tín hiệu với tỉ lệ cường độ
2:2:2:1.
1) Xác định các chất A - J.
2) Giải thích tại sao tert-butyl pyrocarbonate (Bo2O) được dùng trong
tổng hợp G.
3) Tất cả các quá trình động lực học nội và liên phân tử trong chất H và J
trở nên rất chậm ở 140 K. Có bao nhiêu nguyên tử nitrogen không
tương đương có thể được phân biệt bởi phổ 15NMR dưới những điều
kiện này?

442 | Bản quyền thuộc về Tạp chí Olympiad Hóa học KEM
Bài 132
Năm 1992, các nhà Hoá học Đức đã tổng hợp được một hợp chất hữu cơ
mới D là heptacyclo [13.13.21.15 .28.22 .13.27.16.10.113.17.120.40]-hexatriaconta-
1,3(33),6,8,10(34),13, 15,17 (35), 20,22,24(36),27-dodecaene. Biết rằng
phân tử D tính đối xứng bậc cao, và công thức đơn giản nhất là CH. Dưới
đây là sơ đồ tổng hợp D.

1) Xác định công thức cấu tạo của A, B, C, D.


2) Giải thích tại sao trong phản ứng A  B thì methanol được thêm vào
dung môi (tetrahydrofuran THF).
3) Dung dịch D trong tetrahydrofuran được trộn lẫn với dung dịch bạc
triflate CF3SO3Ag, tạo thành các tinh thể trắng E. Kết quả phân tích
nguyên tố của E như sau: (C) = 61.25 %; (H) = 5.00 %; (Ag) = 14.87
%. Xác định thành phàn của E và đề nghị cấu tạo của hợp chất này.

443 | Bản quyền thuộc về Tạp chí Olympiad Hóa học KEM
Bài 133
Hoạt tính xúc tác cao của các phức chất zirconocene (ví dụ như cấu trúc
dưới đây) trong các phản ứng polymer hoá propylene đã dẫn tới sự phát
triển nhanh chóng của lĩnh vực Hoá học này. Hiện nay, các zirconocene có
thể trùng hợp tới 25000 gam propylene trên mỗi gam xúc tác trong 1 giờ
với độ đẳng cấu 97 %. Tuy nhiên, ứng dụng của xúc tác trong công nghiệp
đòi hỏi không chỉ hoạt tính cao, mà còn phải có giá thành rẻ và dễ tổng
hợp.

Năm 2001, Công ty Shell đã đưa ra một phương pháp mới để tổng hợp
các phối tử tương ứng với phức chất này với hiệu suất cao:

1) Xác định cấu trúc của X, X1, X2.


2) Đề xuất cơ chế tạo thành X, chú ý đến phản ứng cộng liên tiếp của các
tác nhân vào hỗn hợp phản ứng.
3) Hợp chất X2 tạo thành bằng cách nhiệt phân chứ không phải bằng
cách xử lí với p-TsOH. Xác định cấu trúc của sản phẩm Z nhận được
trong phản ứng sau và đề xuất cơ chế phản ứng (Z phản ứng với 1
đương lượng bromine.)

444 | Bản quyền thuộc về Tạp chí Olympiad Hóa học KEM
4) Hợp chất Y có hai dạng đồng phân: racemate và meso. Vẽ cấu trúc của
cả hai dạng này.

445 | Bản quyền thuộc về Tạp chí Olympiad Hóa học KEM
Bài 134
Từ thập niên 1990, xúc tác cho các phản ứng hữu cơ bằng phức kim loại
chuyển tiếp đã trở thành một trong những phương pháp chính trong
phòng thí nghiệm lẫn công nghiệp. Một số giai đoạn phản ứng trong sơ
đồ sau được thực hiện với các xúc tác kim loại chuyển tiếp. Trong sơ đồ
này, D và H là các đồng phân, có cùng công thức phân tử C24H12. Cả hai
chất đều có hai tín hiệu trong phổ 1H NMR và 4 tín hiệu trong phổ 13C
NMR. Hoàn thành sơ đồ và xác định công thức cấu tạo của các hợp chất
A - I.

Cho biết các dữ kiện trong phổ NMR


A: 1H: δ 0,26 (s), 13C: 4 tín hiệu
B: 1H: δ 3,17 (s), 13C: 3 tín hiệu
C: 1H: δ 0,42 (s); 7,43 (s) và tỉ lệ cường độ là 9:1
H: 13C: 4 tín hiệu, 1H AA’XX’ (2 mũi đa 1/1) tâm tại 7,46 ppm và 7.22 ppm
I: 1H: δ 4,19 (s), 6,84 (m), 6,92 (m) (1:1:1); 13 C: 4 tín hiệu.

446 | Bản quyền thuộc về Tạp chí Olympiad Hóa học KEM
Bài 135
Vào năm 2001, viên nghiên cứu hoá học hữu cơ, đại học Gottingen - Đức,
đã cùng với các cộng sự tại đại học St. Petersburg (Nga) tiến hành tổng
hợp lần đầu tiên một hydrocarbon A có cấu trúc đối xứng hình tứ diện.
Sơ đồ sau trình bày một vài giai đoạn trong chuỗi tổng hợp đó:

Khi xử lí E với diazomethane khi có mặt palladium acetate thì tạo thành F
với hiệu suất thấp, lặp lại quá trình trên 3 lần có thể tăng lượng F tạo
thành khoảng 80%, chất này sau đó tự chuyển hoá thành A với hiệu suất
khoảng 10%. Chỉ khi tiến hành khoảng 6 lần như thế thì mới thu được
lượng A với hàm lượng khoảng 92 %. Hợp chất F cũng có thể thu được với
hàm lượng khoảng 9-30% khi cho E tác dụng với CH2I2/AlCl3. Khi tiến hành
hydrogen hoá A với xúc tác PtO2 thì lại thu được hợp chất G.
Dữ kiện phổ NMR của một số hợp chất như sau:
A: 1H: 2 tín hiệu (0.15-0.23 và 0,36-0.46 ppm); 13C: 3 tín hiệu
E: 1H 3 tín hiệu (0.27-0.37, 0.78-0.87 và 5.09-5.59 ppm với tỉ lệ cường độ
là 4:1:3).; 13C 5 tín hiệu
F: 1H 3 tín hiệu (0.25-0.36, 0.54-0.60 và 5.04-5.47 ppm với tỉ lệ cường độ
là 4:1:1); 13C 5 tín hiệu
G: 1H 2 tín hiệu (1.04, 2.23 ppm với tỉ lệ cường độ là 6:1); 13C 3 tín hiệu
1) Xác định cấu trúc của nhóm thế R trong hợp chất B.
2) Hãy viết cấu trúc của các hợp chất A, C, D, E, F và G.

447 | Bản quyền thuộc về Tạp chí Olympiad Hóa học KEM
Bài 136
Trong thiết kế tổng hợp các hợp chất năng lượng cao, sự kết hợp các vòng
nhỏ căng và các nhóm chức như NO 2 và N3, là hướng đi đầy triển vọng.
Hợp chất X được tổng hợp đầu tiên vào năm 2001 và đại diện đầu tiên
của các mononitrotriangulane. Triangulane là các hợp chất chứa các vòng
3 cạnh dạng spiro. Quy trình tổng hợp chất X được tiến hành theo sơ đồ
sau:

1) Hoàn thành sơ đồ chuyển hóa trên.


2) Phản ứng giữa chất B và D thuộc loại nào?
3) Gọi tên hợp chất X theo danh pháp hệ thống IUPAC.
4) Viết công thức cấu tạo của dirhodium tetraacetate, biết rằng tất cả
các nguyên tử hydrogen trong phân tử tương đương với nhau.
5) Nếu sử dụng 2-(chloromethyl)butene-1 C' thay vì C thì thu được hợp
chất X’ ở dạng hỗn hợp đồng phân lập thể. Xác định công thức của X’
(không cần quan tâm đến hóa lập thể). Hợp chất X tồn tại bao nhiêu
đồng phân lập thể? Xác định cấu trúc của chúng.
6) Đề xuất một sơ đồ tổng hợp chloride (C) từ tert-butyl alcohol (không
quá 3 giai đoạn).

448 | Bản quyền thuộc về Tạp chí Olympiad Hóa học KEM
Bài 137
Suốt nhiều năm, hydrocarbon X đã thu hút sự quan tâm của các nhà Hoá
học trong việc xác định cấu trúc. X có tính đối xứng của phân tử benzene,
và phổ 1H NMR chỉ có duy nhất một tín hiệu ở 8.8 ppm. Hợp chất này
được tổng hợp lần đầu vào năm 1932, bởi một quy trình nhiều giai đoạn
phức tạp với hiệu suất < 1%. Năm 1940, một quy trình tổng hợp đơn giản
hơn đã được đề xuất, đi từ 7-methtyl-a-tetralone với hiệu suất 1.8 %.

Chú thích: воздух = không khí


Xử lí ketone A với sodium trong ethanol tạo thành sản phẩm B1. Còn nếu
dùng hỗn hống sodium trong ethanol thì thu được sản phẩm B2. Phản
ứng của A với hỗn hống kẽm và hydrochloric acid tạo thành sản phẩm B3.
Khác biệt về hàm lượng oxygen trong A, B1, B2 < 0.5 %.
1) Viết các phương trình phản ứng tạo thành B1, B2, B3. Cho biết sản
phẩm nào được tạo thành trong quy trình tổng hợp hydrocarbon X.
2) Xử lí B với sulfuric acid thì hai sản phẩm đồng phân sẽ được tạo thành,
chứ không phải C. Vẽ cấu trúc các sản phẩm này và xác định sản phẩm
chính, phụ.
3) Vẽ cấu trúc các chất C, D, E, X trong sơ đồ phản ứng.
4) Có bao nhiêu đồng phân lập thể của D và E tạo thành trong quy trình
tổng hợp này?
5) Tương tác giữa hydrocarbon X với 1,3,5-trinitrobenzene tạo thành sản
phẩm rắn F. Xác định cấu trúc của F.
Năm 2005, quy trình tổng hợp hiện đại X đã được công bố, dựa trên cơ
sở ứng dụng các xúc tác kim loại chuyển tiếp. Sản phẩm mục tiêu có hiệu
suất toàn phần khoảng 84 %:

6) Xác định công thức cấu tạo của G, H và I, biết I là đồng phân của X.

449 | Bản quyền thuộc về Tạp chí Olympiad Hóa học KEM
Bài 138
Iodine có thể tạo thành các hợp chất vô cơ lẫn hữu cơ trong đó nó thể
hiện nhiều trạng thái oxid hóa: -1, 0, +1, +3, +5 và +7.
1) Dẫn ra ví dụ về các hợp chất trong đó iodine tồn tại ở các trạng thái
oxid hóa +1, +3, +5 và +7 và trình bày phương pháp điều chế chúng.
Các hợp chất hữu cơ có chứa iodine có thể có cấu trúc vòng bất thường.
Dưới đây là sơ đồ tổng hợp một trong số chúng (hợp chất X):

Mặc dù các phân tử Y và Z có thành phần định tính khác nhau nhưng cấu
trúc của chúng đều rất tương đồng với cấu trúc chất X. Dưới đây là sơ đồ
điều chế Y, Z:

(экв = đương lượng, изб = dung dịch)


Một số đặc trưng của các hợp chất A, C, D, X-Z được cho trong bảng dưới
đây:

2) Xác định cấu trúc các hợp chất A-D, X-Z.

450 | Bản quyền thuộc về Tạp chí Olympiad Hóa học KEM
Bài 139
Năm 1881, A. Hoffmann khám phá ra rằng khi xử lí acetamide với bromine
và dung dịch kiềm dẫn đến sự tạo thành methylamine. Cơ chế của phản
ứng này, gọi là chuyển vị Hoffmann, được cho trong sơ đồ dưới đây:

Nhiều amide tham gia vào phản ứng, tuy nhiên, trong trường hợp nhóm
thế hydrocarbon mạch dài (nhiều hơn 8 nguyên tử) thì các sản phẩm phụ
thường được tạo thành cùng với amine mục tiêu. Ví dụ, khi xử lí lauric
acid (C 11H23CONH2) với bromine và kiềm thì hỗn hợp các hợp chất A, B và
sản phẩm phụ undecylamine được tạo thành.
1) Xác định cấu trúc các hợp chất A, B biết rằng hàm lượng carbon,
hydrogen, nitrogen trong hợp chất A là 72.73 %, 12.12 %, 7.07 % và
trong hợp chất B là 75.00 %, 13.04 %, 7.61 %.
Tuy nhiên, undecylamine có thể nhận được với hiệu suất cao bằng cách
tiến hành phản ứng của lauryamide với bromine và sodium methoxide
trong methanol, và sản phẩm C tạo thành được xử lí tiếp với dung dịch
kiềm.
2) Xác định cấu trúc của C.
Chuyển vị Hoffmann thường được sử dụng trong tổng hợp các hợp chất
có hoạt tính sinh học và cấu trúc bất thường, ví dụ như phân tử X có dạng
hình cầu. Dưới đây là sơ đồ tổng hợp:

451 | Bản quyền thuộc về Tạp chí Olympiad Hóa học KEM
3) Giải mã sơ đồ tổng hợp X, chú ý rằng khối phổ của hợp chất G có một
mũi (MH+ ) ở m/z 571 và chất I chứa 62.1 % carbon, 10.3 % hydrogen
là chất lỏng, trong phổ 1H NMR chỉ có duy nhất 1 tín hiệu.

452 | Bản quyền thuộc về Tạp chí Olympiad Hóa học KEM
Bài 140
Các hợp chất nitro đóng vai trò quan trọng trong hóa hữu cơ. Ví dụ, sự
chuyển hóa của nitrobenzene là một trong những quá trình công nghiệp
quan trọng nhất. Các nitroalkane có tính acid cao; các anion tạo thành
trong quá trình deproton hóa tham gia vào phản ứng với nhiều tác nhân
electrophile. Các nitroalkene dễ tấn công các nucleophile theo phản ứng
Michael.
1) Từ danh sách các tác nhân khử trong Phiếu trả lời, hãy chọn ra các tác
nhân cho phép chuyển nitrobenzene thành aniline. [Bỏ qua]
2) Trong phản ứng của nitroethylene với 2-aminoethano, sản phẩm X
được tạo thành, chứa 34.8 % C và 6.3 % H về khối lượng. Xác định cấu
tạo của X.
3) Tùy thuộc vào điều kiện phản ứng của nitromethane với
benzaldehyde mà 3 sản phẩm có thể được tạo thành: Y (57.5 % và 5.4
% H), Z (64.4 % và 4.7 % H) hoặc W (51.4 % C và 4.8 % H). Xác định cấu
tạo các hợp chất Y, Z, W.
Từ các nitroalkane bậc một, dưới tác động của các tác nhân tách nước
(POCl3, RCNO, AcCl, Ac2O, …), thu được các nitrile oxide RCNO, các chất
này tham gia vào phản ứng cộng vòng với các hợp chất không no a=b, còn
nếu không có tác nhân không no thì chúng tạo thành các dimer vòng
(furoxane).

Các phản ứng có sự tham gia của các nitrile oxide đóng vai trò quan trọng
trong tổng hợp Biotin (vitamin H, coenzyme R) - một tác nhân tăng trưởng
quan trọng có trong mọi tế bào sống và điều hòa protein và chuyển hóa
chất béo ở đó. Dưới đây là hai phương pháp để tổng hợp Biotin ở dạng
racemate.

453 | Bản quyền thuộc về Tạp chí Olympiad Hóa học KEM
4) Xác định công thức cấu tạo các hợp chất A - I.

454 | Bản quyền thuộc về Tạp chí Olympiad Hóa học KEM
Bài 141
Các bromide của phosphorus và hệ Br2/P là những tác nhân hữu hiệu để
đưa bromine vào các hợp chất hữu cơ một cách chọn lọc.
1) Xác định cấu tạo các sản phẩm A, B, C trong phản ứng sau:

Trong tự nhiên, bromine tồn tại ở dạng hai đồng vị với số khối 79 và 81
theo tỉ lệ xấp xỉ 1:1.
2) Ion phân tử (M+ ) của sản phẩm A trong khối phổ tương ứng với hai
vạch (doublet). Tính các giá trị m/z (m - khối lượng, z - điện tích) với
mỗi vạch. Có bao nhiêu vạch tương ứng trong ion phân tử của sản
phẩm B? Tính các giá trị m/z và cường độ tường đội của các vạch này.
Hệ Br2/P được sử dụng để tổng hợp hợp chất hoạt tính cao X, chất này
phản ứng nhanh với nhiều nucleophile và khi không có các tác nhân này
thì chuyển thành các hợp chất I, J, K - một trong số chúng làm mất màu
nước bromine.

Một phương pháp thay thế để điều chế X là nhiệt phân hợp chất Y:

Biết rằng D chứa 77.7 % Br về khối lượng. Trong khối phổ của D, E, F lần
lượt có các mũi bốn, mũi ba và mũi đơn tương ứng với các ion phân tử. Tỉ
lệ khối lượng các phân tử là MX : MI : MJ : MK = 1:2:2:3. Với chất I, phổ 1H
NMR quan sát được một mũi đơn và trong phổ 13C NMR có 3 tín hiệu.
Phân tử I có 3 mặt phẳng đối xứng. Chất J thuộc nhóm lactone. Phổ 1H và
13 C NMR của I và K chứa cùng số tín hiệu với các độ chuyển dịch hóa học

gần nhau.
3) Xác định cấu tạo các chất D - K và X.
4) Phản ứng tạo thành các chất I và J từ X thuộc loại phản ứng nào?

455 | Bản quyền thuộc về Tạp chí Olympiad Hóa học KEM
Bài 142
Năm 1995, nhà hóa học Pháp Remi Chauvin - con trai của Yves Chauvin
(chủ nhân giải Nobel Hóa học 2005) - đã đề ra thuật ngữ “carbo-benzene”.
Carbobenzene là những phân tử được mở rộng, tạo thành bằng cách chèn
các đơn vị C(sp)-C(sp) giữa các liên kết của benzene mà vẫn duy trì sự đối
xứng của phân tử. Các phân tử này đã thu hút được nhiều sự chú ý của
các nhà hóa học, vật lí, sinh học do tiềm năng ứng dụng trong lĩnh vực
quang học phi tuyến tính và quang động học trị liệu.
Dẫn xuất carbobenzene J được tổng hợp vào năm 2008 theo sơ đồ sau:
1) (H3C)3Si Li
PhOC Br2 (C2H5)3N K2CO3
COPh A B C D
CH3COOH C16H10O2 2) CH3I CH3OH 1) C4H9Li
2) CH2O
3) H3O+
SnCl2 1) PhMgBr MnO2 D
J I + H G F E C24H22O4
C54H30 HCl 2) H3 O C46H32O6 Cu2C2 MnO2

1) Xác định công thức các hợp chất A - J.


2) Trong hợp chất H có bao nhiêu tâm bất đối? Hợp chất này có bao
nhiêu đồng phân dia?

456 | Bản quyền thuộc về Tạp chí Olympiad Hóa học KEM
Bài 143
Caffeine là một alkaloid của purine, chất này có tác động kích thích lên các
hệ thống thần kinh và tim mạch. Nó được cô lập từ café và trà, và cũng
được tạo thành từ uric acid U mà có nhiều phương pháp đã được đề xuất
để tổng hợp chất này. Phổ biến nhất là phương pháp Traube, trong đó U
được điều chế từ cyanoacetate qua sự tạo thành các hợp chất đơn vòng
B - D, với D nhận được cả uric acid (hydroxyxanthine) và các dẫn xuất
purine khác là xanthine X và 8-methylxanthine Y.

Hợp chất A có tầm quan trọng đặc biệt trong lịch sử hóa hữu cơ, là sản
phẩm có sản lượng lớn trong công nghiệp hóa chất, được dùng làm phụ
gia thực phẩm. A chỉ có duy nhất 1 tín hiệu trong cả phổ 1H NMR lẫn 13C
NMR. Hàm lượng carbon trong A là 20.0 %.
1) Xác định cấu tạo các hợp chất A - D, X, Y.
Trong công nghiệp, tổng hợp caffeine Z từ uric acid U được tiến hành theo
chuỗi phản ứng sau:

Hàm lượng carbon trong E, F, Z lần lượt là 51. 9%, 34.7 % và 49.5 %. Phổ
1 H NMR của hợp chất F chứa 3 tín hiệu cường độ bằng nhau.

2) Xác định các chất E, F, Z.


Một quy trình tổng hợp công nghiệp khác của caffeine Z là methyl hóa
xanthine X, thực hiện qua sự tạo thành sản phẩm trung gian theobromine
T:

Theobromine là một trong những chất chuyển hóa của caffeine cùng với
các đồng phân theophylline (T') và paraxanthin (P). Trong phổ 1H NMR
của P, T và T’, quan sát được các nhóm tín hiệu sau: P 3.15 (s), 3.84 (d),
7.90 (t), 11.82 (s) ppm; T 3.34 (s), 3.83 (d), 7.97 (t), 11.10 (s) ppm; T' 3.25
(s), 3.44 (s), 8.01 (d), 13.50 (d) ppm. Khi xử lí với trimethylchlorosilane, có
mặt base, thì theobromine tạo thành 2, paraxanthin chỉ 1, còn
theophylline thì không tạo thành dẫn xuất O-trimethylsilyl.
3) Xác định cấu tạo các hợp chất P, T, T’.

457 | Bản quyền thuộc về Tạp chí Olympiad Hóa học KEM
Ngoài các phương pháp tạo thành caffeine kể trên, một phương pháp đi
từ 1-methyl-4-nitro-imidazole-5-carboxylate cũng có thể được sử dụng.

4) Xác định các hợp chất G, H, I.

458 | Bản quyền thuộc về Tạp chí Olympiad Hóa học KEM
Bài 144
Dưới đây là giản đồ chuyển hóa các chất A-K. Trong một số trường hợp,
bên cạnh phản ứng là tên phản ứng hoặc tác nhân được sử dụng.

Xác định công thức cấu tạo các hợp chất B-J.

459 | Bản quyền thuộc về Tạp chí Olympiad Hóa học KEM
Bài 145
Phản ứng Michael dùng để tạo liên kết C-C liên hợp bằng cách gắn C-
nucleophile (Nu) và electrophile E+ vào liên kết đôi hoạt hóa:

Viết công thức cấu tạo của carboxylic acid I và II, được tạo ra bằng phản
ứng Michael dưới đây, biết rằng trong cả hai hợp chất không có carbon
bậc 3.

1) Sử dụng tác nhân electrophile (Е+ ) cho phép tạo ra 2 liên kết C-C mới
và là ví dụ cho chuyển hóa song song tiếp đôi: α-alkyl hóa bằng phản
ứng Michael.
2) Đề nghị một chuyển hóa tiếp đôi để thu được acid III. Biết rằng alkyl
hóa đến bước thứ 2 xảy ra để tác nhân alkyl hóa đến gần trung tâm
nucleophile ở phía nguyên tử carbon có nhóm thế nhỏ hơn (hướng
này ít khó khăn hơn).

Nhà chiến lược tổng hợp hữu cơ kiệt xuất E. Corey đã sử dụng một quá
trình tiếp đôi “Michael-α-alkyl hóa” như là một phương pháp “lắp ráp” có
hiệu quả các cấu trúc phức tạp. Prostaglandin tự nhiên - prostacyclin (IV)
thu được theo sơ đồ sau (R = t-BuMe2Si - nhóm bảo vệ để tăng sự chọn
lọc lập thể của phản ứng):

460 | Bản quyền thuộc về Tạp chí Olympiad Hóa học KEM
3) Xác định công thức cấu tạo các hợp chất B-E. Chỉ ra cấu hình tuyệt đối
của mỗi nguyên tử carbon bất đối trong IV.
Dưới đây là sơ đồ tổng hợp các “khối cấu trúc” ban đầu:

4) Xác định công thức cấu tạo của các chất F - L.

461 | Bản quyền thuộc về Tạp chí Olympiad Hóa học KEM
Bài 146
Hydrocarbon X, là phối tử trong hợp chất phức kim loại M, có thể tổng
hợp theo sơ đồ sau:

Các chất A-D là các đồng phân tạo ra từ 4-clorobut-1-ene bằng phản ứng
[2+2]-dimer hóa, ngoài ra chỉ C có thể tồn tại ở dạng đồng phân quang
học. D là đồng phân dia của C. Khi đun sôi X với dung dịch KMnO4 acid
hóa, tạo ra một sản phẩm hữu cơ duy nhất là acid hổ phách (succinic acid).
1) Viết công thức cấu tạo các chất C-E và Х.
Trong công nghiệp X được tổng hợp bằng cách dimer hóa hydrocarbon
đối xứng Y có mặt xúc tác nickel. Một hydrocarbon đối xứng Z khác cũng
có thể dimer hóa để tạo ra X, nếu phản có mặt xúc tác của phản ứng hoán
vị, mặt khác phản ứng này không thích hợp trong công nghiệp vì sản lượng
thấp và cái chính là khả năng có Z thấp.
2) Xác định cấu tạo Y và Z. Viết công thức các đồng phân đối xứng khác
của Y và Z.
Phức [МХn] thu được khi xử lí [M(acac)2] với triethylaluminum khi có mặt
X, trong đó [M(acac)2] - acetylacetonate kim loại:

Số càng của phối tử X là số liên kết ngắn trong hợp chất đó. Trong quá
trình tổng hợp số phối trí M không thay đổi. Phản ứng đi kèm với sự tách
ra một hỗn hợp sản phẩm khí phụ và tạo ra sản phẩm phụ - hợp chất phức
Q, chứa 10.6% Al và 56.7% C.
3) Tính giá trị của n? Tìm М, biết rằng trong hợp chất phức [MXn] %C =
69.9%. Biểu diễn cấu trúc [MXn].
4) Xác định công thức phân tử của phức Q, viết phương trình phản ứng.
Radialen là các hydrocarbon vòng thú vị, trong đó mỗi nguyên tử carbon
của vòng đều có liên kết đôi exocylic. Dưới đây là công thức chất đầu dãy
đồng đẳng của chúng-[3]radialen.

462 | Bản quyền thuộc về Tạp chí Olympiad Hóa học KEM
Khi sử dụng [MXn] dưới dạng xúc tác oligomer hóa, thu được hydrocarbon
I từ J và K, thuộc loại radialen:

5) Xác định cấu trúc F-K, nếu tất cả trừ H, chỉ có một tín hiệu 1H NMR, và
tỉ lệ khối lượng mol: МI : MJ : MK = 1:2:3.

463 | Bản quyền thuộc về Tạp chí Olympiad Hóa học KEM
Bài 147
Hoán vị (metathesis, có nguồn gốc từ tiếng Hi Lạp meta tithemi - "thay
đổi các vị trí") là phản ứng trao đổi các nhóm nguyên tử giữa các phân tử.
Năm 2005, R. Grubbs, R. Shrok và I. Shovene đã giành được giải Nobel
Hóa học cho những nghiên cứu về phản ứng hoán vị giữa các alkene - trao
đổi các "mảnh" cấu trúc tạo thành các liên kết đôi (A=B + C=D → A=C +
B+D), sử dụng các hệ xúc tác phức chất kim loại chuyển tiếp. Hai ứng dụng
quan trọng nhất của phản ứng hoán vị là: a) tổng hợp các vòng cỡ vừa và
cỡ lớn; b) trùng hợp mở vòng các cycloalkene.
1) Viết phản ứng hoán vị có sự tham gia của: a) ethylene và but-2-ene;
b) dodeca-1,11-diene.
Phản ứng trùng hợp mở vòng norbornene (A) tạo thành polymer A1 - chất
này rất khó tổng hợp bởi các phương pháp khác. Các hệ xúc tác cũng đã
được phát triển, cho phép thực hiện hoán vị các alkyne mà không làm ảnh
hưởng gì tới các liên kết đôi. Trùng hợp B tạo thành polymer B1 được
dùng trong lĩnh vực điện tử.

2) Vẽ cấu trúc A1 và B1.


Trimer của norbornadiene (C) được tạo thành ở dạng hỗn hợp đồng phân
dia. Phản ứng của C với lượng dư ethylene khi có mặt xúc tác hoán vị CM
dẫn tới sự tạo thành hợp chất D, chất này bị oxid hóa bởi 2,3-dichloro-
5,6-dicyano-1,4-benzoquinone (DDQ) tạo thành hydrocarbon S với cấu
trúc không phẳng.

3) Giả sử rằng sự tạo thành các đồng phân dia C xảy ra theo phân bố
thống kê. Hãy viết các cấu trúc của đồng phân dia chính và phụ của C.
Xác định tỉ lệ của chúng.

464 | Bản quyền thuộc về Tạp chí Olympiad Hóa học KEM
4) Hoàn thành sơ đồ phản ứng bằng cách xác định cấu trúc của D và S.
5) D chỉ được tạo thành từ duy nhất một đồng phân C. Xác định đồng
phân này.
6) Xác định cấu trúc của các dẫn xuất monomethyl quang hoạt và không
quang hoạt của S.

465 | Bản quyền thuộc về Tạp chí Olympiad Hóa học KEM
Bài 148
Ngưng tụ là một trong những nhóm phản ứng quan trọng nhất, do chúng
có thể tạo thành các bộ khung carbon phức tạp hơn. Phản ứng ngưng tụ
thường được khởi phát bởi base, một anion được tạo thành sẽ phản ứng
với tác nhân electrophile.
1) Viết các anion được tạo thành từ phản ứng của NaH với các phân tử
sau:

Gần đây, các anion đã được sử dụng nhiều để tiến hành các phản ứng mà
trước đây vốn rất khó để thu được bởi phản ứng ngưng tụ truyền thống.
Các dianion có độ chọn lọc vùng và hoạt tính hóa học cao, với hai tâm
anion kém bền tham gia vào phản ứng với các tác nhân electrophile. Ví
dụ, quá trình tổng hợp radulanin H (hợp chất thiên nhiên có khoảng tác
động sinh lí rộng, kí hiệu là X), gồm 3 giai đoạn tạo thành dianion (tổng
hợp A, B và F).

2) Viết công thức dianion của acetoacetic ether (CH3COCH2CO2C2H5).


3) Hoàn thành sơ đồ tổng hợp chất X, biết rằng: a) chất C chứa 7 nguyên
tử hydrogen, b) các hợp chất D và E có liên kết hydrogen nội phân tử
nhưng chất F thì không có; c) D và F dễ chuyển hóa thành E bởi phản
ứng ngưng tụ Claisen khi đun nóng.
Từ hợp chất E, thu được radulanin E là đồng phân của X.

466 | Bản quyền thuộc về Tạp chí Olympiad Hóa học KEM
Viết công thức cấu tạo các hợp chất J - L và Y.

467 | Bản quyền thuộc về Tạp chí Olympiad Hóa học KEM
Bài 149
Các phân tử thiophene là những khối cấu trúc hiệu quả để sản xuất bán
dẫn hữu cơ do chúng có cấu trúc không linh hoạt và tương đối bền nhưng
có thể dễ gắn thêm nhóm chức. Một dẫn xuất của thiophene - đã được
tổng hợp theo sơ đồ dưới đây vào năm 2011 - được dùng để sản xuất các
bán dẫn hữu cơ.

1) Hợp chất thơm nào - thiophene hay benzene - có hoạt tính cao hơn
trong các phản ứng thế electrophile?
2) Phản ứng nitro hoá thiophene tạo thành một hỗn hợp dẫn xuất
mononitro theo tỉ lệ xấp xỉ 100:1. Viết công thức cấu tạo các sản phẩm
chính.
3) Viết công thức cấu tạo của amine RNH2 biết phổ 13C NMR của nó chứa
3 tín hiệu (phân tử có 3 loại carbon).
4) Xác định các chất còn thiếu trong quy trình tổng hợp, biết rằng phổ 1H
NMR của D chứa 4 tín hiệu với cường độ tương đối là 2:2:3:3.

468 | Bản quyền thuộc về Tạp chí Olympiad Hóa học KEM
Bài 150
Alkaloid crispine A thể hiện độc tính với tế bào (cytotoxicity) cao và được
xem là một trong những tác nhân có hiệu quả nhất trong điều trị ung thư
- điều này đã kích thích việc phát triển các phương pháp tổng hợp crispine
A.

ee là độ trội đối quang (enantiomer excess), nghĩa là chênh lệch hàm


lượng giữa hai đối quang.
1) Xác định công thức cấu tạo của A - I, biết rằng C là hợp chất ba vòng.
Phổ 1H NMR của I được cho dưới đây. Trong Phiếu trả lời, hãy gắn các
tín hiệu trong phổ với từng phần tương ứng trong phân tử I.

469 | Bản quyền thuộc về Tạp chí Olympiad Hóa học KEM
Một mẫu crispine A được tổng hợp bởi một trong các phương pháp được
đề xuất, có góc quay quang [α] D 43.9o (CH3OH).
2) Tính tỉ lệ các đối quang của crispine A trong mẫu.

470 | Bản quyền thuộc về Tạp chí Olympiad Hóa học KEM
Bài 151
Các vinylboronic acid và dẫn xuất của chúng là những tác nhân giá trị trong
tổng hợp hữu cơ. Các hợp chất này bị ghép cặp chéo, có xúc tác Pd0, với
các aryl và vinyl halide (phản ứng Suzuki). Phản ứng này giữ nguyên cấu
hình của liên kết C=C. Tuy nhiên, khi sử dụng các vinylboronic acid để tổng
hợp vinyl halide, thì đáng chú ý là quá trình này có thể giữ nguyên hoặc
nghịch đảo cấu hình, phụ thuộc vào điều kiện phản ứng.
1) Hoàn thành sơ đồ phản ứng sau đây, biết rằng các đồng phân D và E
chứa 93.3 % carbon, hợp chất E có nhiều hơn D 1 mặt phẳng đối xứng.

Phản ứng Suzuli được sử dụng phổ biến để tổng hợp các polyene tự nhiên.
Quá trình tổng hợp bombykol (I) và các đồng phân lập thể Ia, Ib của nó
được cho trong sơ đồ dưới đây.

2) Hoàn thành sơ đồ phản ứng, viết cấu trúc các chất F-O, Ia, Ib.

471 | Bản quyền thuộc về Tạp chí Olympiad Hóa học KEM
Bài 152
Các hợp chất hữu cơ có liên kết N-N không tồn tại nhiều trong tự nhiên.
Năm 1966, hợp chất achiral W (C12H12N2) có hoạt tính sinh học đã được
cô lập từ rễ cây Withania somnifera (nhân sâm Ấn Độ). Hợp chất này được
gọi là withasomnine và là thành phần quan trọng của thuốc Ayurvedic.
Năm 1994, một dẫn xuất dihydro của withasomnine đã được cô lập từ cây
bụi Newbouldia laevis. Hợp chất N này có công thức C 12H14N2, thường
được gọi là newbouldine, tồn tại trong tự nhiên ở dạng đồng phân (S,S).
Dưới đây là các phương pháp để tổng hợp W và (±)-N. Phản ứng cộng
vòng [3+2] là những giai đoạn then chốt trong các tổng hợp này.

B là hợp chất bicyclic dạng mesoionic (nghĩa là hợp chất dị vòng có các
nguyên tử nằm ngoài vòng, với điện tích được giải tỏa và không thể biểu
diễn cấu trúc ở dạng trung hòa). Cả W và N đều có 10 tín hiệu trong phổ
13 C NMR. Dưới đây là phổ 1 H NMR của W:

1) Hoàn thành sơ đồ phản ứng. Xác định công thức cấu tạo của các chất
A - H, N, W.

472 | Bản quyền thuộc về Tạp chí Olympiad Hóa học KEM
2) Gán các tín hiệu trong phổ 1H của withasomnine với cấu trúc tương
ứng.
3) Trong phản ứng của E với 5-(methylsulfanyl) tetrazole có thể tạo ra
chất F với bao nhiêu đồng phân?

473 | Bản quyền thuộc về Tạp chí Olympiad Hóa học KEM
Bài 153
Phản ứng chuyển vị Vinylcyclopropane-thành-cyclopentene (VCR) và
những phản ứng cùng kiểu của các hợp chất dị tố tương ứng được xem là
phương pháp quan trọng để tổng hợp các hợp chất đồng và dị vòng 5
cạnh. Các chuyển vị này có thể được thực hiện trong điều kiện nhiệt,
quang hóa hoặc hoạt hóa xúc tác. Các phản ứng nhiệt và quang hóa xảy
ra qua sự hình thành gốc đôi bền nhất.

1) Xác định công thức cấu tạo của sản phẩm tạo thành từ hai phản ứng
trên, biết rằng: a) cả hai phản ứng đều thuộc loại VCR; b) phân tử Y
có trục đối xứng bậc hai.

VCR là giai đoạn đầu tiên trong tổng hợp toàn phần terperne hydrocarbon
(±)-zizaene (Z):

2) Tính độ bất bão hòa của zizaene (Z)?


3) Zizaene có bao nhiêu đồng phân?
4) Hoàn thành sơ đồ tổng hợp này. Xác định công thức cấu tạo các hợp
chất A - L.

474 | Bản quyền thuộc về Tạp chí Olympiad Hóa học KEM
Bài 154
Các liên kết C-H có tính acid mạnh của nhóm carbonyl được sử dụng rộng
rãi trong quy trình tổng hợp nhiều hợp chất hữu cơ mới và phức tạp. Ví
dụ:

Các hợp chất D1 và D2 là sản phẩm chính khi lượng allyl chloride được
dùng dư gấp đôi. Ozone phân sản phẩm A, sau đó xử lí ozonide với kẽm
và acetic acid, tạo thành formaldehyde và hợp chất E (C 5H8O2). Xử lí C
theo cách tương tự, thu được aceton và F (C 3H4O2). Hợp chất C được tạo
thành từ phản ứng dehydrate hóa B. Phổ 1H NMR của D2, E và F lần lượt
chứ 5, 4 và 2 tín hiệu, với tỉ lệ cường độ tương ứng là 2 : 2 : 1 : 1, 3 : 2 : 2
: 1 và 3 : 1.
1) Xác định công thức cấu tạo của A, B, C, D1, D2, E và F.
Một trong những phương pháp tốt nhất để thực hiện phản ứng alkyl hóa
chọn lọc vùng các ketone là hoạt hóa nguyên tử α-carbon. Ví dụ:

2) Xác định công thức cấu tạo của các hợp chất G - L.
Các nhóm phụ trợ có thể được dùng cho quá trình hoạt hóa chọn lọc các
nguyên tử α-carbon có nhóm thế:

475 | Bản quyền thuộc về Tạp chí Olympiad Hóa học KEM
3) Xác định công thức cấu tạo các hợp chất M - T.
Để tổng hợp các cycloalkanone với hai nhóm thế geminal (trên cùng 1
carbon), thường sử dụng phương pháp “chắn” (blockage) một trong các
nguyên tử α-carbon. Trong hình thì μW là bức xạ vi sóng.

4) Xác định công thức cấu tạo của các hợp chất U - X, Y1, Y2 nếu biết Y1
và Y2 là các đồng phân bicyclic. Y2 không có nhóm carbonyl.

476 | Bản quyền thuộc về Tạp chí Olympiad Hóa học KEM
Bài 155
Một phương pháp hiệu quả để tổng hợp các hợp chất vòng là dùng phản
ứng cộng vòng, trong đó có 2 hoặc nhiều liên kết đôi được tạo thành trong
một giai đoạn. Theo những kiến nghị của IUPAC, các phản ứng cộng vòng
có thể được phân loại theo số nguyên tử tham gia vào phản ứng, theo
kiểu: cộng vòng-(m+n+…)

1) Xác định các công thức cấu tạo của A, B, biết М(А) / М(В) = 0.75.
2) Hãy cho biết phản ứng cộng vòng tạo thành A và B thuộc loại nào?
Có thể tăng hiệu quả của các phản ứng cộng vòng bằng cách dùng xúc tác
phức chất kim loại chuyển tiếp. Cơ chế tạo thành hợp chất A và các đồng
đẳng của nó khi có phức chất MLn được mô tả như sau:

3) Xác định các công thức cấu tạo của D, E với phản ứng được xúc tác
bởi (C 5H5)Co(CO)2; biết rằng: a) E là đồng đẳng của A, phổ 1H NMR có
mũi đôi và mũi đa với cường độ tương đối 6:1; b) tiểu phân trung gian
D có thể được xem là sản phẩm của phản ứng cộng vòng (2+2+1), có
chứa 15.86 % cobalt.
4) Xác định sản phẩm chính của phản ứng giữa A với 2 đương lượng RBr
khi có mặt AlBr3 dưới điều kiện khống chế động học.
Các phản ứng cộng vòng khi có mặt phức chất kim loại chuyển tiếp cho
phép: a) tổng hợp các cấu trúc đa vòng từ các chất đầu không vòng; b) tạo
vòng, đồng thời đưa thêm nhóm chức vào.

5) Xác định công thức cấu tạo của F, G, C 10H12, H, K.

477 | Bản quyền thuộc về Tạp chí Olympiad Hóa học KEM
Có một nhóm phản ứng cộng vòng cho phép tổng hợp các cấu trúc đa
vòng đa dạng, với sự hình thành nhiều vòng chỉ trong một giai đoạn tổng
hợp. Trong quy trình tổng hợp estrone, các vòng ba đã được tạo thành
trong 1 giai đoạn phản ứng:

6) Xác định công thức cấu tạo của L, I, N, O, P, Q, R, S biết rằng: II, P, Q
là các đồng phân; Q có nhiều hơn P 2 vòng.

478 | Bản quyền thuộc về Tạp chí Olympiad Hóa học KEM
Bài 156

Tháng 12/2015, khi nghiên cứu sự bùng phát bệnh não do hạ đường huyết
ở trẻ em tại Muzaffarpur (Ấn Độ), các chuyên gia đã bất ngờ phát hiện ra
sự bùng nổ dịch tễ này không phải bởi lây nhiễm mà là ngộ độc α-amino
acid X (C 6H9NO2) có trong hạt vải. Amino acid này gây sụt giảm đáng kể
lượng đường trong máu, có thể dẫn tới tử vong. Amino acid này được các
nhà Hoá học Anh cô lập lần dầu vào năm 1962. Cấu trúc của nó được xác
định từ dữ liệu hydrogen hoá xúc tác khi có xúc tác Adams. Cụ thể, phản
ứng hydrogen hoá X cần 1 đương lượng dihydrogen, tạo thành hỗn hợp
hai acid Y1 và Y2 là đồng phân dia. Hydrogen hoá X với 2 đương lượng H2
tạo thành hỗn hợp leucine, norleucine và isoleucine. Cấu trúc của X được
xác định rõ từ phổ 1H NMR, trong đó mũi đa tương ứng với hai proton ở
5.7 ppm.

Về sau, các phương pháp tổng hợp acid X đã được phát triển. Năm 2006,
các nhà khoa học tại Đại học Quốc gia Moscov đã đề xuất phương pháp
tổng hợp các đồng phân dia X’ (của X) theo sơ đồ dưới đây. Hợp chất
trung gian quan trọng C có thể được tạo thành theo hai cách khác nhau.
Trong cách thứ nhất, chất đầu là methallyl chloride; còn cách thứ hai thì
dùng methyltriphenylphosphonium iodide

479 | Bản quyền thuộc về Tạp chí Olympiad Hóa học KEM
1) Xác định cấu trúc amino acid X, chỉ rõ cấu hình tuyệt đối của các tâm
chiral (R/S). Gọi tên chất này theo danh pháp IUPAC.
2) Xác định công thức cấu tạo của Y1, Y2.
3) Hoàn thành sơ đồ tổng hợp hỗn hợp racemic các đồng phân dia X’,
biết rằng: a) A’ là hợp chất khí ở điều kiện thường, trong phổ 1H NMR
của nó có 2 tín hiệu còn phổ 13C NMR có 3 tín hiệu; b) hợp chất hai
vòng E chứa nguyên tử phosphorus phối trí 5; c) hợp chất G là dẫn
xuất của imidazole (vòng 5 cạnh chứa 2 nguyên tử nitrogen).

480 | Bản quyền thuộc về Tạp chí Olympiad Hóa học KEM
Bài 157
Phản ứng oxide hoá alkene là phương pháp phổ biến nhất để tạo nhóm
chức trên liên kết đôi C-C trong tổng hợp hữu cơ.

1) Vẽ cấu trúc của hợp chất A và các đồng phân lập thể Ia, Ib nếu biết cả
Ia và A đều có mặt phẳng đối xứng còn Ib thì không.
Sự chọn lọc lập thể của phản ứng epoxide hoá các cyclohexene (có 1 nhóm
thế ở vị trí allylic) phụ thuộc vào bản chất nhóm thế này.

2) Vẽ cấu trúc các hợp chất B-D nếu sự epoxide hóa được quyết định bởi
hiệu ứng không gian (một trong số các phản ứng trên) hoặc sự tạo
thành liên kết hydrogen nội phân tử (phản ứng còn lại), trong đó
nhóm thế là phần tử cho (donor) trong liên kết hydrogen.
Sharpless đã phát triển một phản ứng epoxide hoá chọn lọc đối quang của
các allylic alcohol bằng cách dùng t-BuOOH làm chất oxide hoá và
Me(OR)n làm xúc tác. Trong đó, Me là kim loại chuyển tiếp và OR là một
phối tử chiral. Cụ thể, phối tử này là (+) hoặc (-)-diethyl tartrate [kí hiệu:
(+)-DET và (-)-DET]. Cấu trúc lập thể của (-)-DET được biểu diễn theo công
thức chiếu Newman sau đây:

3) Vẽ cấu trúc của (-)-DET theo công thức chiếu Fischer. Phân tử này là
D- hay L-hydroxyacid?
Hỗn hợp racemic của (+)- và (-)-DET được tổng hợp từ but-2-enedioic acid
(II) bởi phản ứng với RCO 3H, rồi xử lý với dung dịch NaOH.

481 | Bản quyền thuộc về Tạp chí Olympiad Hóa học KEM
4) Vẽ cấu trúc của hợp chất II.
5) Xác định công thức của alcoholate Me(OR)n nếu phổ 1H NMR chứa hai
tín hiệu (mũi đôi và mũi bảy với cường độ tương đối là 1:6) và C =
50.7 %.
Áp dụng quy tắc Sharpless để xác định đối quang oxiran chính, được tạo
thành từ phản ứng trên. Theo quy tắc này, phân tử dẫn xuất allyl alcohol
được biểu diễn theo cách dưới đây. Nếu phản ứng epoxide hoá dùng (-)-
DET thì vòng epoxide được tạo thành trên mặt phẳng alkene; còn nếu
dùng (+)-DET thì vòng được tạo thành dưới mặt phẳng này.

Phản ứng oxide hoá (Z)-tridec-2-en-1-ol (E) bởi hệ t-BuOOH/Me(OR)n/(-)-


DET, tạo thành hợp chất F, là giai đoạn quan trọng trong tổng hợp
disparlure (pheromone của loài bướm đêm Lymantria dispar.)
6) Vẽ cấu trúc các hợp chất E, F. Chỉ ra cấu hình tuyệt đối (R/S) của các
tâm chiral trong phân tử F.

482 | Bản quyền thuộc về Tạp chí Olympiad Hóa học KEM
Bài 158

Các -amino acid vòng với hai nhóm thế ở vị trí α, α là nhóm amino acid
phi tự nhiên quan trọng. Một trong những phương pháp hiệu quả nhất
để tổng hợp các amino acid như vậy là dùng chuyển vị allyl cyanate thành
isocyanate theo sơ đồ dưới đây:

Cụ thể, phương pháp này được dùng để tổng hợp hợp chất Z, là chất ức
chế hiệu quả trong tổng hợp cathepsin C và có hiệu quả cao trong điều trị
chứng giãn phế quản và viêm mạch máu.

1) Xác định công thức cấu tạo của các hợp chất A - I, X, Y, Z. Biết rằng:
a) Tác nhận HATU thường được dùng để tạo liên kết peptide -CO-NH-
; b) Dess-Martin periodinane (DMP) là tác nhân oxide hoá êm dịu; c)
Y là ion lưỡng cực.
Chuyển vị Allyl cyanate thành isocyanate cũng được áp dụng để tổng hợp
ketamine (Ketalar) - thuốc gây mê cho người và động vật.

2) Viết công thức cấu tạo của ketamine và các hợp chất K - Q.

483 | Bản quyền thuộc về Tạp chí Olympiad Hóa học KEM
Bài 159
Pentacycloanammoxic acid là thành phần của phopsholipid 1, tạo thành
màng tế bào của anammoxosome (vi cơ quan trong quá trình anammox.)
Anammox (oxide hoá ammonium kị khí - ANaerobic AMMonium
OXidation) là quá trình vi khuẩn quan trọng trong chu trình nitrogen. Quá
trình này (NH4+ + NO2- → N2 + 2H2O) diễn ra rất chậm và có các tiểu phân
trung gian là những chất độc hại N2H4, NH2OH, cùng với các gốc hoạt
động. Chính vì vậy, màng tế bào anammoxosome được hình thành bởi các
phospholipid bất thường để ngăn chặn sự khuếch tán các chất trung gian
độc hại này vào trong tế bào. Trong giai đoạn 2004 - 2006, Corey và các
cộng sự đã tổng hợp được pentacycloanammoxic acid ở cả dạng racemic
lẫn các dạng đồng phân quang hoạt riêng lẻ. Tuy nhiên, họ không tiếp tục
nghiên cứu về hợp chất thú vị này. Chỉ đến năm 2016, nhóm nghiên cứu
đến từ Đại học Stanford mới phát triển phương pháp tổng hợp các hợp
chất như vậy, gồm cả lipid 1.

Chìa khoá thành công cho quá trình tổng hợp này là việc phát triển dựa
vào phản ứng dimer hoá hydrocarbon F (sơ đồ 2). Cần chú ý đến phản ứng
hydroborane hóa hydrocarbon H, sau đó là ghép cặp Zweifel. Chuỗi
chuyển hoá trong quá trình ghép cặp này được cho ở sơ đồ 1, với ví dụ về
phản ứng của hợp chất cơ boron quang hoạt, vinylmagnesium bromide
và iodine.

Sơ đồ 1: ghép cặp Zweifel

484 | Bản quyền thuộc về Tạp chí Olympiad Hóa học KEM
Sơ đồ 2: Quá trình tổng hợp pentacycloanammoxic acid.
1) Viết công thức cấu tạo các hợp chất A - K nếu biết.
- A, B, C, F, H đều có mặt phẳng đối xứng.
- Hợp chất D có 2 dạng đồng phân dia; cả hai đồng phân này đều có
mặt phẳng đối xứng.
- Hợp chất E có 4 dạng đồng phân dia; tất cả các đồng phân này đều
quang hoạt.
2) Đề xuất công thức cấu tạo của các tiểu phân trung gian X1, X2 và cơ
chế chuyển hoá cuả X2.
3) Đề xuất những yếu tố tiến hoá tác động đến sự xuất hiện loại màng
tế bào ladderane bất thường này của anammoxosomes trong tự
nhiên. Chọn phát biểu đúng:
a) Các tiểu phân trung gian hoạt động mạnh của anammox xúc tác
cho quá trình sinh tổng hợp các ladderane.
b) Bộ khung ladderane cho phép hình thành các màng tế bào siêu
đặc.
c) Các tiểu phân trung gian anammox hoạt động mạnh là tiền chất
trong sinh tổng hợp ladderane.

485 | Bản quyền thuộc về Tạp chí Olympiad Hóa học KEM
d) Ladderane có thể phản ứng với các tiểu phân trung gian hoạt động
mạnh của anammox.

486 | Bản quyền thuộc về Tạp chí Olympiad Hóa học KEM
Bài 160
Năm 1984, nhóm nghiên cứu của John McMurry (nhà hoá học Mỹ, tác giả
của cuốn giáo trình “Organic Chemistry” nổi tiếng, vừa ra mắt ấn bản thứ
9) đã tổng hợp được các hợp chất đa vòng có cầu nối thú vị I và II, theo
sơ đồ sau:

1) Trong tổng hợp của I, chuyển hoá của D thành II được gọi là phản ứng
McMurry. Nếu C 3H6O (III) được sử dụng trong phản ứng (thay vì D) thì
tạo thành hợp chất C 6H12 (IV). Xác định III và IV, biết phổ IR của III có
một dải hấp thụ mạnh ở 1715 cm-1 và chỉ có duy nhất 1 mũi đơn được
xác định trong phổ 1H NMR của IV.
Các hợp chất I và II được đặc trưng bởi kiểu đồng phân lập thể bất thường,
được gọi là đồng phân in/out.
2) Vẽ cấu trúc của Ia và Ib (các đồng phân lập thể in,in- và out,out- tương
ứng của I).
3) Xác định cấu trúc của A-D, RMgCl, II và đề xuất các tác nhân cho giai
đoạn II → I biết phổ 1H NMR của tác nhân Grignard RMgCl (ωC = 59.24
%, ωMg = 10.90%) gồm 1 mũi đơn, 1 mũi ba, 1 mũi ba, 1 mũi bốn, 1
mũi bốn, 1 mũi đa ( δ = 7.5 ppm) với tỉ lệ cường độ tương đối là
2:2:2:2:2:5 và B chứa một liên kết đôi nội vòng (endocyclic). Xác định
cấu hình Z/E của B.

[V] = [Va Vb]


4) Xác định cấu trúc của [V], Va, Vb, X. Đồng phân lập thể in/out nào của
I (Ia hay Ib) có thể thể hiện hoạt tính tương tự?

487 | Bản quyền thuộc về Tạp chí Olympiad Hóa học KEM
Bài 161
Thuật ngữ “synthon” được E.J.Corey đưa ra vào năm 1967. Synthon là
một đơn vị cấu trúc của một phân tử thực hoặc được lí tưởng hoá mà có
thể được đưa vào quy trình tổng hợp hoá học bởi các kĩ thuật đã biết.
Synthon không tồn tại ở dạng một tác nhân cụ thể, nó là hạt hình thức
(ion, gốc hoặc carbene). Mỗi synthon có thể liên quan đến một hoặc nhiều
tác nhân thực, và ngược lại, mỗi tác nhân có thể là “đương lượng tổng
hợp” cho nhiều synthon. Một ví dụ điển hình cho các tác nhân kép như
vậy là formaldehyde, vừa có thể là C 1-electrophile, nhưng khi chuyển hoá
thành 1,3-dithian I thì trải qua phân tích tổng hợp ngược đảo cực thành
C 1-nucleophile. Tác nhân nucleophile thực trong trường hợp này là hợp
chất cơ lithium II.

1) Xác định các chất A-D, F, G, II.


Hợp chất II được ứng dụng thành công trong tổng hợp chọn lọc các hợp
chất đồng và đa nhóm chức.

2) Xác định các chất H, X, J, K, L biết I, J, L và C là các đồng phân, trong


phổ 1H NMR có các tín hiệu ở ~ 10 ppm, mũi đôi (với L và C) và mũi ba
(với J).
Việc sử dụng dithiane I dẫn đến sự phát triển của một phương pháp tổng
hợp “nhanh” hiệu quả các hợp chất tự nhiên đa nhóm chức, ví dụ như

488 | Bản quyền thuộc về Tạp chí Olympiad Hóa học KEM
kháng sinh vermiculin (IV), được phân lập từ nấm Talaromyces
vermiculatus.
3) Xác định các chất M-T từ sơ đồ sau, biết rằng khối lượng mol của IV
thấp hơn 8.64 % so với Q. Các chất T là IV có cấu trúc vòng giống nhau,
phân tử T có số nguyên tử hydrogen lớn gấp hai lần và số nguyên tử
oxygen bằng một nửa so với VI, và số nguyên tử carbon trong T nhiều
hơn 60 % so với VI.

489 | Bản quyền thuộc về Tạp chí Olympiad Hóa học KEM
Bài 162
Sumanene (tiếng Hindi nghĩa là “hoa”) là hydrocarbon liên hợp X (C 21H12),
có thể được xem như một mảnh fullerene. Những nỗ lực để tổng hợp X
bắt đầu từ năm 1993 bằng cách nhiệt phân chất tương ứng A. Tuy nhiên,
trong quá trình nhiệt phân, chỉ có các hợp chất B và C được tạo thành với
hiệu suất thấp.

1) Xác định cấu trúc của sumanene X và các hợp chất A-C.
Quy trình tổng hợp thành công của sumanene X đã được ghi nhận vào
năm 2003. Tổng hợp bắt đầu với hydrocarbon F, có 3 tín hiệu trong phổ
1 H NMR (tỉ lệ 2:1:1) và 3 tín hiệu trong phổ 13 C NMR. F có thể được điều

chế từ phản ứng cộng hợp đẳng mol của hydrocarbon G và khí E (wC =
92.3 %). Giai đoạn tiếp theo là biến tính F để tạo thành một hợp chất G có
2 nhóm chức, chất này dưới tác động của muối copper(I) tạo thành một
hỗn hợp của các hydrocarbon H1 và H2 theo tỉ lệ 1:3 với công thức phân
tử C 21H18. Sản phẩm chính H2 dưới điều kiện của phản ứng hoán vị olefin
tạo thành hydrocarbon J. Còn sản phẩm phụ H1 dưới những điều kiện
tương tự tạo thành hợp chất I, chất này bị oxid hoá tạo thành hợp chất
mong muốn sumanene X.

490 | Bản quyền thuộc về Tạp chí Olympiad Hóa học KEM
2) Xác định cấu trúc các hợp chất D-J.
Sumanene đã được tổng hợp có nhiều tính chất thú vị. Trong phổ 1H NMR
của sumanene ở 25 oC có 2 mũi đôi ở 3.42 và 4.71 ppm, khi đun nóng tới
140 oC thì chúng hợp lại thành một mũi rộng. Để nghiên cứu chi tiết về
hiện tượng này, một dẫn xuất deuterium hoá của Y đã được tạo thành
theo sơ đồ dưới đây. Phổ 1H NMR của hợp chất Y ở nhiệt độ thường có 2
tín hiệu ở 3.42 và 4.71 ppm có cường độ bằng nhau, cho thấy Y tồn tại ở
hai dạng. Tần số chuyển giữa hai dạng này lần lượt là 0.066 s-1 và 0.007 s-
1 ở 318 K và 298 K.

3) Xác định cấu trúc của Y và anion Z.


4) Xác định cấu trúc hai dạng của Y, tương ứng với các tín hiệu NMR: 3.42
và 4.71 ppm.
5) Tính năng lượng hoạt hoá phải vượt qua để chuyển từ dạng này qua
dạng khác của Y.

491 | Bản quyền thuộc về Tạp chí Olympiad Hóa học KEM
Bài 163
Azanthracene là các hợp chất thiên nhiên thu hút được nhiều sự quan
tâm trên phương diện những đặc tính dược lí. Ví dụ, năm 1999, loại cây
Hao-Laam trồng ở Thailand đã được ghi nhận là có chứa Markanin A (I),
có khả năng chống ung thư và sốt rét. Năm 2000, Kalasinamide (II) được
phân lập từ cây Polyalthia suberosa, được sử dụng là tiền chất của
Markanin A. Quy trình tổng hợp đơn giản các hợp chất này từ
naphthoquinone-1,4 đã được các nhà khoa học Đức công bố năm 2009.

Hoàn thành sơ đồ tổng hợp Kalasinamide (II) và Markanin A (I). Xác định
cấu tạo các chất A - J. Biết rằng ammonium cerium (IV) nitrate - sử dụng
trong giai đoạn cuối của quy trình - là một chất oxid hóa mạnh.

492 | Bản quyền thuộc về Tạp chí Olympiad Hóa học KEM
Bài 164
Phân đoạn nhiệt độ sôi thấp của dầu mỏ và quá trình luyện cốc có chứa
hydrocarbon I, được chiết xuất bằng phương pháp tinh cất (tsôi 40.8 oC).
Khi để nguyên, I bị đồng phân hóa chậm thành sản phẩm II, chất này khi
đun nóng lại chuyển trở về I. Đồng đẳng của I, hợp chất III (tosôi 71.3 oC)
được Wilcox điều chế được lần đầu tiên vào năm 1960 qua một chuỗi
tổng hợp nhiều giai đoạn từ 2-methylcyclohexanol.

1) Xác định cấu tạo các hợp chất I - III, A - K.


Một nhà nghiên cứu thử điều chế trực tiếp J từ G bằng phản ứng
dehydrate hóa bởi acid, nhưng lại thu được L thay vì J.
2) Xác định cấu tạo của L.
Trong dung dịch benzene ở 20 oC, I tham gia vào một phản ứng tỏa nhiệt
với các đồng phân hình học của M và N. Trong trường hợp đầu tiên, hai
hợp chất O (85 %) và P (15 %) được tạo thành. Còn trong trường hợp thứ
hai chỉ có duy nhất sản phẩm Q.
3) Xác định cấu tạo của M - Q, biết rằng O, P, Q đều có công thức C 9H10O4.
4) Vẽ (định tính) giản đồ năng lượng của phản ứng I với M, biết rằng O
kém bền về mặt nhiệt động học hơn so với P.

493 | Bản quyền thuộc về Tạp chí Olympiad Hóa học KEM
Bài 165
Tổng hợp các Platonic hydrocarbon - các hợp chất có bộ khung carbon tạo
thành những đa diện đều - vẫn là thách thức với tổng hợp hữu cơ hiện
đại. Dodecahedrane, có công thức đơn giản nhất là CH, là một phân tử có
hình đa diện dodecahedron - 12 mặt (theo tiếng Hi Lạp “δώδεκα” có nghĩa
là “12” và “εδρον” là “mặt”). Nỗ lực đầu tiên để tổng hợp chất này được
R. Woodward tiến hành vào năm 1964. Ông đã đề xuất một ý tưởng -
nghe rất đơn giản nhưng đáng tiếc là không thực hiện được - là dimer hóa
hydrocarbon X. Bản thân hydrocarbon này rất thú vị, chủ yếu là bởi tính
đối xứng cao. X được Woodward tổng hợp theo sơ đồ sau:

1) Xác định cấu tạo các hợp chất A - I và hydrocarbon X, biết rằng: a) B
và C là các đồng phân, có cùng số vòng và không có liên kết C=C; b) E
được xếp vào nhóm hemiacetal vòng; c) Trong phổ 1H NMR của hợp
chất X chỉ có 3 tín hiệu (mũi đôi, mũi đôi-của-mũi ba và mũi bốn) với
tỉ lệ cường độ 6:3:1).
Chất đầu được tổng hợp từ các hydrocarbon Y, Z sẵn có:

2) Xác định cấu tạo các hợp chất Y, Z, J - L. Biết rằng K và L là các đồng
phân lập thể. Chú ý: HBSia 2 là bis(3-methylbutan-2-yl)borane

494 | Bản quyền thuộc về Tạp chí Olympiad Hóa học KEM
Bài 166
Phản ứng dehalogen hóa các α, ω-dihalide bởi kim loại thường được sử
dụng để tổng hợp các hợp chất vòng carbon. Ví dụ:
Br-(CH2)n-Br + Zn → (CH2)n + ZnBr 2 (1)
Trong phản ứng này, với nồng độ dibromide giống nhau thì hiệu suất
cycloalkane tạo thành phụ thuộc vào
a) sức căng của vòng tạo thành;
b) khả năng hội tụ của các đầu mạch dibromide để thực hiện vòng
hóa.
Không giống như cyclohexane, có sức căng vòng xem như bằng 0, đa số
các cycloalkane khác đều đặc trưng bởi các giá trị sức căng xác định. Dưới
đây là các giá trị sức căng vòng (kJ/mol) trên mỗi nhóm CH2 của
cyclopropane, cyclobutane và cyclodecan là:
1.2 6.5 9.2
1) Hãy cho biết các giá trị sức căng trên tương ứng với cycloalkane nào?
Hiệu suất của các cycloalkane trong phản ứng 1 là
(CH2)3 - 80% (CH2)4 - 7% (CH2)6 - 45% (CH2)10 - 8%
2) Xác định xem yếu tố nào (a hay b) có tác động quyết định đến hiệu
suất của sản phẩm tổng hợp của mỗi hydrocarbon: cyclopropane,
cyclobutane và cyclodecane.
Một phương pháp phổ biến khác để điều chế các hợp chất vòng là sử dụng
malanic ether, ví dụ:

3) Xác định cấu tạo của các chất A và B.


Các phản ứng 1 và 2 được sử dụng để tổng hợp hydrocarbon G.

4) Xác định cấu tạo các hợp chất C - G và xác định giá trị của n. Biết rằng
G chứa 3 loại hydrogen theo tỉ lệ 1:1:1.

495 | Bản quyền thuộc về Tạp chí Olympiad Hóa học KEM
Bài 167
Hợp chất quang hoạt A (công thức C 8H14Cl2) khi tham gia vào phản ứng
hydrogen hóa xúc tác, có mặt palladium thì bị chuyển thành hợp chất
không quang hoạt B (C 8H16Cl2). Phản ứng của A với ozone tạo thành hợp
chất trung gian C, chất này khi xử lí với hydrogen peroxide thì tạo thành
chloroacetic acid và carboxylic acid D quang hoạt. Xử lí chất trung gian C
với triphenylphosphine tạo thành các sản phẩm E và F.

1) Biết hợp chất D có cấu hình tuyệt đối R, hãy vẽ cấu trúc lập thể chính
xác của các hợp chất A, B, D, E, F.
2) Vẽ cấu trúc và trình bày cơ chế tạo thành hợp chất trung gian C.
3) Phản ứng của hợp chất A với một lượng xúc tác OsO 4 và lượng dư
hydrogen peroxide dẫn đến sự tạo thành một cặp đồng phân dia G và H.
Vẽ cấu trúc của các hợp chất này và chỉ rõ cấu hình tuyệt đối (R, S) của tất
cả các tâm bất đối.

496 | Bản quyền thuộc về Tạp chí Olympiad Hóa học KEM
Bài 168
Tetrabenzoporphyrin X cùng các dẫn xuất và
phức chất của nó với kim loại, thu hút nhiều sự
chú ý của các nhà hóa học bởi những đặc tính
hữu dụng. Ví dụ, chúng được dùng trong các
transistor hiệu ứng trường, tế bào quang điện
và các thiết bị điện tử khác. Các ứng dụng này
có liên quan đến khả năng của
tetrabenzoporphyrin trong việc tạo thành các
“ngăn xếp” đặc biệt, do sự xen phủ của các π-orbital của các phân tử lân
cận (gọi là sự xếp chồng π-π). Tuy nhiên, hiệu ứng trên cũng làm giảm tính
tan của các hợp chất này, dẫn tới những phức tạp trong quá trình tổng
hợp, đồng thời gây ra khó khăn cho việc sản xuất các thiết bị điện tử phù
hợp.
Năm 2012, các nhà khoa học Hàn Quốc đã đề xuất phương án để hạn chế
những khó khăn này. Họ đã tổng hợp chất I (xem sơ đồ dưới). Ý tưởng
của họ là hợp chất I với độ tan tương đối tốt có thể được dễ dàng đưa
vào bề mặt mong muốn, sau đó bằng cách đun nóng có thể tạo thành hợp
chất X - là kết quả của việc phân cắt 4 phân tử chất khí Y từ mỗi phân tử
I.

1) Gọi tên một chất tự nhiên trong thành phần có 1 vòng porphyrin.
2) Xác định cấu tạo các hợp chất A - I và Y.
3) Dựa vào các hiệu suất đã cho, hãy xác định xem cần bao nhiêu gam
propargyl bromide (3-bromo-1-propyne) để tạo thành 1 gam hợp chất
I.
4) Trong dung dịch, hợp chất D tồn tại ở 2 cấu dạng cơ bản, D1 và D2,
với cấu dạng D1 chiếm ưu thế trong dung môi phân cực, còn cấu dạng
D2 thì trong dung môi không phân cực. Vẽ các cấu dạng này.

497 | Bản quyền thuộc về Tạp chí Olympiad Hóa học KEM
Bài 169
Thực hiện chuỗi chuyển hóa sau với cycloocta-1,5-diene:

Các chất C và D bị chuyển hóa dưới tác dụng của base III ở 20 oC, tạo thành
E và F - các chất này cũng có thể được tạo thành trực tiếp từ chất B dưới
tác dụng của base III trong điều kiện tương tự. Khi đun nóng ở 50 oC, F
chuyển thành G, còn chất E hầu như không biến đổi. Các hợp chất C - G là
những hydrocarbon đồng phân, đối xứng, mỗi 1 mol các chất này phản
ứng với 4 mol bromine, nhưng chỉ với 1.33 mol KMnO 4 loãng ở 20 oC ở pH
= 7.
1) Hoàn thành chuỗi chuyển hóa. Xác định (không cần quan tâm đến hóa
lập thể) cấu tạo các hợp chất A - G. Chú ý: Không cần phải xác định
cấu tạo của các base I - III.
2) Mỗi hợp chất tạo thành trong các phản ứng này có bao nhiêu đồng
phân lập thể?
3) Viết phương trình phản ứng của G với dung dịch potassium
permanganate, được acid hóa bởi sulfuric acid.
Trong phản ứng của E với diazomethane khi có mặt xúc tác, phân lập được
một hydrocarbon H, chứa 6 loại nguyên tử carbon (theo phổ 13C NMR) -
điều này có thể được giải thích bởi một trục đối xứng bậc hai trong phân
tử.
4) Xác định cấu tạo của H, chỉ rõ hóa lập thể tương đối của các nguyên
tử chiral carbon.

498 | Bản quyền thuộc về Tạp chí Olympiad Hóa học KEM
Bài 170
Năm 1903, Tollens và von Marle nhận thấy rằng phản ứng của
acetophenone (C 6H5COCH3) với formaldehdye (CH2O) và ammonium
chloride sẽ tạo thành một amine bậc ba X. Những đặc điểm chính của
phản ứng này, với cơ chế được cho dưới đây, đã được nghiên cứu bởi K.
Mannich - chính vì vậy tên của ông được đặt cho phản ứng này. Để phản
ứng xảy ra hiệu quả, R1C(O)R2 - cần phải phản ứng nhanh hơn hợp chất
carbonyl thứ hai - sẽ phải dễ tạo thành dạng enol. Phân tử R1C(O)R2 không
nên chứa hydrogen ở vị trí liền kề nhóm carbonyl.

1) Xác định cấu tạo của sản phẩm X mà Tollens và von Marle thu được.
Phản ứng Mannich đã được sử dụng để tổng hợp alkaloid lasubin II.
Dưới đây là sơ đồ tổng hợp (hỗn hợp racemic) của epimer ở vị trí số 2
của chất này.

2) Xác định cấu tạo các chất A - G, biết rằng C - không giống như đồng
phân D của nó - có thể tham gia vào phản ứng haloform, nhưng
không có phản ứng tráng gương. B là muối. F chứa 1 liên kết đôi
không liên hợp.
Phản ứng Mannich được dùng cho phản ứng aminoalkyl hóa và các loại
hợp chất khác có vai trò như tác nhân nucleophile. Ví dụ, quá trình tổng
hợp tolmetin - một chất chống viêm được sử dụng trong chữa trị viêm,
nhức khớp - có một giai đoạn aminoalkyl hóa N-methylpyrrole.

499 | Bản quyền thuộc về Tạp chí Olympiad Hóa học KEM
3) Xác định cấu tạo các hợp chất H - K.
4) Trong phản ứng acyl hóa L, bên cạnh K thì một lượng nhỏ của sản
phẩm acyl hóa đồng phân (L) cũng được tạo thành. Xác định cấu tạo
L, biết rằng chất này có 4 tín hiệu trong vùng thơm của phổ 1H NMR:
2 mũi đôi và 2 mũi đơn.

500 | Bản quyền thuộc về Tạp chí Olympiad Hóa học KEM
Bài 171
Năm1985, các nhà khoa học Mĩ đã mô tả một hợp chất X có tính đối xứng
cao, chỉ có 2 loại carbon và 3 loại hydrogen. Hợp chất này được tổng hợp
theo sơ đồ dưới đây. Đồng phân chính của sản phẩm phản ứng Diels-Alder
được chuyển thành hợp chất G bởi phản ứng với methanol khi có xúc tác
acid. Theo các tác giả, khi ozone phân G tạo rồi xử lí ozonide với sodium
borohydride thì thu được hợp chất H. Chất này tiếp tục chuyển thành F,
và cuối cùng là sản phẩm mong muốn X. Không có tính chất lí hóa nào của
F - H và X được đưa ra trong báo cáo này.
Năm 2013, các nhà khoa học Pháp đã nghiên cứu tổng hợp này cẩn thận
hơn và thấy rằng trong các điều kiện được mô tả, thay vì H thì một hợp
chất tricyclic (3 vòng) I (C 17H81O5) đã được tạo thành. Tuy nhiên, khi khử
bằng lithium aluminum hydride (rồi sau đó trung hòa) thì vẫn thu được
cùng sản phẩm F. Chất này sau đó được chuyển thành X. Các nhà khoa
học Pháp đã đề xuất điều chỉnh sơ đồ tổng hợp X, theo đó chất B trước
tiên bị khử bởi lithium aluminum hydride tạo thành C. Các bước sau đó
được cho trong sơ đồ:

Xác định cấu tạo các chất A - I và X.

501 | Bản quyền thuộc về Tạp chí Olympiad Hóa học KEM
Bài 172
Năm 2005, giải Nobel Hóa học được trao cho Yves Schoven, Robert
Grubbs và Richard Schrock vì những nghiên cứu về phản ứng hoán vị
alkene, bao gồm việc phát triển các hệ xúc tác hiệu quả cao dựa trên phức
carbene của molybdenum và ruthenium (các xúc tác Shrok and Grubbs
tương ứng). Kết quả của phản ứng này là sự tái phân bố các nhóm thế
quanh liên kết C=C của alkene theo sơ đồ:

Phản ứng hoán vị alkene là quá trình nhiệt động học thuận nghịch.
1) Xác định thành phần hỗn hợp cân bằng của phản ứng hoán vị propene
ở 298 K, sử dụng dữ liệu nhiệt động cho trong bảng và phương trình
ΔrGo = -RTlnKp. Giả sử chỉ có duy nhất đồng phân trans của but-2-ene
được tạo thành trong phản ứng này.
trans-but-2-
Chất ethylene propene
ene
ΔrGo (298
68.14 62.70 62.94
K), kJ/mol
Trong các tổng hợp dược và hợp chất thiên nhiên, một hướng ứng dụng
đặc biệt của phản ứng hoán vị là tạo thành vòng (hoán vị đóng vòng, RCM
- Ring Closing Metathesis). Nhờ phương pháp này, có thể tạo thành các
vòng đa kích cỡ, kể cả những vòng lớn. Dưới đây là ví dụ về phản ứng RCM
của dodecadiene-1.11:

2) Giải thích tại sao phản ứng này có thể tạo thành sản phẩm mong muốn
với độ chọn lọc cao, bất chấp khả năng xảy ra phản ứng nghịch?
Năm 2004, các nhà khoa học Hàn Quốc đã sử dụng phản ứng hoán vị để
tạo vòng trong quy trình tổng hợp chọn lọc lập thể của alkaloid (+) -
allosedamine (X).

502 | Bản quyền thuộc về Tạp chí Olympiad Hóa học KEM
3) Xác định cấu trúc các hợp chất A - G. Chú ý rằng B1/B2 và C1/C2 là các
cặp đồng phân dia. Trong các phân tử B1 và C1 có 1 tâm chiral với cấu
hình R và tâm còn lại có cấu hình S.

503 | Bản quyền thuộc về Tạp chí Olympiad Hóa học KEM
Bài 173
Các kênh kali (potassium) nhạy cảm với ATP có mặt trong các tế bào nội
tạng, tế bào cơ trơn và các mô cơ xương. Các kênh này tham gia vào các
quá trình sinh lí quan trọng: tiết hormone, co cơ trơn, phóng thích các
chất dẫn truyền thần kinh. Hoạt động của các kênh kali có thể được điều
chỉnh với sự trợ giúp của các chất đặc biệt - gọi là chất hoạt hóa. Các chất
như vậy được sử dụng chủ yếu trong y học để điều trị chứng cao huyết
áp và đau thắt ngực, hoặc làm thuốc giãn phế quản.
Một trong những chất hoạt hóa kênh kali phổ biến nhất là cromakalim
(X), được điều chế theo sơ đồ sau từ hợp chất A.

[Mn]* = phức manganese quang hoạt


1) Xác định cấu tạo các hợp chất A, B.
2) Tại sao giai đoạn đầu tiên của tổng hợp này phải sử dụng phức quang
hoạt?
Vào năm 2006, để thu được một loại thuốc giảm tiết insulin trong tế bào
β tụy, đồng thời ảnh hưởng đến các tế bào cơ trơn, các nhà khoa học Bỉ
đã tổng hợp một số hợp chất có cấu trúc tương đồng, cũng chứa một hệ
dị vòng chroman (dihydrobenzopyran) giống như cromakalin. Dưới đây là
sơ đồ tổng hợp một chất như vậy (chất Y, có hiệu quả và độ chọn lọc cao
nhất):

3) Xác định cấu tạo các hợp chất C - G, Y. Biết rằng:


- Trong vùng thơm của phổ 1H NMR, tín hiệu của hợp chất D cao
hơn so với hợp chất C.
- Trong vùng không thơm của phổ 1H NMR, hợp chất E chỉ có 2 tín
hiệu với tỉ lệ cường độ 3:1.

504 | Bản quyền thuộc về Tạp chí Olympiad Hóa học KEM
Năm 2004, các nhà khoa học Đài Loan đã tổng hợp được Z, cấu trúc tương
tự như cromakalim, mặc dù Z không có hệ vòng chroman.

4) Xác định cấu tạo các hợp chất H - N, Z. Biết rằng Z có chứa 2 vòng
(bicyclic) sáu cạnh.

505 | Bản quyền thuộc về Tạp chí Olympiad Hóa học KEM
Bài 174
Các chất ức chế enzym aromatase đóng vai trò quan trọng trong tổng hợp
estrogen, được sử dụng đề điều trị một số loại bệnh, đặc biệt là ung thư
vú. Tuy nhiên, các hợp chất này không phải lúc nào cũng được sử dụng
cho mục đích tốt. Các vận động viên thiếu trung thực có thể sử dụng
chúng làm doping, làm tăng testosterone. Một trong những loại thuốc
chống ung thư thuộc loại này - cũng thường sử dụng bởi những người
“nghiện” doping - là fadrozole (X). Có hai phương pháp để tổng hợp, được
trình bày trong sơ đồ dưới đây:

Các vận động viên thiếu trung thực cũng thường dùng furosemide (Y). Bản
thân chất này không giúp cải thiện kết quả gì, nhưng là một loại thuốc lợi
tiểu, giúp che giấu việc sử dụng chất kích thích. Quá trình tổng hợp
furosemide được trình bày dưới đây:

Hoàn thành các sơ đồ trêm. Xác định cấu tạo của các hợp chất A - P và Y.
Biết rằng:
a) A là sản phẩm trung gian kém bền, bị phân hủy chậm ở nhiệt độ thấp
và nhanh khi đun nóng;

506 | Bản quyền thuộc về Tạp chí Olympiad Hóa học KEM
b) Trong vùng thơm của phổ 1H NMR - được đo bởi một thiết bị có độ
phân giải không quá cao - thì hợp chất B có 6 tín hiệu: 2 mũi đôi, 2 mũi
đôi mở rộng và 2 mũi đa với tỉ lệ cường độ là 2:2:1:1:1:1;
c) Khi không có xúc tác thì các peracid không thể oxid hóa vòng thơm;
d) Phổ hồng ngoại cho thấy trong Y có một liên kết hydrogen nội phân
tử mạnh.

507 | Bản quyền thuộc về Tạp chí Olympiad Hóa học KEM
Bài 175
Nguồn sáng hóa học là những thiết bị tạo ra ánh sáng nhờ các phản ứng
hóa học. Nhờ có những tính chất như kháng nước, mạnh, an toàn cháy nổ
mà các nguồn sáng hóa học được sử dụng rộng rãi trong các hoạt động
cứu hộ khẩn cấp, lặn biển, thám hiểm hang động, du lịch, cũng như trong
mục đích trang trí. Thông thường, các nguồn sáng hóa học chứa hợp chất
C, hydrogen peroxide và một số huỳnh quang hữu cơ. Nguyên lí hoạt động
của chúng là dựa trên sự oxid hóa hợp chất C bởi hydrogen peroxide, dẫn
tới sự tạo thành 1,2-dioxetane-3,4-dione (X) cực kì kém bền, chất này tự
phân hủy thành khí Y mà không tạo thành sản phẩm nào khác. Trong
trường hợp này, lượng nhiệt lớn giải phóng ra được dùng để chuyển các
phân tử huỳnh quang từ trạng thái cơ bản lên trạng thái kích thích. Quá
trình chuyển về trạng thái cơ bản đi kèm với bức xạ có bước sóng tùy
thuộc vào cấu trúc của huỳnh quang.
Hợp chất C có thể được tổng hợp từ salicylic acid theo 3 giai đoạn trong
sơ đồ sau:

1) Xác định cấu trúc các hợp chất A - C và X, cùng công thức phân tử của
khí Y. Biết rằng hợp chất A chứa 44.05 % chlorine về khối lượng và
không có nhóm thế ở vị trí 4 của vòng thơm. X là hợp chất vòng. Khí Y
làm đục nước vôi.
Các hydrocarbon chứa các vòng thơm ngưng tụ cũng được sử dụng làm
chất huỳnh quang trong nguồn sáng hóa học. Ví dụ hợp chất E (bức xạ ánh
sáng xanh dương), I (bức xạ ánh sáng xanh lục) hoặc rubrene (рубрен)
(bức xạ ánh sáng da cam). Sơ đồ tổng hợp các hợp chất này được cho
dưới đây:

508 | Bản quyền thuộc về Tạp chí Olympiad Hóa học KEM
2) Xác định cấu tạo các hợp chất chưa biết trong sơ đồ phản ứng trên.

509 | Bản quyền thuộc về Tạp chí Olympiad Hóa học KEM
Bài 176
Các phản ứng hoán vị alkene - mà những nghiên cứu về nền tảng lí thuyết
và các hệ xúc tác hiệu quả cho chúng đã mang về giải Nobel Hóa học 2005
cho I. Shoven, R. Grubbs và R. Shrok - đã được đưa vào các bài thi
Olympiad Hóa học Nga và Mendeleev. Ở dạng đơn giản hóa, có thể nói
rằng phản ứng hoán vị alkene diễn ra như là một phản ứng cộng vòng
[2+2], sau đó là phân hủy cyclobutane tạo thành 2 alkene. Cơ chế thực sự
của phản ứng được trình bày dưới đây:

Phản ứng hoán vị alkene được nghiên cứu từ năm 1967, và tới năm 1985
thì ví dụ đầu tiên về phản ứng hóa vị với sự tham gia của alkene và alkyne
- diễn ra theo cơ chế tương tự - đã xuất hiện. Phản ứng này được dùng
trong tổng hợp (±) -differolide, được cô lập từ xạ khuẩn Streptomyces
aurantiogriseus. Ngay cả ở nhiệt độ phòng, hợp chất A bị dimer hóa chậm,
tạo thành (±) -differolide và đồng phân cấu tạo B (hai chất này là đồng
phân cấu tạo).

1) Xác định cấu tạo của A và B.


Về sau, các nhà nghiên cứu tiếp tục tìm thấy các loại hợp chất khác cũng
tham gia vào những phản ứng hoán vị tương tự. Ví dụ như các hợp chất
sắt xúc tác cho phản ứng hoán vị có sự tham gia của alkyne và aldehyde.

510 | Bản quyền thuộc về Tạp chí Olympiad Hóa học KEM
2) Xác định cấu tạo của sản phẩm C trong phản ứng hoán vị nội phân tử
của 2-[(but-2-yn-1-yl)oxy]benzaldehyde.
Bên cạnh những đặc tính vượt trội, các phản ứng hoán vị alkene cũng có
nhược điểm: trong nhiều trường hợp, tạo thành hỗn hợp cis- và trans-
alkene. Nhược điểm này rõ ràng là không ảnh hưởng gì đến các phản ứng
hoán vị alkyne. Sau đó, các sản phẩm hoán vị alkyne có thể được khử chọn
lọc để tạo thành đồng phân cis- hoặc trans-alkene. Gần đây, hướng tiếp
cận này đã được sử dụng để tổng hợp Cucujolide XI, pheromone của cây
bách thảo Oryzaephilus surinamensis ở Thổ Nhĩ Kì.

3) Xác định cấu tạo các hợp chất D - P.


4) Xác định cấu hình tuyệt đối của tâm chiral trong Cucujolide XI.

511 | Bản quyền thuộc về Tạp chí Olympiad Hóa học KEM
Bài 177
Trong hóa học hữu cơ hiện đại, một trong những phương pháp chính để
tổng hợp alkyne đầu mạch là sử dụng phản ứng Seyferth-Gilbert - với sơ
đồ và cơ chế phản ứng như sau:

Nhược điểm của phản ứng Seyferth-Gilbert là việc sử dụng base mạnh
như potassium t-butoxide thường dẫn tới các phản ứng phụ.
1) Đề xuất cấu trúc của một sản phẩm phụ có thể tạo thành từ
phenylacetic aldehyde trong các điều kiện của phản ứng Seyferth-
Gilbert.
Có thể tránh các phản ứng phụ của phản ứng này nếu tiến hành theo biến
thể Ohira-Bestmann, khi đó anion Z được tạo thành tại chỗ (in situ) từ
dimethyl-1-diazo-2-oxopropylphosphonate (còn gọi là tác nhân Ochir-
Bestman, kí hiệu là OBR) dưới tác động của potassium carbonate trong
methanol. Điều chỉnh này cho phép giảm thiếu các phản ứng phụ tới mức
tối đa và thu được alkyne mong muốn với hiệu suất cao. Ngoài ra, phản
ứng còn có thể được tiến hành ở nhiệt độ phòng chứ không cần phải ở -
78 oC.

2) Đề xuất cơ chế tạo thành anion Z từ tác nhân Ochir-Bestman.


Năm 2013, các nhà khoa học Brazil đã sử dụng phản ứng Seyferth-Gilbert
ở dạng biến thể Ohira-Bestman để tổng hợp dược chất CMI-977, một loại

512 | Bản quyền thuộc về Tạp chí Olympiad Hóa học KEM
thuốc đầy triển vọng trong chữa trị bệnh hen mạn tính. Dưới đây là sơ đồ
tổng hợp dược chất này:

3) Xác định cấu tạo các hợp chất A - G. Chú ý rằng hợp chất C không bị
khử bởi lithium aluminum hydride.
Một ví dụ khác của phản ứng Seyferth-Gilbert ở dạng biến thể Ohira-
Bestman là trong tổng hợp chọn lọc lập thể alkaloid (-)-stoamide có trong
cây Stemona tuberosa, được dùng trong y học cổ truyền Trung Hoa.

4) Xác định cấu tạo các hợp chất H - Q, biết rằng O là hợp chất 2 vòng
(bicylic) còn P là hợp chất 3 vòng (tricyclic).

513 | Bản quyền thuộc về Tạp chí Olympiad Hóa học KEM
Bài 178
Chuỗi chuyển hóa sau - được gọi là đóng vòng Robinson - đóng vai trò
quan trọng trong tổng hợp các hệ đa vòng phức tạp, bao gồm các hợp
chất thiên nhiên. Giai đoạn đầu tiên của quá trình này là phản ứng cộng
Michael của enolate ion vào ketone không no α, β; sau đó xảy ra phản
ứng ngưng tụ aldol nội phân tử.

1) Xác định cấu tạo sản phẩm X tạo thành từ phản ứng đóng vòng của
vinylmethylketone với menthone, biết rằng nó chứa một nguyên tử
carbon bậc bốn.

Trong tổng hợp tương tự, thay vì enone, cũng có thể sử dụng các muối
ammonium kiểu R3N+ CH2CH2C(O)R’, chất này có thể dễ dàng điều chế từ
ketone, amine và formaldehyde bởi phản ứng Mannich. Một ví dụ về ứng
dụng của phản ứng đóng vòng Robinson là tổng hợp sesquiterpene
valeranone (Z) - một thành phần quan trọng của tinh dầu nước hoa nữ
lang (valerian) - theo sơ đồ dưới đây:

2) Xác định cấu tạo các chất A - G và Z, biết rằng Z có 3 nguyên tử chiral
carbon.
Rễ cây nữ lang là một loại thuốc an thần, đã được thử nghiệm qua nhiều
thế hệ. Thành phần có hoạt tính sinh học chứa trong đó ảnh hưởng đến

514 | Bản quyền thuộc về Tạp chí Olympiad Hóa học KEM
hệ thần kinh trung ương, có hiệu quả tốt với các chứng đau nửa đầu, mất
ngủ, loạn dưỡng cơ.
3) Rễ cây nữ lang có thể được bào chế và bán ra dưới nhiều dạng. Ví dụ
như cồn thuốc chứa 70 % alcohol (rễ cây nữ lang chiếm 5.57 % về khối
lượng), các viên thuốc chứa 0.02 gam chiết xuất velerian, các viên
thuốc chứa rễ cây nữ lang… Người lớn thường được kê toa 3 - 4 lần
mỗi ngày, mỗi lần 20 - 30 giọt cồn thuốc, uống trước khi ăn 20 - 30
phút. Tính lượng thuốc được khuyên dùng mỗi ngày theo khối lượng
rễ cây nữ lang khô. Ước lượng số lượng thuốc viên tương ứng. Cho
biết khối lượng riêng của cồn thuốc là ρ = 0.920 gam/mL và thể tích
mỗi giọt thuốc là V = 20 μL.
4) Xác định cấu hình tuyệt đối của tâm chiral trong Z theo hệ danh pháp
R, S.
Dưới đây là sơ đồ tổng hợp đầu tiên của valeranone, được tiến hành vào
năm 1986.

5) Xác định công thức cấu tạo của các hợp chất H - R, biết rằng L chứa 2
vòng (bicyclic) sáu cạnh, Q là hợp chất 3 vòng (tricyclic), O là sản phẩm
của phản ứng acetoxyl hóa, J chứa 1 nguyên tử lưu huỳnh.

515 | Bản quyền thuộc về Tạp chí Olympiad Hóa học KEM
Bài 179
Trong thập niên 1960, các nhà khoa học Soviet - trong nỗ lực tìm kiếm
một nguồn nhiên liệu tên lửa hiệu quả cao - đã tổng hợp được hợp chất
syntin (danh pháp quốc tế là 1-methyl-1,2-dicyclopropylcyclopropane).
Trong thập niên 1980 - 1990, hợp chất này được sử dụng làm nhiên liệu
cho xe phóng tên lửa Soyuz-U2. So với các loại nhiên liệu tên lửa lỏng
truyền thống, syntin có những ưu điểm là khối lượng riêng lớn, độ nhớt
thấp, nhiệt lượng tỏa ra khi đốt cháy cực kì cao và cực kì bền - nên thời
gian sử dụng của nó là không giới hạn. Tuy nhiên, do chi phí sản xuất đắt
đỏ nên sau khi Liên bang Soviet sụp đổ, việc sản xuất hợp chất này đã bị
ngưng lại. Dưới đây là sơ đồ tổng hợp syntin từ ethanol:

Chất E cũng có thể được điều chế từ levulinic acid - một sản phẩm của
quá trình chuyển hóa sinh khối - theo sơ đồ sau:

1) Xác định cấu tạo của các hợp chất A - K, biết rằng chuyển hóa của G
thành H thuộc loại ngưng tụ aldol-croton.
2) Khi sử dụng phương pháp tổng hợp này, syntin được tạo thành ở dạng
hỗn hợp các đồng phân với tỉ lệ 56:44 với sản phẩm cis chiếm ưu thế,
dù rằng đồng phân này kém bền hơn so với dạng trans (2.5 kJ/mol).
Sinh nhiệt chuẩn của hỗn hợp này - được sử dụng làm nhiên liệu - là -
133.0 kJ/mol. Xác định sinh nhiệt chuẩn của mỗi đồng phân syntin.
3) Viết phương trình phản ứng đốt cháy syntin trong oxygen. Xác định
hiệu ứng nhiệt của phản ứng đốt cháy syntin và lượng nhiệt giải phóng
khi đốt cháy 1 kg syntin nhận được từ sơ đồ trên. Cho biết sinh nhiệt
chuẩn của: hơi nước là 241.8 kJ/mol, carbon dioxide là 393.5 kJ/mol.
Sự phụ thuộc của sinh nhiệt vào nhiệt độ có thể bỏ qua.

516 | Bản quyền thuộc về Tạp chí Olympiad Hóa học KEM
Bài 180
Parkinson là một trong những bệnh thần kinh phổ biến nhất, ảnh hưởng
tới khoảng 1 % những người trên 60 tuổi. Các triệu chứng đặc trưng của
bệnh này là sự cứng cơ, run rẩy, khả năng vận động kém. Bệnh Parkinson
xuất hiện do sự chết của các tế bào thần kinh chịu trách nhiệm tạo ra các
chất dẫn truyền thần kinh dopamine (2-(3,4-dihydroxyphenyl)
ethylamine).
Ngày nay, bệnh Parkinson vẫn chưa thể chữa được, nhưng có một số loại
thuốc làm giảm các triệu chứng bệnh. Thuốc kháng-Parkinson phổ biến
nhất là “levodopa” - chứa hoạt chất L-α-amino acid X, khi đi vào cơ thể
được chuyển thành dopamine do phản ứng decarboxyl hóa.
1) Xác định cấu trúc của dopamine và hợp chất X. Chỉ rõ cấu hình các tâm
quang hoạt.
Một hướng khác để điều trị bệnh Parkinson là sử dụng các loại thuốc ức
chế hoạt động của các enzyme phá vỡ dopamine. Cac loại thuốc này gồm
rasagiline (Y) và selegiline (Z), ức chế enzyme monoamine oxidase loại B
(MAO-B). Dưới đây là sơ đồ tổng hợp các hợp chất này (trong tổng hợp Y,
thu được hỗn hợp racemic):

2) Xác định cấu trúc các hợp chất A - J, Y và Z. Đối với các hợp chất E - J
và Z, hãy xác định rõ hóa lập thể của các tâm bất đối.

517 | Bản quyền thuộc về Tạp chí Olympiad Hóa học KEM
Bài 181
Các steroid hormone đóng vai trò điều tiết các hoạt
động sống của động vật và con người. Đặc điểm chung
của tất cả các steroid là sự có mặt của một hệ 4 vòng
ngưng tụ, với 3 vòng sáu cạnh và 1 vòng năm cạnh
(như hình dưới).
Ví dụ, một trong những hormone giới tính nữ quan trọng nhất là
progesterone (Z), đóng vai trò thiết yếu trong giai đoạn thai kì. Quá trình
tổng hợp toàn phần đầu tiên, gồm 15 giai đoạn, của progesterone được
V. Johnson công bố vào năm 1971. Quy trình này có thể chia thành 2 phần.
Sản phẩm của phần đầu là (E)-4-methyldec-3-en-8-ynal (X), được sử dụng
tiếp trong tổng hợp progesterone.

1) Xác định cấu tạo các hợp chất A - C, chú ý rằng quá trình chuyển A
thành B xảy ra theo từng gia đoạn, bao gồm chuyển vị Claisen (còn gọi
là chuyển vị Johnson-Claisen).
Phần thứ hai của quy trình tổng hợp được thể hiện trong sơ đồ dưới đây:

2) Xác định cấu tạo các hợp chất D - L, biết rằng G có chứa 2 liên kết đôi
cấu hình trans. Chú ý: Me = methyl; Et = ethyl; Bu = butyl; Ph = phenyl.

518 | Bản quyền thuộc về Tạp chí Olympiad Hóa học KEM
Bài 182
Tháng 10 năm 2016, Kenkichi Sonogashira tròn 85 tuổi. Năm 1975, ông
đã công bố phương pháp ghép cặp chéo giữa các aryl halide và alkyne,
tiến hành trong các điều kiện rất êm dịu với sự có mặt của các xúc tác kim
loại chuyển tiếp.

Trong xúc tác Sonogashira, phức chất Y được điều chế từ muối Z (hóa chất
thương mại sẵn có) và triphenylphosphine. Muối Z được tạo thành bằng
cách đồng kết tinh hai dung dịch halide Q và R. Biết rằng hàm lượng kim
loại chuyển tiếp trong Q, Z, Y là 60.01 %, 36.17 % và 15.16 %.
1) Xác định các halide Q và R, muối Z và thành phần phức chất Y. Đề xuất
cấu tạo của phức chất Y.
Phản ứng của Sonogashira đã được đưa vào “kho vũ khí” của các nhà tổng
hợp hữu cơ, như là một phương pháp đáng tin cậy và phổ biến để tạo ra
các liên kết carbon-carbon sp2-sp. Năm 2003, các nhà hóa học từ Texas
đã xuất bản một bài báo, mô tả quy trình tổng hợp các NanoPutian - các
phân tử hữu cơ có hình dáng giống người, kích thước khoảng 2 nm. Trong
sơ đồ tổng hợp dưới đây của NanoKid, phản ứng Sonogashira được sử
dụng 4 lần.

2) Xác định cấu tạo các chất A - L và NanoKid (N).

519 | Bản quyền thuộc về Tạp chí Olympiad Hóa học KEM
Bài 183
Năm 1992, một số hợp chất có cấu trúc tương tự, gọi là các
decarestrictine, được phân lập từ nấm mốc Penicillium Simplicissimum.
Hiện nay, có 20 decarestrictine được biết đến và kí hiệu bằng các chữ cái
khác nhau, từ A - O (ví dụ, “decarestrictine A1”). Nhiều trong số các hợp
chất này có những tính chất đáng chú ý, do khả năng ức chế tổng hợp
cholesterol. Chính vì vậy, những năm trở lại đây, nhiều nghiên cứu đã
được tiến hành để tổng hợp nhóm chất này. Năm 2015, quy trình tổng
hợp đầu tiên của decarestrictine G, một lactone đại phân tử với vòng 10
cạnh, đã được tiến hành theo sơ đồ dưới đây:

1) Xác định cấu tạo các chất A - E và decarestrictine G (không cần xác
định hóa lập thể). Cho biết: a) Hệ OsO4/NMO được dùng như một tác
nhân chọn lọc cao và hiệu quả, thay thế cho dung dịch KMnO 4 trung
hòa; b) N, N'-dicyclohexylcarbodimide (Cy-N=C=N-Cy) tạo điều kiện
cho các phản ứng ngưng tụ bằng cách kết hợp thêm 1 phân tử nước.
Khác với hầu hết các decarestrictine khác, decarestrictine L không phải
lactone và không có vòng 10. Tuy nhiên, hoạt tính sinh học cao hơn đã
khiến nó thu hút được nhiều sự quan tâm từ các nhà tổng hợp hữu cơ. Sơ
đồ dưới đây là quy trình tổng hợp đầu tiên của decarestrictine L, được
thực hiện vào năm 1993.

520 | Bản quyền thuộc về Tạp chí Olympiad Hóa học KEM
2) Xác định cấu tạo của các hợp chất F - P và decrestrictin L, biết rằng: a)
ở giai đoạn đầu tiên, chỉ có duy nhất 1 vòng epoxide bị ảnh hưởng và
phản ứng diễn ra theo một cơ chế liên quan đến cơ chế SN2; b) hợp
chất L chứa 2 vòng.

521 | Bản quyền thuộc về Tạp chí Olympiad Hóa học KEM

You might also like